Sei sulla pagina 1di 339

CLEC'TIC

EDUCATIONAL

CRIES.
HEEJJiS.
i

'J

^"/^

cC

NEW

PRACTICAL

ARITHMETIC

Revised

Edition

of

the

Practical

JO^^'E'PH" KA'Y/K/R-,:
Ijjie Professor in

YAN
137WALNUT

ANTWERP,
Street,

CINCINNATI.

Woodward

Arithmetic

College.

BRAGG

"
28

Bond

NEW

CO.
Street
YORK.

.^'"Yl"^^^^^

RAY'S

^f^^f^

SERIES,

MATHEMATICAL

^e/^V,
^_

Arithmelic.

Ray'^s

Neiv

Primary

Ray^s

New

Intellectual

Rafs

New

Practical

Ray^s

New

Elementary

Rafs

Higher

Ray's

Test

Arithinetico
Aritlimetic.
Arithmetic.

Arithmetic,

Examples.

Ray^s

Neiv

Elemeittary Algeh'a.

Rays

New

Higher

Ray^s

Plane

Ray^s

Geojnetry and

Rafs

Analytic

Geometry".

Ray^s

Elements

of Astronomy.

Ray's

Surveying

Ray's

Calculus.
i0^d\l\ii^gral\
D"j^'er2ntial

and

Algebra.

Solid

and

Geometry.

Trigonouietry.

Navigatian.

Copyright
1877
BY

Van

Antwerp,

EDUCATION

Bragg

"

Co.

OEfY*

ECLECTIC

VAN

ANTWERP,
CINCINNATI.

PRESS,
BRAGG

"

CO.

ytitM{

PREFACE.

Changes
modes

of

methods

in the

Practical

No

other work

pupilshave
its

Arithmetic

on

of the land,

gained
principles.More

forth from
In view

in

made

schools

and

in the

it necessary

to

revise

our

Arithmetic.

popularity. Teachers
breadth

have

transactingbusiness

Ray's

and

of instruction

are

ever

so

extensive

use

every-where, throughout
familiar with

their arithmetical
than

had

ten

its pages, and

knowledge

thousand

from

editions

of

or

the

spread
wide-

length

millions
the

of

study of

it have

gone

the press.
of these facts,it has been

the constant

aim

in

making

this

revision to preserve carefully


those distinctive features of the former
constituted
the peculiarphilosophical
method
of its
editions,which
learned

author, viz.:

tion
Every principleis clearlyexplained by an analysisor soluof simple examples, from
which
Rule
is derived.
This is
a
followed by graduated exercises designedto render the pupil familiar
with its application.
2d. The
is strictly
philosophical;no principleis
arrangement
the pupil is never
anticipated;
requiredto perform any operation
until the principleon which
it is founded
has first been explained.
The changes made
fall naturallyunder two heads: (1) those which
of instruction;(2)
methods
adapt the book better to the advanced
those which
exhibit present methods
of computation in business.
In the first place,specialattention is invited to the beauty and
elegance of the typogranhy^^^
The^^ffiyantmatter of the volume,
961b;"y
(iii)
1st.

PKEFACE.

IV

definition, the

the

by

of

difference

is

rule,

at

series

running
convenience

of

of

the

indicated

clearly

once

articles, with

numbered

text-book

recitation

for

reference.

for
The

analytic

solutions

All

obsolete

separated.
Beer

Measure

such

as

and

drams,

written

Tables

Cloth

etc.,
is

is

and

Weights
and

all

proper

and

the

such

as

denominations,

Metric

accordance

its

carefully

Measures,

The

in

assigned

been

obsolete

discarded.

presented

and

usage,

of

are

have

operations

Measure,

Measures

extended

widely

and

roods,

and

Weights

of

System
with

its

now

place immediately

Decimals.

after

few

been
and

definitions
has

both

features,
for

have

the

The

publishers

desire

present

edition.
P.

to

accuracy;

real

for
the

has

throughout

to

publishers

been

present

How

improvements.

again

submitted

work

of

to

work

those

August,

engaged

1877.

due

the

wish

to

and

this
in

of

reiterate

object
the

here
and

be

demands.
the

many
in

W.

this

Smith

has

that

the

laborious

and

and

object
scientific

methods,
been

the

revision.

utility with
best

met

Discount

to

M.

to

interesting

embodied

Prof,

the

made

importance

are

practical

age
Percent-

of

been

thanks

features

embracing
far

education.

Cincinnati,

is

combine

to

their

corrections

valuable

many

has

Interest

their

express
and

instances

subject

novel

of

which

mention

Especial

Morgan

subjects

attention

suggestions

conclusion,

In

The

careful

whose

A.

time.

The

many

tions,
Frac-

classification, will

and

subject-matter

many

endeavor

an

applications;

that

teachers

and

of

principles

in

simplified.

expanded,

first

received

been

numerous

of

and

rewritten,

have

much
its

Factoring

as

entirely
rules

been

systematize

with

such

subjects,

have

Mr.

the

the

or

type.

enhances

paragraphs,
and

solution,

with
secured

all
is

responsible

CONTENTS.

OF

TABLE

PAGE

The

Arabic

The

Koman

System

of

System

oe

Notation

20

Notation

Addition

22

Subtraction

31

Multiplication

39
in

Contractions

47

Multiplication

50

Division

54

Division

Short

.
,

Division

Long

59
in

Contractions

64

Division
.

General

Principles

of

Division

67

Numbers

Compound
United

71

States

72

Money

Bills

Merchants'
Reduction

of

Dry

Measure

Rules

for

83

Compound

Reduction

87

Avoirdupois

88

Weight

89

Measure

90

Measure

Square

Time

Measure

Liquid

Solid

84
84

Long

Numbers

or

Cubic

90

Measure

94

Measure

96

(V)

CONTENTS.

Vi

PAGM

Addition

of

Subtraction

97

Tables

Miscellaneous

Numbers

Compound
of

Ill
....

113
115

Time

118

Factoring
To

Find

the Greatest

To

Find

the Least

Common

Common

Divisor

123

....

125

Multiple

....

127

Cancellation

Fractions

131

Principles

135

Reduction

137

of Fractions

Addition

144

of Fractions

Subtraction

147

of Fractions

of Fractions
Multiplication
Compound Fractions

Division

154

Fractions

157
Numbers

Compound

Fractional

159

Practice
Decimal

149

152

of Fractions

Complex

165

Fractions
Reduction
Addition

Decimal

Measures
Land

or

Measures
Measures
Table

178

of Decimals

179

Decimals

of Decimals

Compound

Metric

175

of Decimals

of
Multiplication
Division

168

of Decimals

Subtraction

The

102

....

of Compound Numbers
Multiplication
Division of Compound Numbers

Longitudeand

106

Numbers

Compound

180
.

Numbers
.

183
186

System

189

of

190

Length
Square Measure
of Capacity
of Weight

of Values
........

192

192
193

194

CONTENTS.

vii

PAGK

197

Percentage
for the four

Formulas

of

cases

Percentage.

203

205

Applicationsof Percentage
Transactions

Mercantile

206

Commission

206

Trade

208

Discount

Profit and

210

Loss

213

Transactions

Stock

214

Brokerage
and

Assessments

Dividends

.215

Stock

Values

216

Stock

Investments

217

219

Interest

Simple
The

221

Interest
Per

Twelve

Cent

for the five

Formulas

Compound

Method
cases

of Interest

229
.

Interest

237
237

Annual

Interest

239

Partial

Payments

241

Discount
Bank
True

247
Discount
Discount

247
.256

Exchange

260

Domestic

Exchange
Foreign Exchange
EnglishMoney
French
Money
German
Money
Canadian
Money

261

262
262
'

263
.

263
263

265

Insurance
Fire and

Marine

Life Insurance

265

Insurance

267
.

CONTENTS.

viii

PAGK

Taxes

269

State

and

United

Local

States

Internal

Taxes

269

Revenue

273

Revenue

274
.

Duties

Customs

or

274
....

...

Ratio

.276

Principles

280

Proportion

282
285

Simple Proportion
Compound

Proportion

289
.

Partnership

291

293

Bankruptcy
General

Average

Partnership
Equation

of

with

293

"

Time

294

Payments

295
297

Average

298

Involution
,

300

Evolution
.

Square
Cube

309

Root

316

Mensuration
Surfaces

Measurement

of

Measurement

of Solids
of Mensuration

Geometrical

316
323
328

331

Progressions
Arithmetical

302

Root

Applications

Progression
Progression

331
334

1.'

Article
as,

2. A

3. Arithmetic

Numbers

6. Two

of

and

in two

ways

is

method

Notation

of

Arabic

ARABIC

is

one,

the

art

of

first,by words;
of

expressing

use,

the

is used

in

in

are

system

SYSTEM

6, 7, 8, 9,

OF

numbers,

express

characters, called

ten

its characters

; as,

Arabic
all

our

calculations.

To

Eemark

nimibers,

Notation

The

Roman.

THE

1.

of

expressed

systems

arithmetical

2,

units

more

characters.

by

and. the

or

one

kind;

characters.

System

numbers

of

any

them.
are

second, by
5. A

men.

treats

by

computing
4.

one

cents, nine

of

single thing
dollar, one
pound.

consists

Number

five,seven

is

Unit

apple,

one

one,

1.

the

NOTATION.

Notation

Arabic

ploys
em-

figures; namely, 1, 2, 3, 4, 5,

0.

1.

"

The

Arabic

appear

to

have

System
been

of

N'otation

introduced

is
into

so

called

Europe
(9)

because

bj' the

ARITHMETIC.

PRACTICAL

NEW

RAY'S

10

acknowledged
o-cnv^r.illy

Arabians; tut i'cis now

they originated

that

India.

in

Not'ition
2^; ^T'ae 'A^f'J^hia

Rem.

and

^ClnrvrTio'ii
'%,s^m.

the

2. The

of

Order

Decimal

the

System

'

'

figureis

called

is also

"

place it occupiesin

the

number.

UNITS

3.

unit

1. A

written

is owe,

1.

unit

and

one

more

are

two^

2.

Two

units

and

one

more

are

three^

3.

Three

units

and

one

more

are

4.

Four

units

and

one

more

are

four^
five.

Pive

units

and

one

more

are

six^

6.

Six

units

and

one

more

are

seven.

7.

Seven

units

and

one

more

are

eight,

8.

Eight

units

and

one

more

are

nine,

9.

because

nine

characters

they denote

3. The

cipherand
4. When

called

are

fill vacant

orders.

figurestands

alone

to

5.

significant
figures,

something.
0, called naught,stands

character
is

use

the

singlething

or

UNITS.

OR

ORDER,

FIRST

One

2. These

its

THE

OF

The

for

nothing;

0 is also

called

zero.
a

rightof

number,

it

or

in the

represents one

first

place at
units

more

or

of the firstorder.
5. Units
and

the

of

Iso

1. Nine
is

first order

place they

UNITS

4.

the

OF

units

is called

occupy

THE

SECOND

and

represented by

are

one

the

called

simply units;
units' place.

the

OR

ORDER,
more

figure 1

are

TENS.

called
but

the

ten;
one

it
is

NOTATION.

made

to

is written

Two

tens

are

twenty^ written

Three

tens

are

thirty,

Four

tens

are

Five

tens

are

Six

tens

are

Seven

tens

are

Eight

tens

are

Nine

tens

are

3. When

the

the second

order.

4. Units

of

placethey

the

occupy

1. The

etc.,are
which

right by

10.

20.
30.

forty,

"

40.

fifty,
sixty,

''

50.

"

60.

seventy,

''

70.

eighty,
ninety,

"

80.

^'

90.

number

second

order

is called

TENS

5.

stands

right,it represents one

placefrom

the

'^

figure in

from

thus

ten

2. One

of

place
phice.

units'

in the

writing a

second

the

occupy

11

numbers

are

the

AND

tens'

or

called

in the

second
units

more

tens; and

of
the

place.

UNITS.
,

between

10

and

expressed by representingthe
they are composed.

2. One

ten

and

one

unit

are

One

ten

and

two

units

are

One

ten

and

three

units

are

One

ten

and

four

units

are

One

ten

and

five

units

are

One

ten

and

six

units

are

One

ten

and

seven

units

are

One

ten

and

eight units

One

ten

and

nine

Two

tens

and

Two

tens

and

20,
tens

eleven,
twelve,
thirteen,

and

20

and

written

30,
units

11.

"

12.

^'

13.

"

14.

"

15.

"

16.

seventeeti,

"

17.

are

eighteen,

"

18.

units

are

nineteen,

'"

19.

one

unit

are

''

21.

two

units

are

twenty-one,
twenty-two,

''

22.

fourteen,
fifteen,
sixteen,

PK

NEW

KAY'S

12

ACTIO

TO

]S^UMBERS

AKITHMETIC.

AL

WRITTEN.

BE

Twenty-three; twenty-four; twenty-five;twentysix; twenty-seven; twenty -eight


; twenty- nine.
2. Thirty-seven; forty-tw^o
enty-three
; fifty-six
; sixty-nine
; sev; ninety-four.
; eighty-seven
ty-six:
thir3. Eighty-three;
fifty-one;
ninety-nine;
forty-five;
sixty-two.
seventy-eight;
4. Fifty-five
;
; sixty-seven
; ninety-three ; eighty-one
forty-nine
; thirty-eight.
; seventy-four
-two ;
5. Seventy-six; forty-four
fifty-seven
;
; eighty
thirty-five
; ninety-one
; sixty-three.
1.

TO

NUMBERS

BE

READ.

97.
92.
95.
64.
48.

UNITS

6.
the
tens

OF

1. Ten

figure 1
and

units

THE

tens

THIRD
are

written

one

in

being each

ORDER,

hundred;
the

third

filled with

OR

HUNDREDS.

it is

representedby
order,the orders of
a

cipher.

NOTATION.

the

third

the

of

2. Units

placethey

order

the

between

and

hundreds;

hundreds'

AND

TENS,

numbers

1. The

called

are

is called

occuj^y

HUNDREDS,

7.

13

place.

UNITS.

100

and

200,

and

200

expressed by representing the hundreds,


units of which
they are composed.

300, etc.,are
tens, and

hundred

2. One
written

written

and

hundred

and

one,

ten,

one

ten

are

one

hundred

and

one

ten

and

one

unit

one

and

hundred

written

are

dred
hun-

111.

eleven,written

and

tens

two

are

hundred

one

and

twenty,

120.

hundred,

two

tens, and

1. One

hundred
and
2. One

and

hundred

fifty-six
; seven

hundred

and

hundred

hundred

one

and

and

hundred

hundred

one

and

are

WRITTEN.

thirty;one

fifty
; one

seventy;

BE

TO

and

hundred

five units

125.

written
twenty-five,

NUMBERS

one

hundred

one

are

110.

One

and

and

hundred

One

unit

one

101.

One

One

and

and

sixty;
eighty.

forty;

one

dred
hun-

and
twenty-three
; four hundred
hundred
and
eighty-nine
; one
hundred
and
fifty-eight
; three

forty-seven
; two
hundred
and sixty-nine.
3. One

liundred

and

and
five ; six hundred
five hundred
thirty-four;
and

two;

hundred

three

and

forty-

and
hundred
seventy-eight
; two
and sixty-seven;
eighthundred

ninety.

4. Four

hundred

and

-three
fifty

seven

and twelve; two


eighty-six;nine hundred
and fifty
thirty; four hundred
; six hundred

hundred

and

hundred

and

and

seventy.

RAY'S

14

ARITHMETIC.

PRACTICAL

NEW

four
hundred
.and
fifty-three;
dred
and
hundred
twenty-nine;one huneighty-six;seven
and
hundred
and three; four hundred
six; seven
5. One

and

hundred

and

nine.
TO

NUMBERS

OF

UNITS

by
2. Ten

10000

one

HIGHER

hundreds

1. Ten

8.

1 in the

thousands
hundred

are

fourth
form

READ.

BE

ORDERS.

thousand;
order; thus, 1000.
one

unit

of the

it

is represented

fifth order ;

thousands,a

unit of the sixth

units

order

thus,

order ;

thus, 100000, etc.


3.
next

ten
Invariably,
higherorder.

4. The

from

the

names

of

the

first nine

make

orders

unit

of

8th.

7th.

6th.

Orders.

of

5th.

4th.

3d.

2d.

t
o

'T3
O

T3
o

1^

^
a
o

the

be learned

may

followinoTable

9th.

of any

Ist

NOTATION.

DEFINITIONS

9.

1. The

nine

first nine

15

AND

PRINCIPLES.

numbers

represented by

are

the

figures,1, 2, 3, 4, 5, 6, 7, 8, 9.
2. All other numbers
are
representedby combinations
of the ten figures,1, 2, 3, 4, 5, 6, 7, 8,
of two or more
"

"

J,

0.
3. The

called

numbers

4. The

numbers

odd

with

2, 4, 6, 8,

or

are

that

end

with

1, 3, 5, 7,

or

are

numbers.

5. The

value

6. The

value

depends upon
Rem.

end

numbers.

even

called

that

of

figureis

of
the

it

presses.
ex-

figure is always local; that is,it


in a number.
placeit occuj)ies

principleof local
System of Notation

the Arabic

of units

The

"

the number

value

is Avhat

peculiarly
distinguishes

all other

from

systems that have

existed.

7. The

figureexpresses w^hen it stands


units' placeis called its simple value.
8. The value of a figureis increased
ing
tenfoldby removit one
placeto the left.
9. The
value of a figureis decreased
ing
tenfoldby removit one
place to the right.
number

GROUPING

1. For

10.

the

three
Rem.

by

INTO

ORDERS

convenience

in

diflPerent orders

orders
"

OF

in

PERIODS.

writing and

are

bers,
reading numgrouped into periodsof

each.

number

is

pointed offinto periodsof

three

figureseach

commas.

2. The

the

first three
or
first,

orderp units,tens,
"

unit

period.

hundreds

"

stitute
con-

ARITHMETIC.

PRACTICAL

NEW

RAY'S

16

orders
of three
thousands^ten
group
thousands
constitutes the second,or
thousands, hundred
3. The

second

"

"

period.

thousand

third

4. The

period.
periodsfrom the first to the
the following
learned from

5. The
be

Table

of

6. The

in the

orders

the

constitutes

million

third,or
may

three

of

group

grouping
following

inclusive

Periods.

of

the

twelfth

orders

into

shown

periodsis

Table.
3.

4.
Billion.

Million

2.

1.

Thousand.

Unit.

TU

fl
o

't:!

9
o

a?
^

'-^

IS

en

S B
H
pq

mII

O
3

3
H

NOTATION.

7. It

is

plain that

hundreds

lens J and

1. Write

11.
hundred

and

and

hundred

period is composed of unitSj


iwriod.

that

six

the

in

hundred

six

twenty

million

seven

each

Xumhers

Write

To

of

17

Arabic

and

-four
fifty

billion nine

-one

hundred

and

System,
trillion three

hundred

and

eightythree

thousand
fiftj-four

twenty-one.
-d

g
o

^-

c3

G
o

4,

1,

1) 8

^
G

7,

number
vacant

Rem.

WhP

Begin at the left,and write


composed of himdreds, tens, and
orders with ciphers.
"

"

hundreds

In the left hand


and

tens

are

however,
period,

wanting, the

4,

":
c

MhP

Rule.

vacant

WhP

each

period as a
units fillingthe
"

the hundreds

when
orders

are

not

or

the

filled with

ciphers.
NUMBERS

TO

BE

WRITTEN.

2. Two

thousand; thirtythousand,

3. Five

million ;

4.

sixtymillion ; seven
Eight billion;ninety billion;one

four

hundred

hundred
hundred

sand.
thou-

million.
billion.

KAY'S

18

thousand

5. One

sand

milHon

9. One

One

12.

and

forty-five.
nine

dred
hun-

thousand

sand
thoufil'ty-seven

hundred

one

and

and

and

forty-three

ninety-two.
thousand

one

one;

and

ten;

hundred.

one

thousand

hundred

one

and

hundred

and

twenty-four.

hundred

seven

thousand

one

and

million
vSixty-eight

11. One
one

hundred

three

hundred

nine

thousand

hundred

twelve.

and

10.

dred.
hun-

six thou-

fifty
;

seventy-eightthousand

and

hundred

and

one

eighty-nine.

three

thousand

7. Tw^elve
8. Six

and

thousand

; two

hundred

four

hundred

seven

hundred

two

thousand

G. Three

ARITHMETIC.

PRACTICAL

NEW

ten ;

and

one

thousand

one

thousand

one

hundred

one

and

eleven.
13. Two
14.
15.
16.

thousand

and

three;

four

thousand

and

fifty.

and
tw^enty-six.
Forty-five thousand
hundred
and
two
one.
Eighty thousand
Ninety thousand and one.

17. Four

hundred

and

thousand

ten

two

hundred

and

five.
18. One

hundred

19. Three

thousand

million

and

ten.

thousand

seventy

five hundred

and

nine.

Forty-five
twenty-six.
20.

21. Nine

hundred
hundred

23.

billion two

Forty

25.

Eighty

five hundred

two

hundred
seven

four.

million
ten

hundred

and

billion
and

nine

million

hundred

thousand

eighty-three

and

22. Seven

24. Seven
une

million

thousand

and

ninety thousand.

thousand

and

tw^o.

thousand

and

five.

million
twenty-six billion fifty
and

forty-three.

hundred

and

three

million

NOTATIOJ^.

12. Numeration

is the

accordingto
Read

To

reading of

JYutnbers

654321987654321.

"

in

the

Arabic

1^

HP

numbers

4,

1,

when

pressed
ex-

of notation.

system

1. Eead

19

Sr/stein.

7,

4,

OD

1.

+J

a,

W^p

W^P

W^P

wIp

H^P

and point offthe number


info
Begin at the right,
periodsof three figureseach.
2. Beginat the left,
and read each jjeriod
a number
as
posed
comof hundreds,tens,and units,givingthe name
of the
period.
Rule.

Rem.

1,

"

1.
"

The

left hand

periodwill

sometimes

contain

hut

one

figures.

two

Rem.

2.

"

It is

customarj^to omit the

NUMBERS

2.
3.
4.
5.
6.

TO

BE

name

of the unit

period.

READ.

4923176358.
41582; 763491; 2519834; 375486921;
37584216974; 432685729145; 6253971438267.
1300; 2540; 6070; 8009; 13200; 1005.
682300; 8600050; 3040; 50004; 704208.
7085; 62001; 400009; 2102102; 9001003.

oi

ARITHMETIC.

PRACTICAL

NEW

RAY'S

20

7.

130670921; 6900702003; 23004090701; 9420163070.

8.

570000010326049;

9.

45763000020108000507.

200103478511992485.

800820020802008.

10.

NOTATION.

OF

SYSTEM

ROMAN

THE

DEFINITIONS.

1. To

13.

The

"

is

now

Roman

Notation

ploys
em-

used

System of Notation is so called because it was


It
used by the ancientRomans.
expressingnumbers
mark
the chaptersof books, the dial platesof clocks,

Roman

of

the method

the

letters;namely, I, V, X, L, C, D, M.

seven

Rem.

numbers,

express

to

etc.

2. In

Koman

the

Notation,numbers

arc

expressedin

four

ways.
1st. Each

of

the

seven

letters expresses

numher,

as

lows:
fol-

V, five; X, ten; L, fifty;C, one hundred ;


D, fivehundred ; M, one thousand.
2d. Seven numbers
ters
are
of the letexpressedby repetitions
C.
7, JT, and
Thus, II represent two; Hi, three;
XX, twenty; XXX, thirty;CC, two hundred; CCC, three
I, one;

hundred

CCCC, four

3d. Four
as

numbers
follows:

hundred.

nation,
expressedby a subtractive combiIV, four; IX, nine; XL, forty;XC,
are

ninety.
4th. All other numbers

of
smaller

two

or

more

being always

of

are

the

tions
formed by additive combinanumbers, the
eighteen
preceding

situated

on

the

right of

the

larger

number.
For

example,YI is six; XYII, seventeen; LXXYIII,


hundred
and
one
seventy-eight
; CLXXXIX,
eighty-nine
\
MDCCCLXXYII,
-seven.
eighteenhundred, and seventy

NOTATION.

Write

the

in

21

J^otation,

Koman

1. The

numbers

from

one

2. The

numbers

from

twenty

to

twenty.
to

FUNDAMENTAL

THE

thirty.

RULES.

DEFINITIONS.

14.

1. An

2. Numbers

3. An

12,

are

abstract

number.

whole

either

abstract

number

is

or

concrete.

number

5,

simply, as

20.

4. A
or

is

integer

number

concrete

5. The

number

applied to

one

apple,5 pounds, 12 men.


number
is
of the objectof a concrete
Thus, in 5 pounds, the denomination
1

objects;as

more

is

name

denomination.

its
is

pounds.

or

either

6. Numbers

are

7. A

number

simple

concrete;
8. A

as

3,

numbers

quarts

compound.
is a singlenumber, either abstract
dollars,1 pint.

number

compound

concrete

simpleor

is made

of different

up

denominations

of

two
;

as

or

more

pecks

pint.

primary operationsof Arithmetic;


and Division;
Multiplicatio7i,
namely. Addition^Subtraction,
"

9. There

are

these

called

are

four

the

Fundamental

Rules.

1. If

15.

will

many

Because

spent

how

12

find

and

cents

and

cents

for

cents

slate, and
I

did

cents

spend

and

for

cents

for

cents

an

ball

Joseph

weekly
for

cents

how

book

1. The

dition; hence,

2.

The

cents

for

poems,

and

cents.

22

these

Addition

is the

10

for

examples
of

for

cents

magazine,

cents

process

all

Why?

cents.

monthly
40

did

cents

paper,

cils,
pen-

30

novel

110
in

number

book
copy-

for

spend?

into

Why?

cents

many

daily

operation

numbers

more

of
he

did

for

cents

25

paper,

much

16.

gave

cents.

A71S.

4.
a

17

how

cost?

cents.

for

cents

orange,
:

are

Ans.
gave

how

cents,

Ans.

cents

many

3. John

have?

then

you

Why?
2.

have

you

cents.

is termed

uniting

Ad-

two

or

number.

one

obtained

by

addition

is

the

Sum

or

Amount.
3. When

operation
4.

The

when
are

be

the

is called

sign

placed
to

be

added

numbers

of

Addition

together.
(22)

added

(-|-),called
numbers,

two

thus,

be

simple,

are

the

of Simple Numbers.

Addition

between

added

to

means

j)lus,means
it

that

shows
4

and

more;

that
2

they
are

to

5. The

between
4

8ign
which
6

of

that

are

the

23

that

equality(==:)denotes

it sstands

means

NUMBERS.

SIMPLE

OF

ADDITION

the

equal; thus, the


of

sum

and

quantities
expression

2 is 6

it is

read, 4 plus 2 equals6.


Addition

17.
not

When

exceed

the

sum

of

9, it is written

Table.

the

figuresin

under

the

column

eokimn

added.

does

KAY'S

24

AKITHMKTIC.

PRACTICAL

NEW

Examples.
1. I

own

132

second,

the

the

acres;

the first contains

third,25

240

acres

how

acres:

many

in all?

acres

SoLUTiOiV.
write

of the

units

Begin
which

in

what

write

in

and

7 units,

are

5, and

are

place; 1 and
hundreds' place.

man

is the

sum

of my

3. Find

the

sum

of

4. Find

the

sum

of

18. When

the
or

acres,

132

acres,

4
3

2 5

S142,

and

$387.
8695.
53758.

figures in a
requiredto

figuresare

acres.

another

debts?

the

of

acres,

4321, 1254, 3120.


50230, 3105, 423.

sum

more

are

2 4 0

39

$210, another

one

9, two

and

can

write in tens'

hundreds, which

owe

most

right,and say
units' place; 2

9 tens, which

$35:

in the

not be added

together,
the same
column, so that the figures
convenient
])ositio7i.

order in

same

be

may
at the

write

2. I

orders
different

Since units oi

"

to be added

are

of land

tractH

column

ceeds
ex-

exi)re8S it.

Example.
1. Add

the

Solution.

which

are

1
are

say

(0

no

ten, carried
are

Write

"

Then

column.

numbers

3415, 503, 1870, and

units of the
and

to the

11 tens, which

tens;
are

1 and

5, and

are

) units, written
2

order

same

3, and

are

1 ten, written

written

23, and 4

are

in the hundreds'
2 and

place.

are

are

27

place,and

3, and

are

are

the units'

in

units,
1

10, and

are

are

place,and

3415
503
18

70

922
6 710

10, and 8

hundreds, which
2

10

same

place,and

in the tens'

hundred, carried to the hundreds; 1 and


18, and 5

in the

922.

are

thousands, carried
thousands, written

hundreds,

to the thousands;

in the

sands'
thou-

ADDITION

CaiTying the

be

can

column,

with

the

the

sum

of the

can

be

figuresin

Solution.
sands'

column, the

of the

exceeds

9, the

higherorder.

next

of

adding

the

thou-

the

the

addition

with

^^^

adding the hundreds,


the sum
is 26 hundreds, which
and 6
equal 2 thousands
is 10 tens, equal
hundreds; next adding the tens, the sum
to 1 hundred; and
is 10
finallyadding the units,the sum
have
also to be
units,equal to 1 ten. As these sums
added, this much

complete
19.

the

2.

down

the

same

Begin at

the

the

column

1. Find
In

the

the entire

"

order

the

mnnhers

may

stand

righthand,

of

sum

Add

the

and

in order

to he
in the

add

obtained

units

carry the tens

it,and

Proof.

done

4
10

__1_9

to

6710

Write

1.

"

Place
under

be

must

f."^o

solution.

Rule.

figuresof

work

extra

units'

3415

is 4; next

sum

tens

example.

Commencing

"

column

greater convenience

take the above


first,

column

beginsat the righthand


lowest
order,so that, if

the

any

sum

illustnitc;the

To

of

units

carried to the

"

added.

addition

convenience,the

Rem.

25

is

tens

order

same

For

NUMBERS.

dreds
simply adding tens to tens,hunthe principlethat only units
hundreds, etc., on

to

of the

SIMPLE

OF

added,

by adding

to the next

coluinn.

same

each

that

so

column
each

rately.
sepacohwm

higherorder.

Write

the last column.

columns

downward, commencing

with

of units.

the

sum

of
cokimns

adding long

to retain

the

numbers

placing

them

in

columns,

as

3745, 2831, 5983, and


of

carried.

smaller

3, 2, 1, in the

7665.

figures,it

is necessary

This

be

may

figures under
margin.

done

their

by

3 745
2831
5 9 83

proper

7"ak
9 0 9 2 4

321

liAY'tS

26

AKITHMETIC.

PRACTICAL

NEW

Examples.

(2)

(3)

(4)

(5)

(6)

184

204

103

495

384

1065

216

302

405

207

438

6317

135

401

764

.^-85

348

5183

320

311

573

825

843

7102

413

109

127

403

483

3251

101

43

205

325

834

6044

13.

11

14.

23

+ 22 + 33 -(-44 + 55
+ 41 -I 74 + 83 + 16

15.

45

19

16.

51

48

17.

263

18.

94753

how

many

has

how

man}^

165.

how

many

237.

how

many?

how

many

264.

how

many

843.

9386

258

225.

3456

are

204388.
has

31

days ; February,28 ; March, 31 ;


May, 31 : how
days are there in
many

five months?

20. June

155

+ 93688

January
April,30 ; and
these

+ 321 +

+ 2847

19.

+ 32 + 74 + 55==
4
+ 76 + 85 +

104

(7)

151.

days;July,31; August, 31; September,


153.
30; October, 3l : how many
days in all?
have 151 days, the next 5 have
first 5 months
21. The
has
153
30, and December, 31 : how
days, November
365.
days in the whole year?
many
30

ADDITION

SIMPLE

OF

NUMBEES.

27

bought 4 piecesof muslin : the first contained 50


yards,the second, 65, the third,42, the fourth,89 : how
246 yd.
yards in all?
many
22. I

23. I
a

fourth

$96

how

24. General
lived

S245,

man

one

owe

much

years: in what
the Great
25. Alfred
67

died

A.

Eevolution, 560

American

Eevolution

has

and

43

in the
416

lambs

the

; in

sheep and

second,

sheep and

third,370

lambs:

95

$408

for

did

28. The

barrels

many
he

; and

is their

sum?

the

year

187

first

to

the

to

the

did

the

lambs

65

are

sheep and
; in

sheep

105

lambs

the

fourth,

sheep and lambs


sheep,and 486 lambs.
of pork for $285, 18

many

barrels
for
did

$235,
he

and

buy,
105

first of four

and

1799.

901 ; thence
years ; thence

pay?

third,841

what

30

1732,

die?

in the

how

barrels

23

$144,

how

dollars

the

bought

man

$187,

1775.

1038

27. A

D.

A.

in what

243

has he?

barrels

D.

years:

begin?
of sheep;

flocks

he

314

was

third

$853.

born

was

did

American

$325,
?

owe

year

signing of Magna Charta

26. A

do

Washington

another

numbers

fourth,as

34

and

barrels

how

for

many

bbl.,and

$1072.

is 287 ; "^^^he
second, 596 ;
much
the first three :
as
3448.

29. The

built 1700
Pyramids of Egypt were
years
before the founding of Carthage ; Carthage was
founded
47 years
before and was
destroyed607 years after the
foundingof Eome, or 146 years before the Christian era.
How
the
Christ
before
were
Pyramids
years
many

built?

2500.

30. Add
six hundred
seven

twenty

three
and

thousand
thousand

31. Add

275432

thousand

and

five; fort^^-twothousand

twenty-seven ; 105
and
four; 80079;

; three

hundred

and

three

hundred

and

six hundred.
;

four

hundred

753420.
and

two

thousand

and

KAY'S

28

thirty;

three

million

two

PRACTICAL

NEW

ARITHMETIC.

three

thousand

five ; 872026

and

thouBand

hundred

hundred

and

four

forty-seven.
5851840.

and eighty million


dred
eighthundred
eighthunand
eighty-nine
; seventy-seven million
; 2002002
and thirty-six
and
hundred
thousand; two hundred

32. Add

four

five thousand

million

six

hundred

and

33. North

West
of

34. A

man

$250
for

$120

of his two
what

for

for
and

for

thousand

miles:

for

what

merchant

and

the

for

for

of his

for

carpenter-work,

painting: how

to

his

wife,$3275

of his three

spent $8785

for dress

goods,and $12789
goods at a profitof $878,
for how

much

$12875, bank

worth

$4785.

he worth
38. A

In

stock
one

at its close?

house

has

did

$23702.

began

w^orth

to each

$22350.

he sell the whole?


merchant

much

daughters:

bequest?

sheetings. He sold the dress


of $1250:
the sheetingsat a profit

37. A

work,
stone-

$2700.

to each

$2650

area

building

him?

$7850

the

sq. mi.

for

cellar,$120

$115

lot cost

and

sons,

nine

square

is

lot,$1325

the

digging

bequeaths

8955752

miles;

square

brick-work, $140

and

49003;

15967521

$600

pays

is the amount

36. A

of

area

square

and
plastering,

man

an

continent?

his house

35. A

nineteen

6917246

America,

materials, $30

did

has

Indies, 94523
entire

seven

2155513020.

America

the

and

nineteen.

South

miles;

ninety million

and

hundred

nine

hundred

two

business
worth

year

he

with

$7000

$5600, and

gained $3500

cash,goods
other
:

stocks

what

w^as

$33760.

parlors,each requiring30 yards


of carpet; four bed-rooms, each requiring 25 yards; a
each requiring20 yards:
dining-room and sitting-room,
how
yards are
required to carpet the entire
many
house?

two

200

yd.

An

20.
and

NUMBERS.

in
practice,

excellent

add

is to

accuracy,

SIMPLE

OF

ADDITION

columns

two

example illustrates

order

the

method

29

to

secure

at

once.

readiness
The

lowing
fol-

(!)"
Beginning

with

47, add

77 ; then

the 4 units,

141; and

the

and

the

finallythe
3 hundreds

carry
and 8
are

are
are

the

hundreds'

96, and

60

206, and

351, wnk'h

27744

60

156, and 2
are

30314

(8)

7 tens

above, 216;

above, 315, and


carry

the

Then

and

3 to

are

198,

93

158, and 40
7

273, and

are

70

7892
6 7 79
4865

6234

the 17, and

are

266, and

equal

the 6 tens above,

tens

column.

write in its proper

35249

(7)

are

down

Put

2 units, 317.
the

the

146; then

units, 225^ then


to

81 ; then

making

5 units,

above, which

the 3 tens

are

9347
35117

343, and

place.

3611570

(9)

3223341

(10)

30

62833

RAY'S

NEW

43702

PRACTICAL

97812

ARITHMETIC.

2802803

6685899

21.

If

1,
will

many

Because

Frank

and

apples,

15

apples
cents

from
after

apples

spending
Aiis.

If

you

4.

If

I have

take

l^encil,how

25

from

Subtraction

finding

the

The

Subtrahend

the

.Difference or
When

is called

one

be

sign

less;

that
the

given

Subtraction

meaning

the
on

taken

the

apples.
apj^les.

how

many

Why?

cents.

left?

Ans.

them

for

Ans.

15

5.

lead-

cents.

A71S.

8.

is

examples

is the

of

process

numbers.

two

the

number

the

Minuend;

left

after

less,

subtraction,

Remainder.

the

The

23.

and

are

preceding

called

is

many?

Subtraction

between

number

larger

how

the

in

hence.

difference

the

3.

of

10

how

away,

7,
8

left ?

leaves

twenty

are

many

spend

I have

operation

termed

2.

and

cents,
will

1. The

22.

13, how

from

much

Twelve

5-

give

Ans.

left?

were

to

had

9
left?

have

you

Why?
2.

have

you

numbers

of

Subtraction

placed

number

from

the

operation

of Simple Numbers.

when

the

simple,

are

left; thus,
8, and

is

"

between

the

on

"

read,

numbers,

two

right
5
8

is to
3

minus

called

is

be

taken

notes
it defrom

that

means

minus,

equcds
(31)

5
3.

is

RAY'S

32

NEW

PRACTICAL

Subtraction

When

24.
than

the

ARITHMETIC.

Table.

each

figureof the subtrahend is


correspondingfigureof the minuend.

not

greater

Examples.
1. A

left?

man

having $135, spent $112:

how

much

had

he

SUBTRACTION

Solution.

only

column,
made

Since

"

be

can

Begin
3, which
2, which

put
put

of

units

the
which

the

convenient

right,and

say 2 from

leaves

in

place; I

from

leaves

tens'

place;1

from

leaves

0, and, there being

the

figureson

no

order

same

in

the

subtraction

same

is to be

position.

units'

in

33

of the

order

same

most

the

at

units

NUMBERS.

between

figuresbetween

the

in the

be

may

SIMPLE

the difference

found, write

that

so

OF

^^^

il^
^^

left of

minuend.

subtrahend.
remamder.

this,the placeis vacant.

2. A

farmer

sheep had
3. A

$875:

What

123:

how

many
122.

bought

how

sheep,sold

left?

he

man

245

having

much

is the

farm

he

did

for

$751, and

sold

it for

$124.

gain?

difference

4. Between

734

5. Between

8752

6. Between

79484

and

203.

531?

and

5331.

3421?

54321.

25163?

and

7. Between

25.
than

49528

the

upper,

33113.

16415?

figurein any order


we
arises,which
difficulty

lower

the

When

and

is

greater

will

now

explain.
Examples.
1. James
cents

had

had
he

13

cents; after spending 5, how

manv

left?
1 3

can

6 from

not

be subtracted from

3, but it can

13 leaves 8.

2. From

73

Prac. 3.

subtract

be from

13;

^
"qT

45.

Solution.
1

Take

the

to

put in units' place. Since


but

remain

"

the 3

adding it to

2.

changed,since

tens

from

2 8

and

6 tens

pui

difference is 28.

The

the

8, and 4 from

is taken

which

ten

6 leaves 2.

of the

value

the

cases,

to the number

added

place.

the

from

Subtract

4 5

units; then,
8 units,to be

is taken

ten

tens.

13 leaves

such

In

"

13

7 3

(ten)or

actuallytaking 1 ten from the 7 tens, and


units,the operationis perfi.rmedmentally; thus,

6 from
Rem.

this 1

units.

of

Instead

1.

add

makes

the remainder, 2 tens, in tens'


Rem.

from

and
(tens),

units, which

taken

units,and there will remain

subtract the 5

tens, there

be

not

can

the 7

(ten)from

units

10

units

"

ARITHMETIC.

PRACTICAL

NEW

KAY'S

34

number
is not
upper
from the order of tens is

the order of units.

in

higher order and adding it to the


be subtracted
units of the next lower, so that the figurebeneath
may
is called borrowing ten.
from the sum,
4.
After increasingthe units by 10, instead of considering
Rem.
diminished
the next
as
figureof the up}"er number
by 1, the result
be increased
will be the same, if the next figureof the lower number
by 1; thus, in the previous example, instead of diminishingthe 7
makes
tens by 1, add 1 to the 4 tens, which
5; thus,5 from 13 leaves
Rem.

Taking

3.

"

unit

of

"

8, and 5 from

This process

5.

Rem.

7 leaves 2.

"

the 7 tens,

1 from

their sum,

or

3. Find

Solution
under
same

the fact that

to subtract from

have

having

it lx)th 1 ten

borrowed

and

4 tens,

5 tens.

the

"

we

depends upon

1st

difference between
Method.

"

Writing

805

the

and

637.

less number

8 05

the

order in the
greater,with units of the same
column, it is requiredto subtract the 7 units from

6 3 7
16

5 units.

The

five

because
which

can

it is
leaves

not

be

increased

0; therefore,borrow
7 hundreds

in

tens; then, borrowing 1


the 5 units,9 tens
will be
10

units'

place.

by borrowing from
hundred

hundreds'
ten

in

from
the

from

place;this
the

tens'

10

the

the 8
1

15

figure,

hundreds,

hundred

tens, and

place,and

next

makes

adding

it to

units in the

SIMrLE

OP

SUBTRACTION

NUMBERS.

35

Subtracting 7 from 15, 8 units are left,to be written in units*


place; next, subtracting3 tens from 9 tens, there are left 6 tens,to
from 7 hundreds,
be written in tens' place;lastly,
subtracting6 hundreds
there remains 1 hundred, to be written in hundreds'
place.
If the
5 units
be
increased
2d
Method.
10,
by
say 7 from
0 can
not be
15 leaves 8; then, increasingthe 3 by 1, say 4 from
6 by 1, sa^- 7 from
8
taken, but 4 from 10 leaves 6; then, increasing
"

leaves 1.
Rem.

The

1.

"

and
contains
Rem.

method

second

is

venient,
congenerally used; it is more
when
the upper number
especially

less liable to error,

ciphers.
2. "Begin

the

at

figureis greater than

that

right to subtract, so

the

upper,

1 may

be borrowed

if any
from a

lower

higher

order.
Rem.

3.

If the

"

number, the
from

sum

difference

will be

8 leave 3, then

26, Rule.

"

1. Write

placingfiguresof
2. Beginningat
the one
over
directly
3.
upper
next

If

sum

the loiver

the

is

"

Add

equal to

the

to 5 will

the

less number

the

the lower
one

minuend,

the

less

each

figurefrom
beneath.

the upper, add

to

the

the
work

greater,

column.

same

from it,and
from the next

remainder

to

the

under

write the remainder

figureexceeds
take

added

equal 8.

righthand, subtract

it,and

the

be

greater. Thus, if 5 subtracted

order in the

same

the

subtract
figure,
lower figure,
or

Proof.

equal to

3 added

numbers

of two

ten

carry
upper

subtrahend

one

to

the

to the

figure.
;

is correct.

if the

ARITHMETIC.

PRACTICAL

NEW

RAY'S

36

8209877.

4444444.

from

5. Take

1234567

6. Take

15161718

from

91516171.

76354453.

7. Take

34992884

from

63046571.

28053687.

8. 153425178

"

9. 100000000
10. Take

10001001==?

89998999.

"

17

from

cents

63

12. A

tree

75

feet

37

was

did

horse

the

than

fell

99579930.

carriagecost $137,

11. A
more

53845248==?

13. America

and

S65

horse

cents.

much

how

$72.

carriagecost?

the

broken

feet

high was
long: how high
discovered

was

46

cents.

the

was

the

38 ft.

stump?

Columbus

by

part that
in

1492

345.

years had elapsedin 1837?


many
in the bank
14. I deposited
$1840, and

how

how

dollars

many

15. A

has

man

$7426

16. A

worth

$10104, and
his

debts

debts

owes

paid,how

are

had

and

19. Take

the

how

from

4004

Subtract

22. What
hundred

25. Take

3995996.
million

two

hundred

sand
twenty thou1011924.

and

five

thousand

9238715
9909090009

between

thirteen million two


4101901.

in business

that

much

had

from

and

1595665.

17102102?

found

twelve
861928.

thirty.

and

only $26,967; how


Take

from

invested
he

and

2000687.

one

first year

hundred

nine

million.

is the difference

man

991.

50082.

four

and

four

from

and

23. A

of

1009006

hundred

twenty-two

many

20010.

excess

ten, above

20. Subtract
nine

from

19019

Eequired

thousand

24.

$11

$99989.

17. Subtract

21.

away

left?

he

18.

$2678.

having $100000, gave

man

$1365.

when

$475

out

will be left?

much

the

property

of

the amount

to

drew

1 left?

had

$30,000; at

all his
he

assets

lost?

18126402.

from

19900900900.

the end

of

amounted

to

$3,033.
8887687.
9991810891.

ADDITION

Examples

2. 6723
3. In

479

"

of

Subtraction.

and

183.

228

making

bought goods to
of

payment
he

did

and

sum

the

March,

in

$3000, and

another

still owe?

for $650:

another

being for $250, and

5669.

February, S4375;

notes, whose

three

of the third

merchant

one

much

$4947, how
owe

in

$2675;

$1897 ; after
4. I

347"

"

January, 1876, a

amount

of

Addition

in

37

+ 381 + 625"1098==?

275

1.

SUBTRAQTION.

AND

$1000.
is $1300

note

one

"

is the amount

what

$400.

note?

deposited$450 in bank on Monday; on


Tuesday, $725; oji Wednesday, $1235; on Thursday,
$4675; and on Friday,$1727. On Saturdaymorning he
drew
out
$5935, and Saturday afternoon,^877 : how
5. Mr.

much

money

6. At

Of

Jones

the

had
end

this amount,

he
of

left in bank?
one

$350

for

two

of

acres

I had

I found

year

ing,
for cloth-

for

and
incidentals,

ground :

much

how

the remainder
did

the

7. A

speculatorbought three houses.


$4875 ; for the second,$2250 more

gave
and for the third he gave
all for $20838: how
much
8. A

$16785

man
are

estate ; the

he

two

$1300.

cost?

acres

has

spent $2300.

paid for board, $125

were

$375 for books, $150

$2000.

owns

in

$3725.
did he

He

in bank?

first he

for the first ;


afterward sold them

at

and

depositedin

was

the

than

$5113.

gain?

j^ropertyvalued
personal property,

remainder

For

$49570,

of

$24937
bank

how

which
in

real
much

$7848.

goods for $7895, and


He
sold the
$3 for drayage.
paid $175 for
did he gain?
$2020.
goods for $10093: how much
farmer
10. A
in land,
invested
$10000, as follows:
$5750; in horses,$925; in cattle,$1575; in hogs,$675;
9. A

merchant

bought
and
freight,

bill of

he

did

speculator

11.

$3785
lost

he

he

Friday

on

lost

$1375:

lost

$4625

$6955

gained

much

how

much

he

day,
Tues-

on

Thursday

on

and

$985,

did

day
Satur-

on

during

gain

week?

entire
12.

The

following

for

and

$5

salary,

for

have

$35

at

the

end

board,

sundries.
of

the

paid
How

two

for

salary,
for

$2

weeks?

$7

for

account

received

week,
Jones,

Tom
for

$8

$50

for

Second
to

private

received

$7

sundries.

and

Brown's

week,

clothing,

loaned

washing,

Mr.

18

First

$25

spent

the

$170.

weeks:

two

Brown

he

Wednesday

on

$895;

he

for

how

81075.

gained

Monday

on

tools

tools?

and

implements

in

invest

and

im^jlements

in

remainder

the

and

ARITHMETIC.

PRACTICAL

NEW

RAY'S

38

washing,
$50

for

much

and

board,
did

for

$2
Mr.

$9.

MULTIPLICATION

When
22. What

units

write
2

"

Since

"

tens,and

tens, under

product of
is

25

"

product,215;
set

the

equal to

then

43

and

hundreds

43

12.

25 ?

tens

and

4 3

and

2 5

multiply by
and

the

21

8G

1075

1 ten.

Multiplyingby 5 units gives 5


and multiplying
by 2 tens gives 20
20 times

NUMBERS.

43 X

multiplyby

8
product,

the 2 hundreds

times 43 and

SIMPLE

Multiplier Exceeds

is,20 -f 5,

that

the

the

is the

Analysis.
5

OP

times

43,

times

43; add

4 8 X

=43X20
43X25

them, because

equal 25 times 43.


and write
Hence, multiplyby the units' figureof the multiplier,
the product so that the right-handfigurewill fall in units' place;
then multiply by the tens' figure,
and write the right-handfigure
of the product in the tens' place.

Therefore, in multiplyingby

figureof
Note.
of the

the

product in

ARITHMETIC.

PRACTICAL

NEW

KAY'S

44

the

figureof

order

same

as

any order,write
the multiplier.

the

by
productsof the multiplicand
called pariialproducU.
are
multiplier
The

"

Rule.

General

1. Write

"

plicand,placingfiguresof

the

the

order

in

last

separate figures

under
multiplier

same

the

the miilti'

column.

plier
Multiplythe midtiplicandby each figureof the multithe
in succession,
beginningwith units,always setting
right hand figureof each product under that figureof the
which producesit.
multiplier
will be the
the partial
3. Add
: their sum
productstogether
productsought.
2.

Proof.

Multiplythe multiplier
by
obtained
be
should
the product thus
first product.
23.

"

Multiply2345

the
the

multiplicand.

multiplier.
7035=^2345X
^2345X

2345__ =2
288435

24.

345X1
2345X123

"When

615
3

492

20

there

869

00

Multiply327 by

Remark.

234

469

246
288435

multiplier.
6

multiplicand.
=

12 3 X

40
=123X
300
=123X
=123X2000
123X2345

203.

cipher in the multiplier,


leave it,and multiplyby the other figures,
being careful
to place the right-hand figureof each
partialproduct
under the multiplyingfigure.
6 5
"

the

as

PROOr.

123

12

same

123.

by

SOLUTION.

2 345

multiplicand:

is

82 7
20
981
4

66381

OF

MULTIPLICATION

SIMPLE

NUMBERS.

45

Examples.
25.

235

X13

3055.

34.

624 X

85:

26.

34()

X19

6574.

35.

976

97

27.

425

X29

12325.

36.

342

X364:

28.

518

X34.

17612.

37.

376

X526

29.

279

X37

10323.

38.

476

30.

869

X49.

42581.

31.

294

X57.

32.

429

X62:

33.

485

X76:

43.
44.

45.
46.
47.
48.

will

94672.

124488.

197776.

X536.

255136.

39. 2187

X215:

470205.

16758.

40. 3489

X276

962964.

26598.

41. 1646

X365:

600790.

36860.

42. 8432

X635:

Multiply 6874
Multiply 2873
Multiply 4786
Multiply 87603
Multiply83457
Multiply 31624

49. What

53040.

by
by
by
by
by
by

829.

5354320.

5698546.

5237479.

1823.
3497.

16736642.

9865.

864203595.

6835.

570428595.

7138.

225732112.

barrels

126

of

flour

cost, at $6

rel?
bar-

$756.
50. What

barrels

will 823

of

pork cost, at $12

rel?
bar-

$9876.
will

51. What
a

675

pounds

cheese

of

8775

pound?
52. What

will 496

bushels

of

oats

will he

travel

54. There

yards

travel

man

are

there

are

55. There
if
year:
sail in a

in 152

are

in

yards

209

24

miles

day,

how

mile

one

miles

in

cents.

cents

cents.

many

miles

4256

miles.

how

many

yards.
days in a

367840

miles?

hours
8

in

cents

11904

days?

1760

ship sail

year?

28

24

cost, at

bushel?
53. If

13

cost, at

a
an

day,

and

hour,

how

365

far will she


70080

miles.

and

thousand

20001.

59. A

grocer

for the

cents

the

farmer,

them

it

dollars

150

each

lars
dol-

dollars each,

125
:

cents.

for 75

man

14

when

for 325

each;

did

dollars

he

sold

box

one

dollars

he

sold

much

bought
he

duo

Was

money

for

him

how

lars
dol-

many

$575.

for how

hogs, at
much

of wheat

charged

one

another

bought

more

farmer

of

of

cents.

gain?

dollars:

head

all for

dollars

175

did
24

goods

each, and

them
he

of

all

so

for

three
as

to

sell them?

sheep,at

dollars

250
more

gain
$1781.

head;

head; and 9 cows, at 45 dollars


them
all,he lost 275 dollars : for

sell them?

$754.

65. To

75 X

37 add

85 X

54, and

subtract

5284.

2081.

66. To

69 X

53 add

48 X

27, and

subtract

4279.

674.

67. I
in

bushels

2650

much

horses

horses

merchant

64. A

how

360

bushel, and

selling:how

bought

and

dollars,two

sold it for 55

"

246450

he

63. A

36

cents

for

farmer

did

356

gallon; and
did he gain?
sold

cents.

molasses, containing

much

95

of

farmer?

sold

thirty-

of sugar, each weighing


did he pay
much
: how

barrel

merchant

at

bushel

each,

for

pound

cents

commission

62. A

and

barrels
a

bought

gallons,for 45
cents
a
gallon:how

cents

and

thousand

one

3440

36

and

bought

grocer

61. A

one

4824622807.

seven.

sugar?

60. A

for

and

2020741032.

pounds, for

215

hundred

four

thousand

hundred

Multiplyone
by

one

2048203.

Multiply eighty
and
by sixty thousand
two

twenty-nine by

seven.

57.

58.

and

thousand

Multiply two

56.

ARITHMETIC.

PRACTICAL

NEW

RAY'S

46

bought 50 bags of coffee,averaging 63 poupds


a
did
bag, paying 34 cents a pound : how much
cost?
10719" cents.

SIMPLE

OF

MULTIPLICATION

CASE

the

When

32.

will

1. What
Analysis.

times

5, 15

times

as

15

is

oranges

will

cost

much

as

Cost

of

each?

cents

8 ct.

1 orange,

oranges.
Therefore, instead of multiply-

Cost

of

5 oranges,

4 0

ct.

Cost of 15
by 15, first find the cost of
6 oranges, by multiplying8 cents
by 5; then take 3 times that product for the

ing

factors.

separatedinto

be

cost, at 8

oranges

Since

"

I.

can
multiplier

15

47

MULTIPLICATION.

IN

CONTRACTIONS

NUMBERS.

120

oranges,

of

cost

ct.

15 oranges.

into tico or more


factors.
Separatethe multiplier
2. Multiply the multiplicand
by one of the factors,and
this product by another factor,till every factor is used; the
last productwill be the one required.
Rule.

Kem.

1.

"

"

Do

into which

while

the

whose

sum

not

it may

factorsof a number
separated. Thus, the factors of
15 may
be separatedare
which

confound
be

parts into

equals 15:

as, 7 and

8;

or,

2, 9, and

the ;?rtrts

with

the

15

5 and

are

3,

numbers

any

4.

Examples.

2. What

will 24

acres

cost,at $124

of land

acre?

an

$2976.
3. How
of 1512
4. How

each
5.
6.

far will
miles

per

ship sail

in

weeks^

week?

pounds of iron
weighing 2873 pounds?
Multiply 2874 by 72.
Multiply8074 by 108.
many

56

are

there

at the

rate

84672

miles.

in

loads,

155142

54

pounds.
206928.
871992.

ARITHMETIC.

PRACTICAL

NEW

KAY'S

48

CASE

the

When

33.

IT.

is
multiplier

1 with

ciphersannexed;

10, 100, 1000, etc.

as

Placing one
3) changes the
1.

and

so

on,

thus, annex

the

cipher on
units

into

right of

tens, the

number

into

tens

the number
and, therefore,multiplies
one
cipherto 25, and it becomes

(8,

hundreds,

by ten;
250.

ciphers changes units into hundreds,


the number
tens into thousands, etc.,and
multiplies
by one
two
hundred; thus, annex
ciphersto 25, and it becomes
Annexing

2.

two

2500.

as
ciphersto the multiplicand,
there are
and
the nwnber
thus
ciphersin the 7nultiplier,
formed will be the productrequired.

Rule.

1.
2.
3.
4.

5.
6.

"

Annex

as

Multiply 245
Multiply 138
Multiply 428
Multiply 872
Multiply 9642
Multiply10045

many

by
by
by
by
by
by

100.
1000.

both

When
of the

1. Find

Analysis.

there

4280000.

100000.

87200000.

1000000.

9642000000.

1000000.

10045000000.

are

III.

ciphers at

the

right of

one

factors.

the

"

138000.

10000.

^CASE

34.

24500.

product of

The

625

by

multipliermay

be

500.

considered

as

composed of two factors: 5 and 100, Multiplyingby


"5,the product is 8125; and the product of this number
by 100 is 812500, which is the same
as
annexing two
to the first product.
"'iphcrs

6^5
500
312

5 00

or

MULTIPLICATION

Find

2.

the

OF

of

product

NUMBERS.

SIMPLE

2300

49

170.

2300

Analysis.
of

composed
the

the

2300

factors

two

17

factors

two

number

The

"

and

23

be

may

and

100;

17

regarded

as

and

of

170,

161

10.

2 3
391000

The

of

product
by

23

17,

resulting product
of

product

23

the

product,

both

factors.

Rule.

of
as

the
are

"

at

by

the

and

by

10

17,

and

then

right of

annex

both

be

that

(33);
then
3

is, by

ciphers

the

factors.

the

as

the
the

ciphers

to

right

of

the

ciphers

product

and

finding
3

at

plying
multi-

by

100,

annexing

regarding
to

found

by

product

tire

without

will

170

this

there

as

Multiply

factors

by

2300

on

many

the

right

ciphers

^i^m

..gmm^

35.

If

1.

how

will

many

Analysis.
each
which

each

will

It

"

will

boy

divide

you

have

apples

apples

many

times

many

If

divide

you

many
3.

How

will

each

by

process
is

Because

peaches

called

Ans.
in

which

in

times

times

(i,

3.

are

6.

Ans.

boys,

peaches.

10?
the

Ans.
2

between

equally

have?

times

many

The
solved

Hence,

1.

6?

in

Why?
how

boy

is contained

as

each

give

to

3.

are

How

2.

boys,

have?

boy

require
as

between

apples equally

5.

Why?
Why?

examples

preceding

arc

Division.

DEFINITIONS.

36.

1.

times
2.

one

The

dividend
the

Division

number
divisor

is

number

contained

the

is
is

the

in

number

number

to

how

dividend.

of

process

contained

is the

denoting
in

the

by

be

how

many

another.
which

divided

many

finding

times

to

the
the

divide

; the

quotient

is

divisor

is

NUMBERS.

51

is the

times; here, 3

in 12, 4

3 is contained

Thus,

SIMPLE

OF

DIVISION

divisor,12 the

dividend, and 4 the quotie7it.


3. Since

4. Since

and

is the

at

and

boy

then

After

would

"

has

He

8 cents

give

for the

cents

method

the
made.

buy,

8 cents,

for 1

lemon,

Left,

3d, he would

from

quotient shows

lemon, 2 cents.

may

2 cents.

lemon,

Left,

2 cents,

4th

2 cents.

lemon,

0 cents,

Left,

how

of
the

of

termed

the

same

four

8.

are

short

method

of

number.

subtracted
subtraction

many

times

many

subtracting2

times

be

number

how

3d

performingthis operationis by

8, instead
times, and 4

which

G cents,

4 cents,

4th, he

it is known

subtractions

2 cents.

Left,

lemon, he

for the

of

is the

lemon,

1st

from

2 in 8 four

divisor

number

he

can

4 lem-

left.

subtracted

making many
the

lemons

many

cents.

for the 2d

Therefore,Division
The

known.

left.

natural

times, say

plication
in multi-

factors

2d

subtraction;but, when
be

product 12, the

the

left.

cents

4 cents

2 cents

Lastly,after giving 2 cents


would
have nothing left.

can

how

buy 4, because

can

6 cents

Then, giving 2
2 cents

the

the

factor is

each, will cost 8

giving 2

The

product. Therefore,Biof the factors of a


of finding one

the other

would

have

have

have

times

each?

ons, at 2 cents

The

times, 4

quotient multiplied
produce

factors of

are

process

boy

cents

Analysis.

dividend, to

product,when
37.

and

quotient correspondto

and

; the

insion

four

12

dividend.

the

divisor

in

divisor

is,the

that

12;

are

contained

is

; the

has

subtractions

dividend,

been

have

made

been

38.
1st.

is indicated

1. Division

3)12,

ARITHMETIC.

PKACTICAL

NEW

KAY'S

52

in three

ways:

wliieb

means

that

12

is to be divided

by

3.

which

means

that

12

is to be divided

by

3.

which

means

that

12

is to be divided

by

3.

12

2d.

3d.

12-^3,

2. In

exceed
the

the

using
12,

draw

first
line

quotientbeneath;

curved

line

sign wiien
the

under

if the

divisor

the

the

divisor

dividend, and
exceeds

rightof the dividend, and


quotienton the rightof this.
3. The sign (-^) is read divided by.
on

does

15)45(3
45

Division

"-"
Table.

write

12, draw

place the

Examples.

2)8

not

21-^3=7.

If 7 cents

39.
3

be

boys,each boy
1

cent

The

NUMBERS.

SIMPLE

OF

DIVISION

be divided
would

as

53

equallyas possible
among

receive

cents, and

there would

left,or remainingundivided.

number

left

after

dividing,is

called

the

re-

raainder.
Rem.

of the

1. Since

"

same

will he

the remainder

denomination.

is

part of the dividend, it must

If the dividend

dollars;if pounds,the

remainder

be

he dollars,the remainder

will he

pounds.

Rem.

always lens

is

remainder

2." The

once

divisor;for,if it
contained

be

at

least

in the dividend.

more

Rem.

If

3.

"

dividend

the

and

called Division

operationis

40.

When

merely

the

the

is

written, it
when

is used

simple numbers,

are

the

Numbers.

Division.

division

result

Division

divisor

of Simple

Short

Short

the

than

divisor would

equal to it,or greatei*the

were

ARITHMETIC.

PRACTICAL

NEW

RAY'IS

54

and
performed mentallj^,
Short

is termed

the

divisor

does

Division.
exceed

not

12.

1. How

is

Here, the dividend


tens, and

Now,

Hence,

Dividend.

2 in

hundreds'

which

write

times, which

numbers; 4 hundreds, 6

Quotient.
is contained

in

in

60

''

''

in

^'

"

in

result

separatingthe

in

of three

same

Thus,

composed

units; that is.of 400, 60, and 8.


Divisor.

The

in 468?

is 2 contained

times

many

400

is contained

468

be

can

dividend

into

obtained

200

times.

30

times.

times.

234

times.

without

actually

parts:

(hundreds),2 times,which write


Dividend,
3 times. Divisor, 2)468
place;then, 2 in 6 (tens),
in tens' place;then, 2 in 8 (units),
4
Quotient, 2 34
write in units' place.
4

2. How

many

times

3 in 693?

231.

3. How

many

times 4 in 848?

212.

4. How

many

times

in 4682?

2341.

5. How

many

times

in 8408?

2102.

OF

DIVISION

many

times

in 3693G?

12312.

7. How

many

times

in 88468?

44234.

1. How

Solution.

place; 3

times

many

hundred

the 3 tens, makes


and

units,makes

3. How

1 ten

13

in

the

times

IS

place will
place;the 1 ten

1 in ten's

units,and

units, making

18

placed in

place.

units'

4. How

many

Here, the

solution

ample;

there

5. How

times

is the

being no
many

not

the

with

245

the

in

618?

added

to the 8

in the

in

is 3 contained

206

609?

above

in

3)618

is

quotient figure6

as

3)735

5 times.

tens, their order is indicated

times

735?

is contained

is 3 contained

same

place.

(hundreds),2
contain 3, a cipher

is then

4 3

(hundreds),

is 3 contained

in

3)129

tens'

hundred, united

3 is contained

SoLUTiON.-r-Here, 3 is contained

129?

is

in

in

seven

tens, in which

15 units,in which

placed in ten's

write

left; this 1 ten, united

many

times; as the

in 1 ; but

is 3 contained

over;

in

write in units'

Here, 3 is contained

"

times, and
times

contained

3 times,which
(units),

2. How

with

is 3 contained

(tens),4 times, which

12

in 9

Solution.

times

many

Here, 3 is not

"

in

contained

55

6. How

41.

NUMBERS.

SIMPLE

3)609

ex-

by

20 3

0.

743?

the division of which


there is 2 left,
dividing,
the
is merely indicated by placing the divisor under
remainder; thus, f. The quotient is written thus,
247f; read, 247, and two divided by three; or, 247,
After

with

re7nainder, two.

3)743
24

7"

ARITHMETIC.

PRACTICAL

NEW

KAY'S

56

6. How

many

times

in 462?

154.

7. How

many

times

in 1170?

234.

8. How

many

times

in 948?

237.

Rule.

each

the

the

divisor at

leftof

the

dividend,

them^ and draw a line beneath


dividend.
Begin at the left hand, divide successively
and
write the result
figureof the dividend by the divisor,

tvith
the

1. Wiite

"

line between

curved

order in the

quotient.
remainder
2. If there is a
after dividingany figure,
lower order,and
divide
it to the figurein the next
'prefix
in the

same

before.

as

If the number in any order does not contain the divisor,


order in the quotient,
prefixthe
place a cipherin the same
number
divide as
to the figurein the next lower order,and
3.

before.
4. If there

placethe
Proof.

is

remainder

divisor under

it,and

after dividingthe
it to the

annex

last

figure,
quotient.

Multiplythe quotientby the divisor,and


the remainder, if any, to the product: if the work
will be equal to the dividend.
correct,the sum
Rem.
that

"

"

This

dividend

method
is

of

653

cents

by

3.

SOLUTION.

TROOF.

217

Dividend.

Divisor,

Quotient,

is

proof depends on the principle(36, 4]


product,of which the divisor and quotientare

factors.

9. Divide

add

3)653
2 1 7

3
(151=
2

"
"53

cents

divided.

remainder.

dividend.

DIVISION

Note.

When

"

any

OF

SIMPLE

NUMBEKS.

PARTS

OF

number

is divided

57

NUMBERS.
into two

equal parts,one

of

the parts is called

one-halfof that number.


If divided
into three equal parts, one
of the parts is called 07iethird; if into four equal parts,one-fourth;if into live equal parts,
and so on.
one-fifth;
Hence, to find one-halfof a number, divide by 2; to find one-third^
divide by 3; one-fourth,
divide by 4; one-ffth,
by 5, etc.
4326.
13541

If.
1687601
.

196855.
4311

7^.

1234753f
754065.
1003634.

1830023-V
54841.
3472834.
24. If

bought

25. If 4 bushels
that
26.

cost
oranges
for 894 cents?

of

cents

each,

how

many

bought

for $812?

be
298..

applescost

140

cents, how

bushel?
If flour cost

can

much

is

35 ct.

84

barrel,how

many

barrels

can

be
203.

27. A

is that

28. There

there

are

30. At

bought

months'

months

$47.
how

in 1 year:

years

many

45.

months?
4

are

217.

for $1736?

days in one wxek : how many


there in 734566
are
days?
has been multij^lied
number
32. A
by 11, and
There

31.

work

quarts in 1 gallon: how many gallons


80319.
in 321276
quarts?
barrels of flour can
be
$S a barrel,how
many

There

29.

for

$423

month?

12

are

in 540

there

are

receives

carpenter

much

liow

ARITHMETIC.

FKAGTICAL

NEW

liAY'S

58

weeks

are

"

104938.

duct

is 495

product of

The

33.

numbers

is 9

34. Find

what

number

is the

what

is the

one-half

i)ro-

45.

numbers

two

the

is 3582

one

of

the

other ?

398.

of 56.

28.
'

one-half

of 3725.

Find

one-third

of 147.

49.

37. Find

one-fourth

of 500.

125.

38. Find

one-fifth

of 1945.

389.

Find

one-sixth

of 4476.

746.

one-seventh

of 2513.

35. Find
36.

39.

40. Find

1862^.

359.

.^
*

41. Find

one-eighth

of 5992.

749.

42. Find

one-ninth

of 8793.

977.

43. Find

one-tenth

of 1090.

one-eleventh

of 4125.

375.

one-twelfth

of 5556.

463.

44.

Find

45. Find
46.

eldest

divided

boy

number

did the

them:

of his share

boys;

to his sister

found

195

Daniel

many
48. One-eleventh
192?

cents

of

12.

cents, and

gave
did
275

one-third
she
is

the

what

sister receive?

sister: how

one-eighth of

apples equally among

one-third

gave

47. James
fifth of

144

109.

gave
of

receive?
how

much

to

Daniel

his share

one-

to

his
13.

greater than
1.

DIVISION

SIMPLE

OF

Long
the

When

42,

down,

used

commonly
1. Divide

Solution.

the

dollars

3465

Fifteen

"

work

of the

division

Division.

Long

when

divisor

exceeds

contained

is written

Division

Long

equallyamong

is not

59

Division.

entire

termed

is

it

NUMBERS.

in

12.

15

men.

there will be no
thou(thousands)
; therefore,
sands in the quotient. Take 34 (himdreds)
as
is contained
in 34, 2
Si partial dividend; 15
have 200 dollars each,
times; that ig,15 men
which
30 hundreds
of
requiresin all 15 X 2

15)3465(231
3 0 hund.
4 6 tens,
4 5
15

dollars.

all

hundreds

remain;

(tens)for

hundreds, and

34

to which

bring

down

third

to each

bring down

1 dollar

man

each

in this the divisor

divisor

receives

dollar;that is,231

certain

number

Divisor.

of

hundreds, contimes; the second


it 3

15

Parts.
3000

Quotients
200
3 0

15
3 4 65

1
231

times; the

third part, 15 units,contains


several

ten

dollars.

first part, 30

part, 45 tens, contains

several

dollars,3

4 5 0

tains the divisor

The

for
gives15 (units)
is contained
giving
once,

hundred

times.
The

and

more.

man

has 30 dollars more,

man

the 5 units, which

this process, the dividend


is separated
into parts, each part contain-

the

have

you

of dollars.

45 tens

=^

45, and

and
dollars,

By

partialdividend;

Hence,

the 6 tens, and

require15 X

Subtract

ing

from

second partialdividend.
contains 15, 3 times; that is,each

46

30

hundreds

46

units,

1 5

Subtract
4

is

it 1 time.

parts togetherequal the given dividend, and

partialquotientsmake

up the entire

quotient.

the

60

2. In

NEW

147095

days, how

sands)for the
find

years, each

many

865

Taking 147 (thoufirst partialdividend,

Solution."

we

ARITHMETIC.

PRACTICAL

RAY'S

it will

contain

not

the

)1

(403

4 7005

14

di-

of 365

days?
years,

60

109

109 5
figures.
tracting,
Again, after multiplyingand subas in the precedingexample, and
bringing down the 9 tens,
the partialdividend, 109 (tens),will not contain the divisor;hence,
and bring down
the 5 units;
write a cipher (no tens)in the quotient,
which
contains the divisor
is 1095 (units),
the last partialdividend

visor; hence

four

use

three times.

3. Divide

Rule.
draw

4056

1. Flace

"

312.

13.

the divisor

line between

curved

by

the

on

them, and

leftof

another

on

the

dividend^

the

rightof

the dividend.
2. Find,

how

fewest lefthand
the

times

many

figuresof
place this

divisor^and

the divisor is contained


that

the dividend
number

in

the

in

the

will contain

quotientat

the

right.
3.

Multiplythe

divisor

that

^product under

by

this

quotientfigure;place the

part of the dividend

from

which

it

was

obtained.
4. Subtract

this

remainder

bring down

divide

until
before,

as

figuresabove it; to the


figureof the dividend,and
figuresof the dividend are

productfrom
the next
all the

the

broughtdown.
5. If at any time,afterbringing
down
'

thus

formed
the

cipher in
which

divide

Proof.

"

is too

smcdl

and
quotient,

as

Same

to

contain

bringdown

before.

as

in

Short

Division.

the number
figure,
the divisor,
place a
another figure,
after
a

OF

DIVISION

Rem.

"

dividend

1. The
from

product

which

SIMPLE

must

it is to be

never

NUMBERS.

be

gr eater

"

the divisor; when

than

subtracted; when

figureis too large,and must be diminished.


2. After
Rem.
subtracting,the remainder
than

61

the remainder

is not

must

less than

so,

the
the

partial
quotient

always
the

be

less

divisor,the

quotientfigureis too S7nall,and must be increased.


order of each quotient figure is the same
3. The
Rem.
as
dividend
from which
it was
obtained.
lowest order in the partial

last

"

4. Divide

78994

by

319.

the

21. Divide

48905952

Divide

4049160

22.

$15

24. At

bought

27. A
much

raised
that

was

day, in

is that

The

30.

31. The
the

Divide

thousand

33. Divide

by

colony
109440

35. A

to

if

raised

the

acre:

many

railroad

is $379600

=:=

$360

is 6571435

and
each

capitalof

is divided

into

269

other?

5321.
and

forty-seven

405.

3080.

hundred

and

thousand

one

bought a tract
equally divided,to

10338yyg?_
of land,containing
how

many

160

8288

bushels

how

many

of

acres

did

what

ho

is

is the

share?

37. The

acres.

averaging

corn,

joint-stock
company
shares

of

one

plant?

148
36. The

over.

$1040.

yr.)

entitled?

man

farmer

bushels

56

men

how

bu.

(365 days):

hundred

two

four

684

acres:

year
$136 and

six.

acres

each

was

acres

of

da.

numbers

by

and

107

on

is the

million

thousand

one

34.

10

how

men,

236.

(365

million

hundred

four

corn

gallons:

63

from

what

one

days
days.

89

day?

factors is 1235:

32.

of

$50000

receives
a

19

acre?

one

on

yearly income
is that a day?
product of two

much

how

many

$57.

bushels

9523

President

much

how

acres.

have?

The

how

equallyamong

hogshead there are


hogsheadsin 14868 gallons?
In

29.

be

can

14

divided

will each

man

miles

26

miles?

be

$1083

dollars

28.

land

of

acres

many

231

travel

man

26. If

how

567482^VV

travel 364

will he

973.

how

acre,

328.

12345.

for $3465?

25. If

many

an

4952.

9876.

by
by
by

552160000

23. Divide

ARITHMETIC.

PRACTICAL

NEW

KAY'S

62

acres.

$262275,
value

of

$975.

earth, at

the

equator, is about

24899

miles* in

DIVISION

OF

circumference,and
how

miles

many

heard

miles

238

in 24

once

hours

it turn?

how

feet

many

second

$3731840:

what

$15680.
from

city; the

before

did

the

the

flash

sound

sound

was

travel ?
1127

Light travels

40.

how

the

the

to reach

of 11520000

rate

does

minutes

many

sun

the

at

it

earth,the

miles

41. Subtract

lightof

being 92160000

sun

miles

from

86247

Divide

the

sum

43.

Divide

the

product

the

divide

and

94231

divide

the

sum

the
Multipl}^
divide

the

and

of 46712

difference

the

by

of 497

between

6848
583

by

by
and

2832

104.

515.

71.
987

4081.

add

29.

diflPerence between

product by

4896

and

2384

112.

part

of them

apiece,how

many

received

S7560

for S3885 ; if he
horses

did

he

for

sold

the

sell the

farmer

divide
845.
ence,
differ364.

horses ; he
rest

sold

for $175

second

time?
21

49. A

by

1099.

of 228 + 786 by 95, and


Multiplythe sum
the product by 114.
47. Multiplythe sum
of 478 and
296 by their
and
divide the product by 387.
horse-dealer

678,

87.

46.

48. A

mainder
re-

499.

42.

44. To

TIkvtew.

for

16.

by

49, and

ute
min-

8 minutes.

Examples

45.

feet.

the

require for

distant?

and

1037^^.

long, cost

seconds

24

seen

w^as

cannon

does

63

cost

39. A
of

its axis

on

hour

NUMBERS.

per mile?
feet distant
fort is 27048

the

was

turns

an

railroad

38. A

SIMPLE

horses.

expended at one time $7350 for land,and


each time : how
at another, $4655, paying $49 an
acre
did he buy in both purchases?
245 acr^.
acres
many

refiner

50. A

ARITHMETIC.

PRACTICAL

NEW

KAY'S

64

hogsheads of

58

bought

$77

at

sugar,

for $5742:
how
hogshead,and afterward sold them
each hogshead?
much
did he gain on
$22.
of land,at $26 an
51. A man
bought 240 acres
acre,
giving in payment a house valued at $2820, and horses
horses did he give?
valued
at $180 apiece:how
many
a

19

horses.

of land for $10625,


bought 25 acres
speculator
after dividingit into 125 villagelots,sold each lot
did he gain on
the whole?
On each
$250: how^ much

52. A
and
for

each

On

acre?

lot?

$20625.

CONTRACTIONS

43.

When

1. A

man

the divisor

paid $255

that per

was

Solution.

times 5 acres;
3

acres

$17, the value


factors

divide the

Solution.
twos

ing by 7,
mainder
then

by

are

the

"

and
the

of 4

is 4 ttcos

of land:

acres

factors.

how

much

of
6

3)255

6)85
1 7

gives

the value of 15

acres.

"

the value

of

acres,

the value

of

acre.

acre.

that

3 and

quotientthus

2, Find

is 18

of 1

solution shows

whose

separatedinto

Dollars,

are

value

dividing$85

The

15

dividing$255 by

gives $85, the

acres;

be

acre?
15

"

I.

can

for

$165.

DIVISION.

IN

CASE

$825.

instead of

5, we
obtained

may

by

dividingby the number 15,


first divide by one
factor,then

the other factor.

quotientof 37, divided by

14.

Dividingby 2, the quotient


1 unit remaining. Divid2)37
is
with
re2,
a
quotient
7)18
tioos; the whole

plus 1, or

9.

remainder

^^

and 1
and

over,

4 twos

left.

Rule.

divide

the divisor; then


other

65

by one of the factors of


thus obtained by the
quotient

the dividend

1. Divide

"

numbers.

simplp:

OF

DIVISION

the

factor.

Multiplythe last remainder by the firstdivisor ; to the


productadd the firstremainder ; the amount ivillbe the true
2.

remainder.
Rem.

divisor

the

When

"

can

resolved

be

into

than

more

two

successively.The true remainder


may divide by them
found by multiplyingeach remainder
by all the preceding

factors,you
will be

produced

divisors,except that which


from

2583

4. Divide

6976

5. Divide

2744

6. Divide

6145

7. Divide

19008

8. Divide

7840

9. Divide

14771

10. Divide

10206

11. Divide

81344

12. Divide

98272

by
by
by
by
by
by
by
by
by
by

their

sum

add

the

mainder
re-

63.
32.
28.
42.

132.
64.

72.
81.

121.
108.

CASE

To

To

firstdivisor.

3. Divide

44.

it.

divide

bj^1

with

II.

ciphersannexed;

as

10, 100,

iOOO, etc.
To
the

Had
manner
sam-;

one
thus, 60. On
multiply6 by 10, annex
cipher,
of multiplication,
principlethat division is the reverse
to divide 60 by 10, cut offa cipher.

might have been separatedin


the cipher;6 being the quotient,
5 the remainder.
apply when the divisor is 100, 1000, etc.

the dividend
as

will

Prac-

been

65, the 5

like
The

PRACTICAL

NEW

KAY'S

66

ARITHMETIC.

figuresfrom the right of the


there are
as
ciphersin the divisor ; the figurescut
the quotient.
be the remainder,the other figures,
Cut

Rule."

dividend

offwill

1. Divide

off

as

viany

100.

by

34872

CASE

To

45.
the

divide

divisor,or

1. Divide

on

4072

there

when
the

ITT.
are

rightof

cipherson

the divisor

and

the

rightof

dividend.

800.

by

OPERATION.

Solution.
of the
divide
In
two

Regard

"

factors

800

as

and

100

posed
com-

8, and

8^ 40

in the

margin.
dividing by 800, separate
right hand
figuresfor the
then divide by 8.
as

I|00)40i72

2. Divide

by

77939

Quo...72 Rem.

the
mainder,
re-

8100)40172
Quo...72 Rem.

2400.
operation.

Solution.
24

X 100,

"

cut

the
figures,
then

Since

off the

same

as

2400

equals
two
right hand
dividingby 100;

24100)7

79139(32 1^^1
72

divide

48
by 24.
is
Dividing by 100, the remainder
39; dividingby 24, the remainder is 11.
To
find the true
remainder, multiply 11 by 100, and add 39
the product (Art. 43, Rule); this is the same
as
annexing
figurescut oflf,to the last remainder.

to

the

OF

DIVISION

3. Divide

NUMBEKS.

67

2500.

by

62700

SIMPLE

OPERATION.

25iOO)627|00(25//o"^
Solution.

"

The

as

same

for

the

50

T2T

above.

example

125
2

Rule.

and

"

1. Cut

as

off the ciphersat the lightof the divisor^


from the rightof the dividend.
figures

many
2. Divide the

the

remainingfiguresin
remainingfiguresin the divisor.
3. Annex

the

the

cut off to
figures

the

dividend

by

the

remainder^which gives

triie remainder.

1844"4"A

98.

223jV^.
432^H^.
306/A^V\\687^Wo.

GENERAL

The

46.

values

PRINCIPLES

value
of

of

the

divisor

dividend

may

DIVISION.

quotient depends
and

and
changed by Multiplication
1st. The

OF

be

dividend.

on

the

These

tive
rela-

may

by Division,thus
the
or
multiplied,

be

divisor

divided.
2d. The

3d. Both
both

dividend

dividend

divided, at the

may

and
same

be divided,or

divisor
time.

may

the divisor

be

plied.
multi-

multiplied,or

68

ARITHMETIC.

PRACTICAL

NEAV

RAY'S

Illustrations.
Let

be

24

is 4.

24-^6

divisor;the quotient

6 the

dividend,and

4.

plied
dividend,24, be multipliedby 2, the quotientwill be multi48; and 48-v-6=:8, which is the former
by 2; for, 24X2

If the

4, multipliedby
quotient,

plied
divisor,6, be divided by 2, the quotientwill be multi-

if the

Now,

2.

3;
by 2; for, 6 --2
4, multipliedby 2.
quotient,
=

Take

If the

is the

or
If the dividend be multiplied,
will be multiplied.
quotient

the

example,24

same

dividend,24, be divided

for

by 2;

8, which

"

divided,the
47.

24-f-3

I.

Principle
be

and

24 -=-2

12; and

4, divided by

-f- 6

2, which

the divisor

4.

by 2, the quotientwill

12-h6

former

is the

be

divided

former

tient,
quo-

2.

And, if the divisor,6, be nudtipliedby 2, the quotientwill be


divided by 2; for,6 X 2
2, which is the former
12; and 24 h- 12
=

quotient,4, divided
II.

Prin.

If

"

by

2.

the dividend

the quotient
will
mxdtiplied,,

Take

48.

the

same

If the dividend, 24, and


will not be

the former
And

Prin.
divided

example,24

dividend,24, and
be

former

III.^
"

by

the

-f- 6

; and

divisor,6, be
=

same

4.

6X2

changed; for,24 -f- 2


12;
4, unchanged.
quotient,

If

be

divisor,6, be multipliedby 2, the quotient


=

will not

the divisor

divided,or

be divided.

48
changed; for,24 X 2
4, unchanged.
quotient,

if the

-=4; the

be

both dividend

and

12; 48^-12

divided,

and

by 2, the

6-f-2

divisor be

number, the quotientwill

4;

tient
quo-

3; 12^3

or
jmdtiplied
not be changed.

DIVISION

OF

SIMPLE

Promiscuous
1. In

49.

bags

69

Examples.

$500;
third, $55

are

the

; in

second, $120

NUMBERS.

in

the
what

in the

first,
$96;
in

sum

the

bag?

$229.

2. Four
the

$61

the

for land

paid $1265

men

second

than

4th

than

more

second:

how

the

much

the

first

first;the

did

the

paid $243

third

fourth

less

$79

pay?

man

$493.
3. 1 have

five

the second, 264; the


I

sell

123

are

left?
4. In

850

5. On

the

end

of the

much

Bible

bought
each

8. How

the

in how

31173

verses:
verses

apples

horses

28

63

11. How
for

much
8

die of

for

can

how

much

is

is the

horses

at

ing
remain-

had

he

at

$15095.

$1400:

rest

of the

many

to incur

I fill a

15

days can
362|^.
for

died;
no

loss?

$56.

21

73900;

the

times.

quotient214;

divisor?
and

how

gallon cask, from

gallonseach?

woolen

wounds;

January he

day?

of 148
Multiplythe sum
; divide the product by 23.

pay
each?

fifth,

killed; 586

; in

has

what

70:

gains $8706
in each

10.

to

the

many

are

February,$301 ;
he spends $538:
year?

dividend

certain

9503

men,

he

year

times

many

remainder,
ence

apples;

return?
40101.
man}the
a
speculatoris worth
year

I sell the

must

hogsheadsof
9. A

how

it,by reading 86

7. I

; in

months

I read

how

prisoners;1234

are

first of

ten

6. The

57068

4794

during

sj^ends$237
the

of

army

drowned:

$12307

fourth,97;

stolen

are

157

332.

enemy;

are

186

first bears

the

third,305;

and

428,

an

join the

the

apple trees:

the
345.

56

by

their

cloth,at $6 a yard, will


14 cows
$60 each, and

difier816.
it take
at

185

$45

yd.

Two

12.
70

acres

an

acre

S30

at

how

is

income

My

13.

for

provisions,

$100

for

books,

years

can

year

14.

for
much

for

226.
If

year.

for

40

of

acres

$25

ground
He

acre.

an

$60

at

many

$15

at

sold

an

land?

whole

the

sell

merchant

A
he

yard;

and

share,
in

other

he

get

90

acre

How

acres

stock

17.
mules

to

the

selling.
with

bank:

farmer
of

how

much

to

The

mules

part

the

He

for

mainder
re-

$85

invests

does

then
the

110.
a

dealer
for

dealer
sells

buys

50
and

he

20

horses

the

horses

each,

$150

money,
does

it all

shares

many

$300.

gain?

stock

how

$4

at

purchase?

sold.

The

of

he

$3700.

and

he

share.

share

sends

the

and

each,

last

farmer
be

$175

at

shares

$115

at

did

how

did

cloth

yard,

much

125

shares

$5

an

for

much

of

3^ards

at

how

buys

75

by

yards

yard

broker

16.

250

$6

275

bought

sold
at

clothing,

$6300.

15.

in

$360

how

in

incidentals,

for

gain?

each,

spend

$150

rent,

remainder

the

he

$25

at

13.

at

acres

and

did

$300

bought

80

$4500,

$90

remainder

$10400?

save

man

and

acre,

$1800

and

the

took

man

one

all ?

in

acres

many

farm:

other

tlie

acre,

an

for

$6000

paid

men

ARITHMETIC.

PRACTICAL

NEW

RAY'S

70

for

charging

head

of

deposits

deposit

and

in

bank?

$125

$95

cattle
the

15

at

for

$45

remainder

$2405.

To

Teachers.

Simple

Whole

him

convinces
should

following

in

book

for

author

the

of

of after, Fractions

before, instead

introduced

experience

bers
Num-

learners, Compound

young

in

appropriate

for the

"

reasons:

1st. The
Division

pupils, the

for advanced

that,

be

philosophical, and

is

after

immediately

Fractions

placing

Numbers

arithmetic

higher

in

While

"

operations
of

of

numbers

compound
and

simple numbe/s,

analogous

are

to

serve

Multiplication,and

Subtraction,

Addition,

operations

same

principlesof

the

illustrate

the

to

the

damental
Fun-

Rules.
The

2d.

subject

studying it,

most

furnished

the

by

The

is

pupils require
elementary rules;

this

is

is

discipline than

mental

more

Before

difficult.

and

important

in

general principlesinvolved
of

knowledge

few, and
that

Fractions

the

acquired by

of

study

Numbers.

Compound
3d.

of

Fractions.

introduced,

are

The

examples

they

as

their

should

study, do

fractions

involving

be, with

require

not

other

are

in

exercises

subject.

Teachers, who
Numbers

until

direct

prefer it,can
they

have

studied

their

Fractions

pupils to
as

far

as

defer

Compound

page

159.

DEFINITIONS.

50.
more

pecks
Rem.
must

numbers

concrete

1.

quarts

"

The

belong

to

up

of

two

denominations

different

or

as,

pint.

different
the

of

made

is

number

compound

same

denominations

table;

thus,

of
in

the

number

compound
example

given,

(71)

the

RAY\S

72

pecks

be reduced

may

of

j)arU

are

to

peck.

ARITHMETIC.

PRACTICAL

NEW

quarts

pecks

pints,and

or

7 dollars

pints and quarts


be a compound

the

would

not

number.
in tho
resemble
simple numbers
Compound numbers
the denominations
of compound numbers
respond
corfollowingparticulars:
certain number
of simple numbers, and
of
to the orders
a
denomination
make
unit of the next
one
units of a lower
higher
Rem.

2.

"

denomination.
Rem.
in

-5.
"

each

unit of the next

one

Rem.

4.

"

Weights

and

do make

one

51.

of

units

this; ten

make

compound

Most

United

In

numbers

denomination

lower

higher denomination.
States Money and
the

Measures, however,
imit of the next

1. The

differ from

ten

units

of

simple numbers
do not
uniformly

Metric
lower

denomination

higher denomination.

operationswith

numbers

compound

and
Bediictlon,Addition, Subtraction, Multiplication,

is the

2. Reduction

of

number

of

System

of

changing the
aUering its value.

process

without

are

vision.
Di-

ination
denom-

yards may be changed to feet; for,in 1 yard there are


feet;then, in 5 yards there are 5 times 3 feet,which are 15 feet.
Thus,

3. Eeduction

takes

higher denomination
to

States

to

two

lower.

1st. From

ways:
2d. From

lower

States

STATES

nominati
de-

MONEY.

is the

money

of the United

money

of America.

Table.
10

higher.

UNITED

5"2. United

place in

mills,m.,

make

cent,

"

d.

"

%.

"

10

cents

"

10

dimes

^'

dime,
dollar,

10

dollars

"

eagle,

marked

ct.

E.

Rem.

"

in

United
cents.

United

1.

States,
In

in 1795;

The

money

coined, by the

trade

The

2.

"

1793.

coin

are

of

gold.

shown

1849.

dollar

coins

nickel,silver,and
of each

first

gold dollars,in

standard.
Rem.

was

silver dollars

1794

established,by

money

in

was

73

States

The

1786.

MONEY.

STATES

UNITED

was

the

The

The
were

Gold

coins

and

minted

silver

are

for Asiatic

States

United
name,

Gold

gress,
Con-

authority of

first made

made.

of

act

were

eagleswere
now

both

the

copper
made

legally

commerce.

classed

as

bronze,

value, composition,and

weight

are

in the following

Table.

Rem.
allowed
a

3.

"

by law

grain in

Half

deviation

in

weight of | a grain to each piece,is


in the coinage of Double
Eagles and Eagles; of \ of
Eagles and the other gold pieces:of 1^ grains in all

mill

The

4.

Rem.

"

1. A

53.

of

sum

cents, and, when


the

by

Rem.

dollar

It is used

is

money
in

written

read

Thus,
4

$24.56

dollars

37

carried out

to

read

Rem.

of

tens

dollars,and

dollars

24

56

dimes

cents;

not

is read

$16,375

cents.

the

the

figuresto

leftof

16

the decimal

of dollars; the two figuresto


point,a number of cents; and the

"

If the number

are

eagles
dollars

press
point ex-

the

rightof
third, figure

dollars

dollars

8. Two
9. Four

seven

ciphermust

dollars

hundred
hundred
hundred

^VRITTEX.

eight mills.

cents

six mills.

mills.
,

-three
fifty

dollars three

Twenty

BE

seventeen

six cents

dollars

Forty

7. One

TO

dollars

3. Seven

5. Two

10, a

place.

1. Twelve
2. Six

is less than

of cents

EXAMPT^ES

6.

to

mills.
right,

in the tens'

4.

cent.

used

point,is

number

decimal

to the

decimal

to the nearest

cents.

as

is read

dimes

Hence,
a,

the

the

iness
mills,but, in bus-

5 mills.

cents
4.

and

of cents.

tens

as

and

dollars

Eagles are

3.

dollars

expressed as

is always preceded
figures,

sometimes

are

period (.), called

separate the

calculations.

only in

transactions,the final'result is always taken


2. A

grainsin

sign ($).

Calculations

"

of 2

cent.

coined.

is not

piece; and

iivc-cent

the

pieces;of 3 grains in
three-cent pieceand one

silver
the

ARITHMETIC.

PRACTICAL

NEW

KAY'S

74

cents

five mills.

cents.

two

cents

dollars ten
dollars
dollars

two
one

two

mills.

cents.

cents.
cent

eicfht mills.

be put

MONEY.

UNITED

STATES

EXAMPLES

TO

75

READ.

BE

$18,625

$ 70.015

^6.12

$ 29.00

320.324

$100.28

$3.06

$100.03

$79.05

$150.05

$4.31

$ 20.05

$46.00

$100.00

$5.43

$ 40.125

REDUCTION

of cents
Therefore
Rule.

"

one

annex

2.

"

in

Divide

are

dollar,there

are

cents

number

4.

10

cents

is, cut

Divide

10 X

10

100

there

Therefore,

to

reduce

there

dollar,there

are
are

that

ten ;

is^

"

ten ; thcit is^cut

10
100 X

of

of

the

mills

and
in 1

cents

off

cents

by

dimes

in

dollar;then,

dollars

dollars

to

10

times

100

are

reduce

cents

the number

offtiro figures
from

5. As

by

in 1 dime

dollars

Conversely,to
"

cents

as

right.

"

Rule.

by

cents

mills

of

ber
num-

"

mills to cents

Multiply the number


two ciphers.
that is,annex
Rule.

mills

many

mills

of

cent, in any

as

number

reduce

of

dollars.

as

to

the iiumber
the

in

times

cents

Multiply the
cipher.

there

any

10

are

reduce

figurefrom
3. As

to

there

MONEY.

IT. S.

mills

10

are

to
Conversely,

Rule.
one

there

1. As

54.

OF

one

as

to

many
cents

"

hundred;

dollars-^

by

; that

hundred

07ie

right.

in 1 cent

lOirrrlOOO

and

mills

in

100
1

cents

in 1

dollar;then,

1000
times
number
of dollars there are
any
dollars
mills as dollars.
Therefore, to reduce

in

as

to

many
mitls
"

Multiply the number


three ciphers.
is,annex

Rule.

that

ARITHMETIC.

PRACTICAL

NEW

RAY'S

76

6.

Rule.

"

mills

reduce

Conversely,to

the number

Divide

of
the

is,cut offthree figuresfrom


55.

The

1st. If the
between
2d.

is mills.

sum

the third and


If the

betiveen the second

to

"

Rule.

third

that

dollars may

be

point. Thus,
Put

fourth figures
from

and

thousand;

one

thousand

one

cents

or

by

dollars.

by
right.

Rule.

is cents.

sum

to

mills

decimal

the

simply with

made

of mills

reduction

dollars

of

"

Put

"

from
figures

the decimal
the

right.

the decimal
the

point

point

right.

1. Eeduce

17

ct. to

mills.

170

2. Ileduce

28

ct. to

mills.

280

3. Keduce

43

ct. and

m.

to

mills.

436

4. Reduce

70

ct. and

m.

to

mills.

706

m.

5. Reduce

106

m.

6. Reduce

490

mills

7. Reduce

8. Reduce

14

9. Reduce

104

dollars

10.

Reduce

$60

and

13

11.

Reduce

$40

and

5 ct. to

12.

Reduce

375

ct. to

to

10

cents.

ct.

m.

cents.

49

ct.

dollars to cents.

900

ct.

1400

ct.

10400

ct.

6013

ct.

4005

ct.

to

dollars

to
to

cents.
cents.

ct. to

cents.
cents.

dollars.

83.75

13. Reduce

9004

14. Reduce

15. Reduce

S14

16. Reduce

2465

mills

to

dollars.

S2.46

5.

17. Reduce

3007

mills to

dollars.

$3.00

7.

Reduce

3187

cents

dollars.

18.

19. Reduce

ct. to

dollars.

dollars to mills.

10375

and'*2ct.

mills

to
to

to

mills.

dollars.

S90.04
4000

m.

14020

m.

$31.87.
$10,375.

UNITED

OF

ADDITION

1. Add

56.
dollars
5

cents

Rule.

dollars

mills;

dollars

dollars

mills; 16

1. Write

"

MONEY.

S.

U.

together 4
2

87

20

12

numbers

add

and

as

orERATiox.

$" ct. m.

simple numbers.

in

2. Place

the decimal

the decimal
Proof.

the

point in

under

sum

The

same

2 0

is the

2. What

18

mills; 100

cents;

15

dollars

dollars

dollars

50

cents
cents

; 23

cents

37

4. William

mill; 16 dollars 31 cents


3 mills; and
87 dollars 33
S222.199.

had

the

followingbills for

$29.18; $17.63; $268.95;

collection:

$718.07: how

and

much

5.

Bought

gig

for $200 ;

for $56.93 ; a hat


the amount?
$2.31: w^hat was
clothes

of

6. A

for $43.87 ; a suit


for $8.50 ; and
a
whip for

$311.61.
hundred

five

hundred
what

and
and

is the

^yq

whole

four

lars
dol-

dollars
amount

$2222.22.
five dollars

three

dollars

to

was

watch

him?

7. Add
cents

due

; $304 ; $888.47

fiftycents
due

has

him,
cents; $420.19; one

person

six

$43.75;

$1077.58.

collected?

be

dollars

cents; and 12
S68.45.

mills.

cents

19

dollars

cents?

31

3. Add

of 17

sum

cents; 7 dollars

dollars

3 0

$58749T

pie Numbers.

43

!4

10.005
16.875

of Sim-

in Addition

as

o fi 9

pointsabove.

"

dollars

cents; 10

43

mills;

cents

5 mills.

cents

the

77

MONEY.

STATES

two

mills ;

cents;

cents; thirty dollars twenty

seven

one
seven

dollars

hundred
hundred

mill; $1000.10; forty dollars

four

five mills ;

dollars

mills; and

one

sixty

cent

$64.58

one

7.

$2000.

OF

SUBTRACTION

From

57.

the decimal

under

threo

seven

milla

operatiox.

point in the remainder

the decimal

2. Flace

and

cents

subtract

and

numbers

the

cents

five

Numbers.

Simple

in

as

Write

1.

"

dollars

hundred

one

MONEY.

S.

U.

eighty dollars twenty

mills,tuke
Rule.

ARITHMETIC.

PRACTICAL

NEW

HAY'S

78

$.

et. m.

100.053

pointsabove.

80.207

Proof.

The

"

same

Subtraction

in

as

$19,846

of

Numbers.

Simple

2. From

$29,342

take

817.265.

3. From

$46.28

take

$17.75.

$28.53.

4. From

$20.05

take

$5.50.

$14.55.

5. From

$3, take
$10, take
$50, take

6. From

7. From
8. From
cent

9. B

1 mill.
50

$9,999.
mills.

ct. 5

$49,495.

dollars,take

dollar

one

mill.

one

dollars

one

$998,989.
dollars

1000

owes

$2.97.

et.

thousand

one

and

$12,077.

43

cents; if he pay

cents, how
sixty-eight

much

nine

dred
hun-

=^^%e

still

""

owe?

$99.75.
MULTIPLICATION

58.

1. What

5 mills

each?

OF

will 13

cows

U.

S.

MONEY.

cost,at 47 dollars 12

1.

Multiplyas in Simple Numbers.


2. Put
the decimal point in the same
place
in the product^ as
it is in the multiplicand.
Rule.

"

cents

operation.

$47,125

1_3
141375

Proof.

"

The

Simple Numbers.

same

as

in

of
Multiplication

47125

$612,6

2 5

MultiplyS7.835 by 8.
Multiply $12, 9 et. 3 m. by
Multiply $23, 1 ct. 8 m. by
Multiply $35, 14 ct. by 53.
Multiply $125, 2 ct. by 62.
Multiply $40, 4 ct. by 102.
Multiply 12 ct. 5 m. by 17.
Multiply$3.28 by 38.

2.

3.
4.

5.
6.

7.
8.
9.

10. What
a

MONEY.

STATES

UNITED

$62.68
9.

$108,837,

16.

$368,288
$1862.42,
$7751.24.
$4084.08,
$2,125,
$124.64

barrels of cider,at 1 dollar 6 cents

338

cost

79

$358.2B.

barrel?
Sold

11.

cord:

to

12. At

cords

38

7 ct.

wood,

dollars

75

cost

465

pounds

multiplying7 cents l)y465, mulproduct,Art. 30.


tiply465 by 7, which givesthe same
that 7 cents
But, to placethe decimal point,remember
Note.

"

is the true

of

Instead

cents

operation

4 6

.0 ?

$ 'S^2.61

yards

89

cost

of sugar

multiplicand.

13. What

$218.50.

pound, what

at

it amount?

did

what

of

of

sheeting,at

34

ct.

yard?
$30.26.

14. What

will 24

yards

of cloth

cost,at $5.67

yard?

$136.08.
15. I
the

have

value

sheep, valued

at

$2.69 each

what

sell 691

bushels

will it amount

is

$454.61.

whole?

of the

16. If I

what

169

of

wheat,

at

$1.25

bushel,
$863.75.

to?

hogsheads of molasses, of 63 gallons


$2529.45.
each, at 55 ct. a gallon:what is the sum?
18. What
cost 4 barrels of sugar, of 281 pounds each,
$73.06.
at 6 cents
5 mills a pound?
19. Bought 35 bolts of tape, of 10
yards each, at 1
$3.50.
did it cost ?
cent a yard : what
11 hours a day,
20. If I earn
13 ct. an hour, and work
17. I

how

much

sold

73

will

earn

in

312

days?

$446.16.

sold

21. I

$1.25

What

22.

150

cost

bushels

each,

dollars

10

23. What
24

17

cost

7 mills

cents

per

bags of coffee,of
pound?

OF

DIVISION

To

I.

Case

59.

"

is contained

money

1 mill

81500.15.

acre?

per

at

$67.50.

land, at

of

acres

amount?

is the

what

bushel:

wheat, of

of

bags

18

AKITHMETIC.

PEACTICAL

NEW

KAY'S

80

much

1. How

pounds each, at
$214.14

9.

MONEY.

S.

how

find
in

U.

51

times

many

one

of

sum

another.

cloth,at

7 cents

yard,will $1.75 buy?

OPEllATIQN.

times

175

in

Rule.

"

is contained

Aij many
yards as 7 cents
25.
are
cents, which

Solution."

1. Beduce

both

of

sums

7)175
2 5

to the same

money

denom-

ination.
2. Divide

much

2. How
for 72

in

SimpleNumbers.
rice,at

many

be

bought

towels,at

many

37

5 mills

yards

be

$1,125 per

of

calico,

at

ribbon, at

bushel, how

purchased for

lb.

apiece,
16.

purchased for $3?


6. At
$8.05 a barrel,how
many
$161 purchase?
7. At 7 cents 5 mills each, how
bought for $1.20?
can

and

cents

cents

25

cents

be

8. At

bought

for $6?

yards of
many
bought for $2.80?

5. How
can

be

pound, can

4. How
be

cents

cents?

3. How
can

as

$234?

barrels

3^ard,can
35 yd.
a
yard,
12 yd.
flour will

of

20

many

many

oranges

can

bl.
be

"

16.
bushels

of wheat
208

bu.

STATES

UNITED

Case

II.

number

of

To

"

1, A

divide

MONEY.

of

sum

81

into

money

given

equal parts.
worked

man

for

days

$3.75, what

his

were

daily wages?
OPERATION

Solution.

did he

bushel

Solution.
vided

$3

300

cents.

-^

r=:

of wheat

bushel

got for eadi

gives

were

$1.25.

for $d

) 3.7 5
$T25

how

much

get?

He

"

by
300

daily wages

$3.75

sold 6 bushels

farmer

2. A
a

His

"

quotient $1,

with

divided

by

cents

$9 di-

$9"6.

remainder

operation.

gives a

6)9.00
$1.50

quo-

tient 50 cents.

Rule.

1. Divide

"

2. Put

as

the decimal

Simple Nvmbers,
in
point in the same j^lace
in

the

quotient

it is in, the dividend,

as

1." If

Kem.

Rem.

third
2.
"

it may

Should

3. Divide

how

cipherfor

65

5. If 4

be

dollars

for cents;

remainder

not

and,

exact,
if

essary,
nec-

for 23

$29.61

obtainingtlie mills,
the quotient.

after

equallyamong

per

of land

acres

point

sign -f placed after

the

that

was

decimal

the division

mills.

received

farmer

much

by

dollars,and

is

the

there

be indicated

4. A

dividend

ciphers after

two

annex

the

8 persons.

bushels

bushel?

cost

S8.125.
of wheat:

$1,287 H-.

$92.25, how

much

is that

an

$23,062+.

acre?
6. Make

equal division

an

of $57.50 among

8 persons.

$7,187+.
7. A
much

man
was

received
that

day?

$25.76

for

16

days' work:

how

$1,61.

ARITHMETIC.

PRACTICAL

NEW

RAY'S

82

bought 755 bushels of applesfor $328,425: what


$0,435.
did they cost a bushel?
much
is that a day,
9. My salaryis $800 a year : how
there being 313 working days in the year?
$2.555-|-.
8. I

ten

thousand

is each

man's

10. Divide
what

men:

11. A

did it cost

what

12. I sold

$60:

bought

13.

and

of

kegs

much

oAve

$19.43: what

sum

2. A

and

$1

did

il A

paid
B:

and

4. I went

do

I owe?

B,

6. I
much
7. A

get $50

$3.85,

SI .50:

how

much

twice

much

much

as

his

as

$300.

left?

bought

at

dress
25

was

left?

and

spend

marketing

tea

$1.25, for

had

he

left?

75

for flour

$1.90.
for

$8.10,shoes

cents

yard,and
25
of

$30.50

left in 6 months?

sold
for

sum

B,

$119.

vegetables50 cents,

was

and

$5 ; I spent for butter

of delaine

month,

as

amount?

w4th

money
$20; she

will I have

sugar

as

did I receive?

for $4: what

farmer

$120.85.

$23.75 ; C,

is the

lady had
$5.65,eight yards

shawl

$0,065.

Examples.

35 cents, for

much

pounds each,

235

cost?

pound

market

to

5. A
for

of

$47.50; B, $38.45; C, $15.47; D,

much

cents, for eggs


$1.50: how

16 ct.

$18.38;B, $81.62; C,

me

how

1 cent:

pounds each, for

25

of sugar,

for two

what

more:

and

pound?

barrels

$35.25

owes

acres,

$17,565.

Promiscuous

1.

154

dollars

five

butter,of

that

Avas

for $122.20: what

60.

of

per acre?

15

how

$75.187 -f.

farm

hundred

seven

133

equally among

share?

purchased a

man

thousand

dollars

ct.

it: how

$117.
for $21.75

coffee

he

paid for
$2.50,for spices
:

$12.65.

UNITED

8. T
E

and

do

9.

after

S37.06; B, $200.85; C, S400; D, $236.75,


$124.34; my property is worth $889.25 : how much
than

more

owe

owed

cents

me

of

$109.75.
23

coffee,at

was

he

$400:

bushel:

worth?

am

Bought 143 pounds


paying $12.60, what

45

83

owe

10. A
at

MONEY.

STATES

pound :

due?

$20.29.
435

paid me

what

cents

sum

bushels

of corn,

is due?

$204.25.

11. If B

spend 65 cents a day, how much will he save


in 365 days, his income
$162.75.
being $400?
of 3 bushels
12. Bought 21 barrels of apples,
each, at
did they cost?
35 cents a bushel: what
$22.05.
13. What
cost four piecesof cambric, each
containing
19 yards,at 23 cents a yard?
$17.48.
14. If 25 men
perform a piece of Avork for $2000, and
spend, while doing it,$163.75,what will be each man's
share of the profits?
$73.45.
receive $516 for 43
15. If 16 men
days' work, how
much
does each man
75 ct.
earn
a
day?
16. C earned
$90 in 40 days,working 10 hours a day:
how

much

did

he

17. A

merchant

$500

in

cash, to

how

much

earn

be

hour?

an

has
failing,
goods worth
equallydivided among

will each

Bill

bought
18.

or

or

Account,

sold,with

Bought

9
4

45

17
"What

$1000,
22

receive?

is

their

and

creditors:

$68.18-|-.

written

statement

of my

$0.32 per lb.


1.25

do.

"

.09

do.

"

.20

do.

"

of

articles

entire cost.

and
prices,

pounds Coffee,at
pounds Tea,
pounds Sugar
pounds Cheese

is the amount

m.

BILL8.

MERCHANTS'

ct. 5

22

bill?

$15.33

RAY'S

84

Bought

19.

22

18
25
6

What

yards Silk,
yards Muslin,
yards Linen,
yards Gingham,

yards Muslin,
28"^5iiaTd8
Calico,
23 yards Alpaca,

is the

whole

REDUCTION

$1.75 per

yd. S

''

.15

do.

'-

.65

do.

"

.18

do.

at

$0.13 per

yd.

'-

.09

do.

''

.23

do.

$13.40

COMPOUND

OF

NUMBERS.

MEASURE.

is used

Measure

Dry

at

amount?

DRY

61.

AKITHMETIC.

43

Bought

21.

PRACTICAL

NEW

in

measuring grain,vegetables,

fruit,coal,etc.
Table.
2

Rem.

1.
"

pints(pt.)make

quarts

"

pecks

''

The

measure
cylindrical

21501

cubic

standa7'd iiniiof

18^
inches.

inches

in

quart, marked

qt.

pk.

peck,
bushel,

Dry

Measure

''

'"

is the

bu.

bushel; it is a

diameter, 8 inches deep,and

tains
con-

REDUCTION

by weight,the bushel
givesthe legalweight of a
sold

NUMBERS.

usuallymeasured

articles

2. "When

Rem.

COMPOUND

OF

is taken
bushel

by

the unit.

as

the

85

above

table

are

The

followingtable
articles in avoirdupois

of various

pounds:

To

Teachers.

similar

to the

1. How

"

questionsshould

Numerous

be asked

on

each

table

6?

In

following:
In

many

pints in

2. How

many

quarts

in

3. How

many

pecks

in

bii.?

In

11?

In

13?

In

in

10

bu.?

In

12?

In

14?

In

In

32?

In

In

8?

15?

In

4. How

18?

In

5. How
40?

In

quarts?

In

4?

10?

17?

In

many
In
25?

48?

In

In

5?

7?

In

9?

19?

quarts
56?

pecks

many

In

pk.?

in

16

qt.?

In

24?

64?

6. How

many

bushels

7. How

many

pints in

in

32

qt.?

1 bu. ?

In

In
2 ?

64?

In

In
5?

96?

KAY

St)

precedingexamples show

62. The
To

reduce

reduce
in the

to

quarts
number

the

quarts by
To

ARITHMETIC.

PRACTICAL

NEW

that

"

pints,multiplythe
of pintsin a quart.
bushels

pecks to quarts, or

number

of

ply
pocks,multi-

to

manner.

same

from

Hence, to reduce
multiplyby the

higher to

lower

tion,
denomina-

of units that make

number

unit

one

requireddenomination.
They also show that
of pints
To reduce
pints to quarts,divide the number
of pintsin a quart.
by the number
To reduce
quarts to pecks,or pecks to bushels,divide

of the

"

in the

manner.

same

Hence,

reduce

to

from

lower

to

of units
by the number
requiredhigher denomination.

divide
of the

1. Eeduce

bushels

tion,
higher denomina-

that

make

unit

one

pints.

to

OPERATION.

Solution.

there

because
many

are

bu.

pk. as

8, because

reduce

To

"

To

there

are

pk.

bu. to
in

^ bu.

pk. multiplyby 4,
bu.,

or

times

as

f2pk.

reduce
8

pk, to qt.multiplyby
qt. in 1 pk. To reduce qt.

pt. multiplyby 2, because

to
1

there

are

pt. in

qt.

foTpt.

2. Eeduce

192

pints to bushels.
OPERATION.

Solution.
there

cause

divide
reduce

are

reduce

pt. to

pt. in 1 qt.

qt. divide

by

To

qt. to pk.

reduce

2, be-

by 8, because there are 8 qt. in 1 pk.


pk. to bu. divide by 4, because there are

pk. in

1 bu.

The

two

To

"

higherto

precedingexamples show
a

lower

2)192

8)96 qt.
4)12 pk.

To

3 bu.

that

reduction

denomination^and from

higherdenomination,prove

each

other.

pt.

from

loicer to

7 bu. 3

3. Eeduco

COMPOUND

OF

REDUCTION

NUMBERS.

j)k.6 qt. 1 pt.

87

joints.

to

OPERATION.

bu.

Multiply the bu. by 4,


making 28 pk.,and add the 3 pk. Then
multiply the 31 pk. by 8 and add the 6
qt.; multiplythe 254 qt. by 2 and add the
Solution.

pt.; the

is 509

result

Solution.
2, and
the

509

3 1

there

is 1

remainder

be

must

pk. in

I.

1.
her

FROM

the next

pt. to qt. divide


dividend

the

31

pt. in

the

by

is

reduce

pt. To

2. Add

to

pk. 6 qt.
whole.

operation.

pt.

qt. to

2)509
8)254

TO

HIGHER

which

the

product

in

like

4)31

7 bu. 3

pk.

DENOMINATION.

LOWER

makes
the

qt. 1 pt.
pk. 6 qt.

REDUCTION.

FOR

RULES

lower

pk

bushels.

Multiplythe highestdenomination

of

7 bu. 3

qt.in

pk. divide by 8, and 6 qt. are left. To reduce


pk. to bu. divide by 4, and 3 pk. are left. The
answer
is,therefore,7 bu. 3 pk. 6 qt. 1 pt.

03,

pt. to

left;as

z.

2 5 4

pt.

reduce

To

"

_4

5 0 9

4. Eediice

pk. qt. pt.

"

given,by that num


unit of the higher.

number, if any, of

the

lower

denomination.
3. Proceed

tdl the u'hole is reduced

II.

FROM

1. Divide
denomination
2. Proceed

tillthe whole

icith the

manner

to the

LOWER

TO

the

manner

is reduced

to

obtained,

requireddenomination.
HIGHER

given quantityby that


which makes a unit of the
in like

result thus

DENOMINATION.

number

of

its

own

higher.
thus obtained,
with the quotient
denomination.
the required
next

3.

The

last

Jf this result is the

is, reduce

operation: that

I he

from

denomination

the

to

answer

remainders^if any,

answer.

Eevcrsc

"

the several

with
quotient,

annexed, will be the


Proof.

AKITHMETIC.

PKACTlCAl^

NEW

KAY'S

88

same

as

the

which

it

was

the

derived.

quantitygiven,the

work

is correct.
bu.

5. lieduce

6. Eeduce

7 bu. 3

7. Reduce

3 bu. 1

8. Reduce

384

9. Reduce

47

10. Reduce

95

11. Reduce

pk. 1 qt. to pints.


pk. 7 qt. 1 pint to pints.
pt. to pints.

Liquid

Measure

511

pt.

193

pt.

pk. 7 qt. 1 pt.


bu. 1 pk. 7 qt. 1 pt.
7 bu. 3 pk. G qt.

MEASURE.

LIQUID

64.

pt.

(" bu.

pt. to bushels.
pt. to pecks.
pt. to bushels.
pt. to bushels.

508

290

is used

for

measuring all liquids.

Table.
4
2
4
Kem.
contains

"

The
231

1. Reduce

gills(gi.)make
pints
quarts
standard
cubic

unit

pint,marked

^*

quart,

'"

"

gallon,

"

of

liquidmeasure

pt.
qt.

gal.
is the

gallon,which

inches.

136 pt
gal.to pints.
13 gal.to gills.
2. Reduce
416
gi.
3. Reduce
126 gal.to pints.
1008 pt
4. Reduce
1260 gal.to gills.
40320
gi.
5. Reduce
1120 gi. to gallons.
35 gal.
6. How
8 gal.
gallonsin 1848 cubic inches?
many
7. How
gallons in a vessel containing138138
many
cubic inches?
598 gal.
17

REDUCTION

ordinary

used

is

Weight

Avoirdupois

89

WEIGHT.

AVOIRDUPOIS

65.

NUMBEKS.

COMPOUND

OF

for

weighing

all

articles.
Table.

16
100
20

Rem.
is

"

cwt.,

"

2.

"

ton,

the

At
a

ton.

2. Eeduce

3. Keduce

4. Eeduce

5. Eeduce

6. Eeduce

pounds.
cwt. 75 lb. to pounds.
T. 2 cwt. to pounds.
T. 75 lb. to pounds.
44 lb. to pounds.
cwt.
T. 90 lb. to pounds.

7. Eeduce

cwt.

77

lb. 12

8. Eeduce

cwt.

17

lb. 3

9. Eeduce

1 T. 6 cwt.

to

oz.

lb. 2

11. Eeduce

22400

12. Eeduce

2048000

13. Eeduce

64546

oz.

to

14. Eeduce

97203

oz.

to tons.

15. Eeduce

544272
is the

to

oz.

4803

16. What

200.

to

cwt.

10. Eeduce

oz.

375.
2200.
6075.
444.

10090.

is the
75

lb.?

4444.

ounces.

3475.

ounces.

to

48

lb. to cwt.
lb. to

oz.

total

64

40

weight

lb. 2

oz.

T. 75

lb. 3

oz.

17

tons.

of 52

of

180

T.

17

each
parcels,
9

weight

iron

T.

34

cwt.
3

lb.

T. 4 cwt.

tons.

cwt.

to

cwt.

11

tons.

to

oz.

41666.

ounces.

lb.?

17. What

weiffhin^

T.

"
^

1. Reduce

18

cwt.

avoirdupoispound of the United States


the Troy pound, and contains 7000 grainsTroy.
House
Custom
trades)2240 pounds
(and in some

from

considered

standard

The

1.

lb.,"

2000

or

lb.

''

pound,
hundred-weight/'

make

pounds

determined
Rem.

are

(oz.)

ounces

cwt.

lb.

taining
con-

36

lb.

castings,each
6

T. 15

cwt.

in
lengtli,

or

is used

Measure

Long

IIMET

I C.

MEASURE.

LONG

66.

A KIT

PRACTICAL

NEW

KAY'S

90

in

measuring distances,

direction.

any

Table.

inches

12

feet

16^ feet,

rods

320

yard

standard

The

"

United

the

for

of this standard

unit

is

States

yard,

'-

yd.

'*

rod,

''

rd.

"

mile,

"

mi.

kept

1. Reduce

2. Reduce

yd. 2 ft. 7
yd. 11 in.

3. Reduce

12

4. Reduce

7 mi. 240

5. Reduce

6. Reduce

133

in. to

yards.

7. Reduce

181

in. to

yards.

at each

to

rd. to rods.
rd. to rods.

mi. 31

miles.

9. Reduce

2200

rd. to

miles.

10. Reduce

1 mi.

to

yards.

11. Reduce

1 mi.

to

feet.

SQUARE

has

both

Measure

yd.
5

""("))"

103

in.

263

in.

3840

rd.

2480

rd.

2911

rd.

ft. 1 in.

1 in.

yd.

6 mi.

mi.

280

rd.

1760

y(\.

5280

fL

MEASURE.

is used

length and

figurehaving

square.

standard

mi. to rods.

rd. to

which

The

inches,

2240

Square

yard.

in. to inches.

Reduce

67.

the

preserved at Washington.
state capital.

is

ft.

foot,

lengthis

of

marked

'"

5^ yards,or

Rem.

make

(in.)

in

breadth

equal sides

and

measuring
; that

any

is,two

thing
dimen-

rightanglesis

OF

REDUCTION

A
in

inch

square

is

COMPOUND

square,

NUMBERS.

each

91

side of which

is 1 inch

length.
A

foot is

square

is

yard

square

each

square,
a

side of which
each

square,

side

is 1 foot.
which

of

is 1

yard (3 feet).

Suppose the figureto represent


square yard. It will then be 3

feet

and
9
contain
way,
foot will he 12
feet. Each

each

square
inches

each

way,

and

contain

144

inches.

square
The

of small

number

in

squares

is, therefore,
large square
number
the
of units in
equal to
side multipliedby itself.
one
any

Rem.
which
squares
in area

By

"

is 3

feet

feet,or

each

between

feet,is 6

square

meant

long and

one

foot

figureZfeet square

square

feet; but by

9 square

foot

one

is

square

side of

figure,each

square

feet

is meant

wide; therefore,the difference


and

one

containing 3

square

feet.

Table.
144
Q
9

30J
'^"
160

640

square

inches

make

1 square

foot,marked

1
1

a7"it1
yard,

finnar.^

square

fonf
feet

square

yards

1 square

square

rods

1 acre,

"i

square

1 square

acres

1. Eeduce

Qmior^o

2. Reduce
3. Eeduce

1 sq. mi.

4. Reduce

2 sq.

5. Eeduce

6. Eeduce

960

A.

to

mile,

yd.

to

to

square

rods.

3 sq. ft. to sq. in.

sq. rd. to sq. rd.


sq. rd. to acres.
100

"

yd.

en

^',

sq.

sq. rd.
A.

rod,

inches.
square
rods.
square

yd.

sq.
4 A.

sq. ft

sq. mi

'"

10368
640
102400

sq. in.
sq. rd.
sq. rd.

3024

sq. in.

900

sq. rd.
6

A.

PRACTICAL

NEW

KAY'S

92

7. Eeduce

ARITHMETIC.

8. Eeduce

sq. in. to square


20000 sq. rd. to acres.

9. Reduce

515280

yards.

3888

68.

sq. in. to

4176

A.

right angles.

four
The

of

unit

side is

linear

The

Area

or

See

the

for

unit;as

inches

square
many
3 inches wide?

Explanation."

of

its unit

it contains

in

of

Dividing each

inch,a

contents
Superficial

1. How

and

surfaces,is

square

the

sq. rd.
2 sq. ft.

yd.

sides and

whose

square

foot,etc.

square
a

of

figure,is

tlie

measure.

board

A.

80

four

figurehaving
figurebelow.

measure

of times

number

is

Rectangle

3 sq.

yd.

sq.

yd.

miles.

sq. rd. to square

5 sq. mi. 20

10. Eeduce

sq.
125

inches

long

longer

sides into 4

equal parts, the shorter sides into


equal parts, and joining the oppositedivisions
by straightlines,the surface is divided into
o

squares.
In

that
as

each

of the

is,as many

there

longerrows
as

such rows
many
The whole
number

there
as

inches

are

there

4 squares,

are

in the

inches

are

of squares

longerside;and

there

in the shorter side.

in tbe board

obtained

by multiplying together the


12.
length and breadth; that is,4 X 3

is

numbers

are

Hence,

equal to the product


representingthe

Rule

the

for

Finding

lengthby the

Kem.

"

Both

the

the

breadth ; the

length and

denomination, should, be
2. In

square

floor 16

feet?

Area

feet

made

of

Beetangle.

product uill

breadth, if
so

long

before

and

12

not

be the

in units

Multiply

"

area.

of

the

same

multiplying.
feet

wide,

how

many
192.

3. How

yards

4. How

feet

5. How
and

feet

18

long and

93

cover

20.

two
yards of carpetingwill cover
long and 12 feet wide, the other 21

square

many

one

rooms,

NUMBERS.

yards of carpetingwill
4 yards wide?

square
long and

many

room

COMPOUND

OF

REDUCTION

feet,wide

15

59.

yards

square

many

?
in

ceiling18

feet

long

feet wide?

14

6. In

28.

field 35 rods

long and

rods

32

wide, how^

many
7.

acres?
7. How

will it

much

long and
yard?
square

feet

18

per

8. What
and

feet

18

The

69.

it cost

Area

other

to

$75.

of

plastera ceiling21
cents
yard?
per square
to

17

Eectanglebeing equal to

the

length by

divided

numbers,

two

feet

15

wide, at

the

of
of

will

each
carpet two rooms,
wide, if the carpet costs $1.25

cost

breadth, and

by

either

of

feet

long

S7.14.
the

product
the product
as
them, gives the

(36, 4); therefore,

Rule.

"

If the

area

of

he
rectangle

divided

by either side,

the quotientwill he the other side.

Illustration.
In

Example 1, 68, if the area


the quotient3 is the width; or,
quotient4 is the length.
Rkm.

Dividing the area


really
dividingthe number
"

of squares

In

on

one

inches

Art. 68.
figure,

divide

by
rectangle

of squares in the
of its sides.

dividing12 by 4, the

of square

of

in

be

12

latter is not

rectangle4

12

by 4,

by 3,

inches,but the

long and

the

of its sides,is

rectangle"bythe

4 linear
in.

one

divided

1 in. wide.

ber
num-

ber
num-

See

floor

1. A

is its

what
2

containing132

feet,is

square

11 feet wide:
12 il.

length?

A. floor is 18 feet

what

ARITHMETIC.

PRACTICAL

NEW

RAY'S

94

long,and

contains

30

square

yards:
15

is its width?
field

3. A

containing9

rods

is 45

acres,

in

rods

field 35

4. A

wide, contains

21

acres

what

SOLID

Solid

OR

rd.

MEASURE.

CUBIC

is used

Measure

Cubic

or

rd.

is its
96

length?

70.

length:
32

is its width?

what

ft.

things having length,breadth,

and

in

measuring
thickness; that is,

three dimensions.

Cube

is

solid, having

equal faces,which

are

squares.

Rem.

"

side of

If each

cube is 1 inch

long,it is called a cubic inch; if each side


in
is 3 feet (1 yard) long,as represented
the figure,.
it is a cubic or solid yard.
The
base of a cube, being 1 square
yard,contains 3 X '^ 9 square feet;and
1 foot high on this base,contains 9 solid
18 solid
feet;2 feet high contains 9 X ^
27 solid
feet;3 feet high contains 9X8
=

feet.
or

Also, it may

be shown

cubic foot contains

12 X

that

12 X

12

1 solid
=

1728 solid

or

cubic inches.

Hence, the number of small cubes in any largecube,is


and thickness,
equalto the length,
breadth,
multiplied
together.
Rem.

"

Any solid,whose

solid;boxes

and

cellars are

corners

resemble

generallyof

cube,is

this form.

rectangular

COMPOUND

or

KEBUCTION

NUMBERS.

95

solid are found, as in


rectangular
cube, by multiplyingtogetherthe length,breadth, and

The
the

solid contents

of

thickness.

Table.
cubic inches

1728

27 cubic

feet

cubic feet

128

"

cord.

1.

contains

cu.

ft.

1 cubic

j^ard,

cu.

yd.

8 cord

24J

is

length of

feet make
a

16J

mass

2. Reduce

28

cords

of wood

to

cu.

3. Reduce

34

cords

of wood

to

cu.

4. Reduce

1 cord

5. Reduce

63936

long,5

many

many

8. How

feet

many

wide, 7

cu.

cu.

feet in

cubic

ft.

3584

in.

feet

pileof

cu.

in.

221184

cu.

in.

in.

10. What
feet

40

of excavation

feet

long,12

in

80

the

cost

feet

high,and

ft.

cu.

cellar 8

yd.

cu.

long,15
70 cu. 3'd.

feet wnde, and


30

of

ft.

feet

cords?

many
will be

ft.

yd. 1 cu. yd. 10 cu.


rectangularsolid, 8

deep?
wood

ft.

cu.

7520256

yd. deep?
yards in a cellar,18

in.

cu.

160

yards

cubic

high, how

long,8

in. to

cubic

yd. long,5 3^d.wide,

9. In

to

93312

ft. wide, 4 ft. thick?

7. How

feet

cubic inches.

of wood
cu.

ft. long,1^ ft. wide, and

ft.

cu.

yd. to

pilewhich makes
foot long; hence, it

1 cord.

1. Reduce

6. How

C.

the

high, and

feet

of stone

cu.

^^
"

4 ft. high, make)

feet,and

perch

"'

^^1cord,

foot is 1 foot in

contains

1 ft. high,and

'

cord

16 cubic
"

foot,marked

8 ft. long,")^

It is 4 feet wide, 4

2.

Rem.

1 cubic

"

8X4X4
ft. wide, and

Hem.

make

(cu.in.)

pile of

thick,at S5.50

wood
per

8
C.

feet

80
cord?

$110.
11. What

long,15
or

load?

will be

the

ft. wide, -and

excavating a cellar 24 ft.


ft. deep, at $1.25 per cubic yard

cost

of

$100.

RAY'S

96

PRACTICAL

NEW

MEASURE.

TIME

71,

is used

Measure

Time

ARITHMETIC.

in

measuring time.

Table.

min.

minute, marked
hour,
''

hr.

''

day,

''

da.

'^

1 year,

^'

(sec.)make

60

seconds

60

minutes

''

24

hours

365

days,6

100

years

Also,

hours

make

days
months

Rem.

1.
"

The

exact

(nearly),
'^

1 year,

days
days

366

cen.

marked

week,

1 month

12 calendar

''

century

4 weeks

365

yr.

lengthof

common

leapyear.

the

yr.

year.

solar,or

moan

mon.

''

wk.

tropical
year,

is

days,6 hours, 48 minutes, 46 seconds.


To correct the error
of considering365 days as the lengthof the
year, the followingrule has been adopted:
is not
divisible by 4 consists of 365
Every year whose number
days.
is divisible by 100, but not by 400,
Every year whose number
consists of 365 days.
365

is divisible
centuries,whose number
Every year, except the even
centuries divisible by 400 consist of 366 days.
by 4, and the even
The year containing
366 days is called Leap year, and the extra
day is added to February,givingit 29, instead of 28 days.
Rem.
2. Among
nearlyall civilized nations the year is divided
"

into 12 calendar

January,
February,

1st

months, and numbered,

month, 81 days.

in their

order, as follows:

July,

7th month,

31

August,

8th

**

31

"

September, 9th

"

30

"

days.

2d

"

28

"

March,

8d

"

31

"

April,
May,

4th

"

30

'"

October,

lOth

"

31

"

5th

"'

31

''

November, 11th

"

30

"

June,

6th

"

30

"

December,

"

31

"

12th

COMPOUND

OF

REDUCTION

1. Eeduce

2. Eeduce

7 da. to minutes.

3. Reduce

NUMBERS.

97

7200

hr. to seconds.

da. 3 hr. 44

sec.

min.

10080

min.

to

sec.

seconds.
99843

4. Reduce

5. Reduce

wk.

da.

hr. 40

10

3 da. 4

mon.

seconds

6. Reduce

10800

7. Reduce

432000

8. Reduce

7322

seconds

9. Reduce

4323

minutes

min.

to

minutes

to

11. Reduce

41761

min.

months.

and

sec.

3 da. 3 min.
2 wk.
1

mo.

2 hr.

1 da. 1 min.

OF

is used

WEIGHT.

in

weighing gold, silver,

jewels.
make

24

grains (gr.)

20

pennyweights

12

ounces

The

2 hr. 2 min.

TABLES.

I. MEASURES

Weight

min.

5 da.

weeks.

MISCELLANEOUS

Troy

44644

3 hr.

days.

20280

72.

min.

days.

10. Reduce

to

100000
to minutes.

to hours.
to

minutes.

to

hours.

to

seconds

min.

sec.

Standard

pennyweight, marked

"

1 ounce,

"

in

weight
grains.
is

"Weight

oz.

lb.

'^

the United

used

pwt.

'^

pound,

of all

Unit

pomid, containing5760
Apothecaries

only

States is the

in

compounding

medicines.
20

grains (gr.)make
scruples
''

8 drams
12

ounces

marked
scruple,

dram,

'^

1 ounce,

"

pound,

"

''

"

Troy

9.
3.
5.

lb.

The

also used

followingare

8 fluid drachms
10 fluid

1 fluid ounce,

'*

pt. (octarius)

"

''

gal.(congius)

"

ounces

II. MEASURES

OF

followingmeasures

of them

still used

are

lines

12

in

O.
cong.

and

mentioned

feet

1 pace.

barleycorns
=

1 inch

feet

=1

inches

1 hand.

miles

inches

Surveyors

use

fathom.
'

69^ miles (nearly) 1

1 span.

four

chain

most

specialprofessions:

inch.

=1

f. ".

"

LENGTH.

often

are

f. 3.

drachm, marked

"

pints

The

:
by apothecaries

1 fluid

make
(ordrops)rT\^.

60 minims

AKITHMETIC.

PKACTICAL

NEW

RAY'S

98

rods

league.
degree.

into

long,divided

Ty^^ inches each.


Engineers divide the foot into tenths and hundredths.
The
yard is also divided similarlyin estimating duties
links

of

at the

houses.

custom

degree

is

divided

into

60

nautical

or

geographic

miles.
A

nautical

mile

knot

or

is,therefore,nearly1^

mon
com-

miles.

60

seconds

60

minutes

degrees

360

Rem.

"

each, and

is used

Measure

Circular

The

C)

circumference

into siqjisof

make

in

measuring circles.
marked

minute,

^'

degree,

"

1 circle.

is also

80" each.

divided

into

"

'.
".

quadrants of

90"

12

dozen

"

gross,

"

great gross.

gross

20

things

100

pounds
pounds

of flour

1 barrel.

"

sheets

of paper

20

quires

pork

folded

of lime

"

make

1 barrel.

cask.

quire.

"

ream.

"

1 bundle.

in

leaves

'"

''

"

"

''

''

12

''

"

"

16

"

"

'^

1. Reduce

keg.

beef make

or

reams

is called

Examples

73.

24

gr.

of nails, make

of

240

"

1 score.

"

pounds
pounds

200

doz.

dozen, marked

things make

12

99

TABLE.

12

196

sheet

NUMBERS.

MISCELLANEOUS

III.

COMPOUND

OF

KEDUCTION

in

folio.

quarto,

or

4to.

an

octavo,

or

8vo.

duodecimo, or

16mo.

Miscellaneous

5 lb. 4

12mo.

Tables.

10. Eeduce

64.
Troy to ounces.
5 pwt. to pwt.
9 lb. 3 oz.
2225.
8 lb. 9 oz.
13 pwt. 17 gr. to gr.
50729.
805 pwt. to pounds.
3 lb. 4 oz. 5 pwt.
12530 gr. to pounds. 2 lb. 2 oz. 2 pwt. 2 gr.
4 lb. 5 g 2 gr. to grains.
25442.
41300.
7 lb. 2 " 1 9 to grains.
4 lb. 5 5 7 5.
431 3 to pounds.
3 lb. 4 " 5 5.
975 9 to pounds.
1 lb. 1 g 1 3 1 9 1 gr.
6321 gr. to pounds.

11. Eeduce

2. Eeduce

3. Eeduce
4. Eeduce

5. Eeduce
6. Eeduce
7. Eeduce
8. Eeduce
9. Eeduce

oz.

"

cong.

7 f

to

fluid drams.

4152.

12.

NEW

Eediice

5 O.

6 f.

"

!H69

f. 5

to

13. Keduce

AKITHMETIC.

PRACTICAL

KAY'S

100

f. 3

minims.

to

gallons.

O. 4 f. 3

2 cong.
14. Reduce

41460.

5 f. 3.
324.

yd. to barleycorns.

lines in 1 foot 6 inches?


many
is the height of a horse of 16^ hands?

15. How
16. What

216.

5 ft. 6 in.
field

17. A

how

in breadth:
18. A

cistern

chains

24

measures

many

lengthand

36.

cubic feet 624

267

(Art.64,
gallonsdoes it hold?
many
8" 41' 45" to seconds.
19. Reduce

how

cubic

inches

Eem.).

61"

21. Reduce

915' to

22. Reduce

3661"

gross

of

2000.

223168.
15"

degrees.
to degrees.

cost

31305.

59' 28" to seconds.

20. Reduce

23. What

15 chains

in it?

acres

contains

in

15'.

1" 1' 1".


5 cents

at

screws

dozen?

$3.60.
24. A

is 4

man

25. At

18

and

score

certs

how

10:

old is he?

will 3 bundles

quire,what

90 yr.
of paper

$21.60.

cost?
26. How
a

12mo.

sheets of paper
many
book
of 336 pages?

27. An

Vol.
496

work

octavo

1, 528
in Yol.

of the whole

in Yol.
5:

how

2,

528

much

in Vol.

paper

3,
was

work?

1. What

cost

2 bu.

requiredfor

14.

in 5 volumes

Promiscuous

74.

will be

has
512
used

512

in Vol.
for

in

pages

4,

and

one

copy
6 quires17 sheets.

Examples.

of

plums, at

5 ct.

pint?
$6.40.

2. What

l"eck?

cost

bu.

pk.

of

peaches, at

50

ct.

$7.

3. What

4. At

be

cost

15

ct.

peck, how

'ai
baVl^y,

ct.

pint,how

each

Rem.

"

bu.

91

put

how

of wheat

apples can

were

to pecks,and
quantities

both

spikes,weighing

many

parcelweighing 15 lb. 12 oz. ?


52 lb.
cwt.
8. I bought 44
weighed 9 lb. 15 oz. : how many

l)e

can

bought

pk. 1 qt. 1 pt.


bags containing3 bu. 2
26.
required?

into

bags

many

Reduce

7. How

much
1 bu.

SI.66?

pk.

of

pint?

5 bii.

5. If salt cost

6. I

3* ct.

bashels

many

101

for 83?

bought

with

qt. of

pk.

NUMBERS.

COMPOUND

OF

KEDUCTION

then

divide.

in

each, are

oz.

63.

of

cheese

cheeses

each

did

cheese

buy?
448.

kegs, of 84 lb. each, can be filled


hogshead of sugar weighing 14 cwt. 28 lb.?
10. How
boxes, containing 12 lb. each, can
many
9. How

many

filled from
11. If

long will

cents

13. A

will

of sugar

oz.

125

acres

41

rods

of land

cost,at
$89.

53

sq. rd. to

divide

A.

16

sq. rd. each

wk.

160

square

rod?
square
has a field of
farmer

how

week,

lots of 1 A.

into

be
63.

lb. last them?

10

cwt.

12. What
20

3 lb. 13

family use

17.

lb. of tobacco?

56

7 cwt.

from

how

lots will it

many

make?

13.

14. How
2 ft.

what

do

cu.

cu.

16. What

block

of marble

ft. wide?

high, 2

15. One

in. in

cu.

many

ft. of
ft.

is the

the space

weighing 1000

13824.

weighs

water

weigh?
weight of

of 1 cord
ounces

ft. long,

of

1000

oz.

avoirdupois:
312

quantityof

wood,

each

avoirdupois?

cubic

water

lb. 8

oz.

ing
occupy-

foot of water
4

T.

PRACTICAL

NEW

KAY'S

102

cufei6' fo("tI af! oak

17' jV

ARITHMETIC.

950

weighs

'

avoirdupois:

oz.

7 T.

Ay.bat "l5c""3
oak'\\:eigh?
;Coi\lw,*of;
*'

18*

\^ihd'the *eo"t^of' 63' gallonsof wine,

at

1:^ cwt.

20

cents

$100.80.

pint.
the

19. Find

cost

of

molasses,each

of

barrels

taining
con-

$63.
gal.2 qt.,at 10 cents a quart.
20. At 5 cents
a
pint,what quantity of molasses can
5 gal.
be bought for $2?
dozen
21. How
bottles,each bottle holding 3
many
be filled from
63 gal.of cider?
6 doz.
qt. 1 pt.,can
22. How
kegs,of 4 gal.3 qt. 1 pt. each, can be
many
filled from
58 gal.2 qt.?
12.
heart beat 70 times
a
23. If a human
minute, how
100800.
times will it beat in a da}^?
many
seconds
of February,
in the month
24. How
many
31

2505600

1876?
25. If

ship sail

44

cents

ct.

did

When

contained

much

daily. If

oats

will it cost

to

4440

mi.

oats

cost

feed

farmer

third, 32 bu.

bought

OP

40

barrels
ct.

COMPOUND

numbers

to

called Addition

sold
bu. 3

25

him

of

flour

pound:

how

for

much

$156.80.

the

operationis

1. A

of

miles

many

$40.15.

ADDITION

the

peck

sold it for 5

pound, and
he gain?

75.

hour, how

an

days?

flour dealer

27. A

bushel, how

of 365

year

is fed

horse

miles

2 da. 3 hr. ?

will it sail in 3 wk.


26. A

sec.

pk.:

three

NUMBERS.

be added

the

how

much

compound,

of Compound Numbers.

lots of

pk. ;

are

second, 14
did

the

wheat:

he

bu. 2

sell?

first lot

pk.;

the

ADDITION

Solution.
the

Place

"

column

same

adding,the

and

COMPOUND

OP

units of the

(Art. 17).

by dividingby 4, the

number

Beginning

with

is reduced

to

of

pecks in

being 2 pecks left,write the 2 under


to the
of pecks, and
carry the 1 bushel
bushels;adding this to the bushels,the sum

Rule.

the

of

2.
and

1. Write

the

the

lowest

divide

stim

by

their

3. Write
the

the

the

under
last,

Proof.

Rem.

"

The

the

which

same

column

column

of

is 72, which

72

column.

same

the number

unit

of

numbers,

higher.

the

column

added, and

column.

manner

with

write its entire

as

the

ination
of units of this denom-

the next

under

the next
same

added, placingunits

denomination,add

remainder

quotientto

3. Proceed, in

the

be

to

in the

Begin with

ivhich make

carry

numbers

denomination

same

pecks,
bushels

of bushels.

the column

"

in

bushel,and

there

write under

103

denomination

same

is 6, which

sum

NUMBERS.

all the

colurnns

to

sum.

in Addition

of

Simple Numbers.

nominatio
writing compound numbers, if any intermediate deis wanting,supply its placewith a cipher.
Rem.
for every ten,
2. In adding simple numbers
we
carr}^ one
of the next
because
order always make
ten units of a lower
one
higher; but, in compound numbers, the scale varies, and we carry
1.

"

In

"

one

for the

higher.

number

of the lower

order, which

makes

one

of the next

AKITHMETIC.

PKACTICAL

NEW

RAY'S

104

LIQUID

AVOIRDUPOIS

MEASURE.

WEIGHT.

(8)
T.

cwt.

(9)
lb.

oz.

cwt.

lb.

oz.

45

53

10

16

85

14

14

14

75

15

15

90

13

19

17

18

13

18

74

12

ADDITION

COMPOUND

OF

NUMBERS.

MEASURE.

SQUARE

CUBIC

MEASURE.

(15)

(14)
C.

ft.

cu.

in.

cu.

yd.

cu.

390

50

18

900

15

90

874

45

17

828

20

67

983

46

20

990

MEASURE.

sec.

wk.

mo.

16

18

28

47

13

15

49

59

12

19

16

53

42

18. Five
bu. 1

loads

pk. ;

how

pk.;

measured

pk. ;

18

da. lir. min.

bu.

23

51

40

19

30

37

13

27

18

thus
1

lb. ; 19

13

lb. 7

pk.;

21

bu. 3

22

bu. 1

in all?
raised

corn,

of
400

sold

lb. 11

bu.

bu.

200

oats

oz.

; and

17

bu.

pk.; barley,
pk. ; wheat, 255 bu. 1
bu.

1000

oz.;

pk.;
pk.:

100

hogsheads of
36 lb. ; the second, 4
weighed
third,5 cwt. 19 lb.; the fourth,7 cwt.
7 cwt. 84 lb. : what
did all weigh?
grocer
8 cwt.

21. Add

sec.

in all?

much

20. A

bu. 2

23

farmer

bu.

of wheat

bushels

many

19.

pk.:

(17)

min.
Ill*,

da.

143

in.

cu.

28

(16)

how

ft.

cu.

13

TIME

14

105

17

lb. 13

lb. 9

oz.

oz.

sugar

; 14

the

64

cwt.

75

first

lb. ; the

lb. ; the
33

cwt.

lb. 14
99

fifth,
78

oz.

lb. 6

lb.
; 16
oz.

RAY'8

106

22. Two
mi.

104

rd.

50

23. A

PRACTICAL

has

farms:

sq. in. ; 23

yd.

has

the
the

pilesof

C. 24

second, 16
fourth,29

C. 10

sq.

134

in.;

18

sq.

yd.

ft.: how

141

yd. 5 sq. ft. 116


yd. 8 sq. ft. 88 sq.

sq.

ft.;the

cu.

A.

186

are

88 sq.
the
: in

wood

cu.

first

mi.

17

sq. ft. 119

in.
25. A

rd. due

270

sq. rd. ; in the third,


in all?
586 A. 80 sq. rd.

A.

sq. ft.; 29

yd. 7

sq.

mi.

travels

one

200

in the

sq. rd. ; in the second, 286


much
113 A. 89 sq. rd. : how
17 sq.

place:

same

they apart?

are

man

the

east; the other, 95

due

far

24. Add

ARITHMETIC.

depart from

men

how

west:

NEW

7 C. 78
first,
third,35 C. 127

much

sq.
in.

cu.

ft.;

cu.

ft.;

in all?
88

C. Ill

cu.

ft.

26. I sold 4642

gal.3 qt. 1 pt. of wine to A ; 945 gal.


1707 gal.1 pt. to C; 10206
to B;
gal.1 qt. to D: how
hogsheadsof 63 gal.each did I sell?
many
277 hogsheads 50 gal.1 qt.

SUBTRACTION

When

76.

OF

COMPOUND

numbers

given

two

NUMBERS.

compound,, the

are

operationof findingtheir difference

is called

Subtraction

of Compound Nximhers.
1. I have

67

after

Solution.

bu.

selling34
"

Write

bu. 3

the

less

greater,placing units of the


the

column.

same

but

1 bu.

and

added

leaves 3

being

wheat:

pk. of

pk. can

how

much

will

main
re-

pk.?
number

same

under

the

denomination

in

be taken

from

pk.,
from
67 bu. reduced
to pk.,
pk.,gives6 pk. 3 pk. from 6 pk.
not

taken

to the 2

pk.; 34 bu.

from

66

bu. leaves

difference is,therefore,32 bu. 3

pk.

^_

32

bu.

The

"32

3~

Rem.
the

of

subtraction

NUMBERS.

34

bu.

12

Take

2^

units
2.

of

than

of

0_ 1^

lowest

number

from

But, if the

lower number

the upper,

increase

under

in the

carry

pk. qt.

the

pt.
0

placing
greater,
column.

same

denomination, and, if possible.


the

above

one

number

of

by

as

the next

the lower

to

one

it.
be

denomination

of any

the upper
make
one

as

before,and

greater
units

many

higher;

number

tract
sub-

of

the

higherdenomination.

4. Proceed

Proof.

Rem.

the

that denomination

next

denomination

Begin with

as

in

bu.

same

take the lower


3.

done

pt.

1. Write the less number


the

be

(3)

pk. qt.

From

"

will

simple numbers.

(2)

Rule.

107

by 1, the result
bu. by 1, as is

bu.

67

number

lower

the

increase

to

same

diminishing the

of

Instead

"

COMPOUND

OF

SUBTRACTION

"

"

The

the

in

The

same

same

compound numbers,

in Subtraction

as

resemblance

between

is the

with

manner

same

as

each

denomination.

of

bers.
Simj^leNum-

of

subtraction
in Addition

75,

simple, and
Rem.

2.

Examples,

liquid

gal. qt. pt.

measure.

gal. qt.

pt.

gi.

From

17

43

Take

Hi

0,

23

^^f

108

RAY'S

PRACTICAL

NEW

AVOIRDUPOIS

ARITHMETIC.

WEIGHT.

(6)
T.

cwt.

lb.

T. cwt.

lb.

oz.

From

14

12

50

16

18

14

Take

10

13

75

75

15

LONG

MEASURE.

(8)

(9)

mi.

rd.

From

18

198

Take

11

236

SQUARE

ft.

in.

10

11

yd.

MEASURE.

COMPOUND

OF

SUBTKACTION

NUMBEKS.

109

pk. 1 qt. be taken from a bag containing


hickory nuts, what quantity will remain ?
1 bu. 2 pk. 7 qt.

16. If 2 bu. 1
bushels

of

17. From

bu. take

100

24

bu. 1

pt.
75 bu. 3

18. I

bought
much

remained?

wagon

loaded

oz., how
19. A

the wagon
alone
of the hay?
20. It
has

lb. 4

46

after

with

hay weighs

cwt.

32

67 lb. : what

is the

miles
mi.

41

round

the

rd. what

earth

distance

I gave

22. From

8 C. 50

how

23. A

containing146

86 A.

son

my

taken:

farm

much

cask

sq. rd.

94

ft. of

cu.

A.

how

77.

In
30

1. A

much

12

find the

In

"

was

time

writing

mainder

is 1

vr.

1875
9

mo.

rd.

146

C. 75

cu.

sq. rd.
ft. are

C. 103

ft.

cu.

15

lost ?

gal.2 qt. 1 pt. 2 gi.


sec.

take

da. 4 hr.

12

between

the

yr. 4 mo.
28 da.

46

sec.

between

two
sider
dates, conany
months
1 year.

April 14, 1875, was

February is the 2d month


April the 4th; then, from

da. subtract

mi. 279

3 da. 6 hr. 13 min.

findingthe time
days 1 month, and

Solution.

and

min.

sec.

note, dated

12, 1877

that

5 da. 10 hr. 27

29

remain?

59 A.

11

13 min.

man

containing63 gal.le^J^ed;only 51

gal.1 qt. 2 gi. remained


24. From

is left?

of wine

80 sq. rd. of land.


much
left?
was

how

wood,

lb.

99

cwt.

will

24798
21. I had

weight

after

oz.

lb. ;

66

cwt.

23

100

lb. 8

selling19

26 lb. 12

weighs 8

is 24899

traveled

of rice

oz.

pk. 7 qt. 1 pt.

these

dates, observe
of

the

1877 yr. 2
14 da.
The

year
mo.
re-

paid February
dates.

operation.

da.

yr.
187 7

mon.

12

18 7 5

14

19

2 8

PKACTICAL

NEW

KAY'S

110

ARITHMETIC.

Independenceof the United States was declared


July 4, 1776 : what length of time had elapsedon the
100 yr. 1 mo.
Ist of September,1876?
27 da.
first crusade
ended
3. The
July 15, 1099; the third
tween
crusade, July 12, 1191 : find the difference of time be2. The

these

dates.

91 yr.

Magna Charta
Queen of Scots,was

was

between

of time

these

battle of

William,

what

1688:

6. The

1805;

78.

battle

the

difference

Tor

at

of

Austerlitz

of

fought

was

Waterloo, June

18,

between

number

days

the

yr.

1815:

of

from

10

2,

mo.

the

16 da.

days.

to

Oct. 21.

10
May, there remains 31
days; there are 30 days in June, 31 in July,
in August, 30 in September, and 21 in October;
of days from
then the number
May 10 to

Solution.

October

"

Of

May

da.

21

find

dates in

two

the

December

9 yr. 6

fiijdthe time

23 da.

mo.

between

time

of time.

To

1. Find

of

find the

14, 1066;
Bay Nov. 5,

622

battle

difference

31

Hastings was
Orange, landed
the

w^as

yr. 7
fought Oct.

events?

two

21

of

Prince

Mary,

dates.
371

5. The

27 da.

mo.

signed June 15, 1215


beheaded
February 8, 1587 :

4.

difference

11

21, is 21 -f 30 +

==

"

31

31

30

+ 21

==

164.

2. Find
12.

the number

of

days from

March

17 to

ber
Septem179.

3. A

dated

note

COMPOUND

OF

MULTIPLICATION

NUMBEKS.

April 18, 1877, is

days does it run?


many
dated
4. A
note
Sept. 5,

June

due

HI

20, 1877:

how
how

1878.

6. Find

the

March

of

the

due

Dec.

7,

1877

93.
Oct.

from

days

12, 1877,

to

225.
number

of

from

days

Aug. 20, 1875, to

8, 1876.

201.

MULTIPLICATION

79.

1877, is

it run?

number

the

25,

May

does

days

many
5. Find

63.

When

OF

COMPOUND

NUMBERS.

the

multiplicandis a compound number,


of Compound Numbers.
operationis called Multiplication

1. A

takes

farmer
bu.

pk. :

to

mill

how

much

bags

of
he

had

wheat,

taining
con-

in all ?

for
Begin at the lowest denomination
convenience.
Multiply the 3 pk. by 5, making 15 pk.,
which, reduced, gives 3 bu. and 3 pk.; write the 3 pk.
under
the pecks,and carry the 3 bu.
Then, multiply
the 2 bu. by 5, add to the product the 3 bu., and write
Solution.

each

"

the 13 bu. under

bu.

pk.

3
5

13

the bushels.

1. Write

under the lowest denomination


multiplier
of the multiplicand.
2. Multiplythe loivest denomination
first,and divide the
ivhich
productby the number of units of this denomination
make
unit of the next higher,
write the remainder
under
a
the denomination
to the
and
multiplied,
carry the quotient
product of the next higherdenomination.
Rule.

"

3. Proceed

writingthe
Proof.

"

in

like

entire

The

the

manner

product

same

as

at

in

with

all the

denominations,

the last.

Simple Multiplication.

llA^'S

112

Rem.
and

There

"

of

are

numbers

two

ARITHMETIC.

differences

numbers:

between

of simple
multiplication

it is more
venient
consimple numbers
one
figureof the multiplierat a time; in compound
the eritire tnultiplier
each time.
better to use
2. In
the scale is miiforvi;in compound numbers
it varies

compound
to

PRACTICAL

NEW

use

it is

simple numbers

1. In

the table.

with

Examples.
2.

Multiply2

bii. 1

pk.

qt. 1 pt. by
13

3.

Multiply2

4. If 4

iiour,how

bu.

much

5. Find

the

weighing 8
6. How

cwt.

much

bu.

pk.

pk.

qt. 1

will make

weight
62

of

qt. by 9.
pt. of wheat

12
9

bl.?

hogsheads

pk.

bu. 1

in 7

of

qt.

T.

each

sugar,

loads,each weighing
3

pk. 2 qt.

3 T. 17 cwt.

lb.?

89

bu. 3

23 bu. 2 qt.
make
1 bl. of

58

lb.

hay

6.

16 cwt.

58
10

lb.

cwt.

23

lb.

ship sail 208 mi. 176 rd. a day, how far will
it sail in 15 days?
3128 mi. 80 rd.
8. Multiply23 cu. yd. 9 cu. ft. 228 cu. in. by 12.
280 cu. yd. 1 cu. ft. 1008 cu. in.
99 T. 12 cwt. 6 lb.
9. Multiply16 cwt. 74 lb. by 119.
10. Multiply47 gal.3 qt. 1 pt. by 59.
2824 gal.2 qt. 1 pt.
travels 27 mi. 155 rd. in 1 day: how
11. A
far will
month
of 31 days?
he travel in one
852 mi. 5 rd.
12. In 17 pilesof wood, each pile containing7 C. 98
is the quantity of wood?
ft.: what
182 C. 2 cu. ft.
cu.
39
13. Multiply 2 wk. 4 da. 13 hr. 48 min.
sec.
by
7. If

49

75.

mo.

3 wk.

hr. 48

min.

45

sec.

plantersold 75 hogsheads of sugar, each weighing


6
for
10 cwt.
84 lb.,to a refiner,
ct. a pound.
The
refiner sold the sugar for 8 ct. a pound : how
much
did
he gain?
$1626.
14.

COMPOUND

OF

DIVISION

sold

cotton-factor

15. A

weighing

for the

$24735

cotton

how

bales

425

lb.,for

85

cwt.

NUMBEES.

13

ct.

much

113

of

cotton, each

pound.
he gain?

did

He

paid

$2061.25.
DIVISION

80.

When

The

called

divisor

First.

To

"

is done

denomination

Second.
number

duced
to it.

to

Simple or

two

often

the

Compound

cases:

Number

Compound

one

is

Number.

Compound

by reducing both divisor and dividend to the


before dividing(Examples 6 and 8, Art. 74).

.^ame

into a given
Compound Number
vision.
equal parts. This is properly Compound Di-

of

divide

14 bu. 2

1. Divide

first,so

how

find

To

"

Solution.

compound number,
of Compound Numbers.

either

gives rise

in another

contained
This

be

NUMBERS.

is

Division

mvij

This

Number.

dividend

the

is
operation

COMPOUND

OF

Divide

"

the

that,if there be
to the next

pk.

left;write the

qt. by

3.

highest denomination

operation.

bu.

remainder, it may be relower denomination, and added


a

is contained

3 in 14

under

the

times, and

bushels,and

bu.

pk. qt.

8)14

are

reduce

remaining 2 bu. to pk.,to which add the 2 pk.,making 10 pk.


This, divided by 3, gives a quotientof 3 pk.,with 1 pk. remaining;
which, reduced to qt.,and 1 qt.added, gives9 qt. This, divided by
3, givesa quotient3, wdiich is written under the quarts.

the

(2)
bu.

7)33
4
PPAC. 8.

(3)

pk. qt.

da.

hr.

min.

sec.

5)17

12

56

15

12

11

15

Rule.

its

of

ARITHMETIC.

riiACTICAL

NEW

KAY'S

114

quantityto be divided in the order


place the
denominations,beginningwith the highest;

divisor

the

1. Write

"

the

on

left,
each

Begin ivith the highestdenomination,divide


beneath.
and write the quotient
separately,

2.
ber

If

3.

reduce
afterany division,
add
denomination,and, beforedividing,

remainder

lower

the next

the number
Proof.

The

Each

Rem."

that part of the dividend

67

bu.

from

is of

which

pk.

the

what

casks

is the

bu. 2

weigh 35
weight of each?

hour

traveled

39 mi. 288

rd. in 7 hr.

Divide

8.

490

bu.

9.

265

lb. 10

10.

45

11.

114

T.

18

64

pk.

sq. rd.

by

qt.-^

100.

hr. 45

10

13.

309

bu. 2

14.

127

gal. 3 qt. 1

15.

788

mi.

the

other,87

and

divides

what

is the

A.

bu.

lb. 13

27

pk.

rd.

169
has

min.

oz.

18

sec.

oz.

rate

per

54
3

44

3 bu. 3
63.

-^319."
farms, one

of each

He

of 104

reserves

40

equallyamong
son?

rd.

sq. rd.
5

qt.
oz.

cwt.

-1-54.

-f-23.

"

two

224

pk.

d"a. 3 hr. 5 min.

qt.-^78.
pt. 3 gi.

A. 78 sq. rd.
the remainder
share

lb. 4

2 T. 14

cwt.-^17.

12.

farmer

16.

oz.

5 lb. 5

at what

-^50.

oz.

da. 22

cwt.

22

cwt.

5 mi.

69 A.
2

lb. 12

44

I travel?

did

7.

16

pk. 2 qt. 1 pt.

ewt.

3
t). I

as

qt. 1 pt. by 5.

of sugar

average

denomination

same

it is derived.

13

5. Eleven

to it

Simple Division.

in

as

same

partial quotient

4. Divide

it to

occurs

its denomination.

of

"

num-

17

lb. 11

sec.
oz.

pk. 7 qt.
gal.1 gi.

mi.

151

rd.

A.

117

sq.

rd.;

A. 40 sq. rd.,
his 3 sons
:

50 A,

105

sq. rd.

AND

LONGITUDE

17. A

farmer's

consisted

crop

TIME.

115

bu. 3

of 5000

pk. of
following.He

year, and 7245 bu. 2 pk. the year


bu. 1 pk. and
B022
placed the remainder

one

crib

each

equal amount

containingan

in

how

sold

cribs,

bushels

many

crib?

in each

corn

bu.

528

speculator
bought 6 adjoiningpieces of land,
containing4 A. 80 sq. rd. He divided the whole

18. A
each
into
did

lots,and

54

he

get

19. Add

the

take

sum

divide

8;

for each

the

$5

sq. rd.

how

much

S400.

75

oz..

lb. 14

186

at

lot?

lb. 9

35

them

sold

lb. 14

oz.

productby

lb. 15

oz., 85

multiplythe

from

remainder

1 lb. 5
AND

LONGITUDE

Difference of

81.

by

is the result?

what

64:

oz.;

oz.

TIME.

longitudeand

between

time

different

1^laces.
The

circumference

divided
The

into
sun

360", once

360

the

is
earth,like other circles,
equal parts, called degreesof longitude.
of

the earth,
to pass
entirelyround
appears
in 24 hours,one
day; and in 1 hour it passes

15".

(360" -f- 24
15").
As
15" equal 900',and 1 hour equals 60 minutes
of
of time passes over
time, therefore,the sun in 1 minute
l^' of Si degree, (900'~60=r=:
15').
As
15' equal 900", and
1 minute
of time equals 60
seconds
of time,therefore,
in 1 second
of time the sun
15" of a degree. (900"^ 60
15").
passes over

over

.:=

Table

15"

of

15' of
15"

Longitttde

Comparing

for

of

and

-=.-

1 hour

of time.

^^^

1 min.

of time.

of time.

longitude
longitude
longitude

sec.

Time.

1. How
18"

hr. min.

many

25' :W

Analysis.

PRACTICAL

NEW

KAY'S

116

and

of time

sec.

of
By inspection

correspondto

1 hr. 13

longitude?

of

"

ARITHMETIC.

the

min.

42

sec.

table,it is evident that,

divided by 15, give hours


Degrees ( ) of longitude,
divided by 15, give minutes
Minutes
(^ ) of longitude,
divided by 15, give seconds
of longitude,
Seconds (^^)
"

of time.
of time.
of time.

Hence, if 18" 25^ 30^^ of Ion. be divided


in hr. min.

be the time

To

find

the

and

time

by 15, the quotientwill


correspondingto that longitude.

sec.

correspondingto

difference of

any

:
longitude

Rule.

the

longitude
by 15, accordingto the rule
the quotient
and mark
of Compound Numbers

Divide

"

for Division
hr. min.

sec, instead

Conversely: To
any

of
find

'

"

".

the

longitudecorrespondingto

of time.

difference

Multiplythe time by 15, accordingto the


for Multiplication
of Compound Numbers^ and mark
instead of hr. rain. sec.
product
Rule.

'

"

2. The

what

8. The
4': what
4. The

Cincinnati

between
longitude

difference

difference of

difference of
is 10"

two

longitudebetw^een
of time?

4 hr. 44

longitudebetween

35': what

is the

difPerence of time
min.

20

sec.

between
:

what

hr.

placesis 71"

two

min.

New

16

York

sec.

and

difference of time?
42

is 37

placesis 30*^ :

of time?

is the difference

longitude?

the

"

difference of

is their

5. The

rule

"

min. 20

Cincinnati
is the

and

sec.

adelphia
Phil-

difference of
9"

20'.

AlNl)

LONGITUDE

6. The

of

difference

is 1 hr. 4 min.

Louis

between

time
56

TIME.

sec.

!N^ew York

what

117

is the

and

difterence
16"

longitude?
7. The

of

difference

time

is 5 hr. 8 min.

between

London

what

is the

sec.

is

sun

is

difference

the

As

sun

west, when

meridian

of that

appears
it is

travel

at

any

noon

the time

if EAST
if WEST,

8. When

it is

9. When
time

at

in

time

time

at

the

east

icest of that

place.
tirne than
another,
Therefore,

earlier
of it.

37 min.

12

it is 11

(noon)

o'clock

of the

Columbia

it is 1 o'clock
mouth

of

Ya.,
P. M.

Columbia

at

M.

A.

time

past

sec.

York,

York?

what
1

min.

at New

40

min.

is

P. M.
is

A. M.

sec.

York,

at

noon.

what
Philadelphia,

9 hr. 55

is

20

is the

New

11 hr. 22

St. Louis?

at

at

of New

east

Cincinnati?

at

M.

o'clock A.

o'clock

Wheeling, W.

mouth

when

after noon

Cincinnati,what

at

longitude30"

11. When

the

is

the

toward

east

one

noon

it is 11

10. When

12.

the

from

Philadelphia?

the

the

place is given,the time at another,


their difference
of this,is found by adding
of time;
their difference
by SUBTRACTING
of time.

When

the

i)l^cewhen

any

place,it

place,and beforenoon
Hence, a place has later or
according as it is east or west

I'o

place.

to

of that

the

of

TiME.

IN

(12 o'clock),at

noon

the

on

14^

77"

DlB^FERENCE
It

of

ington
Wash-

and

longitude?

82.

St.

what

sec.

A.

is

M.

in

longitude80" 42' west:


in longitude 124" west:
river,
is the time
at Wheeling, what
river?
10 hr. 6 min.

48

sec.

A. M.

DEFINITIONS.

83.

of

1. Factors

product of
(Art.28, 2).
the

Thus, 2 and

1.

One

2.

"

which

and

X 3 X ^

two

more

bers,
num-

equals the given

number

are

factors of 6, because

are

factors of 30, because


Kem.

number

2X3

or

6; 2, 3, and

are

30-

the number

itself are

not

considered

factors of

number.
Kem.
factors.

2. A

number

"

number

may

be the

Thus, 2 X 6== 12, 3 X 4


of

multiple
is

productof

more

12, and 2X2X^

number

is

than

set of

12.

productof

one

which

the

factor.

Thus, 6 is a multipleof 3; 30 is a multipleof 5.


3. INTumbers

divided

are

into

classes,
prime

two

and

composiie.
4. A

prime

Thus, 5, 11, 17
5. A

prime

are

composite

Thus, 6, 12, 30
6.

number

has

3 is

factor

prime
(118)

factors.

two

or

more

factors.

compositenumbers.
is

umber.
Thus,

no

numbers.

number

are

prime

has

factor of 12.

factor

which

is

prime

FACTORING.

7. A
it is

factor is

Thus,
Rem.
the

3 is

8. Two

or

they have

10

each

are

greatest
numbers

more

Thus,

6 is the

11.

Thus,

multipleof

6 is the least

into

84.

All

the

to each

be

18.

w^hen

other
^

other.

of

two

or

more

divisor of 12 and

(G.

divisor

bers
num-

divisor of 12 and

(C. M.)

C.

18.

D.)

of two

factor.

greatest common

of tw^o

18.

or

more

bers
num-

all of them.

multiple
least

process

of

3.

(L. C. M.)

multiple of

multipleof

is the

2 and

multiplesof

common

common

their

To

prime

common

common

is the

13. Factoring

numbers

multiple

numbers

more

may

factor of 6, 12, and

are

greatestcommon

least

numbers

more

or

common

common

Thus, 6, 12, 18, etc.,are


12. The

two

(C. D.)

is the

common

is any

15.

factor.

common

Thus, 2, 3, and

to each

divisor

is any

or

prime

are

common

when

factor.

common

no

10. The

6 is

of

numbers

more

9 and

smallest

Thus,

factor.

common

9. A

the

Sometimes

"

Thus,

factor of 12 and

common

numbers

more

or

of them.

of each

factor

two

to

common

119

of two

all of them.

2 and

3.

resolving composite

factors.

Find

prime

the

Prime

numbers

or

Jfiniihers.

except

are

odd

bers.
num-

RAYS

120

Rule.

1. Write

"

ARITHMETIC.

PKACTICAL

NEW

odd

the

tuanUrs

in

series 1, ;5,5,

7,

1),etc.
bth

number

eixtse

every

after7

erase

llth number

every

the

and

every
erase

etc.

3. Then

number; after 5 erase


every 7th number; after 11

'M

After 3

2.

numbers

that

remain

the

are

prime

numbers.

KxERCiSE.

85.

Find

"

The

the

prime

numbers

from

to

operationsof Factoring depend

100.

the

upon

following
PRINTIPI.ES.

1. A

factor of

Tlius,5
2. A

Thus,
3. A
that

is

factor

number

30 is

30 and

of

multipleof

is contained

number

multipleof

factor of

exactlydivides

in it G times.

it.

exactlycontains

5 and

number

it.

contains

is

it G times.

factor of

any

midtipleof

number.

Thus,

4. A

being

factor of 6 is a factor of 12, 18, 24, etc.

compositenumber

is

equal to

the

product of

all its

prime factors.
Thus, the prime factors of
In

86.
will

be

30

are

2, 3, and 5; 2 X

resolvingnumbers
found

convenient

facts in reference

to

the

into
to

prime

their

remember
numbers

prime
the

30.

factors it

following

2, 3, and

5.

FACTORING.

1. Two

is

Thus, 2 is
2.

Three

times

of

the

of

sum

its

3.

2457; for

-[-4 + 5 +

18, which

is 6

3.

3. Five
0

factor

when

number

of
multiple

some

or

Thus, 3 is

factor of

number.

even

every

of 4, 6, 8, 10, etc.

factor

is

digitsis

factorof

121

is

factor of

ichose unit

number

every

figureis

5.

or

Thus, 5 is
Rem.

the

Resolve

numbers

30

into

7, 11, 13, etc.,are

into
its

Rule.

2 is

prime

factors.

factor of 30

1. Divide

"

of

Factors.

its Prime

viding
(Art. 86, 1). Di3 being a factor of
30 by 2, the quotientis 15.
15 (Art.86, 2) is also a factor of 30 (Art.85, Prin.
3). Dividing 15 by 3 the quotientis 5, a prime number. Then, 2, 3 and 5 are the prime factors of 30.

Solution."

factors

trial.

by

J^umher

1. Resolve

87.

prime

is best ascertained

not

or

To

factor of 10, 15, 20, 25, etc.

Whether

"

number

the

given number

exactlydivide it.
2. Divide
the quotient
in the same
to divide, until a
quotientis
prime number.

by

any

operation.

2)30
3)15
5

prime number

that unll

3. The

several divisors and

prime factorsof
Rem.
smallest

"

It

will

prime

the

be

obtained

last

and

which

quotientwill

so

tinue
con-

is

be the

given number.

most

number.

the

manner

convenient

to

divide

each

time

by

the

the

Resolve

88.
more

NEW

RAY'S

122

To

followinginto

find

ARITHMETIC.

PRACTICAL

the

prime

their

prime

factors

factors:

two

to

common

or

numbers.
1. What

Solution.

prime
"

Write

factors
the

are

numbers

common

in

line.

to

2 is

30

and

42?

prime factor of both 30 and 42 (Art.86, 1). Dividing by 2, the quotientsare 15 and 21. 3 is a
prime factor of both 15 and 21 (Art.86, 2); and
consequentlyof both 30 and 42 (Art.86, Prin. 3).
Dividing by 3, the quotients5 and 7 are prime to
the common
each other (Art.83, 8). Then
2 and 3 are

operation.

4 2

2)30

Tl

3)15
5

factors.

FACTORING.

123

1. Wiite the

givenmimbers in a line.
2. Divide by any prime number
that will exactlydivide all
manner
oj them; divide the quotientsin the same
; and so
continue to divide until two
or
more
are
of the quotients
prime to each other.
Rule."

3.

Then

What

the several divisors will be the

prime

factors

arc

factors.

common

to

common

2, 3,

5.

"

9
-J,

-J.

2, 2, 3.
2, 3, 3.

3, 3, 3.
2, 2, 5,
2, 3, 7.
2, 2.
2, 3.
3, 3.
2, 5.
3, 5.
5, 5.
2, 7.
11.
13.
17.
19.
23.

89.

Finding the G. C. D.
depends upon the following
Principle.

"

all the
other

factor.

The

G.

C. D.

of

prime factors common

of

two

two

or

or

more

to the

more

numbers

numbers

numbers, and

tains
conno

RAY'S

124

NEW

PRACTICAL

Thus, the G. C. D. of 12 and


2 and

3; it must
greatestC. D.; it can
both

12 and

ARITHMETIC.

contain

both

contain

no

the

6; it contains

18 is

tors
fac-

common

of them, else it would

be

not

other factor,else it would

not

the

divide

18.

1. Find

the G.

C. D.

of 80

First

and

42.

Method.
OPERATION.

Solution.
42

2 and

are

the G. C. D. of

Rule.

1.

"

to 30 and
prime factors common
(Art.88); their product is 6; then
30 and 42 is 6 (Prin.).

The

"

F'uid

the

2)30

42

3)15

21
7

to the

prime factorscommon

given

numbers.

Multiplythem together.
The product will be the greatestcommon

2.

3.

Second
Solution.

Dividing 42 by 30, the


12; dividing 30 by 12, the
6; dividing 12 by 6, the

is

mainder

is

mainder

is 0.

30

and

42.

Then

other, 6

factors

For, 30

7; then, because

each

Method.
re-^

"

mainder

must
to

common

r=

is the

5 and

are

contain
30

and

all

42

operation.

re-

30)42(1

re-

G. C. D.

X 5 a"d

divisor.

3 0

of

T^)

3 0

(2

2 4

prime to
the prime

0)12(2

42; it-is,there-

12

fore,their G. C. D. (Prin.).

by the less,the
greater number
divisor by the remainder,and
so
on, always dividingthe
last divisor by the last remainder,until nothingremains.
divisor.
2. The last divisor nill be the greatestcommon
Rule.

Rem.

1.

"

"

To

Divide

find the

the G. C. D. of two
of the
common

the

G. C. D. of
of them, then

remaining numbers,

and

than

more

so

of that
on

two
common

for all the

numbers, first find


divisor and

numbers;

divisor will be the G. C. D. of all the numbers.

one

the last

FACTORING.

Find

the

numbers

greatest

125
divisor

common

of

the

following

12.
18.
20.
27.
30.
16.
24.
36.
31.
26.
23.
19.
17.
39.

.227.
12.
5.
8.

Finding the L. C. M.
depends upon the following

of

90.

Principle."
tains

all the

TAe

L.

C. M.

two

or

innnbers

more

of

prime factor'sof

two

each

or

more

number

numbers
and

con-

other

no

factor.
Thus, the L. C. M. of 12 and 18 is J^6;its primefactors
and

both

8; it
the

must

contain

numbers;

he the least CM.

all these factors, else it would

it must

contain

no

arc

not

% % 3,
contain

other factor,else it would

not

Solution.

"

The

of 12, 2, 2, and

3; and

prime

factors of the
other

o,

and

36

is the L. C, M.

no

are

of 4, 6, 9 and

factors of 4

prime

2; those of 6

2 and
and

L. C. M.

the

1. Find

ARITHMETIC.

PRACTICAL

NEW

EAY'S

126

12.

OPERATION.

are

and

3; of 9, 3

2X2

3.

Then, the

2X3

12

L. C. M.

are

2, 2, 3,

( Prin.). Hence,

factor

3X3
2X2X3

2X2X3X3

36

OPERATION.

process of

The
the

prime

factors for the

hy
simplified

much
form

and
factoring

of Short

Rule.

the

selecting

L. C. M. is very
operationin the

2 12

3)3

12

Division, as shown.
2 X

2 X

line,

1. Wii'tethe

"

2)4

givennumbers m
prime munher that

2. Divide

by any
two or more
of them.
and
3. Write the quotients

will

undivided

X 3

36.

exactlydivide

numbers

in

line

beneath,
4. Divide

these numbers

in the

same

manner^

and

so

operationuntil a line is reached in which


numbers
all prime to each other.
are
5. Then
the product of the divisors and the numbers
the last line will be the least common
multiple.
the

tinue
con-

the

in

Rkm.

Find

"

"When

the

the least

quotientis 1
common

it need not

be written.

multipleof
24.
36.
40.
30.
72.

CANCELLATION.

7.
8.
9.
10.
11.

12.
13.

14.
15.
16.
17.
18.
19.
20.
21.
22.
23.

10, 12, 15 and 20.


9, 15, 18 and 30.
12, 18, 27 and 36.
15, 25, 30 and 50.
14, 21, 30 and 35.
15, 20, 21 and 28.
20, 24, 28 and 30.
45, 30, 35 and 42.
36, 40, 45 and 50.
42, 56 and 63.
78, 104 and 117.
125, 150 and 200.
10, 24, 25, 32 and 45.
2, 3, 4, 5, 6, 7, 8 and
16, 27, 42, and 108.
13, 29, 52, and 87.
120, 360, 144, 720, and

127

60.
90.
108.
150.
210.
420.
840.
630.

1800.
504.
936.
3000.
7200.
2520.

9.

3024.
4524.

720.

72.

CANCELLATION.

91.

1. I

for them
did

bought

with

pears

oranges
3

at

at

each

cents

each, and paid

cents

how

many

pears

it take?
OPERATION,

Solution

I." 5

cents, the

priceof

is 5, the number

cents

multipliedby

the oranges.

15

divided

are

15

by

of pears.

3)15
5

From
have

we

consideration

the

Principle."
and

then

of this

example

and

its solution

following
A

it
changedby multiplying
productby the mtdtipUer.

mimber

the
dividing

is not

RAY'S

128

the

For

and

solution

"

the

Indicate

division; then,

multiplier3 and
drawing a line across
the result,equal to 5.
The

"

we

the

following

divisor

hy

write

forms

OPERATION.

cancel

or

them; and

product 5 X

offer

may

tion
multiplica-

erase

the

the

Rem.

ARITHMETIC.

PRACTICAL

example, then,
operation:

II.

Solution
and

NEW

dividend

of which

is the

divisor.

2. If 1
with

buy

10 pears

each

5 cents

at

oranges

each, and

at 3 cents
:

how

many

pay

for them

oranges

will it

take ?

Solution.
=

5 is

"

factor of 10, for 10

then, cancel

5X2;

also the

factor

the

divisor

OPERATION.

5 and

5 'in 10

by canceling 10
and
writing the remaining factor 2
above it. The product of the remaining

"==6

factors is 6.

3. Divide

Solution.

and

15 X

5 is a

"

it.

The

of 9 divided

common

factors

is 9, and

the divisor is 4; the

by

4 is 2\.

is

factors

7 is

21, writing

product of the

Cancellation

the

it.

OPERATION.
a

21; then, cancel

it,and

in the dividend

remainingin

10.

factor of 15

below

factor of 14 and

14, writing2 below

14 x

15, writing 3 above

10, writing 2

common

by

common

10; then, cancel

it, and

above

21

--f=-2i

maining
re-

of those

quotient

Therefore,

process
of the

of abbreviation
dividend

and

by omitting

divisor.

CANCELLATION.

Rule.
and

the

Cancel

"

129

to both the dividend

factorscommon

divisor.

dividend

product of
product of

the

2. Divide

the

by

the

the

factors remaining in

in the
factors reynainijig

the

divisor.
result will be the

The

3.

required.
quotient

barrels of molasses,at S13


many
for 13 barrels of flour,at $4 a barrel ?

4. How

pay
5.

Multiply17 by 18, and

divide

by

barrel,will
4.

6.

51.

6. In

15

times

8, how

many

times

4?

30.

7. In

24

times

4, how

many

times

8?

12.

8. In

37

times

times
15, how many
by 40, and divide

5?

111.

Multiply 36
multipliedby 8.
9.

11.

times

36

21 X

productby

the

times

man}^
18

15?

each

13. How

how

divide

15.

is

60.

paid in hogs,

was

did I receive ?

many

46.

yards of flannel,at 35 cents a yard,


6 yd.
yards of calico,at 14 cents?
the quotientof 21 X H X 6 X 26, divided

by 13X3X14X2?
^ 15. The factors
17 ; the

33.
of

dividend

divisors,20, 34,

22

are

21, 15, 33, 8, 14,

and

27

requiredthe

quotient.
V

49.

16. I

bought 21 kegs of nails of


cents a pound ; paid for them
with
35 yards each, at 9 cents
a
yard :
muslin did I give?
17. What
26 X

the

many
for 15

will pay
14. What

and

30

12.

and
together,

^ 12. I sold 23 sheep,at $10 each, and


at $5

product by

6.

5, how
Multiply42, 25, and

10. In

30 X

is the
42 ?

Prac.

9.

quotient of

35 X

pounds each, at 6
piecesof muslin of
how
piecesof
many

95

38.
39

-10 divided

by
If.

NEW

KAY'S

130

18.

the

is

What

quotient

4X9X25?
19.

by

of

26

^3

33^'.

What

the

is

14

What
X

by

divided

35

of

quotient

15

""

21

divided

S^^.

4X6X10X14?
20.

by

ARITHMETIC.

PKACTICAL

is

the

18X20X25?

quotient

of

21

24

28

35

divided

3||.

"33?^"Jt'/^]f^

FRACTIONS.
"%!'"*

'

92.

unit

1st. An

be

may

apple

be

may

divided

into

equal parts ; thus,

equally between

divided

two

boys, by

ting
cut-

three

boys, by

ting
cut-

it into two
2d.

An

equal parts.
apple may be divided

it into three
3d.
any

In

These

equal jpartfi.

like manner,

number

equally among

apple

an

be divided

may

of equal parts.
equal parts into which

unit

may

into/o?/r,^ve,six, or
divided

be

are

called

fractions.
DEFINITIONS.

93.

1.

is

fraction

equal parts

more

one

or

by

words

unit.

2. To

express

fractions

unit

is divided

When

Each

part

Both

parts

When

Each
Two

iifo

-halves,

written
"

i.

f.

equal parts.
written
^.
part is called one-third,
called two-thirds,
f.
parts are
the parts are
called three-thirds,
f.
unit is divided
into four equal parts.
\.
part is called one-fourth, written
unit

is divided

into

three

"

Each
Two

parts

Three
All

equal parts,

one-half,

called

are

two

figures.

"

All

When

is called

into

and

the

called

are

parts

are

parts

two-fourths,

"

"

three-fourths,
called four -fourths,

called

"

are

(131)

f.
f
|.
.

of

KAY'S

132

When

NEW

unit

AK1THMP:T1C.

PKACTICAL

is divided

into

five

equal parts,
Each
part is called one-fifth, written \,
Two
parts are called two-fifths,
J.
Three parts are called threefifths,
|.
Four parts are called /owr-//f
As,
^.
All the parts are
called five-fifths,
|.
unit is divided
into six, seven,
When
a
eight,etc.,
equal parts,each part is called one-sixth,
^, one-seventh,
\, etc.
|, one-eighth,
a

"

"

"

"

fraction is

1. A

94.

expressedin

words

by

two

bers;
num-

the parts,the second names


them;
is called the numerator,
the second is

the first numbers


the

first number

called the
2. A

denominator.

fraction

is

the

above

numerator

expressed in figures,
by writing the
denominator

with

line between

them.
3. The

of the

terms

4. The

is

unit

the

parts

are

When

of each

denominator

shows

divided, and

into

styled the

the

how

equal parts
many
of
numerator, how many

taken.

unit

is divided

part depends upon

the

into

equal parts, the

number

Thus, if apples of equal size be divided,one


another

are

fraction.

denominator

the

95.

and

numerator

size

of the

parts.

into two

equal parts,

equal parts,a third mio four equal parts, etc.,a


larger than a third, a third largerthan a fourth, etc.

into three

half will

be

Hence,
1st. The

less the

number

of

parts

into

which

divided,the greater the size of each part.


of parts into
2d. The
greater the number
unit is divided,the less the size of each part.

unit

is

which

FKACTIONS.

COMMON

96.
or

one

1. A

fraction

more

units.

also be

may

I33

regarded as

part of

Thus:

three boys.
applesmay be divided equallyamong
Each
boy will receive,either one-third of each of the two apples,
of the apples; therefore, ^ of 2 is ". Hence, "
two-thirds of one
1st. Two

or

be considered either as two-thirds or as one-third of two.


may
2d. Two
applesmay be divided equallybetween two boys.
Each

will receive, either

one-halfof each of the two apples,or


of the two
one
apples;therefore, ^ of 2 is |,or 1. Hence, | may be
considered either as two halves or as one-halfof two.
be divided equallybetween
3d. Three applesmay
two boys.
Each
boy will receive, either one-halfof each of the three apples,
one
or
apple and one-halfof another; therefore,J of 3 is |, or 1^.
boy

be considered

Hence, | may

either

three

as

halves

or

as

one-halfof

three.

2. A

fraction

is

3. The

numerator

4. The

denominator

part

of

one

expresses

the

expresses

units.

more

or

ntimber

the

of units.

part

of

each

be

to

taken.

97.

1. A

of
idend

and

fraction

may

division,in

the

also

which

denominator

the

be

the

regarded
numerator

as

an

is the

pression
ex-

dw-

divisor.

Thus:

f is 2 divided by 3; here, the division can only be indicated,


2d. I is 4 divided by 2; in this case, the division can
be performed
exactly,giving a quotient2.
3d. f is 5 divided by 2; in this case, the division can
be pernot
formed
exactly,the quotientbeing 2^.
1st.

2. A
is the

fraction

dividend

is
and

an

indicated
the

division.

denominator

The

is the

numerator

divisor.

KAY'S

134

number

whole

3. A

PRACTICAL

NEW

Thus,

|;

the

value

of

value

1. When
the

|; for
f, etc.

be written

The

98.

fraction

the

value

tli('form

the

for

of

umiierator

2;

may

is its relation

is less than
is less than

to

be written

unit.

the

denominatoi-,

the

denominator,

1.

is

numerator

of the

1 is

by

less tlian 1.

Thus, ^,J,", etc.,arc

the

fraction

numerator

of the

2. When

number

2 divided

be written

may

4 may

the

expressed in

denominator.

1 for the

and

be

may

fraction,by writing

ARITHMETIC.

fraction

equal to
equal to 1.

is

Thus, f,|,f, etc.,equal 1.


3. When
the

the

value

of the

Thus, |,|,f,etc.,are
4. A

proper

5. An
to

or

fraction

is

greater than

fraction

is

greater than

the

givater than

tor,
denomina1.

1.

whose

one

fraction

improper

is

value

is less th'an 1.

whose

one

value

is

equal

1.

6. A

mixed

number

99.

1. A

fraction

Thys, after

is greater than

numerator

apple

is

may

has

been

whole

be

number

divided

divided

and

into

fraction.

equal ])ai'ts.

into two

equal parts,each
half may
be divided into two equal parts; the whole apple will then
be divided into four equal parts; therefore,^ of ^ is \.
Such
expressionsas ^ of ^, ^ of ^, etc., are termed compoimd
an

fractions.
2. A

compound

fraction is

fraction

of

fraction.

COMMON

100.

Fractions

1.

numerator, the
Thus,

They

2. A

sometimes

denominator

^"

are

are

read

oT"

-r"

fractions.

FRACTIONS.

such

in

occur

botli

or

is

whicli

the

fractional.

are

expressions;they

3^ divided

fraction

simple

135

called

are

complex

hy 4, etc.
which

in

one

both

terms

are

entire.
3. A
the

fraction

complex

terms

in

one

whi^h

one

both

or

of

fractional.

are

The

101.

is

operationswith

fractions

depend

the

upon

following
Principles.

'

1. A

the
fraction is multiplied
by multiplying

Thus, if the
in

f;

times

^
as

the

parts

the
as

be

multipliedbj'3, the
size

same

in

as

|,but

divm^dhy dividingthe

fractionis

Thus, if the
third

the

of

are

result will be
there

are

three

many.

2. A

in

of

numerator

numerator.

of

numerator

parts

of

are

the

be divided
size

same

as

hy
in f

numerator.

3. the

but

result will be

there

are

^;
only one-

many.

3. A

fractionis

Thus, if the denominator


be

|; in I there are
only one-third

are

the
as

the
hy multiplying

divided
of

be

multipliedby 3, the
of parts as in |, but

number

same

denominator.
result will
the

parts

large.

4. A

fraction is multiplied
hy dividingthe denominator.

Thus,

if the denominator

|;

in

three

there

times

as

are

the

larsre.

same

of

number

be

divided
of

by 3, the

parts as

in

|, but

result will be
the

parts are

5.

Multiplying

does

twice

are

both

of

terms
as

ARITHMETIC.

of

terms

fraction by

the

same

ber
num-

its value.

change

if both

Thus,
there

not

PRACTICAL

NEW

RAY'S

136

be

multiplied by 2,

parts

many

in

as

J, but they

is

result

the

y"^;in ^

only one-half

are

as

large.

Dividing

G.
does

change

not

if both

Thus,

I there

both

are

of

terms

fraction by

the

number

same

its value.

of

terms

/^

only one-half as

be

divided

parts

many

the

by 2,
in

as

result

will

3^5,but they

are

be

|;

twice

in
as

large.
These

six

follows

be

j)rinciple8may

stated

more

as
briefly,

:
^

I.

fraction

A
1st.

1st.
2d.

III.

2d.

The

By dividingthe
fraction

numerator.

denominator.

is divided,

By dividing the numerator.


the denominator.
By 7nultiplying
The

1st.

multiplied,

By multiplying the

2(1.

II.

is

value

of

fraction

By multiplying both
By dividing both

operations

with

terms

terms

fractions

Subtraction, Multiplicationand

is not

by

by

changed,
the

the

are

same

same

number.
number.

Beduction, Addition,

Division.

COMMON

FRACTIONS.

REDUCTION

102.

Reduction

witiiout

of

OF

FRACTIONS.

is

Fractions

altering;their value.

To

having

reduce

an

changing their

There

CASE

103.

137

six

are

form

cases.

I.

integerto

an

improper fraction,

given denominator.

1. In

apples,how

halves?

many

OPERATION.

Solution.
3

In

"

apples there

apple there
3X2

are

are

halves

halves;then, in

2
=

|X

halves.

Multiplythe integer
by the given denominator
product write the denominator.

Rule."l.
under

the

2. In

3. In

4. In

5. In

6. In

7. In

8. Eeduce
9. Eeduce

apples,how
apples,how
apples,how
apples,how

many

halves?

many

thirds?

many

fourths?

many

fifths?

tenths
inches,how many
twelfths?
feet,how many
4

to

8 to

t^

19

to

thirteenths.

11.

Eeduce

25

to

twentieths.

12.

Eeduce

37

to

twenty-thirds.

fraction.

To

reduce

V'
W
^
W

ninths.

10. Eeduce

104r.

^-ffj
ff

sevenths.

CASE

mixed

II.

number

to

an

improper

RAY'S

138
1. In

31

ARITHMETIC.

PRACTICAL

NEW

apples,how

halves?

many

OPERATION.

Solution.
3

"

applesthere

and

1 half

Rule.

are

1.

"

fraction;to

In

apple there

are

^ halves

are
=

halves; then, in

6 halves.

|X3=|

6 halves

7 halves.

1+ 1

Multiplythe integerby the denominator of the


the numerator, and
under the
the product add

write the denominator.

sum

2. In

4^
2\
2|
51

3. In
4. In
5. In

6. Reduce
7. Reduce
8. Reduce
9. Reduce

It). Reduce
11. Reduce
12. Reduce
13. Reduce
14. Reduce
15. Reduce

apples,how man}^ halves?


thirds?
apples,how many
thirds?
apples,how many
fourths?
dollars,how many
8J to an improper fraction.
12f to an improper fraction.
15f to an improper fraction.
to an
improper fraction.
26^
to an
improper fraction.
3^
to an
improper fraction.
46|
21^i| to an improper fraction.
lyVA ^^ ^'^ improper fraction.
14^^ to an improper fraction.
lOy^ to an improper fraction.

CASE

105.
mixed

1. In

To

reduce

an

f
J
|
^^^-^
^
^^^~^^-p
^"Ml^
|^^|
"Ti^
tVt

ITT.

improper fraction

to

an

integeror

number.

of

apple, how

an

many

apples?
OPERATION.

Solution."
6

halves, there

There
are

are

6^2=3

2 halves

in 1

apples.

apple; then, in

2)6
~3

FRACTIONS.

COMMON

2. In

of

dollar,how

139

dollars?

many

OPERATION.

Solution.

"

There

in 9 fourths,there

Rule.

quotientwill

4. In

5. In
6. In

7. In

-=-

=;

the

1. Divide

"

3. In

are

be the

in 1

fourths

are

2^

^ of
-2^of
-5^of

9. In
10. In

11. Eeduce

12. Reduce
13. Reduce
14. Reduce
15. Reduce
16. Reduce
17. Reduce
18.

Reduce

19.

Reduce

20. Reduce
21. Reduce

ounce,

how

A
both

To

fraction
terms

its value

number.

many
many

dollars?

many

dollars?

many

bushels?
dollars?
ounces?

many

to

dollars?
many
number.
mixed

^\^ to
^^ to
^^^ to
^^U^ to
^j^- to
^^ to
^-^-^to
^-f^ to
^^^- to
-^^ to

mixed

number.

mixed

number.

dollar,how

^^

an
an

number.

mixed

number.

mixed

number.

mixed

number.

reduce

is reduced

8^

oz.

$131

18|.
15|.
25|^i.

199.

number.

I^tot*

IV.

fraction

to

$3|.
S3f.
2^ bu.
^^y^ij^.

l^y^T*
46-j^.
2im.
6^T%-

integer.
mixed

3.

31.

mixed

2.

40.

integer.
integer.

an

; the

apples?
apples?

many

many

CASE

106.

the denominator

the mixed

dollar,how

)9
2^

by

numerator

integeror

an

dollars.

f of an apple,how
-1^of an apple,how
^-^of a dollar,how
y^ of- a dollar,how
^ of a bushel,how

8. In

dollar;then,

to

higher

number.
by the same
(Art. 101, Prin. 5).

higher
terms

This

terms.

by multiplying
does not change

1. Eeduee

to

ARITHMETIC.

PRACTICAL

NEW

RAYS

140

thirtieths.
OPERATION.

Solution.

both terms

Kule.

"

of
2.
the

"

of

30

divided

| by 6, the

1. Divide
the

by

5 is 6.

result is

the

Multiplying

3 0-t-5=:

6X4

ff

denominator
required

by

24

the denominator

givenfraction.

Multiplyboth
result will be

the

the

fractionby
requiredfraction.

terms

of

the

quotient;

FRACTIONS.

COMMON

CASE

107.

To

1. A

fraction

both

reduce

is reduced

its lowest

to

lower

to

its value.

is in its lowest

fraction

and

denominator

does

each

to

by dividing
not change
the

when

terms

prime

are

terms.

terms

This

number.
by the same
(Art.101, Prin. 6).

terms

2. A

V.

fraction

141

ator
numer-

other.

(Art.

83, 8).
1. Eeduce

|^

to

its lowest

First
Solution.

"

is

terms.

Method.
factor

common

24

of

and

30

operation.

(86, 1 ). Dividing both terms of |^ by 2, tbe result


is -^f. 3 is a common
15 (86,
factor of 12 and
of ^| by 3, the result is
2). Dividing both terms
f 4 and 5 are prime to each other.

2 4

1 2

^~30~T5'
_

12

o\
~~

1 5

Rule.

1. Divide' both terms

"

the

givenfractionby any

factor.

common

2. Divide
3. So
terms

of

the

continue

are

prime

fractionin
resulting
to divide until
to each

the

same

fractionis

manner.

obtained

whose

other.

Second

Method.
OPERATION.

24)30(1
Solution.
24 and

30

"

The

is 6.

6, the result

divisor of

greatestcommon

Dividing both
is -|-.

terms

of

|J by

2 4

~6~)2 4(4
2 4

'30

RAY'S

142

Rule.

1. Divide

"

The

1. Two
when

reduce

or

they

2. A
a

of

the

givenfractionby their

two

be in its lowest terms.

fractions

more

or

to

their least

denominator.

common

is

both terms

fractionwill
resulting

To

108.

ARITHMETIC.

divisor.

greatestcommon
2.

PRACTICAL

NEW

common

commx)n

more

have

fractions
the

same

denominator

multiple of

have

denominator

common

denominator.
of two
their

or

more

denominators

fractions

(83, 11).

COMMON

3. The

least

fractions

is the

FRACTIONS.

143
of

denominator

common

least

two

multipleof

common

or

their

more

inators
denom-

(83, 12).
I, and

1. Eeduce

to

their least

de-

common

nominator.
,

OPERATION.

Solution.

multipleof
6, 9, and

The

"

12

is 36

5
"

3 0

3^'

"

Rule.

nnc\
^^^^

3 2

^S'

"

fractionsfor their
2. Reduce

Kem.

by

Art.

Rem.

1.

"

each

2.
"

4,

U*

3T-

"

least

C. M.

the

denominators

of

the

denominator.

common

fraction to

be

of

another

reduced

having this

to the

common

mixed
commencing the operation,
improper fractions (104).

Before
to

3.

Rem.

4." Two

"

denominator

Eeduce

4.

3 3

Integersmust

Rem.

3.

ator.
denomin-

denominator

103,Rule.

be reduced

2.

3)3

the L.

1. Find

"

T2

2)2

12

reduced

be

thirty-sixths
(106).

to

Each

(90).

must

common

the denominators

fraction,then,

-6

least

2)4

Each

fraction must
or

in the

to

more
same

be in its lowest

fractions

may

be

terms

reduced

way.

their least

common

denominator:

numbers

must

(107 ).
to any

common

RAY'S

144

17.
18-

PKACTICAL

NEW

AKITHMETIC.

2i, 3i, 4i, 5.


A, ii, ii, if. If
TTTfJ

!""

2^-

T'

TTT'

A"
TffTtVtT'126"I5"' 12

TT"

ADDITION

Addition

the

sum

two

cases.

of

of

two

or

When

1. Add

and

ITT*

TTslj''
T^VlTTJB^TF'

6 0'

71)^17' "llir'

fractional

more

^17'

luiT'

^TTTTj

process of finding
numbers.
There are

is the

Fractions

the fractions

TTT'

FRACTIONS.

OF

CASE

110.

"

ii' if' ff^T)T7'

109.

Trf

85"

iV' A'

fj

TT""

I.

have

denominator.

common

f.
OPERATION.

Solution."
is
fifths,

The

6 fifths,

sum

are

and
of 1 fifth,2 fifths,

equal to 1^ (Art. 106).

_[-2 _^

6
r=

l
=

FRACTIONS.

COMMON

Explanation.

parts of the

express
and
in

3
one

the denominators

Since

"

145
the same,

are

size;therefore,add

same

as
fifths,
you would add 1 cent, 2 cents, and
in Ihe other,6 cents.
case, being 6 fifths,

the numerators;

1. Add

Rule."

ators
numer-

fifth,2 fifths,

cents; the

the

under

the

sum

sum,

the

write

denominator.

common

Rem.

or
integer,

Rem.

if an
result,

The

"

mixed

The

2.
"

be reduced

improper fraction,must
(Art.105).

number

reduced to its lowest

be

result must

to

an

(Art.

terms

107).

3. Add
4. Add

I,

3^

If

5. Add

4,

4,

i,

I.

22.

Add

-3_

_7_

_8_

iil.

fi

7. Add

-i-V,T%, AtV" a. H,

8. Add
QAHH-JL1113
V.
^UU

the

-=

Explanation.
do
can

not

91
^"^^

if, ||.

2f

fractions

have

not

"

the fractions

Prac. 10.

common

inator.
denom-

operation.

(Art.108),
H
!l; then, the
|f is |f
|| are

Since

II

||

| ^^ff f
ff ih
ff + f| + H
U^^U

the denominators

express parts of the same


be added till they are reduced
not

|i.

denominator

common

ff^f
fi and
of f^, ff,and
sum
equalto 2 f|.
I

"2"0^-

2U?

II.

Reducing

"

2t^
2f

CASE

I, I, and

Solution.
to

if, il

When

1. Add

Hif17

"20"?

2 0?

10. Add

111.

are

the
different,

ators
numer-

size;therefore,the fractions
to

common

denominator.

Bule.

1. Reduce

"

2. Add

the

ARITHMETIC.

PRACTICAL

NEW

RAYS

146

the

fractionsto

their

"

Rem.

2.

and
integral
and
separately

be added

may

fractions

The

"

write the

mon
com-

added

bo

may

separatelyand

united.

then

sums

sum

Integers and

1.

denominator.

common

the

under

numerators^and

denominator.

Rem.

the fractional

their

sums

parts of mixed

numbers

then united.

A-

Solution.
write

the

"

1 to the

the

and

The

under
column

2 is 6.

9. Add

of

sum

the

column
of

\ and

is

|; JrrrlJ;

of fractions and

integers.The

sum

carry

of 1, 3,
"

^'

"'

operation.

2\
3^
6^ A7y.s.

2\.

10. Add
11. Add
12. Add
13. Add
14. Add
15. Add

2A\.
10f|.

16. Add
17. Add
18. Add

1.

19. Add
20. Add
21. Add

21H9f|.

I'KACTIONS.

COMMON

112.

of

Subtraction

There

two

are

cases.

the

When

1. From

the

of
process
fractional numbers.

two

CASE

113.

is

Fractions

between

difference

finding the

FRACTIONS.

OF

SUBTRACTION

U7

I.

fractions

subtract

have

ator.
denomin-

common

^.
OPERATION.

Solution.

2 'sevenths

"

from

sevenths

leaves

"

sevenths.

Explanation.
erators

from

"

express

5 sevenths

in

Kule.

under

"

one

Since

parts of the
as

you

case,

1. From

denominators

are

the

same,

size; therefore,subtract

same

would

subtract

being 3 sevenths,in
the

the remainder

2. From

the

cents

common

num.

2 sevenths

cents; the

mainder,
re-

the other 3 cents.

greaternumerator

write the

from

the

subtract

the

less;

denominator.

| subtract \.
|.
| subtract
| subtract |.
subtract f\.
-^-^
3^ subtract If.

3. From
4. From

5. From
6. From

\.

\.
\.
\.
operation.

Solution.

"

1 from

3.

I; |

J.

"

f can not
equalsf ; f
from

be taken
and

4^

8. From

8-I subtract

23^

are

leaves 1.

7. From

9. From

J; so borrow
f ; | from f leaves

from

subtract

subtract

2|.
3f
17^^

3^

lf_
l^

Ans.

H4|.
^.

KAY'S

UB

PRACTICAL

NEW

ARITHMETIC.

CASK

the

When

114.

II,

fractions

have

not

common

denom-

inator.

1. From

denominator

common

T%

Explanation.
do
fraction
to

not

OPKRATION.

^J
^%

denominators

parts of

subtracted

from

are

the

satne

the

other

the numerators
different,
size; therefore, one
till they are
reduced

denominator.

common

Bule.

to

|-Jleaves

the

express
he

not

can

from

Since

"

fractions

(Art. 108), f

then, f^

H;

|.

Ri^diicingthe

Solution."

and

subtract

yV

1.

"

Beduce

the

2. Fro7n

the remainder

the

fractions to

greaternumerator
icrite the

common

subtract

common

the

ator.
denomin-

and
less,

under

denominator.

.8
15*

fi-

t's
tt-

Solution.
be taken

f; "

"

from

fi^ndI

arc

\ equals |, and | equals |. | can Tiot


f ; so borrow 1 from the 3. 1 equals
|; | from | leaves ". 2 from 3 leaves 1,

OPHUATloN^.

3|

If

FKAOTIo/s.

COMMON

149

If

2H4|

17. From

4^i^subtract

18. From

56^

subtract

421

19. From

60|
97|

subtract

41^^.
48|.

20. From

3^.

subtract

14_i__.

MULTIPLICATION

115.

OF

Multiplication

findingthe product of
1. If 1

apple cost I

19f
48|.

of
two

of

FRACTIONS.

is the

Fractions
or

fractional

more

will 3

cent, what

of

process
numbers.

applescost?
OPERATION.

They will cost 3 times -|of a cent


cent
(Art. 101, Prin. 1). -i/equals

Solution.
=

-L2_

of

Explanation.

hence,

12

apples
^-=y,^-.
"

3 times

2. At

|X f
y-

"

ct.

will

cost

yard, what

will

| -j-| -f |

of

of
^^z.

yard

"'
2f

cent;

ribbon

of

cost?
Solution.

then, I

"

of

^t.
\ of a yard will cost \ of 12;=rij2
will
2
^
cost
times -V" ^ ^^- ( ^^^- )'
yard
"

44.

3. What
pe**

will

of

yard

of cloth

V" X f
"".2^4^44
=

cost,at | of

dollar

yard?

dollar;then, ^ of
a

operation.

dollar.

yard

will cost

times

^3^ i|
=

of

|X

t=

35

NEA^

RAY'S

15a

Explanation.
of

then, }

4.

"

of

of

PHACTICAL

I of

is

dollar

dollar is 3 times

ARITHMETIC.

-^^
=

-^^of

/^ of

dollar

dollar

(Art. 99);
(Ex. 1).

MuUiply f by f

Solution.

"

by
multiplied
pliedhy ^ of

J
4

is the

of 4

(Art.96). f
is f (Art.101, Prin. 1); then, f multiis J of 1
^5 (Ex. 3, Explanation).
same

as

operation.

f n^ |_

^8^

the numerators
of the given
Multiplytogether
fractions
for the numerator of the product.
the denominators
2. Multiply
of the given fractions
together
for the denominator of the product.
Rule.

1.

"

Express integersin

Rem.

1.

Rem.

2.

"

"

Reduce

mixed

numbers

to

convenient
be more
it may
fractional parts separately.

Sometimes
and

the form

Rem.

3.

"

6.
7.
8.
9.
10.

(Art.97, 3).
(Art.104).
the
multiplyby
integral

improper fractions
to

operationand apply the


it is practicable
(Art.91, Rule).
the

Indicate

wherever

5.

of fractions

Multiplyf
Multiply8
MultiplyI
MultiplyI
Multiply5
Multiplyf

by
by
by
by
by
by

Rule

for Cancellation

3.

2\
^
^,
2|,
3|

|.
f.
4.

f
|.
.

operation.

Solution.

plying

the

Indicatingthe operationand apRule for Cancellation


(Art. 91),the
"

12.
13.
14.

Multiply I
Multiply20
Multiply^\
Multiply I

|X f

"

"

result is ".

11.

by 6.
by f.
by if
by 10.

4.
15.

H6.

FRACTIONS.

COMMON

15.
16.
17.
18.
19.

Multiply 12 by
Multiplyj\ by
Multiply f by

|.
f.

Multiply 7 by
Multiply21 by

|.

6-3

^:^

91-

To.

3i.

"

71^

20.

2JL

6.

proper
Reducing 2 J and SJ to imfractions (Art.104), they are | and |.
Multiplyingtogether | and |, the result is
Solution.

151

OPERATION.

--n

Multiply18f by

8.
OPERATION.

144

144

150.

150

21.
22.
23.
24.
25.
26.

27.
28.

Multiply 8 by
Multiply 2^ by
Multiply10| by
Multiply 25 by
Multiply -^%by
Multii:)ly
lOf by
Multiply 64 by
Multiply 8f by

3|.
2|.
7.

8|.
17^.
9.,

8f.
f

29i.
H7^.
215.

15^.
97|.
568.

3f.

:
Multiplytogether

29.

TJ'

2A-

16'

30.

-A2-1"^16'

31.

6|, 2|, 21.


2h 3|, 4f, If
2i,2A, 3i,.lT^.

32.
33.
34.

IT'

i
8'

35.

li
*9-

_3_

8.

10?

9?

^'

^
3'

.5

4'

T'

-^J

9^'

4'

T*
2
3'

fi
""

4091.

49if
22.

36.

37.

116.

PRACTICAL

NEW

KAY'S

152

AKITHMETlC.

f f, If, h h h h 20.
2i, 6|, ^, ^3, 2, f.
Fractioiuil

parts

of

24.

integersare

obtained

by

multiplication.
1. What

is

of 2?
OPERATION.

J of 2 is f (Art.96); then, f
i. i^^l

Solution."
2

times

of 2 is

i=n

3f.
2f
8.
10.

12f
18f
Hi.

Compound fractions (Art. 99)


simple fractions by multiplication.
117.

1. Reduce

Solution.
the

"

result is

of

to

are

3. Reduce

^
f

4. Reduce

5. Reduce
6. Reduce

7. Reduce

^^
f
I

of
of
of
of
of
of

to

simple fraction.

Multiplying | by f (Art.115, Rule),


j%.

2. Reduce

reduced

operation.

2y^*

f to a simple fraction.
|^to a simple fraction.
| of 2f to a simple fraction.
f to a simple fraction.
f to a simple fraction.
4 of 1|^to a simple fraction.

j\

3 3
14

3^
15

"3T'

FRACTIONS.

COMMON

8. Eeduce

of

9. Reduce

of
of

11. Reduce

f
f

of

12. Reduce

of

13. Reduce

of

14. Reduce

f
f
^3
^
^

of

10. Bediice

15. Reduce
16. Reduce
17. Reduce
18. Reduce
fraction.

of
of
of
of

f of i to a simple fraction.
| of f to a simple fraction.
2T
of
fraction.
|- |^to a simple
| of y7_of |f to a simple fraction.
of

simple fraction.
"^,
| of 1^ to a simple fraction.
1
^ of l\l to an integer.
2,
2f of If to an integer.
-/^of 1|^to a simple fraction.
4 of | of 5 to a simple fraction.
\,
ple
| of f of | of f of | of j\ to a simiV*
|

to

Miscellaneous

118.

What

1. Of

2^

2. Of

will be

lb. of

153

the

meat,

Examples.

cost

l^ ct. a
$| a yd.?

at

30f ct.
7 yd.?
S3f

Of 5 yd.? Of
yd. linen,at
Of 61 yd.? 5fyd.?
3. Of 3^ lb. of rice, at 4|-ct. a lb. ?
16 ct.
4. Of 3| tons of iron, at $18f per T.?
$60.
5. Of If yd. of muslin, at $^\ per yd.?
${.
6. Of 21 lb. of tea, at $f per lb.?
$2.
7. Of 5| cords of wood, at $1| per C?
$6|.
8. At the rate of b^ miles an
hour, how far will a man
travel in 7f hours ?
42| mi.
9. I own
I of a steamboat, and sell f of my share :
what
f
part of the boat do I sell?
10. At
$6| per 3^ard,what cost | of a pieceof cloth
containing5^ yards?
$8^.
what?
11. f of I of 161- X I of I- of 15
34f.
12. What
is the sum
of f + J and | X I"?
l^'
'

'

lb.?

=:

RAY'S

154

quotientof

the

yard

process

of

finding

fractional numbers.

two

ribbon

yards of

1. If 3

is the

Fractions

of

Division

FRACTIONS.

OF

DIVISION

119.

ARITHMETIC.

PRACTICAL

NEW

cost

of

what
dollar,

will 1

cost?
OPERATION.

Solution.

"

yard will

cost

of

of

f =f

dollar
^

?Xi-f

(Art.117).

^ is to be divided into 3 equal parts.


f -f-^-f ^.
(Art.101, Prin. 2); for f

Explanation.

will be

2 dollars

be

for

of

dollar

for

of

117); then,
|
yard can be bought.
Explanation.

be

yard, could

hence,
divided

101,

find

to

Prin.

3. At

for 4

2.

by

yard

yard can be bought,


\ of \^^^^ of a yard (Art.
dollar 3 times -^^^-^^ of a
\

it

Were

"

of

yard, what part


f of a dollar?

1 dollar

For

"

for

bought

Solution.
and

Each

part

:==

2. At
can

"

requiredto
for $6, then

bought

the

part of

To

divide

find how
would

cloth

of

operation.

|X|

yards,at $2
many
be divided by 2;

yard that $| will pay for,\ must be


\ by 2, multiply the denominator
(Art.
a

3).

of

cent

for 1

apple,how

can

many

be

bought

cents?

Solution.

bought, and

"

For
for

apple; then, for


6 apples.
I

J of a cent
|, or 1 cent,

4 cents, there

can

\
3

apple can
times |
| of
an

be

bought

be
an

4 times

operation.
"

fX J

^=

4. At

be

f
bought

of
for

for 1

cent

of

cent?

apple,how

many

apples can

FRACTIONS.

COMMON

of

cent

155

apple

an

be

can

operation.

bought, and for |, or 1 cent, 3 times


| of an
apple; then, for \ of a cent J of f
| of an apple
be bought (Art. 117),and for | of a cent 3 times
can
1-1-apples.
f
i

|X 1
f
1
1}

t=r

5. Divide

Solution.

divided

divided

| by
f

"

by

by

Rule.

of

4.

is the

same

of

(Art.96).

operation.

is

j\ (Art. 101, Prin. 3); then, f


sy^^
4Js 5 times j\
Ex.
(Art.
115,
i^
1).

\^

Mulflplythe

"

as

dividend

by

the divisor with

its terms

inverted.

Rem.

1,

Express integersin

Rem.

2.

Reduce

Rem.

"

mixed

the form

numbers

of fractions

to

"

6. If

yard

cost?

7. At

yards
a

of

cent

muslin

each,

cost

how

(Art.97, 3).
(Art.104).

fractions

improper
Indicate the operation and apply the
3.
whenever
it is practicable
(Art,91, Rule).
"

of

many

Rule

lation
for Cancel-

dollar,what

applescan

be

will

Sf
bought

for 3 cents?
8. At

^
be

can

9. If
could

be

10. At

of

6.
a

dollar per

bought

1 orange

for
cost

3^ard,how

many

yards of

^'

Sy% ?
3

cents, what

purchased for ^ a cent?


f of a dollar per yard, how

lin
mus-

part

of

an

orange

\.
many

yards of

cloth

8.
buy for 6 dollars?
11. At
yards of
\ of a dollar per yard, how many
ribbon
be purchased for f of a dollar?
can
3f.
will 1
12. If 7 pounds of rice cost i|.of a dollar,what
pound cost?
$^.
can

you

KAY'S

156

PRACTICAL

NEW

13. Divide

ARITHMETIC.

4^-by If
OPERATION.

Keducing 4^ and
(Art.104), we have | and

Solution.

"

tions
IJ to improperfrac|. Dividing| by J,

li=J

the result is 31.

3|

f4.
40.

fIfIf
8.

^1 I

HA25.

lOf
Aoff
Explanation.
and

in

"

the

^ as
I
simple fraction.

Invert
case

the terms

of the divisor

of both

being

operation.

jXfXIXi"

31. Divide

32. Divide

by f Of 17^.
j\ of | of 12f^ by ^ of 8|.
f of | by f of ^ of 5.
j\ of f of 12^3^by i of 4^^

33. Divide
34. Divide
35. Divide

120.

by

What

division.

of

f by f

of

of 5^

part

one

number

is of

5
6*

iof 20.

another

is found

FRACTIONS.

COMMON

1 is what

1.

Solution.

"

part
is

of

157

of 2?

2; for | of

is

|,or

(Art.

OPERATION.

98, 2d).
is what

2.

:Xi

Solution.
of

"

part

1 is

of

of 3?

3; then,

is 2 times

1=1

3.

OPERATION.
2
1

3.

is what

part

/\

^-

of 3?
OPERATION.

Solution.

4.

times
imoQ

4
J

1 is

is what

Solution.

t-l
=n:| of

"

"

f; then,
8 /-"f
3
of f
I

i
i

is what

6.

f
^
I
3|
I
8|
fi

is what

8.
9.

10.
11.
12.

of
is

of

is

J of -J- i

of 3.

f?

f, and
of |
=

|, or

1, is

of

|, and

times

|-is 2

OPERATION.
2
3

\/

4
3

"

"

5.

7.

3; then,

part
is

of

is what
is what
is what
is what
is what
is what

part
part
part
part
part
part
part
part

of 4 ?
of 5?
of i?
2-

|?

of 5 ?

of

of

5^-

1 5
16-

f?

of 11?
of

I.
9-

_9

|f?

121.

Complex fractions,(Art. 100)


simple fractions by division.
1. Eeduce

-|to

are

reduced

to

simplefraction.
OPERATION.

Solution."

Reducing IJ and

(Art.104), we have {
(Art.119),the result is if.

and

2i

to

tions
improperfrac-

|. Dividing | by J

91
^3

"
"

^
-i

RAY'S

158

NEW

-^

2. Eeduce

PRACTICAL

ARITHMETIC.

to

simple fraction.

^.

to

simple fraction.

y2^.

to

simple fraction.

^.

simple fraction.

^J|.

-^ to

simple fraction.

J|.

to

simple fraction.

|.

-r^ to

mixed

number.

44.

to

mixed

number.

If.

to

simple fraction.

-^

3. Keduce

4. Keduce

"

31

j| to

5. Reduce

21

6. Eeduce

7. Eeduce

"

97

8. Eeduce

8J

9. Eeduce

^^

^^
75

10. Eeduce

Miscellaneous

122.
silk

can

1. At

^ a
bought

be

dollar per
for $3^?

|^.

Examples.

yard, how

may

be

dollar

per

10?

5. Divide
6. Divide

7.

of

3f by f of If
^ of 271 by ^

li by i.

Multiply

of

6^.

pound, how many


be purchased for ^2^?
tea can
3. At
3| dollars per yard for cloth,how
be purchasedwith
can
$42^?
must
4. By what
| be multipliedthat
2. At

yards

many

of

21f

pounds
many
the

of

3|.
yards
11^.
product
26|.
5f
I29.

A-

COMMON

8.

159

JAof 5^ by ^.

Multiply

'T2

^T^

|".

2i

li

^2 by -f

9. Divide

10

FRACTIONS.

Divide

"

bv

FRACTIONAL

"

32-

^.

11.

COMPOUND

NUMBERS.

1. Add

128.

$16j\',$9-1;$53-V;$2j|.
SSB^-V
2. I paid for books
$9|-;for paper, ^4^7^;for a slate,
did I expend?
$|; for pens, $lf; what amount
S15^.
3. Having $50^, I paid a bill of $27-^^:how
much
had I left?
S23Jg.
4. From
$32.31^take $15.12i.
$17.18f.
5. From
^4.621
$5.81^take $1.18f.
Find

the

cost

of

6.

yd. of muslin, at

7.

21

8.

15

lb. of sugar, at
yd. of cloth,at

121

6^

ct.

ct.

yd.

lb.

S1.12f
$1.31f

$47,811
$3.18fper yd.
51 yd. of linen,at $0,621 per yd.
9.
$3.43|.
121
10.
yd. of ribbon, at 18| ct. per yd.
$2.34|.
11.
131 yd. of calico,
at 16| ct. per yd.
$2.25.
12.
101 yd. of cloth,at $3,371 a yard.
$34.59f.
13.
$66.25.
17| dozen books, at $3.75 per dozen.
14. At 18| ct. per yard,how
yards of muslin can
many
be purchased for $2.25?
12 yd.
15. At 371 ct. per bushel,how many
bushels of barley
can
you buy for $5.81|?
15^ bu.
16. If 5 yards of cloth cost $11.56^,
what
cost one
yard?
$2.31^.
17. Seven
share $31.06^ equally: what
is the
men
share of each man?
$4.43f.
,

PKACTR

NEW

KAY'S

IHO

18. Eeduce

5 mi.

19. Keduee

Reduce

20.

15875

22. Reduce

U2634

23. How
take

in

walking

feet.

3 rd.
^

miles.

ft. to

3 mi.

ft. 8 in.

will
revolutions
many
circumference,make in running 65
1 A.

26. Reduce

7506

28. How

each, will

wheel,

sq. rd. 25 sq.

136

in

acres

many

30. Reduce
31. In

field

40^

rd.

yards.
8979 sq. yd.
sq. rd. 4 sq. yd.
4078| in.
long and 32 rd.

the

square

at

moon,

being 238545
A

124.

hr. to
will

time
the

rate

of

Solution.
fraction of
result is

of

2. Reduce
3. Reduce
4. Reduce

5. Reduce
6. Reduce

body

move

cen.

4 yr. 101

da. 4 hr,

fi'om the

earth to

miles per

day, the
21

is reduced

J^

of

To

reduce

peck

to

Rule

to

lower

distance

yr.

24| da.

denomination

I).

the fraction

.}^of a jx'ckto
pint,multiplyby 8 unci by 2.
a pint.
"

31

(Art.63,
by multiplication
1. Reduce

cen.

miles?

fraction

16 sq. rd.
hr.
35064

hours.

to

years

914092

what

man

of 9 ft. 2 in.

8 A.
4

in.

37440.

yd. to

1 A. 88
sq. yd. to A.
5 chains 15 links to in.

Reduce

miles?

wide?
29.

yd. 2

3900.

24. How

27. Reduce

3| yd.

2 miles?

25. Reduce

ft.

2 rd. 2 ft.

2 mi. 80 rd. 2

in. to miles.

in.

10595

rods.

steps, of

many

316800

2 rd. 2 ft. to

yd. to

Ecduce

21.

AKITHMETIC.

inches.

to

mi.

20

AL

of

the
The

i)int.

opkration.

X I X

=-

-^^bu. to the fraction of a quart.


-^^lb. to the fraction of an ounce.
yL.lb. Troy to the fraction of an ounce.
^ rd. to the fraction of a foot.
y^Vo""^' ^^ ^^^^ fraction of a square rod.

|.
^.
|.
|^.
|.

8. Eeduce
9. Reduce

In

result is

the

the

reduction

of

reduce

of

| of

in

denomination,

day

day

in

the

value

of 4

3. Find

the

value

of

4. Find

the

value

of

5. Find

the

value

of

mi.

$|-in
f mi.
^ lb.

ing
is called find-

integers.

tiply
hours, mul-

to

the result is

2. Find

of

an

is 30

in

integers.
integers.
in integers.
Troy in integers.
9

6. Find

the

value

of

7. Find

the

value

of

8. Find

the

value

of

-^^T. in integers.
| A. in integers.
-J of 63 gallonsof

by

fraction

division

1. Reduce
Solution."
tion of
is

2V of

a
a

is reduced

(Art.63,

|
To

of

Rule

100

wine

peck,divide by
peck.

| of
2 and

2. Reduce

qt. to

3. Reduce
luce

ft. to the

Prac.

11.

the

higher

pwt.
75 lb.

8 cwt.

sq. rd.
in integers.

gal.1 pt.

denomination

IT).

pint to

reduce

to

12

oz.

55

12G.

with

integers.

9| hr. To reduce |
to minutes, multiply by 60; the result
I of a day, then, is 9 hr. 36 min.

by 24;
min.

To

"

fraction

lower

fractional part. This

value

the

1. Find

hour

of

to

f.
|^.
|.

number, proceedonly

mixed

of the

value

Solution.

fraction

reducing a

when

the

161

$3!^ to the fraction of a cent.


y^^g^da. to the fraction of a minute.
-^^ bu. to the fraction of a pint.

7. Reduce

125.

ACTIONS.

FK

COMMON

the

of

fraction

pint to the fracby 8. The result

fraction
fraction

of
of

a
a

peck.
operation.

| X ^X i

2?

bushel.
rod.

^.
y

RAY'S

162

PKACTICAL

NEW

ARITHMETIC.

^\ oz. to the fraction of a pound.


rAirlb.
of
the
fraction
ton.
to
a
^
t^Vttf pt.to the fraction of a bushel.
-^^.
^ oz. to the fraction of a hundred-weight.

4. Eeduce
5. Keduce
6. Eeduce

7. Eeduce

2800-

8. Eeduce

in. to the

f
| min.
yf^ ^^-

9. Eeduce
10. Eeduce

To

127.

in Art.

as

1.

of

the

fraction

of

to

the

fraction

of

them

part
to

the

rod.

to

find what

another, reduce

fraction

day.
rwruhundred-weight.

a
a

number

compound

one

^J^

denomination

same

is of

and

ceed
pro-

120.

2 ft. 3 in. is what

part

of

yard?

OPERATION.

2 ft. 3 in.

2 ft. 3 in. =

equals27 in. I yd.


equals36 in. 27 in. are |J of 36 in. fj
equalsj. 2 ft. 3 in.,then, is } of a yard.
Solution.

"

2.

ft. 6 in. is what

3.

pk. 4 qt. is

4. What

part is
part of

5. What
6. What

7. What
8.

of

mile

is 145

of

yard

is 2 ft. 8 in.?

A.

has

man

94

sq.

rd. is what
a

of

farm

rd.,what

part of 6 ft. 8 in.?


part of a bushel ?

yd.=r=36in.

ii

yd. 9 in.
day is 13

15 mi. 123

9. A

37

part
part

what

27 in.

part

of 8

yd.

hr. 30

|
|
^

ft. 3 in. ?

min. ?

rd.?

fj
"

part

of 35 mi. 287

168

A.

of his

rd.?

f.

sq. rd. ; if he sell


farm
will he dispose
28

of?

A\-

10. What
11. 2

part of

qt. lipt. is

pound

Avhat

is

part

7^

of

oz.

1 bu.

^.
1

qt. If pt.?
1 6

12. 1
?
in
1^"

yd.

1 ft.

1^

in. is what

part

of

yd. 2

ft.

8f

1 91 "
5 4T3'

FKACTIONS.

COMMON

To

128.

add

subtract

and

find the value

proceed

of the

fractional

fractions

compound

bers,
num-

in

integersand then
of Compound
Subtraction

and

in Addition

as

163

Numbers.

1. Add

I yd. and

ft.
OPERATION.

Solution.

-|yd. equals 2

"

of
equals10 in.;the sum
in. is 3 ft. 1 in. (Art.75).

2. From

ft. 3

ft. 3 in. and

yd. :rrr2

10

ft.

ft. 3 in.
10

in.

3 ft. 1 in.

da. subtract

in.; | ft.

hr.
OPERATION.

Solution.

hr.

from

"

da.

equals5

equals 50 min.;
5

hr. 20

min.

50

min.

leaves

hr. 20 min.;

I
I

subtracted
hr. 30

da. ==5
hr.

hr. 20

min.

50

min.

min.

hr. 30 min.

(Art.76).
3. Add
4. Add

5. Add

da. and

I
\
I

hr.

wk.

da. and

wk.

da.

7. From

hr. and

\^ gal.and ^2 ^t.
^ da. subtract Jg hr.

8. From

9. From

| lb. subtract ^ oz.


| da. subtract ^ hr.

10. From

1. Reduce
2

Arid

subtract

Promiscuous

y g|j

to its lowest

91

^2

3. From

34^ subtract

4. From

3|

5. Add

of

subtract

j\

and

min

da. 15

min

min.
da. 6 hr. 40
3

1^.
^ of 3^.
| of -j^.

sec

qt. 1 pt. 2 gi.

18 hr. 36 min.

SA.

129.

-^

hr. 45

hr.

5
6. Add

16

40

sec.

55 ct.

2 hr. 34

min.

^oz.
17| sec.

Examples.

terms.

a-

RAY'S

16^

G. Add

PRACTICAL

NEW

2.1-and

1| -^

niiinber

7. What

xVRITIlMETIC.

-^ 3i

divided

2f|.

by f

will

give

10

for

tient?
quo6.

number

8. What

will

multipliedby |

10

give

for

product?
9. What

number

the
itself,

number

the
itself,

sells

is

will be

sum

is worth

boat

of his share

which

that,from

will be

remainder

10. What

11. A

is

if you

16|.
^ of

take

16?

28.

that, to

which

if you

add

j^ of

20?

14.

S900 ;
what

merchant

part

has

he

owns

left,and

of

it,and

what

is it

worth
$375.
3^ left,
^ of my share for

worth?
12. I

of

j^

own

is the

$1944|: what

ship, and sell


whole ship worth?
3 cents is | of 2 cents?
a

13. What

part

14. What

part of 368

15. From

|^

of

810000.

|.
i|.

is 170?
the

subtract

of

sum

^, y^y,and Jj^y.
1 007

y\.
-^^of ^^ of 4yV
17. From
2%| ^ f subtract | -^ |f
rods in y^^of an
18. If I ride 2044
hour, at that rate
how
far will I ride in 1|4 hr. ?
8468 rd.
19. What
part of 1^ feet are 3^ inches?
|.
20. Two
men
bought a barrel of flour ; one paid S3^,
and the other $3| : what
part of it should each have ?
One
^^^,the other y^^^.
21. A has $2400 ; | of his money,
+ $500, is | of B's :
16. From

1 subtract

"

what

has

sum

22. John

B?

Jones

daughters,the
The

younger

share
estate

of
:

the

find

$1600.
divided

latter
son

his

received

share

$2200,
share

of each

among

sharing equally

elder, w^hose
the

estate

Avith

which
was

was

^f

daughter.

of

sons

each

-^
the

and

other.
of

the

whole

$1356^.

130.

An

aliquot part is

Parts

Aliquot

The
used

exact

an

of

of

divisor

number.

100.

of aliquotparts of 100 are often


followingmultiples
184=^3^,37i=a, 40=f, 60=f, 624=t, 75=f, 87*=^

1. What

will 24

yd.

of muslin

cost

at

25

ct.

yd.?

OPERATION.

Solution.
be

as

2. I

many

"

25 ct. is

Since
dollars

many

as

there

spent $1,121

yd. did

for

\ of

dollar,the

yards. \

are

muslin

at

cost

will

4)24
$ 6

of $24 is $6.

12|

ct.

yd. :

how

bii}?
OPERATION.

Solution.
he

8 times
=

"

as

Since
many

V2h

ct. is

yards

as

of

there

a
are

dollar,there will
dollars.

8 times

9" yd.

9yd.
8. What

1"^

cost

12^-yd. of

ribbon

at

]8| ct.

yd?

S2.34g.

KAY'S

166

ARITHMETIC.

PKACTICAL

$2.25 for muslin

4. Paid
did I

NEW

at

18J ct.

how

yd.

buy?

12

will 5^

5. What

6. Paid

for books

at

cost

S0.62i

at

$3.75

buy?
80 gal.of

will

wine

how

many

17| doz.
$2.37^a gal.?

at

cost

yd.?
$3.43f.

dozen

did

doz. books

linen

yd. of

$66.25

7. What

yd.
yd.

many

$190.
number

8. A

of

$39

that

so

each

there?
men
are
many
barrels of flour cost at

$4.87^:how
9. What

divide

men

will 36

one

ceives
re-

8.

rel
bar-

$8.33^a

$300.

?
10. How

bought for
11. What

will

A.

cloth

land

sq. rd. of

60

yd.

$1.33^ a

at

cost

at

can

be

185

yd.

$16.50

acre?

an

Solution.
4

of

yd.
$246. 66|?
many

A.

A., 40

Since

A.

$1G.50,

costs

Since

$16.50X4:= $66.

cost

sq. rd.
cost

"

sq. rd. =

A.

operation.

$16.50

160
The

sq. rd. will be J of $16.50=


The
cost of 20 sq. rd. will be

of 40

$4,121.
of the
total

of 40 sq.

cost

is

cost

66.00
J

J
J

$2.06^. The

rd.,

or

$66 -f $4.121 -f

of $16.50

of

4.12^

$2.06}=

4.12^

2mI
$72.18J

$72.18|.
12. At
for

$18.33^per

acre,

how

land

much

can

be

mA.

$229,162?

13. What

$125.60
land

will

$250
be

can

$6.20

bu.?

120

sq.

rd.

of

land

cost,

at

$1475.80.

lot,containing50

bought
6

15. What

A.

11

acre?

per

14. At

bought

will 83

150

ft.,how

much

for $10000?
A.
bu.

141
3

sq. rd. 28 sq. jd. 108


pk. 2 qt. of grass seed

sq. in.

cost,at

$519.63|.

PKACTICE.

$0.75

At

16.

17.

What

18.

At

for

will

SI.

50

bu.

223

yd.

3|

how

yard,

$1.75

at

cost,

bushels

raany

pk.

cloth

much

li

qt.

yard?

be

can

bought
4f

What

will

lb.

45

12

butter

of

oz.

20.

21.

$0,121
for

the

is

much

be

can

sugar

lb.

23-i

$2.93f

What

how

pound,

per

of

cost

T.

of

cwt.

wool

at

37^

ct.

$1837.50.

pound?
22.

$0,371

at

cost,

yd.

$17.15|.

At

bought

pt.

$6.12f

pound?

per

be

can

S7.12I-?
19.

how

bushel,

S167.50?

for

bought

167

What

is

the

of

cost

readers

100

$3.90

at

dozen?

$32.50.
23.

What

is

the

of

cost

knives

dozen

3f

at

$20.25.

dozen?
24.

and
in

$5.40

37^

lb.

butter,
and

money,

pound:

sold

farmer

how

the

many

6^

at

doz.

$0.37^

chickens,

in

remainder

pounds

pound

per

of

sugar

he

he

apiece,
$36

received
at

sugar,
did

$0,331

at

get?

$0.12^
32i

per

lb.

The

131.

rightin

orders

units

1. The

that
3.

the

orders

in the

Since

number

tenth;

is 1

unit

the

like

In

1 unit.

is 10 times

the

the

from

law

same

order

of decrease.
the

separated from

be

order

1.111,
point is

from

second

order

from

is 10

1 to the

riu;ht

J^.

the unit is 1

tenth,the second
times

order

yi^.

the unit is 1 hundredth, the third

thousandth; for yi^ is


it may

manner

unit,the

for 1 unit is 10 times

hundredth; for ^^

the unit is 1

from

hundred,!

point (.).

first order

the

times

be continued

the 1 to the left of the

point is 1

the

units

is 10

DECIMALS.

may

rightby

Then,

4th,
the

OP

by

2d. Since

order

ORDERS

follows

of the

3d.

1 ten

tlie order

Ist. Since

from

1 ten. and

the

toward

2. Let

1111, 1 thousand

mmibcr

is 10 times

132.

left to

from

tenfoldratio.

Thus, in the
hundred

integers decrease

of

be shown

10 times

yoVo-

that 1 in the fourth "n"der to

right from the unit is 1 ten-thousandth; 1


is 1 hundred-thousandth; 1 in the sixth
riglit

in the fifth order

order

is 1

to

millionth,

etc.

Rem.
as

well

"

A
for

number
the

of figuresother
consisting

purpose

(1G8)

of illustration,

than

1, might be used

DECIMAL

The

4.

relative

FRACTIONS.

the

position of
the

to

unit

169

decimal

integral and

ia exhibited

in the

phices

Ibllowinii;

DTACiRAM.

1.
\A

^^y^.

.^c)V^

/-

5. The

first order

first order
order

the

on

the

on

the

on

left of

the

rightof

unit

the
is

the

leftis hundreds;

unit

is

tens,the

tenths; the second

second

order

the

on

rightis hundredths,etc.

DEFINITIONS.

133,

1. A

decimal

fraction,

or

decimal,is

tenths,hundredths,thousandths,
etc.,written
of

one

or

more

like the orders

integers.
2. A

decimal

tenths

to

3. The

decrease

is

( )

point

placed

distinguishthe

fraction.

decimal

increase

from

orders

left to

right

the

from
same

before

the

order

rightto left,and
as

the

orders

of

similar

to

integers.
4. The

the

names

134.
iiro"?

names

of

of the

Conversion

the

orders

of decimals

correspondingorders
of

^^ decimals.
^^^'"""

the

common

of

are

integers.

fractions

"^, j^-^,

RAY'S

170

1.

yi^
j\

PRACTICAL

NEW

is

are

written

.1

written

.2

.4

TIT
5

.5

TIT

Hence,

ARITHMETIC.

i^*/ienthe

denominator

10, fAere

is

is

decimal

one

order.

2.

^^ wi-itten

Yw^

is written

in the

.Hence,when

.01; there

being no

tenths,a cipher

order.

vacant

the denominator

is

100, there

are

mal
tiro deci-

orders.

3.
no

is
yo^oTj

written

.001 ; there

hundredths, ciphersare

Att

.003

.005

Hence,

4. In

when

the

^i*e

written

.008

.009

is

1000,

there

manner;

OUTFO

TTr"77"00^

is

written

.006
.007

TOGO

denominator

and

orders.

vacant

orders.
like

tenths

T7T0"

.004
Tirxro

no

in the

written

lOOlF

decimal

being

.0001
.00001
.000001

are

three

FRACTIONS.

DECIMAL

the

same

the

common

5.

of orders in the decimal is always


of
of ciphersin the denominator

number

the

Hence,

the number

as

fractio7i.
and

j\

Hence,

written

jW

j^-^are

tenths

and

thousandths

of

numerator

there

decimal

8. The

hundredths;

as

of

name

sandths
thou-

as

sandths
ten-thou-

number

it

presses
ex-

point.
before

the

numerator,

in them.

right hand

the

is the

order

name

of

decimal.
To
1. Write

135.

Decimals.

Wj^ite
hundred

two

and

Number
Explanation.

1111

read

are

is the

orders

vacant

are

written

ciphersare

decimal

the
disregarding

The

read

are

.11

'^

tenths,hundredths, thousandths, and


etc.
read as ten-thousandths,
are

6. The

the

1111

f\V(^

hundredths

tenths, hundredths,and

7. If

171

"

figure5

must

be

stand

must

hundredths

is phicedbefore
therefore,

2. Write

in

and
the

"

Write

the

order

and

Number

Explanation.

numerator,

265,

as

an

integer.

(134, 8); then,


be tenths; the decimal
2 must
point,
figure2 (133, 2).
the

hundred

two

.265.

Written.

the

First, write

thousandths.
sixty-five

thousandths

millionths.
sixty-five
.000265.

Written.
numerator,

265,

as

an

integer. The

(134, 8); then, 6 must be


be ten-thousandths, and ciphersmust
hundred-thousandths, 2 must
be written
in the orders
thousandths, hundredths, and tenths (134,
7 ); the decimal
pointis placedbefore 0 tenths (133, 2 ).

figure5

must

stand

in the order

millionths

KAY'8

172

3. Write

NEW

PRACTICAL

huiidrod

two

ARITHMETIC.

hundredths.
Hixty-fivc

and
NuMJJKR

Explanation.

figure5

the

the decimal

in the order

four

huiidred

4. Write

1. Write

"

2. Place
order

Note.
orders

as

so

right,and
Rem.

to

be

the

the

in

the

rendered

2; then write

point.

integer.

an

that the

as

Ex.

name

familiar

of

name

of

the

right

the decimal.

with

in supcession,
both
readily,

the
from

decimal
loft to

left.
decimal

prefixciphersto
When

2.

same

them

rightto

as

as

point so

should

name

When

"

Rem.

to

decimal

the

the numerator

shall be the

from

1.

necessary

dred
hun-

two

498.000265.

the left of the decimal

the decimal

Pupils

"

Written.

First write

"

integer,
placingit at

hand

integer.The

an

miUlonths.
sixty-five

Explanation.

Rule.

as

and
ninety-eight

and

NuMBKR

the

2G5,

numerator,

stand

2.65.

hundredths; then, 0 must be tenths;


point,therefore,is placed between the figures2 and 6.

must

and

Write

"

Written.

is

proper

the numerator

decimal

is

fraction

it is sometimes

( Ex. 2 ).

improper fraction, the decimal


point is placed between two of the figuresof the numerator
( Ex. 3).
In a mixed
3.
Rem.
number, the decimal point is placedafter the
units order of the integer( Ex. 4 ).
"

an

"

Write
5.
G.

7.

the

numbers:

Twenty-six hundredths.
himdredths.
Thirty-five
Eighty-sevenhundredths.

8. Four
9. Five

10.

followingdecimal

hundred

and

nineteen

hundredths.

thousandths.

thousandths.
Fifty-four

11. Three

hundred

and

four

thousandths.

FK

DECIMAL

thousand

12. Seven

ACTIONS.

173

hundred

two

and

ninety-three

thousandths.
13.

Twenty-five and

14.

Tw^o

15.

Four

hundred

thousandths.

forty-seven

five ten-thousandths.

and

thousand

hundred

one

and

twenty -five

ten-

thousandths.
16. Mne

hundred

17. Nine

hundred

18. Six
19.

-thousandths.
thousandths.

hundred

Twenty

and

five hundred-thousandths.

thousand

three

hundred

and

four hundred-

thousandths.
20. Seven
21. Two
22. Three

millionths.
hundred

and

hundred

three

millionths.

thousand

and

four

millionths.

24.

Twenty-four ten-millionths.
and six ten-millionths.
Eighty thousand

25.

Two

hundred

26.

Two

hundred-millionths.

23.

27. Nine

millionths.

and

hundred
million

hundred-millionths.

seven

28.

Twenty

29.

One

million

ten

thousand

and

30. One

million

ten

thousand

and

twenty

31. One

hundred

32. One

thousand

and

33. Two

hundred

and

34. Two

hundred

35. Two

and

thousand

thousand

six and
one

and

one

one

three

hundred

dred-milli
hun-

ionths.
mill-

ionths.
hundred-mill-

thirty-seventhousandths.

thousandth.

twenty-fivethousandths.
units and twenty-fivethousandths.
hundred
nine
and
twenty-nine

millionths.
36. Two
nine

thousand

nine

hundred

millionths.

37. One

million

and

five hillionths.

units

and

twenty-

NEW

HAY'S

174

PRACTICAL

ARITHMETIC.

38. Two

hundred

and

39. Two

hundred

units and

40.

and
Sixty-five
the

Change
3

4.1

six thousand

and

five inilUonths.

fractions

2 3

17

ten-hillionths.

4 1

to

decimals:

5 3

tVh' iWtt' tWttj AVtf' tVV^-

A*:{

two

followingcommon

'^^^ tW'

ten-biUionths.

two

10

5 3

Read

To

136.

1. Read

Number

Read.

5 0 3

Decimals.

.2G5.

"

Two

hundred

and

thousandths.
sixty-five

is
Disregardingthe decimal point,the number
hundred
and sixty-five;
this is the numerator
of the decimal
two
(134, 6). The righthand order of the decimal is thousandths;this
is the name
of the decimal
(134, 8).

Explanation.

2. Read

"

.000265.

Read.

Number

Explanation.
hundred

The

right hand

Number
and

and

Read.

as

"

an

2. Give

hundred

and

order

is

millionths.
sixty-five
decimal
the

point,the

numerator

millionths;this is the

name

number

of the

"

Two

and

of the decimal.

hundredths, or
sixty-five

two

hundred

hundredths.

Disregardingthe decimal pointyread


integer.
the name
of the righthand order.

1.

is

decimal.

2.65.

sixtyfive

Rule.

Two

Disregarding the
this is
sixty-five;

"

two

3. Read

"

the

ber
num-

DECIMAL

Note.

Before

"

righthand
Rem.

"

commencing

order should
A

or
fraction,

mixed
as

Eead

the

4.

.028;

an

FRACTIONS.

to read

be ascertained

number

improper

be
may
fraction

the

175

decimal, the

(135, Note,

read

either

as

under

an

numbers

followingdecimal

13.0008241094710947.

8.
9.
10.
11.
12.
13.

Change
16.
17.
18.
19.
20.

137,

the

followingdecimals

to

operations with

decimals

and
Subtraction,
Multiplication

REDUCTION

138.
without

Keduction

of

alteringtheir

OF

Decimals

value.

fractions,

common

.9; .13; .19; .29; .37; .73.


.91; .347; .513; .691; .851;
.007; .0207; .00079; .001007.
1.36; .3421; .03401; .0900.
.001; .5302; 8.01; .000053.
The

15.

7.

Rule).

(Ex. 3).

14.

6.

of the

integerand

.341; 2.327; 50.005; 184.173.


.0003; .0625; .2374; .2006; .0104.
3.0205; 810.2406; 10720.0905.
.00004; .00137; .02376; .01007.
.001768; .040035; 70.360004.
.1010101; .00040005; .00100304.
.31456; .000133; 60.04; 45.1003.
357.75; .4928; 5.945; 681.0002.
70.1200764; 954.203; 38.027.
1007.3154; 7496.35491768.
.00715; 3.00005; 28.10065701.

5.

name

are

.917.

dition,
Reduction^Ad-

Division.

DECIMALS.

is
There

changing
are

four

their form
cases.

KAY'S

176

PRACTICAL

NEW

ARITHMETIC.

CASE

139.

Annexing decimal

1.

I.

ciphersto

an

integerdoes

not

vdlue.

change its

Thus, 7.00 is the

sume

as

7; for 7.00 is 7 and

no

hundredths

(Art.

136, Rule).
2.

of

an

Conversely: Omittingdecimal ciphersfrom


integerdoes not change its value.
1 of this

Number
and

2 to Case

evidentlycorrespondsto

case

1.

right

Case

I, Art. 103,

does

not

III, Art. 105.

CASE

140.

the

II.

Annexing ciphersto

decimal

a.

change

its value.
Thus, .70 is the

same

as

.7; for y^

y^o^.

"

Conversely: Omittingciphersfrom
decimal does not change its value.
2.

1 of this

Number
and

2 to Case

evidentlycorrespondsto

case

reduce

To

1. Eeduce
Solution.
fraction is

.75 to

"

"

75

decimal

to

IV, Art. 106,

case

hundredths

fraction.

common

fraction.

common

j^j^^.-^^^reduced

written

as

to its lowest

operatiox.

common

terms

(Art.

.75

i%=l
1. Write

2. Reduce

III.

107),isf.
Bule.

right of

V, Art. 107.

CASE

141.

the

the decimal

as

common

the fractionto its lowest terms.

fraction.

-^^-^

FKACTIONS.

DECIMAL

177

2. Reduce

.6 to

common

fraction.

8. Reduce

.25 to

common

fraction.

4. Reduce

.375 to

common

fraction.

5. Reduce

.035 to

common

fraction.

6. Reduce

.5625

7. Reduce

.34375

8. Reduce

.1484375

9.

to

fraction.

common

to

to

T6"-

fraction.

common

fraction.

common

|.
\.
f.
2-^^.

JL9_
12

4.02

Express

as

integerand

an

8-

fraction

common

^5U-

Express 8.415

10.

an

as

integerand

CASE

To

142.

reduce

1. Reduce

to

IV.

fraction

common

fraction

common

to

decimal.

decimal.

Annexing a decimal cipherto 3, it is


3.0; 30 tenths divided by 4 is 7 tenths, and 2 tenths
remaining. Annexing a cipher to .2 it is .20; 20
hundredths
divided
The
by 4 is 5 hundredths.
Solution.

"

operation.

4 ) 3.00
.7 5

is .75.

result

Explanation.

"

is 3

divided

by

(Art. 97 ). Annexing

decimal
a

cipherto 3 does not change its value (Art.139). Annexing


cipherto .2 does not change its value (Art.140).

B.ule.

"

1. Annex

decimal

ciphersto

the numerator.

2. Divide
3.

there

by the denominator.
Point
off as many decimal
decimal ciphersannexed
are

orders

iyi the

quotientas

to the numerator.

2. Reduce

to

decimal.

3. Reduce

f to

decimal.

.625

decimal.

.28

4. Reduce
Prac.

2V
12.

^^

.8

decimal.

.075

decimal.

.9375

*^

decimal.

.0008

to
-^^j^
^l^ to

decimal.

.0225

decimal.

|^to
^ to
^\ to

decimal.

.83

decimal.

.09 +

decimal.

.12

5. Eeduce

to
-^jj
^| to

6. Eeduce

7. Eeduce

9. Eeduce

10. Eeduce
11.

*^

yrsir

8. Eeduce

Eeduce

12. Eeduce

ADDITION

143.
the

of two

1. Add

Solution.
four

of

Addition

sum

AKITHMETIC.

PRACTICAL

NEW

KAY'S

178

OF

DECIMALS.

Decimals

is the

decimal

more

or

.00390625

decimal

Write

numbers

the

be

points may

in

units 5, 9, 2, 3 in the first cohimn

finding

813.9762.

that

so
a

the

operation.

column, the

3 7 5.83

left,the

4 9.62

to the

to the right,
8, 6, 1, 9 in the first column
etc.;then, adding as in simple numbers, placethe

5 842.196

decimal

7081.6295

tenths

point in

the column

Rule.

and

the

sum

of decimal

"

1.

1 and

6 under

7
3

813.9762

points.

Write

figuresof

between

-|-

numbers.

375.83; 49.627; 5842.1963;


"

of

process

-\-

the

the

same

ninnhers
order

so

that the decimal

may

stand

in

the

points
same

cohunn.
2. Add

in

simple numbers.
3. Place the decimal
point in
of decimal points.
2. Add
3. Add

as

the

37.1065; 432.07; 4.20733;


4

and

8 and

11.706.

ten-thousandths;

hundredths;

and

under

sum

28

9404

the column

485.08983
and

35

sandths;
thou-

sandths.
hundred-thou40.19944

FEACTIOISIS.

DECIMAL

the

4. Find

and

units

hundredths;

25

tenths; and 35 hundredths.


21.611; 6888.32; 3.4167.
6.61; 636.1; 6516.14; 67.1234; and

and

units

of

sum

179

10.

5. Add
6. Add

6913.3477
5.1233.
7231.0967

7. Add

8 tenths ; 43

and

thousandths;

19

8. Add

and

45

thousandths

and

11

and

hundred-

71

ten-thousandths

4327

and

432

thousandths;
and

1000

10. Add

and

33

and

1599.69834

thousandths;

41

hundredths;

27

719906

thousandths; 61 and 793 ten7794 hundred-thousandths; 6.009;

ten-thousandths.

16

and

and

94

millionths

thousandths; 32

969

milUonths.
Add

11.

103

ten-thousandths;

and

dred-millionths;42
hundredths

9099

and

millionths;430

ing

Subtraction

the

difference

1. From

decimal

be

numbers
in

and

1 in the

first column

to

and

in the

first column

to

mal
under

point
the

in

in

is the

decimal

remainder
of

decimal

of find

461.5738.
so

that the

column, the
the

process
numbers.

two

units

left,the tenths

operation.

7 2 9.835

the

right,etc.; then,
simple numbers, place the deci-

the

column

two

subtract
the

points may

as
subtracting

99

DECIMALS.

Decimals

between

Write

"

OF

of

729.835

Solution.

and

.0038885

SUBTRACTION

144.

hun-

ten-thousandths;.00035; 35 millionths, and

35

ten-milHonths.

35

and

999.99999999

; 220.0000009.

12. Add

100.

and

204

401

432

and

100

1001

and

151.49251

ten-tiiousandths.
9. Add

and

133.333

thousandths;

and

; 74

thousandths.

204

19

93

31 hundredths

and

between

points.

and

4 61.5
2 6 8.26

7 38
12

RAY\S

180

Rem.
as

The

"

and

PRACTICAL

phicein
cipher(Art.140).

1. Write

"

ARITHMETIC.

ten-thousandth

occupiedby

Rule.

NEW

the

the

figuresof

numbers

the minuend

order

same

that

so

regarded

the decimal

stand

may

be

may

in

points

the

same

column.
2. Subtract

simplenumbers.
the decimal point in the
3. Place
column
of decimal points.
in

as

subtract

remainder

under

the

2. From

97.5168

3. From

20.014

4. From

5.03

5. From

24.0042

6. From

170.0035

7. From

.0142

8. From

.05 subtract .0024.

.0476

9. From

13.5

subtract

5.463

subtract

subtract

38.25942.
7.0021.

2.915

13.7013.

subtract

subtract

13.0119

2.115.

subtract

subtract

59.25738

10.3029

68.00181.

102.00169

.005.

.0092

8.037.

10. From

11. From

29.0029

12. From

5 subtract

.125.

13. From

1 thousand

subtract

.00003.

subtract

2.99997

19.003.

9.9999
4.875

1 ten-thousandth.

999.9999
14. From

15. From

25

subtract

1 millionth.

thousandths

take

MULTIPLICATION

145.

Multiplication

findingthe product of
146.

OF

of

25

Placing the decimal


upon the following

millionths.

.024975

DECIMALS.

Decimals

numbers

.999999

is the

process

of

involvingdecimals.
point in

the

pends
product de-

FRACTIONS.

DECIMAL

181

Principle.
The
the

nvmber

number

of

decimal

decimal

of

in the

orders

orders

both

in

is

product
the

factors.

Thus, let the factors be .2 and

.03; then, the number


product will be three. For, .2=z^q and .0o

orders in the

product of

the

But,

rX

in which

Too

.2

.03 will be the

and

To^o"

"

there

by

T^oo

three decimal

are

equal to

of decimal

y3_. then,
as the productof ^^ by y^^
Therefore, .2X-03- .006,

same

:.006.

orders.

Examples.

147.

Multiply2.149 by

1.

6.34.
OPERATION.

Solution.

by

"

Multiply as

in

2140

simple numbers,

2.1 4 0

634.

6.3 4

There

three

are

decimal

decimal

orders

in

decimal

orders

in the

the

2.

by

are

four

the

"

in

orders

in the

be

cipheron

be

five

fore,
There-

orders

in

"

point between

Write
7 and

it is in 2.075.

2075

5, two

0 6

6447
1 2804

.035.

numerator

(Art.134, 6)

the result is 0660.

.0276, and
must

be

three
seven

There
decimal
decimal

product (Art.146); three ciphers,then,


Therefore, omitting the
prefixedto 0660.
the right(Art.140, 2) the product is .000066.

Multiply2.075 by

"85

] 3.6 2 4 6 6

.035; hence, there

Solution.

than

35;

numerator

orders

two

13.62466.

Multiply the

decimal

must

product (Art.146).

Multiply.0276 by

276

3.

6.34; hence, there

product is

Solution.

must

orders in 2.140, and

.0000660

100.

and

place the
placesfarther to

decimal
the

right

operation.

2 0 7.5

RAY'S

182

Rem.

"

decimal

ARITHMETIC.

multiply 207.5 by 100, annex


point two placesto the right.
the
Multiplytogether
SimpleNumbers.

1.

"

in

as

2. Point

off as
decimal

are

Rem.

PRACTICAL

To

Kule,

there

NEW

1.
"

When

many

orders in both
the

is less than

numerators

decimal

number

of

the number

cipher and

mcve

of

the

numerators

orders

in

the

the

mals
deci-

jiroductas

factors.
figuresin

of decimal

the

orders

product of the
required,
prefix

ciphers. (Ex.2.)
Rem.

After

placing the

decimal

point,omit ciphers at the


rightof the decimal part of the product. (Ex.2.)
To multiply a decimal
Rem.
3.
by 10, 100, 1000, etc., remove
the decimal pointas many
placesto the right as there are ciphersin
the multiplier.If there be not
enough figuresannex
ciphers.
2.
"

"

4.
5.
6.
7.
8.
0,
10

11.
12.
13.
14.
15
IG,
17,
18.
19.
20.
21.
22.

Multiply33.21 by 4.41.
Multiply32.16 by 22.5.
Multiply.125 by 9.
Multiply.35 by 7.
Multiply.2 by .8.
Multiply.02 by .4.
Multiply.15 by .7.
Multiply125.015 by .001.
Multiply.135 by .005.
Multiply1.035 by 17.
Multiply19 by .125.
Multiply4.5 by 4.
Multiply.625 by 64.
Multiply61.76 by .0071.
Multiply1.325 by .0716.
Multiply4.87 by 10.
Multiply5.3 by 100.
Multiply17.62 by 100.
Multiply1.01 bv 10.

146.4561
723.6
1.125
2.45
.16
.008
.105
.125015
.000675
17.595
2.375
18.
40.

.438496
.09487
48.7
530.
1762.
10.1

FKACTIONS.

DECIMAL

23.
24.
25.
26.

183

Multiply.0001 by 100.
Multiply1 tenth by 1 hundredth.
Multiply1 hundred by 1 ten-thousandth.
Multiply43 thousandths by 21 ten-thousandths.

.01
.001
.01

.000090a
27.
28.

Multiply40000 by
Multiply.09375 by

1.064.

OF

DIVISION

148.

Division

of

quotientof

the

Placing the

149.

the

upon

.09975

DECIMALS.

is the

Decimals

numbers

two

.04

1 millionth.

process

of

finding

involving decimals.

decimal

pends
quotientde-

the

in

point

following
Principle.

The

number

the number

of

of

decimal

decimal

in the

orders

orders

in the

quotientis equal to

dividend^less

the

ber
num-

in the divisor.
of decimal
by .03; then, the number
For .006=:y^%^ and .0?)=:j|}o;
orders in the quotientwill be one.
the quotientof
as
then, the quotientof .006 by .03 will be the same
fore,
There; ^^^^^l-^,= :2.
I'o
To%o divided by ^f^. But, \o%o ifo

Thus, let .006 be

divided

.006

-^-

.03

.2, in which

there

is

one

decimal

order.

2125

5.

Examples.
1. Divide

150.
Solution.
There
decimal
orders

are

"

Divide

three

order in
in

the

quotientis 4.25.

2.125

as

decimal

in

by

.5.

simple numbers
orders

.5;hence, there

in
must

by

2.125, and
be two

one

decimal

quotient (Art. 149). Therefore,

the

operation.

.5)2.12 5
4.2 5

2. Divide

.048

Solution.

by

the numerator

in

.048, and

There

3. Divide

the

operation.

decimal

orders

.006).0

.006; hence,

in

in the

4 8

quotient(Art.

quotientis 8.

by

.3

orders

orders

48

(Art.134, 6)

three

are

decimal

decimal

no

Therefore

149).

the numerator

6.

three

will be

there

.006.

by

Divide

"

ARITHMETIC.

PRACTICAL

NEW

KAY'S

184

.004.
OPERATION.

Solution.

Annex

"

ciphersto .3; then

two

solve

.004

).800
75"

in Ex. 2.

as

4. Divide

Solution.

83.1

Annex

"

(Art. 140, 1) in
formed

Solution.

far

is

Solution.

or

order

in

by

Write

"

4 and

in Ex.

as

may

decimal
be per-

operation.

)8

3.1 00

20.7 7 5

1.

3.

one

475.(1

point between

more

to

ciphersto

in Ex.

as

the

carry

the dec-

division

as

operation.

3)2.1 10
,7 03-|-

1.

100.

4756

7, two

and

place the
placesfarther to

decimal
the

operation.

4.7 5 6

left

it is in 475.6.

Rem.

To

"

Rule.

2. Point

number
the

divide

1.

"

numerator

in

by

the

the division

that

wanted; then solve

6. Divide

the

ciphers to

solve

Annex

(Art.140, 1)

as

than

order

2.11

"

4.

two

exactly;then

5. Divide

imal

by

4.756

Divide

by

100

the

of the divisor
off as many
of orders in

divisor.

prefixa cipher;thus, .04756.

of the dividend by the


simple numbers.
orders in the quotient
as

numerator
as

in

decimal

the dividend

exceeds

the

number

FKACTIONS.

DECIMAL

Eem.
the

When

1.

"

same

as

the

the
number

of decimal

number
in the

185

orders in the

dividend

divisor, the quotientis

an

is

integer

(Ex. 2).
"

less than
make

When

2.

Rem.

the number

them

Rem.

the

same

When

8.

"

the
in

number
the

by annexing ciphersto
the

"

divisor.

point as

orders in the

divisor,for convenience

division is not

of decimal
any requirednumber
4.
To divide a decimal
Rem.

decimal

of decimal

many
If there be not

the

dividend

is

in

pointing off,
dividend
(Ex. 3).

exact, it may

be

continued

to

places(Ex. 5).
the
by 10, 100, 1000, etc., remove
placesto the left as there are ciphersin the
prefixciphers(Ex. 6, Rem).
enough figures,

liAY'S

186

PRACTICAL

NEW

30. Divide

.08

31. Divide

by 7.
11.1 by 32.76.
.0123 by 3.21.

80.

by

33. Divide

DECIMAL

.001

1.5

32. Divide

151.

ARITHMETIC.

COMPOUND

decimal

1. Reduce

.05

to

Rule

lower

.3388278

.00383177

denomination

I).

decimal

the

gal.to

NUMBERS.

is reduced

(Art. 63,
by multiplication

.2142857

of

pint.

orKRATION.

.06
Solution.

"

To

pint,multiplyby

reduce
4 and

.05

by

gal.to

decimal

result is .4

The

2.

the

of

pint.

.20

2_
.4

2. Reduce

.035

3. Reduce

.0075

4. Reduce

.005

5. Reduce

.00546875

the decimal

pk. to

of

bu. to the decimal

yd.

the

to

A.

decimal
the

to

of
of

a
a
an

pint.

.56

pt.

quart.

.24

qt.

inch.

decimal

of

To

find the

value

of

decimal

in

square
.875 sq. rd.

rod.

152.

.18 in.

integers(Art.

125).
1. Find

the

value

of .3125

bu. in

integers.

OPERATION.

Solution.

by 4;

the

"

To

result

reduce
is 1.25

.3125

pk.

bu. to
To

pecks,multiply
reduce
.25 pk. to

.8 12 5
4

quarts,multiplyby 8; the result is 2 qt. Therefore,


.3125 bu.

1.2 5 00

equals1 pk. 2 qt.

2:00
2. Find

the value

of .75

3. Find

the value

of

54

yd. in integers.
.3375 A. in integers.

fl. 3 in.
sq. rd.

DECIMAL

of .7 lb.

the value

4. Find

FRACTIONS.

187
in

Troy

integers.
8

the value

5. Find

bii. in

of .8125

6. Find

of .33625

decimal

division

by

is reduced

(Art. 63.

1. Reduce

.64

qt.

integers.
140 rd. 4 yd. 1 ft. 2.4
cwt. in integers.
33

153.

pk.

of .44 mi. in

the value

7. Find

pwt.

integers.
3

the value

oz.

pt. to

to

Rule

II).

the

decimal

in.

lb. 10

oz.

higher denomination

of

gallon.

OPERATION.

Solution.

"

To

gallon,divide by

reduce
2 and

pt. to the decimal

.64

by

4.

The

result is .08

of

2).6

gal.

).32

708
2. Reduce

.72

the

qt. to

decimal

of

bushel.

.0225
3. Reduce

.77

yd.

to

the

decimal

of

mile.
.0004375

4. Reduce

5. Reduce

6. Reduce

.25 pt. to

.6

pt. to

the

the

.7 rd. to the

decimal

decimal

decimal

Promiscuous

154.
per

1. What

yard,

2. What

yard, and

and

is the

of

of

of

bu.

mi.

gallon.
.03125

gal.

.009375

bu.

.0021875

mi.

bushel.

mile.

Examples.
cost

of 9

yd. flannel,at $0.40


S12.60.
yard?

yd., at S0.75 per


is the cost of 2.3 yd. of ribbon, at $0.45 per
1.5 yd., at $0,375 per yard?
$1.5975.
12

NEW

KAYS

188

3. What

PKACTICAL

is the

of

cost

ARITHMETIC.

16.25

yd.

cloth, tit $2.6875

of

$43.671875.

yard?

per

At

4.

$0.75

bushel, how

per

much

wheat

be

can

$35.25?

for

5. At

$2.5625

6. What

yard, how

per

much

cloth

be

can

will

lb. of

50

cwt.

hops

cost

7. What

per

pk.

will

cost, at

corn

$9.296875.
A.

13

115

sq.

of land

rd.

10.

for

At

per

bushel, how

much

$4.32

If 63

12.

Add

13.

Add

14.

From

15.

From

acre,

per

how

much

land

gal.of

wine

cost

$49,

what

From

.41

17. Find

subtract

yr.

subtract

da.

the

value

What

19.
mi.

is

per

At
135

the

of

115

sq. rd.

$360.88

2 ft. 10

in.

qt.
2

rd.

.5

pt.

ft. 4 in.

343

da.

hr.

48

in

hr.

min.

40

48

min.

sec.

integers.

3^d. 2

da.
ft. 3

13

hr. 48

in. of

min.

tubing,

yard?

$690.35

cost?

gal

hr.

.16

of .3 yr.

cost

be

pk.
bought

464

in.

.5 hr.

109

$0.16

will

3^d.,1.07 ft. and 8.92 in.


.75 qt.
.625 gal. and
1.53 yd. subtract
2 ft. 3.08

18.

A.

.34

.05

bu.

can

13

be

can

29

18

16.

corn

$9.296875?

$59,265?

11.

$17.28

cost, at

$237.06.
$0.3125

bought

17

of

qt.

acre?

9. At

for

bu.

14

bushel?

8. What

per

$21,125.

will

per

bought
38.4 yd.

$3.25

at

hundred-weight?
$0,625

bii.

47

$98.40 ?

for

at

boui^ht

$55.
per

long?

mile, what

is the

cost

of

road

$12027.19140625.

THEjMETRreiSYSTEM.

DEFINITIONS.

1. The

155.

meter, the
Eem.
at the

unit

The

"

Metric

French

close

obligatoryin

the

called

so

the

from

system is based.

originatedthis system

the

of

which

upon

is

System

last

centurj^,and

The

metric

of

its

weights and
in

use

measures

became

France

legal in nearly all


system is now
civilized countries, and, in several,it is making its way
rapidlyinto
1841.

in the United
In 1866, its use was
States,by
generaluse.
legalized,
in
act of Congress. It is
general use by scientific men
throughout

the world.

2. All
a

the

units

of

the

from

simple manner
1st. The

Meter

2d. The

Ar

other

the

measures

derived

are

in

Thus,

meter.

Length. It is the base


of the Metric System, and
is very
ionth
nearly one ten-mill(.0000001)part of the quadrant extendingthrough
Paris from
the equator to the pole.
is the

the

is

unit

side is 10
square whose
3d. The
is the
Liter
wdiose
which

contents

are

of

unit

of

Land

It is

Measure.

meters.

unit

of

capacity. It

equivalent to

cube

is

vessel

edge

of

It

is

the

.01

meter.

the

is .1 meter.

4th.

weight

The
of

Gram

is

the

cube

of pure

unit
water

of

Weight.
whose
edge is

(189)

190

KAY'S

NEW

name

of

each

the

unit

3. The

its relation

to

rilACTlCAL

Thus:

27

ARITHMETIC.

denomination
oi' the

1st. The

indicates

at

once

measure.

uf

luinies

the

louver dononiiiiutions

formed

of the unit the


by prefixingto the natiie
Latin
and dcci (.1).
mdli
ctntl (.01),
numerals
(.001),
of a
is one
thousandth
For example, a miUimeter
are

meter;

is
centigi-a^n

deciliter is

2d.

The

hundredth

one

tenth of

one

of

names

of

and

gram;

liter.

liiffhrrdrnoiiiinatidiis

the

arc

formed

miiiuTal.- (Iil;ii
by prefix!
Hi;- to tin- unii tin- a ml;
For
(10),heldo (100),/.v/o i l("n(" mimI inur'in i KMHXh.
i- I'^n nirt.T.-;
hrl;ii_,r,frri" ..nc
a
example, a dekameter
.

hundred

is one
liters;a kilocjrain

is ten thousand
"ni/ria7neter

4. Since

in the

etc., units
unit

of

Ist. A

of

Metric

System 10, 100.


denomination

lower

higher denomination,
number

places to the
multiplier.

is reduced

ination

right

number

the

left

ination
nom-

innny

ciphersin
iiKiriEu

the

drncnn-

jxiinfa.^

are

thai.

d,

d.s
j/oi/it

are

deciniai
there

as

make

i.owf.h

"i

to

1000,

it follows

drci/jud

is reduced

by removing the

places to

in

there

as

and

grams;

meters.

by removing the

2d.

tliousand

iiKinij

ciphersin

the

divisor.

OF

MEASURES

156.

The

the United

Rem.
3 ft. 3|

is the

Meter

States at 39.37

1." Its

in.,which

length is
may

also

unit

LENGTH.

of

length;

it is

legalin

inches.

littleless than

be remembered

as

1.1

the rule

or
nearly
yards,

of ike three threes.

THE

Rem.

The

2.
"

Rem.

"

If
shown

are

in the

graving
en-

oppositepage.

Standard

3.

its divisions

and

decimeter

the

on

SYSTEM.

METKIC

States,and copieshave

meters

have

been

furnished

provided by

l)een

to the several

the

United

states.

Table.
10

millimeters,marked

}0

centimeters

JO

decimeters

10

meters

10

dekameters

10

hektometers

10

kilometers

are

mm.,

centimeter, marked

cm.

and
kilometer.
chieflyused are the meter
tances;
meter, like the yard, is used in measuring cloth and short disthe kilometer
is used in measuring long distances.

Rem.
The

The

"

measures

1. Eeduce
Solution.

"Write

5.638
To

"

and

5638

orders farther to

to

m.

reduce

centimeters.

meters

centimeters, multiply by

to

3 and
8,
place the decimal point between
the right than it is in 5.638 (Art. 155, 4, 1st).

A71S, 563.8

2. Eeduee

3642.9

Solution.

"Write
orders

"

To

to

m.

reduce

and

36429

place the

to

decimal

kilometers, divide

To

reduce

1000.

"Write

Solution.
10

X 100

Dm.

"

by

it is in

3.6429

Solution.
100

1000.

point before

5.6 dm.

"

To

reduce

Write
them

1000.

Km.

to centimeters.
dekameters
427 and

to

annex

centimeters,multiply by
cipher (Ex. 1 ).
A71S. 4270

4. Reduce

cm.

point between 3 and 6, three


3642.9 (Art. 155, 4, 2d).
Ans.

4.27

two

kilometers.

meters

farther to the left than

3. Reduce

100.

cm.

to hektometers.

decimeters

56, prefixtwo

(Ex. 2).

to

hektometers, divide

ciphers,and

place the

by

10

decimal

A71S. .0056

Hm.

J92

PRACTICAL

NEW

RAY'S

5. Eeduce

30.75

6. Eeduce

4.5 Km.

7. Reduce

75

8. Reduce

.025

Dm.

9. Reduce

36.5

dm.

10. Reduce

at

119.6

to

meters.

to

m.

dm.

2.5

centimeters.

.365

Dm.

48750

cm.

MEASURE.

SQUARE

of Lund

unit

m.

.075

decimeters,

to

cm.

4500

dekameters.

to

OR

3075

meters.

to

is the

Ar

centimeters.

Km.

.4875

LAND

157. The

to

m.

mm.

ARITHMETIC.

Measure;

it is

legal

yards.

square

Table.
100 centars, marked
100

Rem.
is very
Rkm.

1.

"

An

For

its subdivisions

is 100

ar

square

meters, marked

other

surfaces,squares

measuring

Ha.

"

Xhe

m^,

hektar

of the

meter

and

used.

are

262.5

2.625

a.

to

centars.

2. Reduce

397.8

a.

to

hektars.

3. Reduce

2500

ca.

to

hektars.

4. Reduce

3.8

a.

to

Liter

quarts,Liquid

.25
380

unit

Capacity: it

of

is

Table.
cl.,are

marked
deciliter,

dl.

"*

liter,

"

1.

10 liters

"

""

Dl.

10 dekaliters

"

"

HI.

marked
centiliters,

10

deciliters

Ha.
ITji.
m^.

legalat

measure.

10

ca.

CAPACITY.

OF

is the

3.978

meters.

square

MEASURES

1.0567

a.

acres.

1. Reduce

158. The

marked

1 hcktar,

"

ars

nearly 2|
2."

1 ar,

ca., are

dekaliter,
1 hektoliter,

THE

1." The

Rem.

etc.,in
is used

the liter and

are

hektoliter.

nearlya quart; it is used in measuring milk, wine,


moderate
quantities.The hektoliter is about 2 bu. 3i pk.;it
in measuring grain,fruit,roots, etc.,in largequantities.

use

Instead

2.

Rem.

are

used

commonly

measures

103

SYSTEM.

liter is very

The

to

METRIC

"

of the milliliter and

the cubic centimeter

their
Rem.

and

the

it is customary
kiloliter,

the cubic meter

equivalents.
For measuring wood
3.

the ster is

"

(marked m^), which

used; it is a cubic

meter.

1. Eeduce

2.456

1. to

2. Reduce

873.5

1. to hektolitem

3. Eeduce

1.83

HI. to deciliters.

1830

4. Eeduce

2400

cl. to dekaliters.

2.4

5. Eeduce

1400

1. to

1.4 m^

centiliters.

cubic

MEASURES

159. The
15.432

Gram

is the

245.6
8.735

meters.

OF

unit

cl.
HI.
dl.
Dl.

WEIGHT.

of

Weight;

it is

legalat

grains.
Table.

kilogram,and
The gram
is used in mixing medicines, in weighing the
metric ton.
preciousmetals, and in all cases where great exactness is required.
is the usual
The kilogram or, as it is commonly called,the "kilo""
articles generally; it is very nearly
and coarse
weight for groceries
2i pounds Av.
The
metric ton is used for weighing hay and other
than our ton.
it is about 204 lb. more
heavy articles;
Rem.

"

The

weights commonly

"

Prac.

13.

used

are

the

gram,

194

KAY'S

NEW

PRACTICAL

1. Eediice

1428.06

2. Eeduco

.28

3. Reduce
4. lieduce
5. Reduce

IGO.

legaland

approximate

presentedin

the

The

"

1. How

"

in. which

reduced
are

to

is used in

In 4 meters
are

there

157.48

are

M.T.

65.4
1.927

are

of
in

eg.

Dg,

nominat
those decommon

solvingthe followingexamples.

4 times

in., 157.48

integers of higher
yd. 1 ft. 1.48 in.

1.7135

g.

yards,feet,etc., in

many

Solution.
39.37

legalvalue

Kg.

280

values

System which
following

Table

Note.

1.42806

Kg. to grams.
1713.5 Kg. to metric
tons.
.00654 Hg. to centigrams.
192.7 dg. to dekagrams.

The

are

kilograms.

g. to

of the Metric
use

ARITHMETIC.

in.

tions
denomina-

m.

THE

2. What

is the

METRIC

value

SYSTEM.

lb. in

of 36

I95

kilograms?
OPERATION.

In

as

are
are

many
contained

are

16.329

there

pounds
kilograms as 2.2046

Solution."

36

times

2.2 0 4 G

)3

( 1 G.3 2 9 +

2 2046

which

in 36

6.0 0 0 0

139540
1322

7 6

72640
66138
65020
44092
2 0 9 2 8 0

198414

is the

3. What
4. How

of 20

value
hektars

many

in

Km.

5. What

is the

value

of 49

m.?

6. What

is the

value

of 15

g.?

7. How

many

hektoliters

8. How

many

cords

9. How

many

long

and

10.

32

.5

in

square

in

and

how

1. What

234.5

rd. 4
9

bu.?

64.75-|-Ha.
yd. 3.13 in.

pwt. 15.48
14.8+

sters?

500

in

yards

roll of

is the

sq.

8.4536

gallons?

of

sum

127

gal.

cl,
1.

56.65

will

merchant

many

4. A

meters

block

thick

cubic

yd.

be

the

cost

of

45

Ha.

of

land,

at

$3.32

S14940.

3. A

how

m.

dl.,4.87 1.,1563

ar?

an

C.

Examples.

dl.?

2. What

HI.

paper
5.382

many

gr.

137.95

wide?

m.

1. are

161.

42

Miscellaneous

m.

acres?

160

mi.

12.4274

cost

meter?

of

paid $457.92
did he buy?
marble

$.864

what

.72

m.

for

cloth,at

S3

meter:

152.64

long,.48

is the

cost

m.

of the

wide,

and

marble

m.

.5
per

$5.'

196

RAY'S

5.

how

How

did

these

wood

for

8tcr?

of

SI. 195
in

oats

4685

each

sacks,

HI.

7496
of

Kg.

346.75

coffee

for

$194.18

what

kilogram?

per
nickel

$0.56
coin

5-cent
would

what
hiid

coins

that

was

of

stcrs

HI.?

diameter:

in

380

hektoliters

bought

The

8.

much

1.6

pay

ARITHMETIC.

bought

many

containing
7.

PRACTICAL

manufacturer

$454.10:
6.

NEW

in

weighs
be

the

and

g.

cm.

of

enough
in

meter

is

of

weight

make

to

row,

length

250

How

9.

much

lining
of

made

garment

1.85

6.5

m.

of

m.

wide

cloth

will
1.25

it

take

g.

for

wide?

m.

4.39+
How

10.
the

distance
A

11.

it

The

12.
how

for

the

22.4

wide

mm.

between
of

.715

two.
m.

the
of

is

towns

each,

length

mi.
of

72
one

rd.

difference

take

step.

yd.

1.7

the

similar
in.

for

metric

example
the

distance,

what

Km.:

13.24037

must

between
a

measures,

Km.

mm^.

to

18518

system
8

649.6

illustrate

common

substituting

and

To

Dayton,

to

96.56-|-

long

mm.

"

our

Cincinnati

cover?

steps

distance

Note.

29

distance

many

that

is

map

from

miles.

60

being

does

space

kilometers

many

m.

walk

steps.

and

system

may
and

be

given,

28.15

in.

162.

1.

Any

cent

per

of

number

is

yi^

is

so

msiuy

dredths
hun-

of it.

Thus,
Rem.

1 per cent

Per

"

2. The

Thus,

is read

two

number

i^from

sign of

3. In all
in

cent

of

per

the Latin per

is

cent

fiveper

1st. As

%,

centum,

read

is

cent

y^^,etc.

the hundred.

by

per cent.

cent.

operationswith

ways:

it,2 per

of

per
a

be

cent, it may

expressed

fraction; 2d.

common

As

decimal.

Thus

the

Rem.

1.

reduced

followingexpressionsare

"

to

Per

cent, which

is expressedas

equivalentexpressionsby

4i

4^ ^

Tqq"

equivalent:

Arts.

mixed
121

number,
and

142.

9
which

may

be reduced

to

^qqI also, i\ ^

.045.

no7)

may

be

Thus,

RAY'S

198

Express
decimals

the

following as

and

.10

^-^and

.15

yV

2.
8.

20%

and

.20

4.

30"^

f^

and

.30

5.

70

-1-

and

.50

6.

2i%

i^

and

.025

Rem.

ARITHMETIC.

fractions

common

and

as

10%

1.

PRACTICAL

NEW

2.--C()inmon

fractions

Art. 108, snul tluMi read

How

many

as

per cent.

cent

per

be

may

reduced

Thus, ^

to

.165 or 16"

equivalentto

are

hundredths

the

by

ol

following

fractions ?

163.

1. Percentage

embraces

the

various

operations

with

per cent.
2. In Percentage three

the

Base^ (2) the

3. The

Base

considered.
are
quantities
Rate^ and (3) the Percentage.

is the

number

upon

which

the

per

(1)
cent

is estimated.
4. The

fraction

common

5. The
cent
6.

may

is

Rate

be

or

as

Percentage

of the

Any

the
a

when

cent

per

expressed as

decimal.

is the

resujt of taking the

per

base.

two

found.

of

these
There

quantitiesbeing given, the


are

four

cases.

third

PEECENTAGE.

CASE

1. What

base

the

Given

164.

is

199

I.

and

the

rate,

to

find

the

centage.
per-

of 32?

25%

OPERATION.

SoLUTiox."

i of

25^0is i (Art.162).

2. What

is

32 is 8.

2b"f^ ^
=

of 162?

7%

OPERATION.

Solution."

^^is

.07

(Art.162). Multiplying 1G2 by

16

.0 7

.07,the result is 11.34.

1 1.3 4

Rule.

the

Multiplythe

"

base

by

the rate; the

product will

be

percentage.

Rem.
or

"

Whether
as

rate

expressionis
given example.
3. What

is

4. What

is

5. What

is

6. What

is

7. What

is

8. What

is

9. What

is

10. What

is

11. What

is

12. Wliat

is

What

is

14. What

is

15.

What

is

16.

What

is

best

should

must

be

which

is

decimal,

of

13.

the

1%
2%
3%
^%
3f%
4%
5%
H%
^%
H%
8%
8i%
10%
12i%

tion,
frac-

be

expressedas a common
of judgment. That
matter

simplestor

most

convenient

form
in

the

of 278?

2.78

of 180?

3.6

of 97?

2.91

of 165?

5.5

of 240?

9.

of 140?

5.6

of 118?

5.9

150?

8.

of 250?

15.

of 450?

30.

of 11?

,88

of 384?

32.

of

57?

5.7

of 292?

36.5

of

HAY'S

200

17. What

18. What

19. What

"

20. What

21. What

What

28. What

24. What

25. What

26. What

"

27. What

"

28. What

29. What

30. What

What

22.

31.

15% of 95?
17% of 53.4?
of 11.2?
m%
20% of 9.85 ?
25% of 43?
33J% of 0.93?
45% of 5.7 ?
50% of 38.75?
i% of 456 ?
8% of 464?
of 144?
A%
125% of 36?
208% of 650?
450% of 12?
1000% of 24.75?

CASE

the

Given

165.

AUITUMETIC.

PRACTICAL

NKW

base

14.25
9.078
2.1

1.97
10.75
2.31
2.565
19.375
2.28
1.74
.63

45.
1352.
54.
247.5

II.

and

the

percentage, to

find the

rate.

1. What

per

Solution."

2. What

per

of 8 is 2?

cent

OPERATION.

is

of 8

cent

(Art.120). \

of 56

is

25^^.

is 3.5?
OPERATION.

Solution.
IS

"

Explanation.
as

"

One

.56 is contained

-R^lQ,

result

3.5

.0625 is 6] %.

.0625.

cent

Dividing 3.5 by 56, the

"

1. Divide

will be the rate.

-^

5 6 "..0 6 2 5

.0 6 2 5
per cent
times

the

of 56 is

.56; then

3.5 is

=r

as

6] ^^
many

per

in 3.5.

percentageby

the

base;

the

quotient

3. What

per

4. Wlaat

per

5. What

per

6. What

per

7. What

per

8. What

per

9. What

per

10. What

per

11. What

per

12. What

per

13. What

per

14. What

per

15. What

per

16. What

per

17. What

per

18. What

per

Given

166.

the

and

rate

the

percentage

to

the

find

base.

1.

15

is

number?

of what

25%

OPERATION.

Solution."
of

^,is | (Art.162).

is

4.93

is 4 X

number

the

number,

some

2.

25

17%

of what

Since
15

15 is

25^,
15X4

00,

60

number?
OPERATION.

Solution.
number

(Art.162). Since some


multiplied])y.17 givesthe product 4.93,

the number

Rule.

"

17

oi is .17

is 4.93 divided

"

Divide

will be the base.

the

bv

.17

or

17/,=.17
4.9 3 -t-.I 7

29

29.

percentage by

the

rate; the quotient

202

167.

RAY'S

NEW

Given

the

base and

of the

PRACTICAL

ARITHMETIC.

the

and

rate

sum

find

percentage, to

or

the

the

difference

base.

number, plus 35% of itself,


equals675:

1. A

what

is

the number?
OPERATION.

Solution.
itself

.35 of

number

the

2. A

The

of

equals 1.35
is 675, and

divided

is G75

the

35 % is .35.

"

number,

35

plus
1

it; then, 1.35 of


number

the

by 1.35, or

number

itself

=.3

5=:1.3
-f-.3

6 75-^-1.3

5 00

500.

minus

5%

of

w^hat is

itself,
equals 57:

number?
OPERATION.

Solution."

minus
c!^is 2V- The number
^^^
of itself equals ^" of it; then, ig of the number
is 57,
3

2^0of

it is 3, and

the

number

is 20

Rule.

the

uV

"

"

Divide

is

number?

the

1 minus

"

=^

^i^
2^

,^ X H

difference
by
3. 721

\%

times

CO.

sum

by

plus the rate, or

the rate; the

3^ greater than

quotientwill

certain

divide

60

the

be the base

number;

what

is
700.

PERCENTAGE.

is

4. 68

less than

66%

5. What

203

number?

what

increased

number,

200.

of

by 25%

amounts
itself,

2125?

to

1700.

6. What

diminished

number,

of

by 6%

is equal
itself,

7.52?

to

8.

7. 8250

37J% greater than what number?


fraction,less 10% of itself,
equalsf ?
number?
than what
20% more

is

8. What
9. 6.6

is

168.

Formulas

Let

Four

for the
the

represent
Then,

Cases
the

base, r

Case

I.

Case

II.

Case

III.

]) -^

Case

IV.

^""==6. *-^^=

b X

J) ^b

how

much

2. A
per

did

3. A

rate, and

j^ the

centage.
per-

6.

\"r

S800

Examples.

in bank

drew

and

of it:

36%

out

$512.

I left?

had

man

cent

$3000

had

Percentage.

^=b.

1. I had

5.5

=p.

Miscellaneous

169.

-:f^.

=^r.
r

1 +

of

6000.

$300;

he

have

merchant

after

40%

in the bank

25%.
deposits,leaving

of his

what

what

spent $225,

left?

withdrew

remaining

had

he

did

amount

draw?
he with-

'

$2000.
4. A

grain

40%

was

dealer
than

more

sold

for 56

corn

it cost

him:

ct.

what

bushel, which
the

was

bushel?
5. A

than

cost

per

40 ct.
man

the

sold
horse

cost:

horse
what

for $175, which


did

the

horse

was

cost?

12J%

less

$200.

G. A

12^^
7. A

owed

9. A

than

more

invest

cent

per

of 250

farm

his

in bonds?

then

were

many

takes

for toll 6

ground:

what

per

12. A

farmer

which
were

was

there

13. When

the

cent

real

A.

222

in

35%

how

much

112

sq. rd.

one

year:
216.

qt. from
a

every

5 bu. of

he take?

does

ow^ning45% of
G0% of what

tract

he

of

3i%-

land,sold

owned:

in the tract?

gold dollar

in

flock?

miller

11. A

15%.
bonds,

rd.,which

sq.

neighbor owned:

in the

of

$15000.

A. 86

neighbor own?
flock of IGO sheep increased

10. A

acres

owned

he

his

did

wheat

did

much

farmer

12^%

acres,

what

paid $425:

75% of his estate in


it, amounting to $5000,

of

remainder

how

estate:

how

$500;

remains

the

land

$91.87^.
lie

unpaid?
speculatorinvested

8. A

was

pound?

per

man

the debt

and

ARITHMETIC.

PRACTICAL

grocer bought 4 sacks of coffee of 75 pounds Ccach;


what
lost by waste:
the remainder
wortli
was
was

35 cents

at

NEW

KAYS

204

how

many
2000

is worth

7%
gold

more

540

than

A.
the

in
are
$371.29 in
greenback dollar, how^ much
$347.
greenbacks w^orth?
he spends 18% of it for
'4. A's salaryis $800 a year;
and
rent, 15% for clothing,
12% for
23% for provisions,
sundries:
how
much
does he save
$25G.
annually?
answered
15. A pupil at an
examination
17 of the 20
?
: what
questionscorrectly
per cent did he make
85^
16. 2 bu. 3 pk. are
33J% of what number?
8 bu. 1 pk.
number
of pupilsattending school on
17. The
a
tain
cer37; this w^as
day was
7|% less than the number
.

enrolled

how

many

w^ere

enrolled

40.

gold dollar weighs 25.8 grains Troy; 10% of


tain
grains of pure gold does it conalloy: how many

18. A
it is

23.22

PERCENTAGE.

19. The
G.

20.

than
how

of

of whicli

grams,

copj^er: what

him

it cost

per

per

75%.

sold

man

1.25

is the

copper?

many

cost

piece weiglis5

tlie remainder

nicl":eland

are

cent

five-cent

205

horse

for

if he

had

a
:

sold

it have

would

cent

$150, which
the

25%

was

horse

been

more

him?

for

more

$200,

than

it

66f%.

APPLICATIONS

OF

PERCENTAGE.

DEFINITIONS.

1. The

170.

into
of

two

those

time; (2)
2. The

of Percentage may
applications
classes:
(1) Those without the
with

the

element

be

vided
di-

element

of time.

of the first class


importantapplications
are
(1) Mercantile Transactions,
(2) Stock Transactions.
3. The most
of the second
class
important applications
are
(1) Interest;(2) Discount.
4. Percentageenters
less extent
into
to a greater or
the calculations
of Exchange,Insurance, Taxes, Equation
of Payments,etc.
5. The principles
of Percentage apply directlyto application
with
the fol^of the first class in accordance
lowing
most

"

pr"^
.

Jyi/

GENERAL

RULE.

Rule.

"

1.

Ascertain

base, percentage, and


2. Note
3.

Apply

the
the

the
their

quantitieswhich
sum

or

their

correspondto

difference.

required.
quantitiesgiven and the quantities
ample.
of Percentageto the given exproper case

KAYS

206

NEW

AKITHMKTIO.

PKACTICAL

TRANSACTIONS.

MERCANTILE

DEFINITIONS.

171.

1. Mercantile

and

to

the

chase
pur-

sale of merchandise.
is the value

2. Price

3. Merchandise
retail

relate

Transactions

is

of any

bought

thing in

and

mone}^
sold at wholesale and

at

priceof

in

prices.

4. The

priceis

wholesale

the

merchandise

quantities.
5. The
small

price is
quantities.

the

"Wholesale

buy

Rem.

retail

merchants

prices. Retail dealers


the

to

6.

users

The

or

chief

price of

distribute
of

consumers

Mercantile

merchandise

sell merchandise

and

merchandise

sale
whole-

at

of every

in

tion,
descrip-

it,at retail prices.


Transactions

involving an
(1) Commission; (2) Trade

of Percentageare
application
and Loss.
Discount,and (3) Pj^ofit
Rem.

Wholesale

"

merchants

buy

and

or
through agents, who receive salaries,
buyers at wholesale are sometimes
purchases;and merchants
usuallymake

their

sell merchandise

largely

commission, for their

allowed
a

ices;
serv-

discounts

or
profit,

upon their
sufier a loss,in

transactions.

COMMISSION.

172.

1.

business
Rem.
called

"

of
The

the

2. A

An

agent

is

person

intrusted

with

the

another.

person

who

employs

the

agent, in reference

to

him, is

principal.

commission

for another.

merchant

buys

and

sells merchandise

COMMISSION.

Kem.
his

"

own

and

1.

factor is

control

Kem.

2.

termed

agent who

an

is intrusted

and

name,

207

his

by

and

buys

sells merchandise

with
principal

the

in

possession

of it.
The

"

the

to whom

person

merchandise

is sent

consignee;the

while

3. The

who
sends
person
itself is called
the merchandise

the

is

commission

it is termed
a

sold

the

is

signor;
con-

consignment.

paid

sum

to be

for

agent

an

transactingbusiness.
4. The

charges

are

incurred

expenses

by

in

agent

an

transactingbusiness.
5. The

net

the
6. The

commission

value

is the
the

net

is the

proceeds
of

and

the

proceedsis

base

business

in the

commission

the

after deducting

charges.

materials

base; the

remaining

sum

acted
trans-

is the

percentage;
percentage.

less the

and

Examples.
1. An
much

agent whose

upon

2. An

sale of

auctioneer

goods amounting
did he

receives
much

4. An

received

$460

to

for

$11.50
what

$240?

to

commission

$8.12^
per

for

5. What

of

2^^
selling25

barrel

did

agent receives

deducting his
expend?

commission

are

the

$210

barrels

which

with

of

commission

of

molasses:

commission

for

$13.

molasses?

sell the

he

merchant

commission

to

buy goods :

what

5^

must

sum

$200.
net

proceeds

on

sale

amounting to $180, at 4% commission?


6. A lawyer received
a
$11.25 for collecting
debt?

lot of

sellinga
cent

per

|12.

2^%.

after
he

goods amounting

how

receives

5^,

receive?

3. At

how

is

commission

being 5^,

what

was

the

amount

of

goods

$172.80.
debt:

his

of

the

$225.

KAY'S

2(18

7. An
and

NEW

PKACTICAL

receives

agent

commission

8%

at

ARITHMETIC.

$1323.54
:

what

to

of

cost

cover

goods

is his commission?

$98.04.
8. A

commission

bbl.; 175
at 25
feathers,

bbl.

per

does

sum

he

merchant

ct.

remit

2. A

from

$5178.83.

DISCOUNT.

be

may

assumed

an

price is

net

pork, at $15

the owner?

1. Merchandise

discount

bbl.

flour,at $7 per bbl.; and 1456 lb.


is 3%: what
per lb.;his commission

TRADE

173.

sells 250

sold

or
list,

fixed

at

price or

net

at

regular,
price.

pricefrom

which

discount

no

is allowed.
3. A

list,or

regular, price is

by

seller

the

as

established

an

basis upon

which

sumed
price,as-

to

calculate

discounts.

The

4.

is the deduction

discount

from

the

or
list,

ular,
reg-

price.
Rem.
to

time

In

1.

"

trade,the

the wholesale

(1

purchaserdepends upon

of discount

amount

) the amount

and
purchased,

granted
(2 ) the

of payment.

Rem.

In

2.

"

of
price-list

lines of

some

the

goods

the discounts

dealer; in others,

from

the

are

made

from

of
price-current

the
the

market.

regard to time, sellingfor cash means


payment as
be delivered.
the goods can
as
soon
in a certhat the payments are to be made
tain
Time
purchasesmeans
time after the purchase the time varies with different lines of
goods.
Rem.

3.

"

In

"

5. The
or

as

so

Thus, 20

price.

discount
many

is

expressedas

so

many

per

cent

off

off.

off,or
(fc

20

off,means

at

discount

of 20 %

from

the

DISCOUNT.

TRADE

6. There

2d. Two

off.

5 cL off

and

Thus, the expression20


The

Thus,

The
off

7. The
the

12^ cj^off;\ and

means

the

priceof

5 off

the

the

fraction.
5 off.

of

sum

price of

the

of

discounts.

common

base ;

less the

base

and

and

discount

successive

33i

means

percentage;

pricepaid is

or

three

seller is the

is the

or

the remainder.

from

5^

expressedas

is sometimes

per cent

discounts

buyer

of

the

Rem."

first,a

means,

price,and then a discount


expression25, 10, and 5 cL oif,means
from

20^

5^,

as

discounts.

successive

more

or

singlediscount;

1st. A

be:

may

209

all
the

percentage.

Examples.

bill of

1. A

amounted

goods

being allowed, what


being allowed i and
3. I paid $1430.75
discount

5. Sold

discount

doz.

20

of 10,

$325.20, how

to

6.

for

Bought

of
the

of

face

he

$459.27.

pay?

which
groceries,
bill: what

cost

Avas

40, 10, and

the

$390.45

at

5 off: what

and

25

$548.

feather

10, and

much

100

was

$1475.

goods
list price?

the

did

lot

$180.40.

goods?

bill?

bill of

4. A

for

off

$725.16,the purchaser

to

off,what

the

from

of the

amount

was

for the

bill of articles amounted

2. A

3%

paid

was

$225.50 list; 20%

to

10%

was

doz.

my

his discounts

price per

stay bindings,at
off:

7^^

the

dusters,giving

what

did

purchaser
amounting
$60.

dozen?

60

I pay

ct. per

dozen,

for them?

$29.97.
7. A
dozen

retail dealer
for $50

for them

in

what

he

did

buys

and
list,

the
pay

usual
per

case

of

slates

containing 10

gets off 50, 10, and 10^ ; paying


additional 2^ :
time, he gets an
dozen

for

the

slates?

$1.98.

HAY'S

210

PROFIT

1. The

174.
2. The
Rem.

The

"

and

the

the

is the

consumer

to the

cost

i)rice
paid for goods.
the price received
for goods.

retail dealer

chant;
sellingpriceof the meris the selling
price of the

dealer.

wholesale

3. Goods

usuallysold

are

4. The

they

is

price

to

cost

LOSS.

AND

is the

cost

selling

ARITHMETIC,

PRACTICAL

NEW

profit is what

at

the

profitor

goods

at

loss.

sell for

than

more

cost.

5. The

loss

is what

cost

is the

the

goods

sell for less than

they

cost.

6. The

percentage;and
of the base

the

base; the profitor

sellingprice is

and

the

the
sum

loss
or

is the

the

ence
differ-

percentage.
Examples.

merchant's

1. A

$40

is

10%

2. Prints

yard:

for how

that

much
6

cost

is the

what

3. A

profiton

per

does

ct.

he

yard

cloth which

piece of

sell it?

sold

arc

$44.
for 5

of loss?

cent

cost

ct.

16f%.
pound,

coflPee at 27 cents per


by retailing
24
gains 12^% : w^hat did it cost per pound?
4. Sellinga lot of goods at a loss of 4%, the loss
the entire lot was
$15.30: what did the goods cost?
grocer,

ct.
on

$382.50.
5. To

make

profitof 37^%,

dry -goodsmerchant
6. A

75

ct.

sell shawls

bookseller

sells

w^hat is his

gain per

7. What

is the

cost

profityieldsa profitof

at

what

price must

that cost $8?

grammar

$11.

for 90 ct. which

cent?

of tea, which, when


5 ct. per

pound

sold

at

cost

20%.
6J%
80 ct.

PROFIT

8. A

of 18f%
profit

what

price per yard


of 16f% ?

the

was

would

211

barrel,making

per

have

$4.

cost?

lost

yard, and

$1.35 per

silk at

9. Sold

apples at $4.75

sells

grocer

LOSS.

AND

sold

10^

it to

what

at

make

profit

'

$1.75.

peddlerbought a stock of goods


disposedof them at a profitof 25% : how
10. A

11. If

sells for $1.75:

sold

gained 25%,
he

did

much

13. A

of cloth
the

the

cloth

money

on

on

an

the

by

atlas,which

he

lost

25%

he

16f%.
one

on

how

$20.

transaction?
the

5 ct. per

yard, and

10%

other

the

reduced

from

ct.

is his per cent of profit?


horses for $150 each ;
two

and

lose

25

merchant

cloth

on

makes

what

dealer

12. A
he

much

$218.50.

bookseller

and

$874,

make?

he

did

for

price of a certain piece


his profit
thereby reduced

to

8%

what

was

the

cost

of

$2.50.
yard?
14. A
speculatorbought 10000 bushels of corn, at 60
ct. per bushel; in a few days, corn
advancing in price,
then
he sold 7000 bushels,at 65 ct. per bushel ; corn
at 55 ct!
fallingin price,he disposedof the remainder
of the
per bushel : what
per cent profitdid he get out
per

transaction?
15. A

'^i%.

speculatorin

real

estate

for

$12000, which

sale afforded

the

cost; he then

invested

he
he

was

lose

obligedto
by the two

sell at

him

sold

house

and

lot

profit of 33J% on
the $12000
in city lots,which
did
loss of 33 J % : how
much
a

$1000.

transactions?

Miscellaneous

Examples.

lisher
purchasesbooks from the pubat 20% off the list price;if he retail them
at the
list price,what wnll be his per cent of profit?
25%.
175.

1. A

bookseller

KAY'S

212

2. A

lb.,at 2^
advance, what will

ct. per

12^^

ARITHMETIC.

5 luilf-chests of

bought

grocer

45

at

PRACTICAL

NEW

$4.25

at

4. Sold

had

received

12^^

374

or

retail it at

^20.12.
profit?
men's
boots,containing

his

of

discount

ct.

each

on

each,

be

off for cash ; I retail

pair: what is my profit?


of hats containing 3 dozen, on

case

if he

lb.

cash

of
Bought 5 assorted cases
pairs each, for $45 per case, 5^

them

of 74

off for

8.

12

tea

10^

hat:

and

what

made

the

was

$41.25.
which

profitof
wholesale

merchant's

$120.
j)rice
per case?
5. A merchant
bought 100 packs of pins,of 12 papers
each, for $1.00 per pack, 00, 5 and 5% off; if he retail
of $23.90,lor how
them
much
to make
as
a profit
so
a

paper will he sell them?


6. I sent

merchant

$9.50
of

$17.25:
did

7. A

$125

8. A
of

York

flour,100 bbl.,to

commission

he

I make?

contractor

apiece;the

$10450:

of

disposed of the flour at


was
barrel,his commission
2-^% with charges
flour cost
if the
$7.50 per barrel,how
me

was

hhd.

New

in

per

much

car-load

5 ct.

$159.

bought 80 horses for government, at


mission
$200, and the agent'scomfreightwas

such

what

per

that
cent

was

merchant

commission
sugar,

the

1500

lb. each

horses
the

cost

the

commission?
sells

government

2^%-

consignment of 50
net, at 10^ ct. per pound;
signor
charges $22.50; the cona

2% and
clears 14^ by the transaction:
what
did he pay
9 ct.
per pound for the sugar?
9. A dealer in notions buys 60 gross shoe-strings,
at 70
for 50, 10 and
^t. per gross, list,
5% off;if he sell them
what will be his profit? $10.77.
at 20, 10 and 5% off list,
buttons
for 25, 10 and
10. Bought 50 gross of rubber
5% off; disposedof the lot for $35.91,at a profitof 12^ :
the list priceof the buttons per gross?
what
$1.00.
was
his

commission

is

TRANSACTIONS.

STOCK

213

TRANSACTIONS.

STOCK

DEFINITIONS.

17G.

1. Stock

sale of

and

Rem.
other

stocks,bonds,

the

to

purchase

gold.

form

of transferable

shares.

capitalof banks, of railroad,insurance,telegraph and


companies is held in this way.
The

"

3. The

Stockholders

share

4. A

Rem.

share

"

is

the

are

of the

owners

stock.

usually^100.

is sometimes

quoted, in the

are

and

the

capitalin

is

2. Stock

relate

Transactions

New

$50

York

or

other

some

market,

number.

invariablyas

$100

Stocks
to

the

share.

5. A

is

bond

certain

of money

sum

Rem.

1.

Bonds

"

at

given

Rem.

rate

the notes

are

corporate bodies which


bear

promise, under

written

are

of interest and

seal,to

pay

time.
specified
of the Government
allowed
are

to issue

payable

and

ous
of the vari-

them; usuallythey

within

specifiedtime.

In

2.

quoting United States bonds, the different issues are


1st. By the rate of interest;as 6's,5's,4J's,
distinguished,
4's; 2d. By
the time at which
they mature; as 5-20's, which are payable in 20
be paid after 5 years.
The
5-20's
also distinguished
are
years, but may
by the date of their issue,as 5-20's of 1868. Bonds of the
"

Funded

Loan

Rem.

o.

company
"

Adams

6.

Rem.

"

bear

The

5%

bonds

interest,and

later

ones

4J and

4^.

of the
corporationstake the name
issues
which
them; as, "Chicago and Northwestern,"
Union
Express," Western
Telegraph,"etc.

of

local

"

Currency
"

National'

is the

It consists
Bank

of

notes.

paper

money

notes,
legal-tender

of the

called

country.
"greenbacks,"and

KArS

214

7. The

given
Hem.

stocks

bonds

and

is the

quotationsfor stocks, bonds, and gold are

The

the currency

8. The

ARITHMETIC.

value

face of them.

the

"

of

value

par

on

PRACTICAL

KEW

all based

on

dollar.

Stock

chief

transactions

involvingan application
of Percentageare
(1) Brokerage^
(2) Assessments and
Dividends^(3) Stock Values^and (4) Stock Investments.

BROKERAGE.

177.

1. A

is

broker

who

agent

an

and

buys

sells

stocks,bonds, gold,etc.
Rem.

Persons

"

broken^;the

buy and sell stocks


by the "operator."
is the

2. Brokerage

business,and

the
3. The

in stocks

"operate"

par

value

paid

sum

the

is calculated
is the

usually do

in kind

latter

authorized

are

who

on

and

through
as
they

so

amount

for transacting

broker

the par

value.

hase^the brokerage

the

2"^r-

centage.
Examples.
1. A

broker

and

bought

Hudson

for

Eiver

stock

shares

75

me
:

New

York

requiredthe brokerage

J%.
brokerage

Eock

Island

for

stock

buying

$6.25

was

shares

50
:

what

of
was

cent?
3. At
an

Chicago
the

per

1^.
\% brokerage a

investment

buy?

at

$18.75.

2. The
and

tral
Cen-

in bank

broker
stock

received

how

many

$10

for

shares

making
did

he
40.

ASSESSMENTS

broker

4. A

AND

shares

17

buys

DIVIDENDS.

215

Milwaukee

and

St. Paul

preferredstock : what is his brokerage,at J^


of Vermont
95 shares
5. The
brokerage on
is the per cent?
stock is $11.87^: what
received

broker

6. A

$9.50,or

Pacific

buying Union
purchase?

stock

the

An

1.

Central

-J^.

shares

many

did

for
he
38.

DIVIDENDS.

AND

ASSESSMENTS

178.

$4.25.

brokerage of ^%,

how

is

assessment

of money

sum

paid by

stockholders.

Kem.

In

"

the formation

of

business,the stock subscribed


assessments

are

require. The
2. A

Rem.

made

from

is

The

"

transaction

time

to

said to be

business; the

receiptsafter deducting all


of the net earnings.

are

company

earnings

net

but

holders.
stock-

in
receipts

its total
are

the

to

the

is left of the

what

dividends

The

expenses.

at once;

in installments.

paid

money

of any

of the business

the needs

as

paid for

of

sum

transaction

usuallyall paid for

is not

earningsof

gross

of the

for the

company

time

stock is then

dividend

paid

are

out

Examples.
1. I

own

dividend

2. A

man

insurance
3. A

what

35

of

mining
Mr.

bank

of

4^,

stock;
will

what

of

assessment

an

pays
stock:

does

shares

how

shares

many

declares

company
Jones

receive

$140.

7^%,
does

or

he

$300, on

his

own?

40.
of

dividend
owns

80

15^

shares

of

$1200.

stock?
4. A

clare
de-

I receive?

who

bank

if the

man

60

owns

declare
company
much
stock will

he

shares

dividend
then

of

own?

of

railroad

stock

if the

5^ payable in stock,how
03

shares.

KAY'S

216

5. A

gas

NEW

PRACTICAL

has

company

what

does

cent

per

capitalstock

$15700, and

earnings are

gross

1. The

is the

priceat

market

$lt)0000; its

the

$4500

ally
annu-

stockholders?

1%-

VALUES.

of

value

which

of

its expenses

it pay

STOCK

179,

ARITHMETIC.

stocks,bunds,

and

gold

they sell.

is above pnr, or at a premium, when


it sells for more
it sells
than the par value; stock is below jxir, or at a disconni, when
for less than the par value.
Rem.

Stock

"

2. The

value

market

estimated

at

certain

of

per

and

stocks, bonds,
of the

cent

is

gold

])ar value.

gold dollar is worth 106J^ of


New
York
tral
Centhe currency
dollar,or is at a premium of 6| ^.
and Hudson
that the stock of this railroad
River, 91 J,"means
sells for 91^ ol of the par value, or is at a discount of Sh 4).
that the

Thus, "gold, 106^,"means

"

3. The

the

value

is the

percentage;the

market

par

base; the premium


value,the amount

or

or

discount

is

difference.

Examples.
1. What

Harlem,

will be

the

of

cost

150

shares

($50 each)

of

$10500.
139|, brokerage \% ?
2. Bought $8000
in gold at 110, brokerage ^^ : what
did I pay for the gold in currency?
$8810
broker
sells 50 shares
3. My
of Chicago and
North
$2475 : at what
western, brokerage ^% ; he remits me
per

cent

4. What

Bonds
5. I

of

at

did

the

stock

will be

the

sell?
cost

49f^.
of

25

1000-dollar

1867, at 114J, brokeragei%

paid $1560
brokerage\% : how

for

Milwaukee

many

shares

did I

U.

S.

$28593.75.

"

5-20

St.

Paul,

buy?

at

19i
80.

STOCK

6. When

gold is

INVESTMENTS.

105,

at

what

217

is the

value

in

of

gold

in

dollar

currency?
^^2T ^^'
the value
of a
7. When
at 112J, what
was
gold was
dollar in currency?
88|-ct.
worth
8. In 1864, the
only
"greenback" doilar was
the price of gold?
280.
35f ct. in gold: what wa^
age
9. A merchant
paid $8946.25 for gold, at 105, brokera

l^")w much

\^c

sells

broker

My

10.

did

gold

"

he

buy?

certain

$8500.
of

amount

$25734.37^? His brokerage,at


the price of the gold?
was
$15,621: what

remits

me

STOCK

180.

1.

The

gold,and
xV%' ^^"
103.

INVESTMENTS.

is the

income

annual

profitfrom

the

investment.
The

Eem;"
coin

its

or

of

States

United

the

is

bonds

in

equivalent.

2. The
is the

most

from

income

cost

the

of

investment

base; the income

is the

'percentage.
Examples.

$39900

1. If I invest

be

be

my

6^

bonds,

at

$39900

in

6% bonds,

at

105,

ifI

$2280.
$39900

invest

in

6%

bonds, at 95, what

IT. S.

6^

5. What

will be

$2520.

income?
4. What

will

what

income?

my

will

what

par,

$^394.

income?
my
2. If I invest

3.

in

is

bonds,

when

gold is ^105?

gold is

in currency

income

investing $5220

income

man's

in

U.

8. 5-20

who

20

owns

$1284.

107?

would

1000-dollar

receive

man

6^ bonds,

at

by

116, when
^283.50.

NEW

RAY'S

218

What

6.

108,
7.

If

Central

8.

What

9.

How

of

G^
what

37^,

Michigan

on

of

cent

per

investment?

my

of

16^.

S.

U.

in

income

of

S's
in

$1921

1881,

118,

at

when

currency,

$40120.
shares

many

brokerage

105,

at

cents,

per

113?

at

was

but

me

on

annual

an

4g%.
dividend

invested

sum

S.

105?

cost

receive

U.

do

annual

an

which

do

yielded

gold

receive

ARITHMETIC.

income
is

gold

stock,

income

at

when

of

cent

per

yield

PRACTICAL

of

\%

stock

bought

each

on

and

95J,

at

will

transaction,

sold

yield

"^

of

profit
10.

12.

be

must

of

U.

S.
will

be

bought

If

bonds

6^

to

realize

an

75^.

income
can

for

paid

8^^?

When

yearly
that

100.

What

income
11.

$925?

87

pay

income

of

bonds

4%
be

received

for
for

in

from

gold

what

iOG,

at

the

bonds

$188.

$4982?
railroad

w^hat

7^,

quoted

are

bonds

per

that
do

cent

yield
get

on

an

nual
an-

vestment?
in-

my

%%.
13.

annual
realize

What
income

6%?

could
of

afford

7^

to
to

pay

invest

for

my

bonds

money

yielding
so

an

as

to

116f.

DEFINITIONS.

181.
Rem.

1. Interest

The

"

2. The

is

3. The

is the

The

borrower

Rem.
"from

the

"When

date"

the

lender

allowed

are

his

promise

is made

to

draw

frequentlyinserted
of the

interest

note

is

pay

for

note

the

to

pay

IG, 1877.
Clu^*lesThomas,

interest

at

James
note

to

terest.
in-

O., June

dollars,with

hundred

face

Legal

which

form:

common

promise

received.

"

of

use

principal and

written

always gives

after date

year

5. The
6.

of

Dayton,

order, five

value

lender.

time.
specified

$500.00.

or

for the

money

is

at

following is

The

One

to the

borrower

sum

note

of money

sum

money.

the

paid by

of money.

use

paid.

amount

"

for the

paid

is the

promissory

certain

Rem.

is

principal

interest

4. A

interest

is money

interest

from

after the word

is the

interest

Q.

8^,,for
Dean.

date, the words


"interest."

principal.
at

per

cent

b}^ law.
(219)

that

is

220

KAY'S

Rem.
and

NEW

PRACTICAL

ARITHMETIC.

The

per cent of interest that is legalin the different States


is exhibited in the following
Territories,
"

TABLE.

NAME

OF

STATE.

NAME

Alabama
Arizona

60/, 10/.
lOf, Any.
Nebraska
10/. 12/.
Nevada
10/. Any.
New
Hampshire... 6/.
New Jersey
6/.
New
Mexico
6/. 12/.
Montana

Any.
6%

California

Any.

Colorado

Anv.

Connecticut
Dakota

12 fo

Delaware

New

6^

Idaho

Indiana
Iowa
Kansas

Kentucky
Louisiana
-^

Maryland
Massachusetts

Michigan
Minnesota

Mississippi

When

law.

the per

cent

the

first column

If

stipulatedin

in the second

8/.
12/.

Rhode

Island

South

Carolina

6/. Any.
7/. Any.
Tennessee
10%
6/.
Texas
8/. 12/.
7"/o 12/.
United States
6/.
6^
Utah
10/. Any.
.."T.
Vermont
Any.
6/.
Virginia
0/.
Wash. Territory...10/. Any.
6/.
7fc lOfc West Virginia
7/. 10/.
12/, Wisconsin
Wyoming
10%
12/. Anv.

lOfc

Maine

6/f

Oregon
Pennsylvania

lOfc

Illinois

Carolina

Ohio

Any.
7"foAny.

Georgia

York

North

District Columbia.
Florida

STATE.

MiBSouri

8^"

Arkansas

OF

column

mentioned

of interest is not

gives the
the note,
mav

per

cent

per

cent

be collected.

that

in the note
may

of interest

or

tract,
con-

be collected
as

high

as

by

that

INTEREST.

SIMPLE

7. Usury
that

than
Rem.

"

charging interest
allowed
by law.
is

It will

be

from

seen

at

table

nearly half the

in

abolished
practically

the

221

cent

per

above

that

usury is
Territories.

States and

subject of Interest may be divided


(2) Compound Interest^
(3) Annual
Simple Interest^
(4) Partial Payments.
8. The

SIMPLE

182.

1. Simple

greater

into

now

(1)

Inter est

INTEREST.

is interest

Interest

the

on

principal

onl}^
Simple interest
is due.
principal
Rem.

"

2. In

(1) the
the

Simple Interest
principjal,
(2) the

due

four
per

three of these

Any

not

can

be collected

till the

quantitiesarc considered,
and (4)
cent, (3) the tirne,

be found.

There

quantitiesbeing given,the
are

183.

Given

find the

1st.

the

r.

the
principal,

time

and

the per

cent,

interest.

When

1. Find

fourth

Jive cases.

CASE

to

and

interest.

3.

may

is not

the

the

time

interest

is

one

of $25

7jear,

for 1 yr., at

6%.
OPKnATlON.

SoLtJTioN."
one

year

^^ is

!f)6

(Art.162, 3). Then,

is the unit of time, the interest

$25 X. 06 ^.$1.50.

since

2 5

for 1 yr. is

.0 6

IJO

NEW

KAY'S

222

2. Find

the

PRACTICAL

interest

ARITHMETIC.

of 818.75

for 1 yr., at

6"%.

OPKRATION.

15)18.75(1.25
15

2). Then, the interest for


15

3, Rem.

6|^ois ^, (Art.162,

Solution."

1 yr. is $18.75

3 7
3 0

-i-

7 5

$1.25.

r=

V b

3. Find

the

of S215

amount

for 1 yr., at

G%.
OPERATION.

$2 15
The

Solution."
U

interest of $215

$12.90; then, the

for 1 yr. ni 6

is $2\ry -\-$V2/J0

amount

.0 6

fo

1 2.9 0

2 1 5

$227.90.

$ 2 2 7.9 0
Hule.

Rem.

Multiplythe principal
by the

"

"

Find

To

find the amount

the

add the

rate.

and
principal

interest.

interest
V

4. Of

$200

5. Of

$150

6. Of

7. Of

Find

$85 for 1 yr., at


$7200

S%.

$16.00.

5^.

$7.50.

7%.

$5.95.

for 1 yr., at

GJ^.

$450.

the amount

8. Of $28.20
9. Of

$45.50

10. Of

$420

11. Of

$857

12. Of
13.

for 1 yr., at
for 1 yr., at

Of

14. Of

for 1 yr., at
for 1 yr., at
for 1 yr., at
for 1 yr., at

$96 for 1 yr., at


$2000 for 1 yr., at
$164

for 1 yr.. at

8J%.
10%.
^%.
9%.
8^%.

U^.
m%.

$30.55.
$50.05.

$442.40.
$934.13.
$104.16.

$2090.
$184.50.

SIMPLE

2d.

When

1. Find

the

SonjTioN.

the

time

$3.50; then, the

is Two

of $50

interest

The

"

INTEKEST.

223

More

or

for 3 yr., at

Years.

7%.

interest of $50 for 1 yr., at 7^, is


interest for 3 yr. is $3.50X3=

operation.

$^0
.0 7

$10.50.

3.5 0

Rem.

is sometimes

It

"

multiply the per cent


above example, the per
2. Find

convenient

more

and

time

the

$ 1 0.5 0

together. In

for 3 yr. is 21.

cent

the amount

first to

of

$225.18 for 3 yr., at

4^%.

OPERATION.

Solution."

The

interest of $225.18

is $10.1331;
A\c/ci
S

$10.1331 X

then, the

and

$30.3993;

$30.3993 -I-$225.18

interest

for \ yr., at
for 3 yr. is

$ 2 2 5.1 8

is

9 0 0 7 2

the

amount

^0

$255.58.

112

4|

5 9

roX3~3T
Rem.

In

"

business,it is customary

final result to the

ample,
nearest

Rule.

by

the

57

cents

58

"

9 mills

3. Of
4. Of

5. Of
6. Of

7. Of
Find
8. Of
9. Of

and

Thus, in the

3 tenths

of

mill

the

the interest

of

for

one

2 2 5.1 8

are

year.

9 3

2 5 5.5 7 9 3

this
3Iultiply

years.

interest

S65

for 4 yr., at

5%.
$300 for 2 yr., at 6%.
$275 for 3 yr., at 6%.
$187.50 for 4 yr., at 5%.
$233.80 for 10 yr., at 6%.

the

3 0.39

1. Find

the

ex-

cents.

givennumber

Find

unit.

nearest

take

to

$13.

$36.
$49.50.
$37.50.
$140.28.

amount

$45

for 2 yr., at 8%.


$80 for 4 yr., at 7%.

$52.20.
$102.40.

KAY'S

224

PRACTICAL

NEW

ARITHMETIC.

11. Of

$237.16 for 2 yr., at 3f%.


$74.75 for 5 yr., at 4%.

12. Of

$85.45

13. Of

$325

14. Of

$129.36

15. Of

$8745

10. Of

1. Find

$105.96.
$377.65.

for 4 yr., at

the

the

$89.70.

lor 4 yr., at 6%.


for 3 yr., at 5|%.

4f%.
8%.

for 2 yr., at

When

3d,

$254.95.

Time

interest

$10144.20.

J^iunhev

is atiy

of $24

$152.

for 9

of Months.
6%.

nio., at

OPBRATION.

I.

Solution

$24 for 1 yr.,


mo.

is

II."

which

the

Rule.

"

part of

is

mo.

of 6

36

The

mo.

interest of
for 9

h of
est
inter-

then, the

of $1.41, whieh
mo.

is 72

is \ of 72 et.,

Then, the interest

ct.

mo.

mo.

for 9

mo.

the interest for

one

year.

given number

of

7honths

=r$1.08.

1. Find
this

The

arc

mo.

is $1.44;
65^,

interest for 8

is 36 ct.

is 72 ct. +

year.

is $1.08.

(Art. 130)6

are
year, and 3 mo.
of $24 for 1 yr., at

ct.,and

ji

6%, i^ $1.44; then, the interest

at

interest for 6

ijof

are

mo.

J of $1.44, which

Solution
u

"

as

the

Take
is

such

part of

a
a

year.

Find

the interest

2. Of

$300

for 1 mo.,

at

3. Of

$240

for 2

mo.,

at

4. Of

at

5. Of

$ 50 for 5 mo.,
$ 86 for 3 mo.,

6. Of

$ 50

for 4

mo.,

at

at

6%.
S%.
6%.
6%.
8%.

$1.50.
$3.20.
$1.25.
$1.29.
$1.33.

SIMPLE

Find

the

INTEREST.

amount

7. Of

S150.25

for

G mo.,

at

8. Of

S360

for

7 mo.,

at

9. Of

$204

for 11

mo.,

at

10. Of

$228

for

mo.,

at

11. Of

$137.50 for

mo.,

at

12. Of

S759()

mo.,

at

1. Find

10

for

When

^th.

225

the

Time

$370.50.

7^.

$217.09.
$238.26.

6%.
6%.
8%.

$143.00.
$8102.40.

JWoinher

is any

the interest of $288

$156.26,

8%.
5%.

for 24

da., at

of Days,

5^.
OPERATION.

"288

Solution

da.

I." 24

of $288

for 1 mo.,

interest for 24 da. is

4 of

are

terest
in-

$1.20; then, the

5^^^is

at

The

month.

| of $1.20, which

.0 5
1 2

is 90 ct.

) 1 4.4 0
5 ) 1.2 0
.2 4

_4
.9li
Solution
of

(Art.130)

month, 6 da.

da.

II."

at

mo.

of 6 da.

are

15 da. is

the

which

The

5^, is $1.20;
of

-i

interest

is 12

da. is 60

Rule.

part of this

month, and

as

ct.

288
.0 5

then, the interest for

-[-12

)1 4:40

is 24 ct.,
of

interest
ct.

"

^Q

24

ct.,

for 24
.9 6

ct.

the interest for


the

1 2

terest
is 60 ct.; the in-

Then, the
24

OPERATION.

interest of $288 for

of $1 .20,which

1. Pind

"

for 3 da. is

ct.

ct.

of

$1.20, which

for 6 da. is
and

are

da.

are

15

given number

one

Take

month.

of days

is

such

part of

a
a

month.
Rem.

"

In

computing interest,it is customary

1 month.
Prac. 15.

to

regard 80 days as

RAY'S

22G

Find

the

ARITHMETIC.

PRACTICAL

NEW

interest

2. Of

S360

for 20

at 6%.
dii.,

$1.20,

3. Of

S726

for 10

tit
dti.,

$1.21

4. Of

$1200

for 15

5. Of

$180

for 19

da.,at
da.,at 8%.

6. Of

$240

for 27

7. Of

$320

for 21

8. Of

$450

for 25

Find

$3.00
76

da., at 7%.
da., at 5%.
da., at 10%.

ct

$1.26
93 ct.

$3.13.

the amount
$100.80

for 28

10. Of

$150

for

11. Of

$360

for 11

12. Of

$264

for

13. Of

$900

14. Of

$430

9. Of

Sth.

6^.
6^.

When

the
or

$264.40.

for 14

da.,at 7^.

$902.45.

for 19

da., at 4^%.

$431.02.

Time

is Tears, Months,

any

18

the

$150.38.

6^.

Two

these

of

First
1. Find

$101.19.

da., at 5%.
da., at 5%.
da., at
da., at 6%.

and

Days,

Periods.

Method.

of

interest

$360.66.

$360

for 2 yr. 7

da.,

25

mo.

8%.

at

OPERATION.

$360
Solution

I." The

interest of $360

1 2 ) 2 8.8 0

for

.0 8

$28.80; then, for 2 yr.


the interest is $57.60; for 7 mo., or /^

2 8.8 0

1 yr., at

of

year, the interest is


da., or f of a month,

25

$2.
25

8^^,is

$16.80; and
the

Then, the interest for


da. is $57.60 +

$16.80 + $2

for

5 7.6 0

interest is

1 6.8 0

yr. 7
=

mo.

$76.40.

2.0 0

$7 6.4 0

2.4 0

7_
$16.8

) 2.4 0
M)
5

$2.0 0

simple:

INTEKE8T.

227

OPERATION.

Solution

II."

The

(Art. 130).

of $360 for 1 yr., at

is

8^^,

est
inter-

28.80, and

$?

for

half of a
or
yr. it is $57.60; for 6 mo.,
year, the interest is $14.40, and for 1 mo.,
2

^ of 6 mo., it is $2.40; for 15 da., or ^ of


month, the interest is $1.20, and for 10

or
a

da.,or

interest

of

month, it is 80 ct.

for

yr. 7

25

mo.

7.6 0

Then, the

da. is $57.60 -f

$14.40 + $2.40 + $1.20 + $0.80

mo.

4.4 0

mo.

2.4 0

da.

=:

1.2 0

15

$76.40.

10 da.

=1

.8 0

$7 6.4 0
I.- -1. Fhid

Rule

the interest

for

each

period,and

add

the results.
Second
2. Find

the

of

interest

/^

Method.
$120

for

yr.

20

mo.

da.,

6%.

at

OPERATION.
3 0

Solution.

6|

mo.

of

are

of $120

6^c,will

at

da.

20

"

are

of

Q2
a

"

S
2_0.

mo.;

terest
Then, the inyear.
20 da.,
for 4 yi-. 6 mo.
a

be

$120 X

-06X41

$32.80.
40

.02

32.80

Rule

of

II.

"

Reduce

the months

and

days

fraction

year.

2.

Midtiplythe principalby the rate, and


product by the time expressedin years.
Kem.

to the

"

Indicate

the

operationas

far

as

is

multiplythe

and employ
practicable,

cancellation.

3. Find

the

interest

of $150

for 4 yr.

mo.,

at

6^.
$37.50.

KAY'S

228

Find

ARITHMETIC.

the interest of

4.

S375.40 for 1 yr. 8 mo.,

5.

$ 92.75

6.

$500

7.

fit

for 3 yr. 5 mo.,

at

S37.54.

G^.
6^.

$19.01.

mo.

18

da., at

6^.

$560

for 1 yr. 1
for 2 yr. 4

mo.

15

8%.

$106.40.

8.

$750

for 4 yr. 3

mo.

6^.

$192.00.

9.

$456

mo.

18

10.

$216

for 3 yr. 5
for 5 yr. 7

da.,at
da.,at
da.,at

mo.

27

11.

$380

for 3 yr. 9

mo.

Find

tlie amount

13.
14.

$205.25

15.

$150.62

16.

$210.25 for 2 yr. 7


$ 57.85 for 2 yr. 3

17.

18. Find

for 2 yr. 8
for 3 yr. 5

the

interest

April 19, 1849,


Rem.

"

To

19. The

at

find the time

interest

$34.00.

5%.

$79.04.

da., at
da., at 15%.

$122.22.

6%.
6%.

$366.00.

10^.

$215.18.

of

$300 for 3 yr. 8 mo.,


$250 for 1 yr. 7 mo.,

12.

to

PRACTICAL

NEW

of

at
at

$273.75.

mo.

15

da., at 6%.

mo.

12

da.,at

mo.

20

mo.

23

da., at 7%.
da., at 5%.

of

$150, from

5^. $176.60.

interest of

28, 1845, at
21. The

$20.50.

between

$240,

dates, see

two

from

Art. 77.

February 15, 1848, to

$180, from

$23.04.

May 14, 1843, to August


$28.84.

of

$137.50,from

July 3, to

28, at
March
24.

amount

of

$125.40, from

March

amount

9, 1848, at
The

amount

1, to August
$130.64.

Si%.

23. The

November

$4.95.

27, at 9^.
22. The

$64.54.

6%.

7^.

interest

$249.09.

January 9, 1847,

April 27, 1849, at 8^.


20. The

$238.60.

of

$234.60, from

August 2, 1847,

$242.02.

5J^.
of $153.80,

July 24, 1847, at 5%.

to

from

October

25, 1846,

to

$159.55.

SIMPLE

twelve

The

184.

Explanation.

it is 2

1 ct.; for 2 mo.,

cents

there

as

it is 2

there

Rule.

of $1 for
are
days.

Call

"

findinginterest.

for any

time,

any

at

3 mo.,

it is 3 et.,etc.

number

of months,

is $0,001,
12^/^,

da.,at

da.,it is 3 mills,etc.

interest
as

of

12%,

at

\2(ij,is $0.01, or
Hence,
VloL, is

at

as

many

mofiths.

are

mills; for

The
mills

ct.; for

interest of $1 for 3

The

229

interest of $1 for 1 mo.,

of $1 for

ifiterest

The

of SI

interest

The

"

method

cent

per

find the

Ist. To

INTEREST.

any

the months

number

or

mill; for

da.,

Hence,

of days,at 12^,is J

as

yna.ny

cents, and, one-third of the days

mills.

Rem.

Find

"

Reduce

the

months.

to

years

of

interest

$1, at

1. For

mo.

12

da.

2. For

mo.

18

da.

12^,
$0,094
$0,046,

3. For

mo.

12

da.

$0,074

4. For

mo.

da.

$0,091

5. For

1 yr.

mo.

6. For

1 yr.

mo.

27

da.

$0,179,

7. For

2 yr.

mo.

21

da.

$0,277

8. For

3 yr.

mo.

12

da.

$0,434

9. For

yr.

mo.

15

da.

$0,505

10. For

mo.

da.

11. For

mo.

17

da.

12. For

10

mo.

13

da.

$0.16,

1 yr.

mo.

da.

For

yr.

mo.

20

da.

15. For

yr.

mo.

29

da.

13. For
14

$0.020J
$0.055f
$0.104J
$0.141J
$0.336"
$0.419"

RAY'S

230

To

2d.
per

NEW

PRACTICAL

find the

interest

ARITHMETIC.

of

$1,

for

time

any

at

any

cent.

1. Find

the interest

Solution.

mo.

terest, at 69^, will he

2. Find

SI, for

2 yr. 5

18

mo.

6q^ is i of 12^. The interest of $1


is $0,296; then, the in18 da.,at 12^^,

"

for 2 yr. 5

of

^ of $0,296, which

the interest of

operation.

is $0,148.

$1, for

yr. 7

da.,at

20

mo.

).29 G
~ATs'

da., at

operation.

Solution."
3 yr. 7

est,at

20

mo.

8^0is ^ of Ufc The interest of


is $0.436";then, the
da.,at 12^^,
be

will
Sfo,

$0.43G",which

of

$1 for

3) .4 3 6^

TTSJ

inter-

is $0,291^.

.29lj
-1.

Find

this

as

the

interest

Rule.

fart of
Find

the interest,
at

the

givenper
of

cent

12^, and
is

take

such

of 12%.

81,

For

mo.

24

da.,at 6%.

$0,039.

For

10

mo.

15

?0.043f.

For

11

mo.

18

da., at 5%.
da., at 9^.

For

For

yr-

For

$0,087.

mo.

da.. at

$0.07 U.

mo.

12

da., at

$0.1 9G.

yi*- 10

mo.

17

10^.

For

yi^

mo.

11

da., at
da., at

$0.388yV
$0.299|f.

10. For

yi'-

mo.

24

da.. at

4^.

$0,226.

3d. To
any

per

find the
cent.

interest

of

any

sum

for

any

time, at

INTEREST.

SIMPLE

1. Find

the

Solution.

for 1 yr.

S25,

interest of

The

"

of

interest

231

$1 for 1 yr. 5

18

mo.

da., at

18

mo.

da., at 12t/c,is $0,176; then, at 6%, it is $0,088.


$2.20.
Then, the interest of $25 will be $0,088 X 25
==:

2. Find

the

of

interest

$134.45, for

1 yr. 7

da.,

15

mo.

S%'

at

OPERATION.

yr. 7
at

Then,

it is $0.13.

Sfc,

for

$1

is $0,195; then,
1^^^.,

da., at

15

mo.

of

interest

The

Solution."

$134.45 will be $.13 X

the

134.45

).l9

4.45

$13

.0 6 5

interest of

J_3
403

1 3445

.13 0

$17.48.

35

$ 1 7.4 7 8 5
Rule.

the

"

1. Fhid

the

multiplythis by

of $1, and

interest

given sum.

Rem.

"

Find

Take

the

3. Of
4. Of

5. Of
6. Of

7. Of

either f^ictor for the

$40,
$50,
$120,
$200,
$500,

for 6

mo.

21

for 8

mo.

24

for 10

mo.

for 11

mo.

$7.28

15

da., at 6%.

$11.50

$48.75, for

1 yr.

10. Of

$7G.32, for

1 yr.

10

Of

12. Of

$1.34

12

9. Of

11.

convenient.

da., at 6%.
da., at 9^.
da., at 7%.

8. Of

the

is most

interest

for 1 yr. 3
$750, for 1 yr. 5

Find

multiplieras

mo.

mo.

27

da., at
25

mo.

$19

da., at 3%.
da.,at S%.
3

mo.

$3.30

$89.50
$5.14

6%.

da., at

4^.

$5.81

amount

$600,
$900,

for 2 yr. 1
for 2 yr. 4

mo.
mo.

da., at 5%.
10 da., at 6%.

$663.25.
$1027.50.

PRACTICAL

NEW

RAY'S

232

yr. 7

14. Of

S86.25,for 2
S450, for 3

yr.

15. Of

$534.78, for

13. Of

ARITHMETIC.

17

da., at 9%. $10G.G7.


13 da., at 8%.
$565.30.
22 da., at 4%.

mo.

mo.

yr.

mo.

S609.17.

S1200, for

16. Of

3 yr. 11

CASE

Given

185.

1. The

the

for

of $225

the

was

and

jkm* cent

the

terest,
in-

time.

interest

$66: what

$1675.

da., at 10%.

II.

the
principal,

the

find

to

15

mo.

certain

time, at 4%,

was

time?
OPERATION.

The

Solution."

4^,

yr., at
terest

times

for

is

interest of $225

$9;

as

years

many
in 66, which

what

$225

then, $66 will be the in-

as

2. In

fori

is

is contained

7 yr. 4

7J,or

7J

$9.0 0

mo.

7J

will $500

amount

time, at 10%,

9)66

.0 4

yr.

7 yr. 4

to

mo.

$800?

OPERATION.

800
-The

Solution.

will be

then, $300

is contained

50

will

he

$800

interest of $500

The

$300.

interest

for 1 yr., at
interest for as many

the

times

^^^

$500=

"

^^^

is $50;
10^^,

in 300, which

10

as

years

) 5 00
^0

is 6.

50)T00
6

3. In

what

time,

at

8%,

will

any

principaldouble

itself?
Solution.
doubled

8^
many

in

principal

itself when
100

comes

"

the

Since

^.

yr., it will

years

in 100, whicli

as

the
be

interest
interest
100 f^ in

is contained

is VJ.\.

..r

V2

vr.

has
be-

mo.

8)100

is

as

times
6

operation.

12^

yr.

12

yr. 6

mo.

2^

INTEEEST.

SIMPLE

Rule.

for
prineipal

Rem.

"

the

If there be

"

it to months

4. I lent

what

the interest

oj the

and

amount

given, subtract

are

the

to find the interest,

fractional

part of

in

year

the

duce
result,re-

daj^s.

$200, at 6%,

the

long was
5. In

principaland

the amount

principalfrom
2.

given interest by

year.

one

If

1.

Kem.

the

1. Divide

"

233

received

and

how

$36 interest:
3

lent?

money

will $60

time, at 5%,

amount

yr.

$72?

to

4 yr.
6. In

what

time, at

will any

be
principal

doubled

16 yr. 8
7. A
how

lent

man

at

8%,

received

and

what

mo.

$90 interest:
3

it lent?

long was

8. In

$375,

time, at 9%,

will $600

amount

to

what

time,

at

10%,

will

10

long will

it take

$250, at 6%,
2

interest?
11. How

long

will it take

$60,

12. How

was

6%?
13^ The

it take

amount

on

$400,

at

7%,

treble

2 yr. 5

In what

time, at 9%,

will $700

amount
3 yr. 6

15. How

one-half, at

long
8% ?

16. In whut

will

time, at

it take

any

itself,

yr. 4
how

$68.60:

was

to

27 da.

mo.

33

interest

18 da.

mo.

to

to
principal

any

it loaned?
14.

yr. 3

3 yr. 9

long will

12

mo.

mo.

"

amount
3 yr. 11

long
da.

24

da.

increase

principalto

$1200

mo.

$924.70?

to

6 yr. 3

10^;,will

yr.

yield $34.50

to

6%,

at

$73.77 ?

at

mo.

principaldouble

any

itself?
10. How

yr.

$798?

3 yr. 8
9. In

to
mo.

mo.

$1675?
15 da.

KAY'S

234

PKACTICAL

NEW

ARITHMETIC.

CASE

find the

to

the

Given

186.

1. A

the
principal,

time

the

and

interest,

cent.

per

merchant

for 1 yr. 8

III.

interest for the

paid $30
what

mo.:

the per

was

of

use

S300,

cent?
OPKRATION.

Solution."

1 yr. 8

mo.

Since

the interest for

terest

for 1 yr. is $18.

1|, or J

are

1 yr. 8

yr.

yr. is $30, the in$18 is ^3^of $300;

mo.

-Y-X |

"

yr.
18

^y*^ -^^
=r:

'

^^

Qfc (Art. 162).

are

what

2. At
in 20

of 100

Bule.

fc
=

what

this is

per

4. A

the interest

of

$310: what

was

was

what

the
per

and

paid $48

interest:

of

use

paid

cent

will

$1000

yr. 4

8^.
da.

24

mo.

cent?

cent?

cent?

for the

per cent?

for 2

per

what

per

the

was

of

the

interest

$250
per

interest

find what

year, and

one

4^.

was

in 12 yr. 6 mo.?
8. The
amount
was

100-^2

pay?

of
the

for

for 2 years

what

mo.:

amount

7. At

the

operation.

principal.

paid $200

5. The

what

the

did

broker

6. $23.40

itself

doubled

100 o^. Since

become

$000

cent

for 2 yr. 6

itself

%.

3. I borrowed

yr.

Qfc'

yr. is 100 o^, the interest for 1 yr.

1. Find

"

cent

per

principaldouble

any

principalhas

"

the interest has

^V

will

cent

per

interest for 20
is

yr?

Solution.
when

i^jf

was

10%.
for the

use

of

$260 for 2

4^%any

principaldouble

itself

S%.
$175

for 3 yr. 7

mo.

was

$250.25:

12%.

INTEREST.

SIMPLE

9. The

of

interest

is the

$61.20: what
what

10. At

12. The

the per

was

of

interest

the

double
principal

for

$650

da.

yr.

da. is

18

mo.

$640

6^.

for 6 yr.

was

what

$110.40:

cent?

2^%-

the

Given

is

itself in

cent?

per

CASE

187.

12

mo.

9%.

of

is the

yr.

8%.

will any

per cent
10 da.?

$746.20: what

cent?

per

11 yr. 1 mo.
11. The
amount

for

8450

235

time,

IV.

and

cent

per

find

interest,to

principal.

1. The

interest

for

3^-.,at

is $27:

6%,

what

is the

?
principal
Solution."

^0 is -5%(Art.162, 3).

Since

for
of the principal,
-5^^
2 yr. it is -^-^
of the principal. Then, ,^ of the
principalare $27, 2V ^^ ^^" principalis $9, and
the principal
is $225.

operation.

the interest for 1 yr. is

2. The

interest for 3 yr., at

9%,

J^^X f

-j3_
2^5

225

X ^"^
=^

is the

is $21.60: what

?
principal
Solution.
for

"

^oL

is .09.

Since

the

interest

for 3
principal,
mul.27,the principal. Then, the principal,
tipliedby .27,is $21.60, and the principalis
1 yr. is

$21. 60

"

Rule.

interest
3. The

.09,the

.27

"

9 oL =r.09
.09

3 =.27

$21. 60---. 27

80

$80.

Multiply the rate


by the product.
interest

?
principal

operation.

yr. it is

by

for 3 yr., at

the

5%,

time, and

is $8.25:

divide

what

the

is the

$55.

4. The

the

PRACTICAL

NEW

RAY'S

236

for 3

interest

ARITHMETIC.

$841.25: wliat is

is

5%,

yr., at

$2275.

princi2)al?

5. The

for

interest

1 yr. 4

at

mo.,

6%,

is $2.20: what

is the

principal?
What
at 5%,
principal,

6.

$28.25.
will

produce a yearlyinterest
$20475.

of $1023.75?
7. What
in 1 yr. 6
8. What

27

mo.

20

da.?

interest

for

mo.

9. The

is the

$9.41: what
10. The

interest
is the

$28.38: what

the

interest

$240.
will

the

produce $525.40

interest

$474.40.
2

yr.

11

mo.

da., at

4%,

for

yr.

is

$90.

principal?
24

mo.

da., at

6%,

principal?

CASE

Given

produce $30.24

da. ?

at 9%,
principal,

in 12 yr. 3

188.

will

at 8%,
principal,

is

$82.50.

V.

time, per cent,

and

amount

G%,

will

find

to

principal.

1. What

principalin

yr., at

amount

to

$650?
Solution.
1 yr. is

"

6^

is

-^jj.Since
for
principal,

the interest for

5 yr. it is j\ of
3^0""f*the
the principal,
and
the amount
is \^ of the
principal. Then, }| of the principalare $650,
y^^of the principalis $50, and the principalis

amount

"

Multiplythe rate by
by 1 -\-the j)roduct.

2. What

$435?

(jcL

^-^
A
,3yXf
}g -f fV^if
=

50

IMx^$='jOO

$500.

Rule.

operation.

principalin

the

yr., at

time, and

5%,

will

divide

amount

the

to

$300.

INTP]REST.

COMPOUND

3. The
the

5%,

yr., at

is $571.20:

what

interest?

4. The

the

for 4

amount

237

S95.20.
for 6 yr., at

amount

is $532.50:

7%,

interest?

5. The
what

6. The
is the

for

yr.

mo.,

at

is

8%,

$285.48:

6%,

at

mo.,

is $690:

interest?
for

is the

is the

189, Formulas

yr.

24

mo.

da.,

yr.

27

mo.

da., at

interest?

for the five

I.

Case

II.

Case

TIT.

Case

lY.

Case

Y.

-f-

(6 X 0

t)
i -^{rXt)
~

Rem.

1.

"

r.

'

b,

INTEREST.

of the

tlie

Sometimes

^"

"

(^-X 0

the

Interest

addition

rate,and

^--^"^-^^^b.
1

COMPOUND

yearlyb}^the

the

Xt=^i.

(i^

Compound

is

of Interest.

cases

b Xr

4%,

$134.94.

t the time, r
represent the principal,
interest.
Then,

Case

is

$520.

In

7%,

at

principal?

for

amount

$914.94: what

190.

what

$90.

amount

8. The

is

$234.

for 2 yr.

$643.76: what

i the

j)rincipal?

amount

7. The

Let

what

$157.50.

amount

is the

is

principalis increased

interest.

interest

is

added

semi-amiually,or

quarterly.
Rem.

the end
with
note.

2.

"

The

of each
face

way

year,

equal

to

in which
to

both

take

the

interest is
up

the

old

legallycompounded is,at
and give a new
note
one

principaland

interest

of the

former

'

1. Find

PRACTICAL

NEW

KAY'S

238

the

compound

ARITHMETIC.

interest

of

for 3

$300

3^r., at

6%.
Solution.

$18; the

The

for

of

$318

is

$19.08; th'

$10.08

amount

is

$3 3 7.0 8

0 6

.0 6
2 0.2 2 4 8

3 00

3 3 7.0 8

357^3048

-3X8.

for

.0 6

$20.2248;

3 0 0

19.08

$57.3048

is $20.2248 +

$357.3048.

$300

"

firstyearj

is

$57.30.

the

Find

1.

"

Then,

interest

compound

Kule.

$3 0 0

18.0

$337.08.

6^^,

$357.3048

the

6^^,

is

amount

the

$18+

interest

interest of $337.08

$337.08

operation.

yr., at

$318

yr., at

the

is

amount

$318.

r^

The

6^, is

for 1 yr., at

of $300

$300

interest

The

"

and

amount

make

of

it the

the

for
givenprincipal
principalfor the second

year,
2. Find
make

the amount

it the

given number
3. From

the last amount


will

remainder

Rem.

of this principalfor the


principalfor the third year, and
of yearns.

When

1.

be

the

the

subtract the

compound

second
so

on

year^

for

the

givenprincipal
; the

interest.

interest is

or
payable half-yearly,
quarterly,
find the interest for a half, or a quarter year, and proceed in other
respectsas when the interest is payable 3'^early.
the time
Rem.
is years, months, and days, find the
2. When
"

"

for

amount

months

and

the

the interest
compute
years, then
days,and add it to the last amount.

Find

the

2. Of

$500,

for 3 years.

3. Of

$800,

for 4 3^ears.

amount,

at

on

this for the

6%, compound interest,


S595.51.

$1009.98.

INTEREST.

ANNUAL

Find

the

4. Of

$250,
$300,
$200,

5. Of
6. Of

compound

239

interest

for 3 yr., at
for 4 yr., at

$47.75.
6%.
$64.65.
5%.
6%, pajablesemi-annually.

for 2 yr., at

$25.10.
7. Find

the

of

amount

for 2 yr., at

$500,

20%

interest,payable quarterly.
is the

8. What

$738.73.
of

for 2 yr.

$300,

6%?

at

mo.,

interest

compound

9.

What

mo.

15

$47.19.
is the

da., at

10. What

pound
com-

interest

compound
6%?

is the

semi-annually,for
11. What

is the

compound

interest

3 yr. 6 mo.,

at

at

$500, for

interest

compound

$762.52.

6%?

between

diiference
on

for 2 yr.

$1000,

$171.35.

of $620

amount

of

simple interest

4 yr. 8 mo.,

at

and

6%

$16.49.

ANNUAL

191.
and

Annual

each

on

Rem.

1.

Rem.

2.

reads

with
3.
"

notes; these
and

4.

interest

annual

are

given

The

"

annual

interest

in 4 yr. 8
at

mo.

24

the

on

principal,

semi-annual

quarterly.

or

be collected when

the note

or

bond,

payable annually."

The

interest is sometimes
at the

time

same

interest

having

on

as

representedby interest
the note for the principal,

due.

paid when

interest notes, called coujwns;


presentedfor payment when

1. ]^o

interest

after it is due.

interest

interest may

interest if not

draw

Rem.

annual

Annual

"

Rem.

is

Interest

This interest is sometimes

"

"

INTEREST.

bonds

these

are

is sometimes
detached

from

representedby
the bond

and

the interest is due.

been

da., on

6%, payable annually.

paid, find
note

for

the

$400,

amount

with

due

interest

KAY'S

240

Solution.

NEW

The

"

PKACTICAL

ARITHMETIC.

interest of

24 da.,at
for 4 yr. 8 mo.
terest
6^,is $113.60. One annual in-

$400

of $400, at
first annual

The
mains

unpaid

6^c"is

$24.

interest

re-

yr. 8

24

mo.

da.; the second, 2 yr. 8


24 da.; the third, 1 yr. 8
da., and

24
24

the

mo.

The

fourth, 8

da.;

for 8 yr. 11
this is $12,864.

-f $113.60 -I-$100

2. Find

annual

no

each

to the

interest
on

interest

will
principal,

for

for

the

been

be the amount

paid,find

S800, with

amount

Interest

at

in

having
5

yr.

on

been
a

due

in 6 yr.

on

paid

note

10%, payableannually.
interest having been
4. No

interest

due.

the amount
at

due,

due

8%, payable
S1007.36.

interest
due

of

unpaid.

annually.
3. The

sum

interests will be the interest

two

having

note

annual

one

the time

principalfor
paid.

interest remains

the

of

sum

the

of

interest is

annual

this,added

in 3 yr.

the interest

the interest of

3. The

2. ^o

$526.46.

1. Find

"

during xchich

and

be

then, is $12,864

amount,

the times

mo.

$24

on

Bule.

mo.

interest must

da.; hence,

reckoned

mo.

bond

for

for

yr., find

$750, with

the

interest

8997.50.

paid for 4 yr.,


for $10000, with

find

the

interest

$2150.
5%, payableannually.
due
interest having been paid,find the amount
5. No
Sept.1, 1877, on a note for $500, dated June 1, 1875, with
interest at 6%, payable semi-annually.
$571.10.

at

PAYMENTS.

PARTIAL

on
demand,
received,
Morgan, or order,twelve hundred
6%, payableannually.

value

For
G.
at

No
due

Wis.,May 12, 1873.

Milwaukee,

[S1200.]

6.

241

promise to pay John


with interest
dollars,
H.

having been paid, what


20, 1877?
note, Sei:)tember

interest
this

on

7.

New

[$1500.]

W.

Slocum.
the

was

amount

$1545.66.

Orleans, La., October 10, 1872.

May, 1877, for value received,I


dred
promise to pay Andrew^ Jackson, or order, fifteen hundollars,with interest,
payable annually,at 5%.
the

On

first

day

of

George
No
at

having

interest

been

paid,what

Quitman.

amount

$1872.75.

maturity?
is the

8. What
interest

$1000

on

will be

9. What

for 5 yr., at
due

U.

S.

on

6%

$36.

be

ning
citybonds run6%, payable semi-annually,
$3580.50.
paid?
bonds
4 per cent
is payable

quarterlyin gold; granting that the income


might be immediately invested,at 6%, w^hat
income
with

gold

at

bonds

1000-dollar

20

on

than

the

2. The

1. A

receiptof
Prac.

16.

on

them

would
to

in

the
5

yr.,

PAYMENTS.

partial payment

it

from

$4798.50.

face,paid on

by indorsing

amount

105?

PARTIAL

192.

annual

six 500-dollar

on

at

interest

simpleand

between

difierence

3 yr., with interest


not
if the interest should
10. The

due

was

is

of

sum

money,

less

note.

partialpayment

the back

of the

note.

is

acknowledged

KAY'S

242

3. The
of the

TKACTICAL

NEW

ARITHMETIC.

the

of

consists

indorsement

date

and

amount

pajMuent.
rule of the

4. The

in reference

to

Supreme Court
Partial Payments, is

of the United
as

follows

States,

partialpayments have been made, apply the


of the interest
payment, in the firstplace,to the discharge
When

'^

then

due.

If

^'

toward

the payment

the

exceeds

the surplus goes


interest,

the subsequentinand
terest
dischargingthe principal,
remuinis to be computed on the balance of principal

ing due.
the

If

"

be less than

payment

the

the surplus
interest,
of
the

but
principal,
tintil the period
interest continues
the former principal,
on
the interest due,
exceed
when the payments, taken together,
then the surplus is to be appliedtoward
and
discharging
the principal
interest is to be computed on the balance,
; and
as
aforesaid.''Kent, C. J.
interest must

not

taken

be

to

augment

"

Rem.
nor

This

"

rule is founded

payment shall draw

For

value

Warren,

interest

at

6%.

this

note

On

that
principle

neither interest

Boston, Mass., May 1, 1875.

received,

Alonzo

the

interest.

[SIOOO.]

1.

on

or

demand, I promise to pa}^ to


thousand
dollars,with
order, one
on

William

partialpayments

were

Murdock.

indorsed

as

lows
fol-

November

25, 1875, $134; March


7, 1876, $315.30;
August 13, 1876, $15.60; June 1, 1877, $25; April 25,
the amount
due on settlement,
w^as
1878, $236.20. What

September 10, 1878?

PAYMENTS.

FAUTIAL

from

time

"The

Solution.

1, 1875,

May

to

25, 1875, is

November

the

da.;

mo.

24

of

$1000

243

interest
is

for this time

$34; the payment, $134,


exceeds

the

interest; the

is

$1034; $1034

amount

$134

"

the

$900,

sec-

iirincijiol.

ond

from

time

The

1875,

25,

7, 1876, is

March

this

for

$900

the

$15.30;

to
mo.

of

interest

da.; the

12

vember
No-

is

time

payment,

$315.30, exceeds
the

terest;
in-

the

is

amount

$915.30; $915.30"1315.30
third

the

=r$600,

jirin-

cipal.
from

time

The

7, 1876,

March

August

to

1876, is

interest

of $600

mo.

da.; the
for

is $15.60; the

time

13,
this
ment,
pay-

$15.60, equals the

interest;the

$615.60; $615.60"
^^

$600,

the

is

amount

$15.60

fourth prin-

cipal.
time

The

13, 1876,
is 9

18

mo.

of
time

to

is

from

August

June

1, 1877,

terest
da.; the in-

$600

for

this

$28.80; the payment,

$25, is less than


the

interest;continue

princij^al.

to

find

the

interest

on

the

fourth

The

June

from

time

PRACTICxVL

NEW

KAY'S

244

1, 1877,

the

is $32.40; the

$661.20; $661.20
The

da.;

of the payments,

sum

is

amount

ffth principal.
September 10, 1878, is 4

April 25, 1878, to

due

is $9; the amount

interest of $400 for this time

the
";la.;

24

mo.

$261. 20 ==$400, the

"

from

time

10

interests,$01.20; the

the

of

sum

April 25, 1878, is

to

the interest of $000 for this time

$261.20, exceeds

AKITIIMETIC.

mo.
on

15
tlement
set-

is $409.

RULE.

each

I. When
1. Find

of

the

the time

exceeds

the

interest.

to

the date

for
givenj^rincipal

this time.

equalsor

payment

the date

from

of

the note

first
payment

2. Find

the amount

3. From

this amount

is the second

time

of the

the date
5. Then

subtract

so

second

one

or

of

the

firstpayment

to

payment.
the

to the date

on

II. When

the date

from

proceed with

anil
first,

the payment ; the remainder

principal.

the

4. Find

the

of

second

of

principalas

settlement.

payments

more

with the

are

less than

the

same

the

interest.
1. Continue
until

date

Then

amount;

Rem."

payment

is

exceeds

equalsor
2.

to

find the interest


reached,when the
the

subtract

the remainder

sum

the

on
sum

of

the

principal
payments

of the interests.
sum
of the payments from
is th?. next

the

principal.

that the
it may
be determined, by inspection,
this can be done, it is not
is less than the interest;when

Sometimes

necessary to find the intermediate time and interest,but interest may


be found to the date when
it is apparent that the sum
at once
of the

payments

exceeds

the interest.

PARTIAL

For

with

received,I promise to pay


demand, three hundred

value

order,

on

interest

at

w^as

due

and

Sargent,
fiftydoHars,

on

; December

Gordon.

October

1, 1876, ^44;

January 1, 1877, S26

the amount

Edward

James

6%.
March

Indorsements:

SIO ;

245

Boston, Mass., July 1, 1875.

[S350.]

2.

or

PAYMENTS.

1, 1876,

1,1877, $15.

settlement,March

What

16, 1878?
$306.75.

3. A

of $200

note

is dated

January 1, 1875.

ment:
Indorse-

the amount
due
was
January 1, 1876, $70. What
$150.52.
January 1, 1877, interest at 6% ?
Indorsements:
of $300 is dated July 1, 1873.
4. A note
was
January 1, 1874, $109; July 1, 1874, $100. What
the amount
due January 1, 1875, interest at 6%?

$109.18.
ments:
May 10, 1870. IndorseSeptember 10, 1871, $32; September 10, 1872,
the amount
due November
$6.80. What
was
10, 1872,
5. A

interest
6. A

of

note

of

note

was

$132.30.

$200

is dated

March

5, 1872, $20; December

June
What

is dated

6%

at

$150

the

amount

due

June

5, 1871.

ments:
Indorse-

5, 1872, $50.50.

5, 1874, interest

at

10%

$189.18.
7. A

June

What

was

ments:
January 1, 1875. Indorse1, 1875, $6; January 1, 1876, $21.50
due July 1, 1876, interest at 7%'
amount

of $250

note

the

is dated

$248.40,

August 1, 1874. Indorse


ments:
February 1, 1875, $25.40; August 1, 1875, $4.30
due July
the amount
was
January 1, 1876, $30. What
$138.54.
1, 1876, interest at 6% ?
Indorse
9. A note
of $400
is dated March
1, 1875.
ments:
September 1, 1875, $10; January 1, 1876, $30
8. A

note

of

$180

is dated

PRACTICAL

NEW

RAY'S

24G

ARITHMETIC.

What

July 1, 1876, $11; September 1, 1876, $80.


the

March

due

amount

1, 1877,.iuterest

at

was

6%?
8313.33.

ments:
April 16, 1876. IndorseJanuary 1, 1877, $20; April 1, 1877, $14; July
25; 1877, $10; July4, 1878,
16, 1877, $31; December
10. A

What

$18.

8%

at

note

of

is dated

$450

the

was

amount

due

dorsemen
InJanuary 1, 1870.
May 1, 1870, $18; September 4, 1870, $20;
16, 1870, $15; April 10, 1871, $21; July 13,
of

note

1871, $118;

is dated

$1000

December

1, 1873, interest

partialpaj'ments are made on


running a 3'ear or less, the amount

accounts

found

commonly

by

1. Find

the amount

the date

2. Find

of

of

note

due

Credits:

the

RULE.

on

the date

from
principal

of

each

of

the

payment from

the

of

its date

the

principalsubtract

to the

of $320

is dated

Jan.

of

sum

Indorsements:

1, 1876.

Nov.

16, 1876, $100.


Jan. 1, 1877, interest at 6^

account

the

payments.

of

$540

was

due

What

was

the

$186.45.

March

1, 1877.

May 1, 1877, $90; July 1, 1877, $100; Aug. 1,

1877, $150; Oct. 11, 1877, $180.


due

is

settlement.

the amount

May 1, 1876, $50;


An

due

settlement.

the amounts

2.

of

the amount

3. From

amount

and

notes

the

MERCANTILE

1. A

$663.80.

6^?

at

the

was

When

193.

of

What

23, 1871, $324.

October

due

amount

to

interest

$466.50.

December

date

1, 1879,

11. A

note

June

settlement

Jan.

1, 1878,

W^hat
interest

was

at

the

S%?

amount

$39.

DEFINITIONS.

194.

2. There

True

is interest

1. Discount

kinds

two

are

in

advance.

discount,Bank

Discount

and

Discount.

DISCOUNT.

BANK

1. Banks

195.

Rem.

"

These

business

2. An
bank

"

notes

(2)

sorts

business

of notes.

accommodation

The

lends

is

note

accommodation

the

made

the

after

common

form

of

an

accommodation

date, we,

Second

National

dollars,for

value

or

either

Bank

of

20,

received.
S. West.
B.

Sharp.

18 /

/oi

^^^

1877.

promise to
Chicago, 111.,^ve

W.

January"^

note:

of us,

O.

Due

the

to

money.

followingis a

Ninety days
hundred

paper,

payable

Chicago, III., October


to

(1)

notes.

frequentlytermed

are

^500.

pay

two

on

money

paper.

which

TvEM.

lend

notes, and

accommodation

and

of

paid

1878.
,

(247)

RAY'S

248

3. A

PRACTICAL

NEW

ARITHMETIC.

is

payable to an individual.
be nefjotiahle
business
note
not negotiable.
or
may
is one
that
be bought and
can
negotiable note
note

business

4. A

5. A
sold.

Rem.

The

"

aro
followinu:;

of business

Buffalo, N. Y., March 21, 1877.


promise to pay Charles II. Peek, two

$200.
I

demand,

hundred

value

dollars,ibr

received.
W.

G.
This
and

is

note

bears

2d.

payahle only

to

H.

it is due

Peck;

St.

date, I promise to pay


thousand
dollars,for value

after

year

words

order"

"or

transfers it,he must


of it.

3d.

This

bears

note

pay

make

indorse

this note

it
"

his

King,

received.
B. Archer.

negotiable. If

that is,write

name

David

King

the back

across

interest tillafter it is due.

no

Washington,
or

before

the

first

Durand,

Amos

with
dollars,

of

May,

bearer,

or

interest at

day

10^

C, August 10, 1877.

D.

from

one

words
This

"

or

note

bearer

"

make

this note

bears interest from

1878.
hundred

date,for
John

The

David

to

JYeiotiable.

$150.
On

once,

Louis, Mo., May 1, 1877.

Elmer
The

at

JS^egotidble,

order, one

or

Charles

Clinton.

datt?.

interest from

$1000.
One

notes:

JV'ot ne^otiahlc.

1st.

On

forms

common

promise to
and
fifty

value

received.

Sherwood.

without
negotiable

ment.
indorse-

date, it being so specified.

DISCOUNT.

PANK

249
A

6. A
the

is payable,

note

7. A

the

in
specified

time

note

matures,

the

nominallydue, at

or

end

of

note.

is

or

legally
due,

three

after

days

the

time.
specified
three
8. The
days
days of grace.
Rem.

Banks

1.

"

lend

after the

only

money

time
specified

short

on

are

called

time; rarelybeyond

months.

To

find

ivhen

1st. When

the

Rule.

three

Count

"

note

is

time

the

matures:

expressed in days :

days from

the date

of

the note and

add

days.

2d. When
Rule.

the time
Count

"

the

is

expressedin

months

the

from

months:
date

and

three

add

days.
Rem.
the

In

2.
"

Delaware, Maryland, Pennsylvania, Missouri, and

District of Columbia,

the

day of

discou7it is the

first

day

of the

time.
Rem.

When

8.
"

in bank

note

is not

paid at maturity, it goes to


fact,made out in legalform,

protest that is,a written notice of this


by a notary public,is served on the indorsers,or securit^^
"

The

9.

bank

is

discount

simple

taken

interest

in

advance.
10.

The

11. In

(1)

The

proceeds
Bank

is the

Discount

money

four

received

on

quantitiesare

the

note.

considered:

/ace of the note, (2) the per cent, (3) the time^

and

(4) the discount.


12. Any
three of

fourth

may

be

found.

these
We

quantitiesbeing given,
will

consider

two

cases.

the

PRACTICAL

NEW

KAY'S

250

ARITHMETIC.

CASE

the time

find the

to

When

1st.

the
the

1. Find

of

note, the

the

discount

and

does

note

date

the

hear

not

due, bank

when

cent, and

per

proceeds.
interest.

discount,and

ceeds
pro-

note, discounted

followingaccommodation

the

of

face

the

Given

196.

I.

6%:

at

Mobile, Ala.,

$700.
after date

Sixty days

National

First

the

pay to
hundred

of

Bank,

25, 1877.

promise

to

Mobile, Ala., seven

received.

for value

dollars

either of us,

or

we,

June

Charles

Walker.

Walter

Smith.
OPERATION.

Solution.

The

"

note

is due

August

/oy
^ '

1877

6^, is

at

$0.0105 X
$700

$0.0105, and
700

$7.35

"

interest of $1 for 63

The

(Art.78).

$7.35;this

the

interest

of

days,

7 00

is

$700

is the discount ;

nTTTF
.0
1 0

73

then,

5 0 0

7 0 0 0

0~

7*3
^

$692.65, the proceeds.

ir92ir5
Rule.

the
2.

this is the

given time;
From

remainder

Find

the interest

1. Find

"

the

face of

is the

the

on

hank

the note

the

the note

for

the discount ;

the

face of

discount.
subtract

proceeds.

date

when

due, bank

discount, and

ceeds
pro-

of
2. A
and

note

of

discounted

$100, dated
at

6%.

June

20, payable in

August

60

days,

/22' SI.05, $98.95.

BANK

3. A

of

note

251

October

dated

$120,

discounted

and
(lays,

DISCOUNT.

months,

note

of

and

discounted

dated

$140,

of

note

at

and

months,

$180,

January 15, payable in


6^.

discounted

April 10, payable


4%.

at

of

note

of

note

days, and

$375, dated

discounted

in 60

of

note

days, and

dated

in 90

of

note

discounted

days, and

$1200,

in 90

note

of

days,and

discounted

at

the
and

date

dated

discounted

when

proceeds

of

30

^g $2.06,$372.94.
,

10^.

at

^^24'
^^1' ^^^^^"

January 11, 1872, payable

6^.

April
Find

in

February 20, 1877, payable

dated

$1780,

$8.50, $241.50.
y^,

$9.45, $590.55.
^^^^5

May
10. A

February 12, 1876, paj^able


at 9%.

April
9. A

in

6%.

at

$600,

1, payable

August 4, payable

September
8. A

8^.

at

May
7. A

in

$3.66,$176.34.
^^/^3,

December

dated

discounted

and

months,

$250,

$2.87,$137.13.
^^jig,

dated

October
6. A

30

^7^4,
S0.88,$119.12.

May
5.

in

8^.

at

November
4.

12, payable

$27.59, $1752.41.
^^/i3,

due, time of discount, bank


notes:
the following business

count,
dis-

NEW

KAY'S

252

PRACTICAL

San

Francisco, Cal., Sept.15, 1876.

11.

[$600.]

One

3^ear after date, I


E.

Abel

Worth,

promise to
First

the

at

hundred

Francisco,Cal.,six

ARITHMETIC.

pa}^ to

National

dollars,for

value
M.

of

order

Bank

George

Discounted

the

of

San

received.

Burgess.

May 21, 1877, at 10%.


1877, 120
^^/-^g,

Sept.

days, S20, $580.

Nashville, Tenn., May 8, 1877.


[SIOOO.]
Ninety days after date, I promise to pay Albert
12.

Kirk,

or

for
dollars,

thousand

order, one

value

Jacob
Discounted

June

22, 1877,

[S1500.]

Six

months

48
/g,

received.
October

January

the

When

1. Find

the

discount,and

25, 1877,

^^3,1878,
note

dale

at
81

due,

proceedsof

the

months

after

O., eight
received.

6^.
days,$20.25,$1479.75.

time

hundred

the

of

discount, bank

followingbusiness
O., January 5,

date, I promise

Stuart,at

Watson.

interest.

Dayton,

of Charles

value

hears

when

$800.
Six

days,$8, $992.

Orlando

Discounted

2d.

Simmons.

Pittsburgh,Pa., July 10, 1877.


after date, I promise to pa}'' Alex.
M.
bearer, fifteen hundred
dollars,for value

13.

or

received.

6%.

at

August

Guthrie,

E.

Dayton

note:

1877.

to the order
pay
National
Bank, of Dayton,

dollars,with

to

interest
Francis

at

6^,

Murphy.

for

DISCOUNT.

BANK

Discounted

Solution.

July

April 15, 1877, at 8^.

The

"

^/g1877.

is due

note

The

time

of discount,

April

from

is 84

days.

for 6

mo.

The

253

The

July 8,

to

of $800

amount

is $824.40.
da.,at ^o]^,
of $824.40 for

discount

bank

15

The

is $15.39.
days, at '^c/^,
$809.01.
are
j^roceeds

84

Rule.

1. Find

"

2. Find
Eem.

the

discount

and

the note

for the giventime.


proceeds of this amount.

followingexamples, remember
days.

February

has

29

Find

the

date

when

due, time

of

note

at

leap years

discount,bank

of

count,
dis-

May 20, 1875, payable in 6


6%, and discounted September

^^/23
1875, 75

November

1 year,

in

$150, dated

months, with interest


9, 1875, at S%.

3. A

that

proceedsof

and
2. A

of

the bank
In

"

the amount

note

with

of

$300,

interest

days,$2.58,$152.

dated

August 5, 1876, payable in


S%, and discounted
April 16,

at

1877, at 6%.

August
4. A

note

of

$450,

1, 1878, with interest


1877, at 10%.

January

^g,1877, 114
dated
at

March

6%,

and

^^4,
1878, 144

days, $6.16,$318.04.
4, 1877, due

discounted

January
August 13,

days,$18.90,$453.60.

KAY'S

254

5. A

1878, with
at

of

note

Pll ACTIO

NEW

$650,

interest

at

May 16, 1876, due Sept.1,


9%, and discounted April 25, 1878,

6%.

6. A

note

of

20, 1875,

interest

of

note

1876, with

at

6^,

and

discounted

May 1,

Jan. 17, 1876,

10%.

8. A

note

1878, with

y4,1876, 108

of $2400, dated

interest

at

8^,

and

days, $44, $1422.50.

Oct. 16,

1876, due

discounted

Jan.

1,
July 26, 1877,

10%.
January

'^/ji^,
1878, 162

days, $118.51,$2515.09.

Macon, Ala., October,15, 1877.


One
Moore,
year after date,I promise to pay Adam
hundred
dollars,with interest at
order,thirty-five

[$3500.]

9.

6%,

Joseph

received.

for value

Discounted

Stephens.

May 15, 1878, at 9^.


1878, 156
^^j^g,

October

days, $144.76,$3566.99.

Frankfort, Ky., 3Iay 10, 1875.


One
year after date, 1 promise to pay Henry Warren,
with
interest at S%. for
order, six thousand dollars,
10.

or

days, $14.71,$867.99.

$1400, dated July 19, 1875, due

interest

$767.29.

September 1, 1875, payable


discounted
at 10^, and
cember
De-

Y4,1876, 75

May

or

days,$17.26.

132

8%.

at

March
7. A

dated

$840,

months, with

in 6

at

ARITHMETIC.

dated

Sept.Y4,1878,

at

AL

value

[$6000.]

Amos

received.

Discounted

November

May

21, 1875, at

^^L,1876, 174

E. Burton.

10^.

days, $313.39,$6170.61.

BANK

DISCOUNT.

CASE

find

to

what

to

ceeds

per
face of the

the

1. For
note

the

Given

197e

II.

cent, the time, and

proceeds,

days hence, must


discounted
at 6%,

90

bank, that,when

the

note.

due

sum

255

give

the

pra

S177.21?

will be

OPERATION.

1.0 00

3)93
bank

The

Solution."

discount

the

is $0.0155,
days,at 6^^,
$0.0155
proceeds $1

$0.9845.
ceeds

-0155

.0155

of $1 for 93

and

)"03

.9845

"

"

Then, $177.21 is the

of 177.21

.9845

--

.0845)177.21(180

pro-

9845

$180.

787CO

78760

1. Find

Rule."

the

givenper
2. By this
2. The

proceedsof %l for

the

given time

divide the

6^,

givenproceeds.
discounted

note

what

$197.90:

were

was

bank

the

face

at

for GO
of

the

$200.

note?
3. For

what

discounted
will be

at

must

sum

bank,

for

be made,

note

90

days, at 6%,

so

that

the

$400.
be

must

at

bank

the

face of

for

note, that

months^

at

8^,

months,

proceeds

the

$225.

proceeds of
and

the

counted
dis-

when

be $217.35?

5. The

when

proceeds

$393.80?

4. What

may

at

cent.

proceeds of

days, at

the

discount

note
at

are

(3^

$352.62, the
what

is the

time

face?

S360.

KAY'S

256

6. 1 wish

what
at

be

must

6%,
7. I

borrow

to

ARITHMETIC.

PRACTICAL

NEW

the

from

$400

bank

must

sum

give my

days:

discounted

note, that,when

face of my

receive this amount?


I may
bank
wish
obtain from
to
a

for what

for 30

$402.21.

$500 for 60

note,

8^

at

days:

discount?

$507.10.
8. I wish
from

money

use

for 6

$1500

bank,

must

sum

to

at

give my

discount

note

if I

months;
of

obtain

can

what

for

10%,

realize this amount?

to

$1580.33.

February 19, 1876, payable January


discounted
ber
Octo1, 1877, and
bearing8% interest,was
$1055.02: what
12, 1876, at 6fc; the proceedswere
9. A

note

face of the

the

was

dated

$1000.

note?

DISCOUNT.

TRUE

1. The

198.

given i)er cent,


2. The

present
Rem.
True

interest

which, being on

money,

true

worth

will amount

the

"

the

as

the

of

sum

at

note.

the

between

difference

running

given time

same

of the

amount

Notes, d^^bts,and

1.

to

is

note

for the

is the

discount

and

of

worth

present

note.

accounts

are

discounted

by

Discount.

Rem.

2.

"

Banks

sometimes

discount

by the

method

of

True

Discount.

the face of the note, the time, and


cent, to find the present worth and discount.

199.

Given

1. Find
note

of

the present worth

$430.50, due

and

in 2 yr. 5

mo.

the per

discount,at 6^,
18

da.

of

TRUE

Solution.
2

yr. 5

of

amount

present worth

is 430.50 --1.148

is $430.50

$375;
$375

"

$1

6^o,is

da., at

18

mo.

the

Then,

The

"

DISCOUNT.

of
and

257

for

$1,148.
$430.50

the

count
dis-

$55.50.

$5 5.5 0

2. Find

of

note

the

present worth and discount,at 8^, of


$500, due in 3 yr., and bearing interest

6%.

Solution.

"

The

$500 for 3 yr.,


The
at

of

6^^,is $590.

of

$1 for 3 yr.,
8^^,is $1.24. Then, the
amount

present worth
^

at

amount

1.24

discount

of

$590

is 590

$475.81; and
is $590

"

$475.81

the
=

$114.19.

590
4 7 5.8 1
1 1 4.1 9
Prac. 17.

80

at

HAY'S

258

Kule.

1. Find

"

PKACTICAL

NEW

the

givenper
2. By this

ARITHMETIC.

0/ SI for

the amount

the

given time

at

cent.

the

divide

of

amount

the

this is the

note;

present worth.
3. From

worth;
Hem.

the

When

the

is the

3. Find

same

subtract

not

bear

interest,of

does

note

face of the

the

as

the

4. Find

present

worth

in 2 yr.
present worth

the

$300, due

of

note

note

present

the

course

note.

and

discount,at

$224, due

of

note

the

the

this is the discount.

"

amount

of

amount

of

G^,

8200, $24.
and

in 2 yr., and

discount,at 6^,

bearing interest

of

8^.

at

$310.71, $37.29.
the

5. Find
debt

$675, due

of

6. Find

10%,

present

of

7. A

in 5 yr.

10

present worth

the
an

worth

and

of

dated

$800,

discount

and

of $368.75.

of

$500, $175.

mo.

account

note

discount,at 6^,

for 5 mo.,

at

$354, $14.75.
September 10, 1876, due

posed
disbearing interest at 6%, was
present worth, at 10%, July 19, 1877:

January 1, 1878, and


of for the
what

present worth

the

was

at

this

date

and

the

count?
dis-

$825.65, $37.15.
merchant

8. A

bought

bill of

goods amounting to
is worth
$775, on 4 months' credit: if money
10% to
$750.
him, what might he pay for the goods in cash?
8
9. Bought a bill of goods, amounting to $260, on
will
is worth
"months' credit: if money
6%, what sum
pay

the

10. A

$2480:
cash:

debt

$250.

in cash?

merchant
he

can

if money

gain by paying

buys
have

is worth
cash?

bill of

months'

goods amounting

credit,or

only 10%

to

him,

to

5%

off,for

what

will he

$45.47.

DISCOUNT,

TRUE

the

Find

11.

S956.34,

one-third

and

third

one

of

latter

12%

for

1878,
25,
was

yr.,

the

bank

count
dis-

S2.45.
for

did

he

house,

without

mo.,

much

and

the

is

7%?

at

$1122

10

offers

2,

by
A
and

1877,

if

lose,

cash,

in

He

interest.

of

worth

is

money

or

chose

for

cash,

installments

equal

be

will

considering
the

gain

dated

interest

worth,

the

to

$620.

$2000,

payment

or

the

at

interest:

what

10^,

in

$8000

cash?

bearing

present

farm

without

years,

w^orth

note

his

three

in
3

paying

by

sell

to

and

be

to

the

what

grace,

discount

true

yr.,

payable

1,

money

15.

in

$12.

man

of

buyer

of

him?

$10296,

end

one-third

yr.,

of

days

offered

how

to

14.

in

payable

debt

8870.60.

the

was

in

paid

three

for

man

$1221,
the

the

$535,

be

of

5^,

at

yr.

between

difference

13.

in

Omitting

12.

worth,

present
to

259

July
at

the

of
at

8^,
present

this

date,

4,

due

1876,

May
October

cancelled

was

worth
and

6^

at

the

what

discount?

$2223.08,

$68.92.

1,

HANG

Xe

200.

1. A

order, from

draft,

one

bill

or

exchange,

of

another,for

to

person

EI.

is

certain

written
amount

of money.
Rem.

is called the
the bill is drawn
whom
person
upon
is called the payee.
the person in whose favor it is drawn
it is
the draft is to be paid upon
2. When
presentation,
The

1.

"

drawee;
Rem.
called

"

time, it is called
2.

of

time

is the

two

are

the

in the
Rem.

method
bill of
of

sorts

exchange

of

certain

of

making a payment by
exchange.
exchange: domestic or Inland,

"

The

followingis
is commonly

$500.

sight,
pay

dollars,for

takes

place between

localities

form

bill of

country.

same

which

value

common

termed

of

rfraftor

an

Charles

(260)

inland

change,
ex-

check:

Cincinnati, O., May 1, 1877.


to John
Jones, or order, five hundred
of
received,and charge to account
Silas

To

end

foreign.

4. Domestic

At

paid at

draft.

draft,or

3. There
and

Exchange

means

it is to be

sightdraft;when

Smith

"

Co., Xew

York.

Thompson.

EXCHANGE.

exchange takes place between

5. Foreign

The

"

localities in

countries.

different

Rem.

261

following is

form

common

of

foreign bill

of

exchange:
"500.

Cincinnati, O., May

At

sight of

of the

this

and

tenor

same

first of

order, five

or

received,and

hundred

Smith

James

"

third
roll,
Carvalue

of

account

Stanley
To

1877.

exchange (second and


unpaid),pay to Amos
for
pounds sterling,

date

charge to

1,

Bingham.

Co., London.

in dupliforeignbill of exchange is usually drawn


cate
called a set of exchange;the different
or
triplicate,
copies,termed respectivelythe firstssecond,and third of
riage
exchange,are then sent by different mails,that miscarWhen
is paid, the
or
one
delay may be avoided.
A

others

void.

are

6. The

of

acceptance

b}' the
KEM.-Abill

drawee

is

with

to

bill of

his name,

it when

pay

accepted by

exchange

the

ment
agree-

due.

drawee's

writing the

the face of the

across

is the

word

bill;the bill

cepted,"
"ac-

is then

an

acceptance.

201,

To

find

the

cost

face of

or

domestic

bill of

exchange (Art. 170, Eule).


1. What

is the

cost

of

sight draft

on

New

York

for

$1400,

at

2. What

S2580,

at

^%

is the

\%

$1407.

premium?
cost

discount?

of

sight

draft

on

Boston, for
$2567.10.

RAY'S

262

is the

3. What
which

is the

4. What

5. What

is

the

sight draft
premium?
sight draft on
a

$5680, payable
and

7. What

Wheeling,

on

$375.40.

Chicago,for
82778.04.

of

sight draft, which

discount?

in

60

cost

$1876.

of

cost

draft

New

on

Orleans

for

days, exchange being at ^%


$5649.08.
6% ?

interest

is the cost of
in

face

is the

G. What

of

cost

\\%

$1852.55, at

premium,

of

ARITHMETIC.

discount?

1%

at

payable

face

$375.87, at J%

cost

$2785,

PRACTICAL

NEW

draft

IS'ew York

on

for

$1575,

30

days,exchange being at ^^% premium,


and interest 6%?
$1578.13.
face of a draft,payable in 60 days, is $2625;
8. The
exchange being at \\% premium, and interest 6%, what
is the

cost

of the

draft?

$2636.69.

FOREIGN

202,

of the

money
Rem.
Great

Foreign

"

The

EXCHANGE.

bills

of

country in

foreignexchange

of the United
and

Britain, France, Germany,

ENGLISH

The

unit
4

Rem.

"

English money
1
farthingsmake
pence

20

shillings

The

usual

silver,crown
and

farthing.

"

coins
=

''

are:

be

States is

the

paid.
chieflywith

Canada.

is the
penny,

pound sterling.
marked

shilling,
pound,

gold,sovereign

s., half

three-penny;

to

in

MONEY.

of

12

drawn

exchange are
which
they are

crown,

copper,

the

''

s.

".

"

florin ==2
penny,

d.

", and
s.,

half

half

ereign;
sov-

penny,
sixshilling,
penn}^

and

EXCHANGE.

FRENCH

The

10

is the franc, marked


money
1 decime.
make
centimes

10

decimes

francs; silce?-piecesfor
5, 2, and

are:

1 centimes.

of

unit

divided

into

Rem.-

"

MONEY.

The

German

100
usual

coins

piecesfor 2, 1,
pennies.
Canadian

The
standard

Rem.
or

"

below

The
the

The

par
commercial
The

par

quoted
gold.

at

The

of two

par

is the

value

about

be

of

above

quoted
varies from
$4.83 to $4.90 gold.
It is usually
is $0,193.
of the franc
dollar
5 fr. 14f centimes, equal to one

value

of

the

of

the

quotations,
always
to

of the

comparative value

foreignexchange may
Quotations are always in gold.

par value.

value

cents, corresponding

countries.

value

commercial

value

is

currency.

exchange

coins

which

mark,

gold piecesfor 20, 10, and 5 marks;


^ marks; nickel piecesfor 10, 5, and 1

is in dollars and

States
of

par

the

are:

and

money

United

is

money

pennies(pfennige).

silver

with

1 franc.

^'

GERMAN

The

ft\

gold piecesfor 100, 40, 20, 10, and 5


5, 2, 1, h, and \ francs; bronze piecesfor 10,

coins

usual

The

"

MONEY.

of French

unit

Rem.

263

pound

is $0,238.

mark

for

four

Its

is $4.8665.

marks,

The

cial
commer-

from

vary

$0.95

$0.98.
To

find the

cost

or

face of

foreign bill

of

exchange :

RAY'S

264

1. What

ARITHMETIC.

sightbill on London, for


York, exchange being at $4.87?

will

in 'New

cost

PRACTICAL

NEW

"500

IOh.,

OPERATION.

10

S.r=:".5
5 0 0.5

Solution.

Since

"

20

s.

10

"1,

s.

".5.

r.=

4.8 7
If

is worth

"1

X 500.5

$4.87, "500.5

worth

are

$4.87

3~5T35

$2437.44.

=z

40040
20020

$2437.435
2. How

large a bill on London


$1808.04, exchange being at $4.88?
Solution."
as

pounds

many

$1808.04
in

Since

times, with
remainder

by
"870

$4.88,

remainder.

4.8 8

Reduce

)1

8 0 8.0 4

870

3440

the

341o

20.

4.88

10

times.

The

bill will

be

(3 7 0

244

shillingsby multiplying
is contained
in the product

to

for

1464

times

contained

is

bought

OPERATION.

for

bought

is contained

It

$1808.01.

is worth
be

can

$4.88

as

"1

be

can

20

for

4.88) 4

80(

10

s.

488

10s.

3. What

will

bill

on

London

for

being at $4.86?
4. How
large a bill on London
$2130.12,exchange being at $4.88?
5. What

will

being

bill

fr. 15

on

Paris

centimes

8s. .cost,exchange

$4327.34.
can

cost
to

"890

be

for 1290

$1?

bought

for

"436

lOs.

change
francs,ex-

$250.49.

largea bill on Paris can be.bought for $1657.60,


8553 fr.22.
exchange being at 5 fr. 16 centimes?
6. How

7. What

will

bill on

Berlin

cost

for 12680

exchange being $.97 per 4 reichsmarks?


Frankfort
8. How
can
large a bill on
$1470, exchange being at .98?

reichsmarks,
$3074.90.

be

bought
6000

for
m.

DEFINITIONS.

203.

1. Insurance

of money,
to pay
the occurrence
on
2. The

policy

sums
agree, for specified
to the person
insured

Companies
certain

of

amount

certain

is the

event.

written

the

given by

contract

company.
Eem.

The

"

companies

sometimes

are

3. The

insured

persons

are

the

policyholders.

The

the underwriters.
stj^led

is the

premium

called

sum

the

paid to

company

for

insurance.
4. Pire

in

case

in

of loss

Marine
case

sum

by

for

certain

amount

indemnity for a certain


dangers of navigation.

amount

is

loss'bythe
Insurance

at the

indemnity

fire.

Insurance

of

6. Life

is

Insurance

death, or

is

an

at

agreement
certain

time

to

pay

in

the

specified
life,of the
a

insured.

FIRE

204-.
a

certain

The

AND

premium

percentage

MARINE

INSURANCE.

in fire and
of

the

amount

marine

insurance

insured

is

(Art. 170,

Eule).
(265)

RAY'S

260

Rkm.

"

value.

NEW

PRACTICAL

ARITHMETIC.

Insurance

companies will
insurance is commonly

The

1. What

is the

f of its value,
costing$1 ?
at

Solution.

"

cost

the

seldom
upon

insure

" or ^ of

property at its full


the value.

of

insuringa liouse worth $3375,


and the policy
premium being H^

of the value

of the

premium is U^;^
of $2250, which
is $33.75;adding $1,
is
the cost of the policy,the sum
house

is $2250.

$34.75; the

The

cost

of insurance.
3 4.7 5

2. What

is the

cost

f of its value,the
costingS1.50?
at

3. A

is

insuringa house worth $5000,


premium being ^^, and the policy
of

$20.25.

valued

$12600,

and

the

goods at
S14400; " of the value of the store is insured at f^ and
^ the value of the goods at 2% ; the cost of the two policies
the total cost of inis $1.25 apiece: what
surance
was
store

at

$209.50.
4. A

man

owns

manufactory valued

at

$21000,

and

worth
will it cost. to insure
$7200: what
dwelling-house
the manufactory, at ^ of its valuiB,
at IJ^, and
the house, at its full value, at f^, the two
policies
$23G.50.
costing$1.25 each?
5. A man's
dwelling,valued at $5600, was burned ; it
had been insured, in a certain company,
20 years, for f
much
of its value, at l^% : how
did he receive from the
than the sum
total of the annual premiums?
more
company
a

$2940.
6. A

for

man

$3600,

secures

furniture

policyof insurance,on his house,


for $1600, and
library$800; the

INSURANCE.

is

premium

of the

cost

is

its

of

its

the total cost

value, at 1J%

of insurance

is $151.25

worth

of

cost

the

$4500, for ^

policywas

of

$1.25

of insurance?

cent

per

; the

$15000.

house

insuring a

$32.75 ; the

the

was

$53.75.

and

of

cost

value, was

what

is the

is the hotel valued?

sum

8. The

what

policy $1.25:

insured, for |

policycosts $1.25
at what

of

cost

insurance?

hotel

7. A

mid

^^,

267

^^.

of $1000 on
his house,
farmer, with an insurance
$1500 on his barn, in the Yermont
Mutual, pays an

9. A

and

annual

of $3.50

assessment

what

is the

of the

cent

per

premium?

-J-^%.
LIFE

205.

1. Life

kinds

INSURANCE.

(1) lifepolicies,
(2) endowment
life policyis payable at the

2. A

of

policiesare

Insurance

principal

two

policies.
death

of the

person

insured.
3. An

time, or
Rem.

death

at
In

"

annual

life

the

1. A

the

within

the

premium

company's

the

amount,

his insurance.

effects

premium,
at

man

occurs

dependent, in

he

annual

if it

insurance

payment,
when

show

policy is payable

endowment

at any

age

annual

of

at

specified

this time.
is

commonly
the

upon
The

tables

of

age
of

regular

dividual
in-

the

company

age, for $1000 of insurance,

insures

40

premium

his life for $5000 ;

$1000,

on

for

life

this age, is $31.30; if he dies at the age of 70,


much
will he have
paid the company?
money

policyat
how

OPERATION.

Solution.
is

$31.30, on

the

amount

--=$4695.

"

Since

the

annual

$5000 it

is $31.30

paid, in

30

premium
X

yr., will

on

$1000

$31.30

$1^6.50; then,

be

$156.50 X

30

5
15

6.50
8 0

$r6"5.W

RAY'S

268

2. Mr.

PRACTICAL

NEW

ARITHMETIC.

takes

Harris, aged 35,

endowment

policy
life insurance
for $10000, payable in 10
in a
company
premium on $1000, at bis
years ; the cost of the annual
age, is $105.58 : if he lives to receive the endowment, what
will be the cost of the paid-uppolicy,
without
interest?
out

an

$10553.
the

3. At

annually,is $47.18
policy,payable in
end

50, the

of

age

20

policy of $8000
life plan?
the
of

amount

annual

of

age

how

endowment

an

on

$1000

have

been

endowment

44,

plan

at

much

dies

for

after

than

more

his

of

this age,

man

insures

man

in favor

$12000

if the

will

more

the

of

cost

than

he

his

the

paid on
by the

wife;

the

the

the

company's
policy,is $36.46

life

life to

of

payment

paid out,

will

5 premiums,

his widow

$9812.40

receive ?
the

5. At

of

age

policy for $5000,


$19.89

; at

$2123.20.

premium

$1000

on

much

life policy,
payable

is $60.45

years,

by

4. At

$1000; the

on

how

of 20 years,

of

cost

$1000

on

him

will it cost

21, a

which

upon
if he

to

the

lives to the

keep

up

takes

man

young

annual
of

age

out

life

premium
75, how

is

much

his insurance?

$5370.30.
the

6. At

of

age

for

$1000, payable in

of

$104.58 ; what
at

the

30,

will

end

10

to

3 ears,

be

of the

secure

the

endowment

an

costs

an

amount

policy
premium

annual
of

the

ten

time, allowing interest

at

ments
pay-

6%

$1390.91.
the

7. At

age

of

38, a gentleman

took

out

policyfor

plan,paying annually $29.15 on $1000.


After keeping up his premiums for 15 years, he suffered
had
much
he paid out,
his policyto lapse: how
money
$3882.78.
allowing interest at 6% ?
$6000,

on

the

life

DEFINITIONS.

206.

1.

for the

coimtry

is

tax

of

support

the

paid by

money

government

or

citizens
for other

of

lic
pub-

purposes.
2. A

tax

is either

3. A

direct

4. A

tax

is

tax

one

the

upon

or

indirect.

which

is levied

the

upon

son
per-

citizens.

of the

property

or

direct

person

is called

poll tax;

upon

property, a property tax.


5. An

levied

the

upon

6. The
to

their

State and

business
of the

taxes

Local

1. The

chieflyfrom
Kem.

"

Some

lands,from

of the

direct

revenue

in

some

way,

is

citizens.

ence
States,considered in referof two
are
classes,(1)
purpose,

Taxes; (2)

States

United

for State

Revenue.

TAXES.

LOCAL

AND

money

which,

one

United
and

nature

STATE

207.

is

tax

indirect

and

arises

local purposes

taxation.

accrues

to the State

from

the rent

of school

licenses,fines,etc.

(269)

the

2. For
Real

as

of

purposes

Estate

3. Real

PKACTICAL

NEW

KAY'S

270

and

is ([classed

taxation, property

Personal
is

Estate

ARITHMETIC.

Property.
which

property

is

fixed,as

lands,

houses, etc.
4. Personal

furniture,merchandise,
5. The

which

that

is

Property

movable,

as

etc.

worth

is the estimated

valuation

is

of the

erty.
prop-

The

Rem.^
"

the

In

tax.

some

first be

must

is generally
the basis upon which to estimate
the polls
tax upon
states, however, the specific

vnluation

subtracted; in Massachusetts,

sixth part of the tax

is

assessed upon
the polls,
vidual;
provided it does not exceed $2 for each indiin Vermont, the basis is what is called the Grand
List,which
is ascertained
each

valuation

by dividingthe

100

by

and

adding $2

for

poll.

6. The

valuation

is

made

by

called

officer

an

an

assessor.

Rem.

This

"

contains

the

To

The

each

rate

dollar

out

list called

of the persons

names

of their

208.

official makes

to be taxed,

roll; it

assessment

an

along with

the valuation

property.

find the

rate

of taxation.

of taxation

is

of

property,

taxable

expressedas
or

mills

so

as

many
such
a

per

on

cent

of it.

1.

The

$1049905
a

certain

property
; there

year

are

of
483

certain

persons

the total taxes

town

of the
each

tow^n

poll-taxbeing $1.50

for

rate

of taxation

property?

the

valued

at

subjectto poll-tax.In

the

upon

is

person,

are

$13323.36
w^hat

is the

TAXES.

271

poll-taxis
$1.50x483r=$724.50;then, the
Solution.

The

"

is

^ ^^^

$13323.36"

property

tax

$724.50

$12598.86.

'

the

since

tax

on

$12598.86, the
$12598.86
12

'

is

1 '^^ 2 3.3 6

$1

is

7 2 4.5 0

tax

mills,or

$1049905
on

1049905

--

'"^

Then,

1 0 4 9 9 0 5

$0,012,

)1

2 5 9 8.8 6

4 9 9 0 5

10

^c-

(.01

2 099810

2099810

Multiply the tax on each poll by the number


of polls; the product is the poll-tax.
the total amount
2. From
of tax subtract the poll-tax
;
Rule.

1.

"

is the

the remainder

the

3. Divide

is the rate

Rem.

Of

"

property tax by

2. A

build

valued

at

where

course,

of

tax

there

is

$2500

$618000,
the

and

there

no

assessed
The
there

28

are

certain

upon

property

subjectto

persons

the

be

4 mills

rate

$18409.44:

is

6.4 mills
4. The

total

valuation

was
Wisconsin, for j.874,

this
the

valuation
hundredth

property

$421285359;

$656491.61:

was

of

of

mill?

what

in

was

1.56
.

the

the tax

of

$1, or 1%.

on

Upon a valuation of $2876475 the tax


what
is the rate?
being no poll-tax,

district

district is

of the

will

poll-taxis $1, what

taxation?
3.

quotient

specificpoll-tax,the total
immediatelyby the valuation.

is

is to be divided

school-house.

poll-tax:if

the valuation ; the

taxation.

of the tax

amount

to

of

property tax.

on

State

$1.
of

levied upon
the
mills

rate

to

on

$1.

KAY'S

ITl

209.

To

I. A

tax

PRACTICAL

NEW

among

the

is aBsessed

upon

apportion the
of

$1373.64

tax

is valued

property of which

AR1THMETI(\

tax-])ayer8.
the
village,

$748500 ; 57 persons pay


of $1.25 each; find the rate of taxation,and
a
poll-tax
tax table to $9000.
construct
a
at

TABLE.

TAX

Rate, 1.74 mills

Rem.
is

such

to construct

in any

column

table.

farther than

Turner's

1. James

pays

$1.

In order to facilitate the calculation

"

customary

it out

on

poll-taxfor

It is not

the nearest

property

person'stax,

necessary to carry

at

$7851, and

is his tax?
OPERATION.

Solution.

$12.18;

on

"

By

table, the

$800, $1,392;

$1, $0,002; then,


$1,392 -f-$0,087 +
tax.

Then,

the

James

The

the

$0,002

on

$7000

$50, $0,087; and

on

tax

tax

on

$7851

is

7851

on

is $12.18 +

$13.66;this is his property


is
$1.25 X 2
$2.50.
poll-tax

Turner's

=r

tax

is

it

mill.

is valued

what

persons:

of each

$13.66 + $2.50

at

$16.16.
16.16

he

UNITED

Explanation.

It is evident

"

REVENUE.

that

the

the rate, 1.74,and

multiplying$7851 by
Brown's

2. John

STATES

273

operationis equivalent to
adding the poll-tax.

is valued

property

at

$2576,and

he

S5.73.
poll-taxfor 1 person : what is his tax ?
3. Henry Adams'
property is valued at $9265, and he
what
for 3 persons:
is his tax?
$19.87.
pays poll-tax
pays

Clarke's

4. Amos

pays
5.

poll-taxfor
Emily Wood's

is her

property is valued

1 person:

what

$8367

at

tax

be raised

to

is valued

in

of

Eate

Mill's

property

3.816

is valued

Young's property

rate

construct

at

is his tax?
2. Samuel

able
; its tax-

find the

mill,and

table to $90000.
1. William

city is $64375

$16869758

at

thousandths

to

mills

O'Neil's property is valued

at

$83612

Adolph Meyer's property

is valued

at

$72968

Ganot's

is valued

property

1. The

indirect

taxation
from

Internal

sale of

what

what

at

$69547

what

REVENUE.

STATES

United

revenue

lands,from
the

$265.39.

UNITED

2. The

what

$278.45.

is his tax?

the

$1.

$319.06.

is his tax?

and

tax

$217.04.
at $27543 : what

is valued

is his tax?

from

of

$105.10.

3. Charles

210.

on

$56875:

is his tax?

5. Louis

what

$14.56.

property
taxation

he

$9.53.

tax?

II. The

4.

and

is his tax?

is valued

property

$4759,

at

States
; it

tax

consists

Duties

Revenue

upon

postage stamps,

Revenue

or

of

Internal

wholly
Revenue

Customs.

arises from

certain
etc.

arises

the

sale of

lic
pub-

manufactures, from

KAY'S

274

3. Duties

rilACTICAL

NEW

Customs

or

taxes

are

goods imported

on

foreigncountries.

from

INTERNAL

1. The

211.

will the

postage

thereof:

disposed of

are

receive

government

for

ship
town-

$28800.

is 3 ct. for each


is the

what

SI. 25

at

sq. miles?

containing36
2. Letter

REVENUE.

public lands

what

acre:

per

IJ

ARITHMETIC.

postage

half-ounce,or
on

letter

tion
frac-

weighing

oz.?

3. The

postage

what

thereof:

fraction

on

books

is 1 ct. for each

is the

postage

on

book

ct.

oz.,

or

ing
weigh-

1 lb. 5 oz.?

11

ct.

is 70 ct. per gallon: what


spirits
])roof
barrel of 40 gallons?
is the tax on
$28.00.
a
5. The
tax on
cigarsper 1000 is $5 : how much does
the price of a singlecigar?
this enhance
^ ct.
beer is $1 per barrel of 31 gal. Each
6. The
tax on
in malt
wholesale
dealer
liquorspays a specialtax of
$50, and each retail dealer a specialtax of $20 ; in a
certain
12 wholesale
dealers,250 retail
citythere are
4. The

tax

dealers,and
bbl. : what

on

annual

the
is the

manufacture
to

revenue

DUTIES

212.

1.

Duties

are

OR

of

of

government?

beer

is 30000

$35600.

CUSTOMS.

two

kinds, specificand

ad

valorem.
2. A

specific

duty

is levied

upon

the

quantity of

the

goods.
duties,allowance is made (1 ) for waste
levying specific
called draft,(2) for the weight of the box, cask,etc.^containingthe
of liquors,imported in casks or
goods, called tare. The waste
age.
barrels,is called leakage)that of liquorsimported in bottles,breakGross weightis the weight before deductingdry ft and tare; net
weight is the weight after deductingdraft and tare.
Rem.

"

In

DUTIES.

3. An

ad

levied

duty is

valorem

'

the

upon

275

of the

cost

goods.
Rem.

The

"

cost of the

goods is shown by the foreigninvoice,or


by appraisementat the custom-house.

is determined

be

4. Duties

must

Kem.

duty

The

"

which

coin.

computed on the net weight and


in the foreigncountry. The dutiable
estimated, is always the nearest exact

of the article

cost

paid in

the

is

duty is
dollars,pounds, etc.
1. The

it

the total

on

value
number

upon
of

weight of a hogshead of imjiorted


sugar
is 1760
is the duty at
lb.; allowing 12^% tare, what
S26.95.
If ct. per pound?
2. A
manufacturer
imported from Spain 40 bales of
pound:

what

each, tare
the

was

^%

duty, at

the

ct. per

cost

45

was

pound

ct. per

and

valorem?
3. A

of

lb.

of 400

wool,
ad

gross

$2052.

merchant

the

imported a

in France

ware

"

4. A

The

365.15

was

Germany
marks,

and

imports a

1317.04

was

commission

in U. S. money,

glassware;
francs,the

the

mark

and

case
:

$34.40.
value

of books

case

cost

5^ what
reckoning the

S. money,

marks,

6^

the

the commission

being $85.58, the dutiable

total cost

book-seller

of

case

57.15 francs,and
chargeswere
the duty at 40^ in U.
was
franc at 19-\ ct.?
Rem.

10^

; their cost

in

and

case

what

is $86.00.

was

charges 34.36
the duty at 25^

being estimated

at

23.8

ct. ?

$85.25.
5. A

merchant

imports six

cases

of

woolen

cloth,net
"500, cases

weight 1500 lb. ; the cost in England was


and
charges "8 48. 6d., commission
2^^ : what was the
in U. S.
duty, at 50 ct. per To. and 35% ad valorem
$1637.25.
estimatingthe pound at $4.8665?
money,

DEFINITIONS.

213.

1. Ratio

is the

their

by

The

"

given in

the

that

2^3;

"

in several

is, 6 is

pressed
ex-

numbers

times

of

departments

change the treatment


Ray's Arithmetics.

of

between

of

is indicated

2.

ematics
math-

ratio

as

by writing

them.

the

ratio of 2 to 6.

numbers

two

numbers

two

to

of two

is read

The

3.

editions

ratio

sign ( : )

Thus,

2 is 6

to

it advisable

former

The

2.

established custom

makes

of

quotient.

Thus, the ratio of


Rem.

relation

are

"

styled the

terms

of

the

ratio.
The

4.

second

yd.
ratio

to

can

(276)

the

antecedent,

and

the

consequent.

is 3,
a

is called

ratio

between

ratio between
To

denomination.

same

the 2
A

S2 is 3,

the

the

term

5. 6

$6

first term

ft;

not

ft.

exist

two

numbers.

numbers

concrete

two

find

abstract

yd.

ft. and

of

ft.,reduce

2 ft. is 3.

between

because

KATIO.

they can
Hence,
1st. The

reduced

be

not

the

of

terms

277
the

to

ratio

denomination.

same

be

may

either

abstract

or

concrete.

2d.

the

Whe7i

terms

concrete, both

are

be

must

of

the

denominatioji.

same

3d.

The

6. Eatios

7. A

is

ratio

either

are

ratio

simple

Thus, 2
8. A

G is

always

singleratio.

ratio.

ratio

compound

number.

compound.

simple or

is

simple

abstract

an

consists

of

two

or

simple

more

ratios.
2

Thus,

6 ^

"

!Q

9. In

ratio.

compound

MS

Eatio

three

considered:

quantitiesare

and (3) the ratio.


(2) the co7isequent,
being given, the third may be found.

antecedent

Any

of these

Given

214.
1. What

the

is the

terms,

find

to

of 6 to

ratio

the

(1)

the
two

ratio.

3?
OPERATION.

ratio of 0 to 3 is 6 divided

-The

Solution.

by 3,

2.

equal to

2. What

ratio of

is the

"

to

:
--

3
3

^?
OPERATION.

Solution.
or

"

The

ratio of

to

f^is|

divided

by |,

I multipliedby f equal to |.
,

Rule.
Rem.

"

"

be reduced

Divide

the antecedent

When

the terms

to the

same

are

by

the

of different

denomination.

I = f
I "^ f

consequent.

denominations, they

must

278

KAYS

NEW

is the

ratio of

What

AKITIIMETIC.

PKACTICAL

3. 12

to

3?

4,

4. 30

to

5?

6,

5. 35

to

7?

6. 56

to

8?

I-

7.

5 to

10?

8.

7 to

21?

2.

9. 12

to

18?

10. 15

to

20?

11. 15

to

25?

^.
J.
|.
f
f

12. 25

to

15?

l|

What

is the

23. S18

If

If
^"

ratio of
3.

$6?

to

24. 54

days

to

days?

6.

25. 96

men

to

12

men?

8.

to

17

bu.?

13.

bu.

26. 221

27. 1 fl. 9 in. to


28. 5

yd.

1 a. to

Given

215.

the

in.?

7.

5 fl. 4

in.?

ratio and

the

3.

consequent, to

find the

antecedent.
1. 7 is the

ratio of what

number

to

4?
OPERATION.

Solution.

equal to
Rule.

"

The

number

is 4

multipliedby 7,

4 X

"

=^28

28.

"

Multiply the consequent by

is the

ratio of what

|-is the ratio of what


-j^is the ratio of what
2|-is the ratio of what
45^ is the

ratio of what

the ratio.

number

to

13?

52.

number

to

27 ?

15.

number

to

52?

28.

number

to

24 ?

63.

number

to

If?

7^

EATIO.

7. 3 is the

ratio of what

to

75

of what

to

ratio

is the

8.

279

9. 2.6

is the

ratio of what

216.

Given

the

ratio

ct. ?

lb. 8

oz.

3 lb. 15

$4.

to

the

and

$2.25.
oz.

$10.40.
find

antecedent,to

the

consequent.
ratio of 45

5 is the

1.

what

to

number?
OPERATION.

Solution.

The

"

number

is

45 -=-5

by 5,

Rule.

Divide

"

is the

the antecedent

ratio of 56

2.

3.

is the

ratio

4.

2f
7|

is the

ratio of

the ratio.

what

to

of 42

by

what

to

23f

number?

to

find

value

the

of

14.

number?

what

60.

number

is the ratio of $27.20 to what

To

217.

Find

the

vahie

of the

$3.60.

ratio.

compound

compound

8i

6
1

9.

equal to

5.

divided

45

ratio

2)

3 f

"

OPERATION.

Solution."

The

product of the antecedents 6


and 9 is 54, the product of the consequents 2 and
3 is 6; then, the value of the compound
ratio is
54 divided
by 6, equal to 9.
Rule.

of

"

the

Divide

the

'product

"

thus, in the above


to

the

5 4

2X3=

5 4-^6=

the antecedents

by

the

uct
prod-

consequents.

Multiplyingthe
together,evidentlyreduces
Rem.

of

6X^

simple ratio

example
54

togetherand
compound ratio to

the

antecedents

6.

the
the

compound

ratio

6
g

"
"

consequents

simple

2 ^
o

one;

"is equivalent

Find

the

ARITHMETIC.

PRACTICAL

NEW

RAY'S

280

value

3,

2. Of

the

compound

ratio

3. Of

the

compound

ratio

o^f! of [

8.

4. Of

the

compound

ratio

f-

5. ^n
Oi

the

.1

6. Of

compound

the

the

"

24

men.

,,

6.

2)

7:3^

ratio

compound

I
; f^^^^
^^u^
5

7. Of

men

it"

ratio

if[

*"
ratio

compound

.'^

10^.

9:5)
The

218.
a

fraction,the
to

the

Thus, ill 2

101)

have

we

ratio

antecedent

to

correspondto
the

of

the terms

numerator, the

quent
conse-

denominator.

and

numerator

of

terms

3 the

the
the

ratio is

", in

consequent

which
the

the

antecedent

denominator.

Hence

is the

(Art.

following
Principles.

I. A

ratio is

1st.
2d.
II. A

multiplied

the antecedent.
By multiplying
By dividingthe consequent.
ratio is divided

By dividingthe antecedent.
the consequent.
By multiplying

Ist.
2d.
III. A

ratio is not

1st.
2d.

changed

both terms by the


By midtiplying
By dividingboth terms by the same

same

number.

number.

KATIO.

To

219.

reduce

1. Eeduce

16

ratio

24

to

281

its lowest

to

its lowest

terms.

terms.
OPERATION.

The

Solution."
both

ing

terms

C. D. of

G.

of 16

and

16

24

24

by 8, it

becomes

both

terms

of

is
2

8; divid3

8)16:24
2~:

(Art.

218, III, 2d).

Rule.

Divide

"

the

ratio

by

their greatest

divisor.

common

To

220.

clear

1. Clear

1^

25

to

its lowest

terms.

5.

30

to

its lowest

terms.

3.

51

to

its lowest

terms.

3.

133

to

its lowest

terms.

7.

125

to

its lowest

terms.

3:5.

279

to

its lowest

terms.

ratio

of

fractions.

2\ of fractions.
operation.

Solution.
3

is

"

The

L. C. M.

6; multiplying
9

Rule.

"

14

both

of

terms

denominators

the
of

K]

2\ by

2 and

ymdtipleof

both

terms

the denominators

of

~^T~1"4

(Art. 218, III, 1st).

Mvltiply

U:2^

comes
6, it be-

the

of

by the least
fractions.

ratio
the

mon
com-

9.

-zz:'.
z^*;"-''-O,

DEFINITIONS.

221.

1.

of two
Thus,
is

is

Proportion

expressionfor

an

the

equality

ratios.
2

4 and

may

form

for
proportion,

the ratio of each

\.
2. The

proportionis

indicated

by writing :

between

the ratios.
Thus,

is read

is to

as

is to

6.

4. In

proportionis either simpleor compound.


both the ratios are
a
simple proj^ortion
simple.

Thus,

3. A

5. In

is

simple proportion.

proportionone

compound

or

the

both

ratios

compound.

are

Thus,

G.

!4

"

"

."Q

's

compound

Every proportionconsists

7. The

first and

fourth

proportion.

of four terms.

terms

of

proportion

called the extremes.


(282)

are

PROPORTION.

the

called

the

three

other

4 and

are

10.

When

number
other

is said

proportionare

3; and

said

is

6 is

be

to

: :

fourth proportionalto

G, the extremes

fourth

numbers

three

be

to

in order.

taken

proportion2

Thus, in the
means

of

terms

means.

last term

9. The

third

and

second

8. The

283

6; the

proportionalto 2, 4, and

form

3.

proportion,the second
the
proportionalbetween
a

mean

2 and

are

two.

Thus, in the proportion 2


2 and

between

8, 4 is

proportional

mean

8.

The

222.

operations of proportion depend

the

upon

following
Principle.
is

Thus,

In

"

the

proportion 2

proportion 3 !
the

same

4}

be

may

shown

^|2 X

for any

product of the
Kill
extremes, the c/uotlent

1st.

the

the

equal to

in

proportionthe product of
productof the means.

2d.

Tf

the means,

223.

the

X ^ X 8

other

means

==

three

terms

fourth.

8?

3 X

be

tremes
ex-

"i

the

X 4 X ^i

-^nd

4 )"

proportion.

3;

Hence

divided

(36, 4),

by

one

of

be the other extreme.

be
product of the extremes
the quotientwill be the other

Given

1. What

6, 2 X 6

the

If

the

every

of

divided

by

one

of

mean.

to
proportion,

find the

RAYS

284

ARITHMETIC.

PRACTICAL

NEW

G and 4,
product of the means
of the extremes,
is 24; then, 24 divided by 8, one
equals8, the other extreme
(222, 1st).

Solution.

"

The

'^

2.

what

'

10

operation.

GX4

24

24--8.

5|,
I ?
12]

"

operation.

Solution.
4 X
the

of

"

tremes,
product of tlie exX 12, divided by 3 X 10, one

The

equals 8,

means

the

other,

4
=-8

mean

(222, 2d).

Rule.
name

Rem.

Divide

"

by

"

the

the other

Indicate

the

'product

of

the

terms

of

the

same

given term.
operation and

cancel

whenever

it is practicable

(91).

24.
14.
3.
10.
G.

18.
20.

21.

15.

A-

I3^.
9.

PKOrOKTlON.

285

when
crete
Proportion,
appliedto the sohition of conproblems,has been styled The Rule of Three,''
because
three
terms
are
given to find the fourth. The
of Proportionwas
use
formerly so extensive that it was

224.

"

called

often

Bide.''

problem by proportion consists

of

parts:
statement; that is,the proper

The

1st.

the

Golden

of

solution

The
two

The

"

numbers

into

of

proportion.
operationof finding the requiredterm.

The

2d.

arrangement

in a proportion,it is customary,
arranging the numbers
the number
or
quired
quantityrethough not necessary, to make
to the other three; then, the first three
a fourth pi'oportional
of the proportionalways are
given to find the fourth.

Rem.

"

terms

In

I.

1. If 2

yd.

PROPORTION.

SIMPLE

of cloth

$4,

cost

will

what

cost?

yd.

OPERATION.

Solution.

quired,or

"

Since

fourth

term

the

number

re-

proporis $4.
term
"

Since

what?

of the

tion,is dollars,the third

the cost of 6

^X
^

yd, will be greater


than the cost of 2 yd., 6 yd. is the
second term
of the proportion,
and 2 yd. the first term.
Dividing
the product of 6 and 4 by 2 (Art. 223, Rule), the requiredterm
is $12.

Rem."

In

to the number

this
of

example,the

number

of dollars is in

yards; that is,the greaier the


greater the number of dollars they will cost.

number

direct ratio

of

yards,the

KAY'S

286

2. If 3

days

men
men

ARITHMETIC.

cellar in 10

dig a
dig it?

can

can

PliACTICAL

NEW

days, in

how

many

OPERATION.

Solution.

quired,or fourth
tion,is days,the
Since

men

of

third term

dig
days than

1 0

3 X

is 10 da.

what?

/l^
0

/,

men,

proportionand 5 n)en the first term.


10 by 5 (Art.223, liule),
the required

3 and

is 6 da.

term

Kem.

this

In

"

less the

number

Rule.

of

example, the number

1. For

"

the

the

the

second

the

third;

be less than

the

will

men

4. If 3

an

inverse ratio

the

fourth

which

required.
of

gueat'er

the

term

is to be

the

fourth term

when

less,

number

the

two

greater
is to

the third.

first;the

3. If

and

the

3. Divide

is in

the mimher

as

term, tcrite the

remainingnumbers, when
than

days

term, write that

third

denomination

same

2. For

of

of men,
that is,the yreaicr the number
of men;
of days in which
they will dig the cellar.

the number

the

of the

term

Dividing the productof

is

re-

the cellar in

is the second

to

number

of the propor-

term

will

less number
men

the

Since

"

j^^oductof
quotientwill be

men

can

dig it in
yd. cloth

the
the

second

$8, what

5. If 5 bl. flour cost

$30, what

6. If 3 lb. 12

cost

oz.

tea

third terms

fourth tenn,or

dig a cellar
6 days?
cost

and

in

number

days,how

12

by
quired.
re-

many
(i

cost
cost

$3.50,what

6
3

yd.?

816.

bl.?

$18.

cost

11 lb. 4 oz.?

$10.50.
7. If 2 lb. 8

buy

oz.

of tea

cost

$2, w^hat quantitycan


6 lb. 4

for $5 ?

8. If 4

hats cost

you

$14, what

cost

10

hats?

oz.

$35.

PROPORTION.

9. If 3 caps
10. If
11. If
12. If

69

287

cents, what

11

cost

purchase 8 yd. of cloth,how


will $48 buy?
14. If $48 purchase 12 yd. of cloth,how
be bought for $32?
can
13. If $32

15.' A

receives

man

should

much

If

16.

what

17. If 60

men

will

18. If 15

$152

$48.
$32.

yards
12.

yards

many

8.
how

work:

work?

$32.
in

24

days, in
16 days.
da., how

in 8

240.

25 ct.,what

co^

$15.75.

many

months'

months'

for 4

of pepper

oz.

for 19

perform a piece of work


12 men
perform it?
perform a piece of work
perform it in 2 days?

can

men

have

he

men

time

many

$2.53.

caps?
4 yd. cloth cost $7, what
cost 9 yd. ?
12 yd.?
8 yd. cloth cost $32, what
cost
cost 8 yd.?
12 yd. cloth cost $48, what
cost

cost

6 lb.?

$1.60.
19. If

gal. of

molasses

$2.70, what

cost

$11.70.

gal.?
20. If 5 cwt.

21. If 11

lb. of sugar

cost

22.

$253.80.

yd. of

If 90

will 450

build

$2.50, what

cloth cost

will be

the

cost

$1,871

of oats

horses

supply 40

da.,how

build

wall

long
da.

30

supply them?

men

in

15

da., how

many

men

18.

it in 5 da.?

If 15

much

bu.

bu.

23. If

24.

will 35

$42.12, what

yd.?"'

of 11

can

85

25 lb. cost?

cwt.

26

cost

bu.
can

corn

25. If 3 cwt.

of
be

had

for 30

pay

corn

for 140

lb. of sugar
cwt. 25 lb.?
25

bu.
cost

bu. of potatoes,how

potatoes?
$22.60, what

70 bu.
will be

"

the

cost

26.
4

ft. 6

If

of 16

$113.

3 ft. long, cast a


perpendicularstaff,
is the height of a steeplewhose
in.,what
a

180

measures

27. If

man

shadow
120

ft.?

perform

shadow

journey in

60

ft.

da.,traveling9

NEW

KAY'S

288

lir.each

how

in

day,

hr.

ARITHMETIC.

PRACTICAL

days

many

day?
merchant, failing,
paid

traveling12
28. A

owed

60

ct.

S22()0,and

the

dollar of
what

$1800:

$250; C $375.10; D $500; F $115;


$G12.12,goes to his creditors: how
on

each

on

S1320.
A

merchant, having failed,owes

29. A

his

bl.,what

4-ccnt

will it

da.

after,B

day:

loaf

weigh

will this pay

much

$250 for G

when

oz.

flour is $6

when

weigh

on

starts

in how

many
33. If William's
labors

he

property, worth

mo.

how

flour is $8
bl.?

12

hr.

travels 27

journey,and

travels the

and

days will
services

are

day, what

ought he
day?
$|, what

mo.

mi.

3G

A?

overtake

$15"

oz.

day ;

road

same

worth

mi.

I lend

long should

$300 to compensate the favor?


starts

$1080.

30 ct.

31. I borrowed

32. A

did

$800.30;

dollar?

30. If the

by
45.

receive?

each

it

perform

He

his debts.

he

can

7
a

21.
a

when

mo.,

receive

for

4|^
he labors 12 hr. a
$91.91^.
mo., when
34. If 5 lb. of butter cost
cost f lb.?
$3%.
35. If 6 yd. cloth cost $5f what
cost 7f yd.?
$6|f
cost | bu. ?
3G. If J bu. wheat
cost $f what
$^.
If
cloth
what
?
37.
$|.
cost 2 yd.
cost $2^,
If yd.
will $31J
much
38. If $29f buy 59^ yd. of cloth,how
buy?
G2iyd.
will be
39. If .85 of a gallonof wine cost $1.36, what
the cost of .25 of a gallon?
$0.40.
a

to

40. If 61.3
cost

42.

cost

$44.9942, what

will

of 1.08 lb. ?

41. If

yard

lb. of tea

of

of

yard

of cloth cost

yard

of velvet

yd.?
43. A

wheel

has

the

$0.79.

cost?
If

be

35

cogs ;

$|, what

will

^V

^^

^^.
cost $4|, what
cost
17f
$178.38J.
smaller wheel
working in

PilOFOKTlON.

revolutions

many
cogs : in how
will the smaller gain 10

it,26

If

44.

of the

largerwheel
28f.

revolutions?

instead

grocer,

289

of

use
gallon,

true

measure

will be the true measure


what
of 100
gill,
of these false gallons?
96J gal.
be 1142 feet per sec, and
45. If the velocityof sound
in a person
of pulsations
70 per min., what
the number
counted
is the distance of a cloud, if 20 pulsations
are
of seeing a flash of lightningand
the time
between
3 mi. 22G rd. 2 yd. 2\ ft.
hearing the thunder?
46. The
length of a w^all,by a measuring line,\v^s
deficient

ft. 8

643

by

in.,but

the

long,instead of
the true length

men

found

was

to be 25 ft. 5. 1

feet,its sup2)osedlength:

25

of the wall?

II.

1. If 2

225.

line

654

wliat

was

ft. 11.17

in.

PROPORTION.

COMPOUND

men

in.

$20 in 5 da.,what

earn

sum

can

\.i}--

what?

in 10 da.?

earn

OPERATIOIS^.

Solution."
ber

Since

the

requiredor fourth term of


proportion is dollars, the

the

third

is

term

men

can

earn

greater

than

is in the second

men

Since

$20.

of dollars

than

in 5

the

da.,10 da.

first term.

product of

2 and

2. If 6

men,

high, and
build

wall

80

I'rao. 19.

2 0

120

of the

term

0X|0X2O
?X^

ber
num-

proportionand

greater number

in the first

men

of dollars

can

be earned

proportionand 5
Dividing the product of 6, 10, and 20 by
(Art.223, Rule),the requiredterm is $120.
is in the second

in

2 ft.

men,

term; and since in 10 da.


in

num-

10

da., build

thick,in
ft.

how

long,2

ft.

da.

of the

term

many

wall

20

days

high, and

ft.

the

long, 3

could

15

3 ft. thick?

men

ft.

RAY'S

290

NEW

PRACTICAL

ARITHMETIC.

OPERATION.

Solution.
rumber

days, the
build

of

15

in

is

the

second

build

to

build
and
1

..

in the

^x^^X^X
,^y/^f^y^jj/\

second

since to build
to build

than

ft.

first term;

and

take

wall

of

80 Tt. long will take

wall

20

ft. long, 80

less number

in the second
since to build

days than to build


the proportion and

days than to
the proportion

wall

wall

3 ft. thick

will

2 ft. thick,3 ft.is

ft. in the

since

of
of

term

ft. is in

20 ft. in the first term;

of

term

, "
l^

high will
ft. high, 2 ft. is

^XX ^ P

proportionand

greater number

second

days

wall

3 ft. in the
e

94

men

of the

term

wall

of

what?

and
proportion,

greater number
t

10

nun,-

in the first term;

men

2 0

can

less

in

of the

men

than

days

men

term

15

is 10

term

15

wall

ber

third

Since

days.

the

required,or fourth
the proportion,is

of

term

Since

"

first term.

Dividing the product of 6, 80, 2, 3, and 10 by the product of 15, 20,


is 16 da.
3, -nd 2 (Art. 223, Rule),the requiredterm
Rule.

0^ the
2.

1. For

"

the third term, \trite that number

denomination

same

Arrange
ratio

each

if

as

as

pair of

the number
the

numbers

which

required.
forming the

with the third term, theyformed

is

pound
com-

simple

proportion.
3. Divide

the

product of the numbers in the second and


tnird terms by the productof the numbers
in the firstterm ;
the quotient
will be the fourth term
number
or
required.
3. If

man

day: at the
da.,walking 8
a

4. If
many

can

br.

men

spend

in

in 2

rate, bow
a

build

spend
mo.?

72

hr.

far will

be

travel

in

10

240
rods

18

da., by walking

day?

build

20

mi.

24

same

men

men

5. If
men

16

travel

of

fence in 12

days, bow

rd. in 8 da. ?

$150

in 8

mo.,

mi.

96.

bow

mucb

will

15

8^37.50.

PARTNERSHIP.

6. I travel

217

I travel

can

in 9

7. If $100
in

of

days

hr. each

what

mo.,

lb. be

8. If 100

carried

lb. be

10100

9. To

12

carry

will it cost
10. If

to

18

J mi.
$75 gain
$3.37f

will

Hum

far will

10

in

60

lb. 400

75

cwt.

men,

ct.,how

for 20

mi.

20

da.,build

15

what

thick, in

$168.
wall

could

time

mi

what

mi., costs $57.12:

75 mi.?

tons

far

511

for $60.60?

carry

ft.

high, 4

carried

how

hr. each?

11

in 12

gain $6

of

days

mo.?

ft.

in

mi.

291

rd.

40
20

long,5
build

men

high,and 5 ft. thick?


58|f da.
in 6 days, of 10 hr. each, dig a trench
11. If 180 men,
200 yd. long, 3 yd. wide, 2 yd. deep, in how
days
many
100 men,
can
working 8 hr. a day, dig a trench 180 yd.
24.3
long,4 yd. wide, and 3 yd. deep?
wall

87 rd.

ft.

long,8

PARTNERSHIP.

226.
for

1. A

the

called

firm,or

2. The

or

property contributed
3. The

with

assets

the

are

property

4. The

liabilities

5. The

net

and

Such

each

house,and

capital,

association

an

of business.

transaction

is

Partnership

by
the

the

amounts

firm

capital is the

an

is

partner.

of

amount

money

or

firm.

of all kinds
of

persons

association

member,

stock, is the

of

due

firm, together

it.

belongingto
are

its debts.

between
difference

the

assets

liabilities.

1. A

and

engaged in trade;
B's,$300; they gained $100: find
Solution"

The

whole

capitalwas $200;
each partner'sshare.

A's

capitalis $200 + $300

=r

$500; of this A

fgg
| of the capital;hence, A's gain
|ggr=|, and B owns
be f of $100:= $40, and B's gain will be ^ of $100=: $60.
"

owns

will

Rule.

$200

::

$100

$40, A's share;

$500

$300

::

$100

$00, B's share.

such

jmrtnefsstock

each

Or,

Rule.

into

"This

rule

parts having

General

is
a

the whole

gain
stock.

$500; then,

or

loss,as

'

partnefsstocky
gain or loss.
partner's

applicablewhen

given

stock is to each

loss to each

gain or

whole

the

part of
is part of

the whole

As

"

is the whole

Rem.

$200 -f $300

$500

Take

"

capitalis

whole

The

Or, Solution."

so

ARITHMETIC.

PRACTICAL

NEW

K/iY'S

292

ratio to

each

requiredto
other; as

in

divide

sum

Bankruptcy,

Average, 6tc.

with
a
partnership,
capitalof
^800 : A's part is S300 ; B's,$500 ; they gain $232 : what
is the share of each ?
A's,$87 ; B's,$145.
3. A's stock was
$70; B's,$150; C's,$80; they gained
2. A

and

$120: what

was

form

each

man's

share

of it?

B's,$60 ; C's,$32.
4. A, B, and
C traded together:A put in $200; B,
$400; C, $600: they gained $427.26: find each man's
share.
A's,$71.21; B's,$142.42; C's,$213.63.
5. Divide
3 persons,
the
that
$90 among
so
parts
shall be to each other as 1, 3, and 5. $10,$30, and $50.
6. Divide $735.93
in the ratio of 2, 3,
4 men,
among
5, and 7.
$86.58; $129.87; $216.45; $303.03.
A's, $28

7. A

person

left

an

estate

of

$22361

to

be

divided

children,in the ratio of their ages, which are


3, 6, 9, 11, 13, and 17 yr. : what are the shares?
$1137; $2274; $3411; $4169; $4927; $6443.
8. Divide
$692.23 into 3 parts,that shall be to each
other as ^, f, and f
$127.60; $229.68; $334.95.
among

BANKRUPTCY.

293

BANKRUPTCY.

227.
debts

Rem.
of

an

divide

is

Bankrupt

when

who

one

has

failed to pay

his

due.

of

bankrupt are usuallyplaced in the hands


whose
them
into cash, and
duty it is to convert
assignee,
the creditors.
the net proceeds among
The

"

assets

A
owes
$175; B, $500; C, $600;
failing,
D, $210; E, $42.50; F, $20; G, $10; his property is
1. A

worth

man,

what

$934.50:

will

A's, $105;

is
what

worth

$492.45

will each

3. Mr.

creditor

share?

E's, $25.50;

F's,$12.00; G's, $6.

what

can

he

pay
67

get?

his property

$1 ; and

on

ct.

on

$1;

A, $156.78; B, $117.25;C, $218.42.


failed in business,owing $37000.
His

Smith

the

assignee sold

stock

how

for expenses:

creditor's

$234; B, $175; C, $326:

owes

man

each

C's,$360;
D's, $126;

B's,$300;
2. A

be

much

for
did

and

$25000,
he

pay

on

charged $4650
the

dollar?

55%.
AVERAGE.

GENERAL

228.

General

Average

the owners
among
by casualties at

sea.

A, B,

1.

wine.

obligedto
must

each

and

of

owned
cast

sustain?

method

shipand

cargo,

freighteda

48,
45

is the

tuns

ship

36, and
overboard

24

how

of

apportioning

losses occasioned

with
tuns

much

108
;

tuns

they
of the

A, 20; B, 15; C, 10

of

Were

loss
tuns.

2. From

$15000,

at

what

$1125:

loss of

the

ship valued

there

ARITHMETIC.

PRACTICAL

NEW

RAY'S

294

at

SI 0000, with
overboard

thrown

was

divide

the

Solution.

"

valued

at

The

work

TIME.

WITH

built
B

days, and
money?

goods

the

was

and

1. A

work

at

valued

cargo

generalaverage, and what was


valued
at $2150?
A, whose goods were
General average, 4J%; A's loss,$9G.75.
%

PARTNERSHIP

229.

of 4

wall for $82;

men

days :

5 da.

men

had

how

equalsthe

should

work

men

they

of 4 X

S"

7 da.,equals the work

of
da.; and the work of 3 men
1 da.; it is then requiredto divide $82 into two
3X7, or 21 men
ratio to each other as 20 to 21; hence, A's
parts, having the same
$42.
$40; B's part is f| of $82
part is If of $82
20

or

men

:=r

2. A

put

trade

in

they gained $24:


Solution."

$60

for 5

into two

gives A

what

$50 for 4

B, $60

$50 for 4 mo.;


each

w^as

equals$60X5
parts having the same
$9.60, and
I of $24
mo.

for

$300

ratio
B

as

share?

man's

equals $50X4

mo.

$200

mo.

200

to

of $24

for 5 mo.;

for 1 mo.;

and

Hence, divide $24


300,

or

2 to 3.

This

$14.40.

Multiplyeach partnefs stock by the time it was


employed; then take such part of the gain or loss as each
partner's
product is part of the sum
of all the products.
Rule.

3. A

"

and

hire

for $54

pastures 23
horses 27 da.; B, 21 horses 39 da.: Avhat will each pay?
A, $23.28|;B, $30.71i.
for 8 mo. ; C,
4. A put in $300 for 5 mo.
; B, $400
$500 for 3 mo.:
they lost $100; find each one's loss.
A's, $24.19i|; B's,$51.61^; C's,$24.19^.
a

pasture

OF

EQUATION
5.
6

and

A, B,

what

B,

da. ;

30

cows

shall each

6. Two

hire

PAYMENTS.

pasture

cows

40

formed

for $18.12: A

28 da. :
C, 8 cows
A, S5.40; B, $6; C, S6.72.

partnershipfor

pastures

da. ;

pa}^?

men

295

16

mo.

put

in, at first,
$300, and, at the end of 8 mo., $100 more
;
B put in, at first,
$600, but, at the end of 10 mo., drew
out
$300; they gained $442.20: find each man's share.
A's, $184.80; B's, $257.40.
7. A

and
7

and

What

B, $500.

entitle

to

mo.

partners : A

are

him

half

to

the

mean

or

due
1. A

$4 for 6
24

equalsthe

1.

"

2. Divide

Rem.
the
sum

"

the

making

method
two

of

or

finding

more

ments,
pay-

and
be

sums

$4,

due

in 6

neither

that

paid so

mo.

6 mo.;

interest

$1 for 6
be

==

==

2 4

$6

30

$4

$2 +

interest 30

on

-j-24

mo.

operation.

the interest

$1 for 6 X

on

on

equals

mo.

mo.

^-

payment

by

y^O
5

mo.

the

time

to

elapse

due.

the

of

the

quotientwill

be

When

equated time
of the

is the

$2 for 3

on

Multiplyeach

till it becomes

'

then, the interest


must

of

$720.

year'sprofits?

in 3 mo.,

interest

on

end

loser?

mo.

hence, $6

Rule.

of

both

$1 for 3 X

interest

in at the

times.

due

on

mo.;

equalsthe
mo.;

the
The

"

the interest

==

$2,

period can

Solution.

on

will be

party

different

at

put

mo.,

PAYMENTS.

payments
time

average

owes

wlvdt

at

of

Equation

the

OF

EQUATION
230.

must

sum

for 12

in $800

put

sum

one

of the

productsby the sum


the equatedtime.

payments

is reckoned, its

payment, this

must

is due

on

the

of

day

product is 0; but,

be added

with

in

the others.

the payments

from

which

findingthe

2. A

owes

find the
3. A

4. A

$2,
^8,

mo.,

time

of

5. A

buy

of

of

and

that

to

are

find

mo.:

due

12

mo.:

in

of

in 10

rest

due

at

mo.

is the

what

time

from

of the

Mean

mean
mo.

as

average

paid

mo.

now

June

mo.

time

the

time.

days
is the

Ju\y 30th.

May

1, it is

first

days from

is the average

60
follows,on
15th,$200: what

payment,
that of the second; then,
90

in

for the ment


paydifixirent times, any date

reckon

to

buys goods

Counting

to the

what

1 fifth to be

goods ;

May 1st,1848, $100;


time of payment?
"

paid

1 fourth

in 6 mo.;

mo.

be

the

May 1st,

operation.

$100 X
$200 X

^"0"

6000

105=^21000

$300

) 27000
^90

bought goods

on

90

as
days credit,

Ist,$300:

what

follows:

April

is the average

payment?
3. A

on

in 8

third

which

'2d,1853, $200; June


of

goods ; S250

$750

sums

merchant

days to
equated time is
is,July 30th.

2. I

of

mo.

findingthe Average or

105

the

in 8

tal^en from

be

days

due

all?

several

Solution.
60

$4,

mo.

; the

mo.

paying
In

average

mo.;

sums.

and

mo.,

in 8

payment.

worth

$200

2 fifths in 5

credit:

in 5

remainder

payment?

of

1. A

both

worth

$300;

time

may

paying

$6, due

the

231.

and

payment.

8 mo.;

time

mo.,

in 5 mo.,

$500

owes

6. I

of

of

buys $1500

ARITHMETIC.

in 4

due

time

mean

in 2

due

time

average
B
owes

find, the

PRACTICAL

NEW

KAY'S

2m

time

Aug.

6th.

merchant

bought goods as follows: April 6, 1876,


$1250; May 17, 1876, on 4 mo., $4280; June

mo.,

21, 1876,

on

payment?

mo.,

$675:

what

is the

average

time

of

Sept.12, 1876.

AVERAGE.

297

AVERAGE.

is the

232.

Average

average

priceof

their

mix

pounds

lb.,worth
worth

50

ct.

of

known.

are
jDrices,

of

tea, worth

lb. :

or
findingthe mean
the ingredients composing

when

mixture,

it,and
1. I

method

what

is

40

ct.

with
lb.,

lb. of

worth

-40

1.60

-50

rrr

3.00

the

mixture

?
OPERATION.

Solution.

4 lb. at 40 ct. per lb.

"

$1.60, and

then, 4 -f-6

lb. at
=:^

50

lb.

10

ct.

worth

$3.00;
$4.60;hence,

are

worth

are

are

10

) 4.60
"746

Rule.

Divide

"

the

whole

by

cost

will
ingredients
; the quotient

whole

the

be the average

number

or

of

price.

mean

6 lb. of sugar, at 3 ct. a


lb.,with 4 lb.,at 8
be worth?
will 1 lb. of the mixture
lb.,what

2. Mix
ct.

5 ct.
3. Mix
and

25

lb.,at

40

lb. sugar,
25 ct.

12

at

ct.

lb.,25 lb.,at

is 1 lb. of the mixture

what

18

worth?

19|
4. A

5. I have

the

stood
A.
3

rest,$9 each

6. On

M.

as

from

find the

average

the

mercury

certain

till 1 P.

da}^,55"

follows:

till 7 P.

ct

gal.water, w^ith 12 gal.wine, at 50 ct. a


40 ct.
worth?
is 1 gal.of the mixture
worth
30 sheep: 10 are
$3 each ; 12, $4 each ;

mixes

gal.:what

ct.,

M.,

from

M., 70"

to

was

the

sunrise?

till 10

; from

73" ; from

what

sunrise

day

mean

M.,

1 till 3

thermometer

the

in

A.

7 P. M.

S5.

value.

at

P.

03"^

from

10

75" ; from
of the next

M.,

till 6 A. M.

temperature

of

the

da}^,
62X".

DEFINITIONS.

233.

1. Involution

into itself
2. A

one

or

3. The
4. The

second

5. The

third

power,

by taking the
Thus,
Rem.
square
power

X
The

"

is the

second

three

the

or

or

power

cube,

three

power
of

as

as

product obtained

or

factor.

cube

of 2.

is called the square, because the


two

equal

factors

cube, because the solid

"

16, is the fourth

32, is the fifth power


is the sixth power
of 2, etc.

(298)

third

cube

is the

equal factors (Art.70).

Thus, 2 X 2 X 2 X 2
==

of

contents

of

area

The

(Art. 68).

of a number
higher powers
the fourth power^ fifth
respectively
power

of 2.

square

6. The

X 2 X

tained
product ob-

factor.

is the

times

involution.

is the

twice

8 is the third power

product

is called

product of

number

number

itself

square,

number

4, is the second

==

number

or

power,

by taking the
Thus, 2 X 2

product obtained by
is the

first power

times.

more

is the

power

of
multiplication

is the

of

denominated

are

power

sixth power
of

2; 2X2X2X2X2X2

2; 2 X

etc.

X 2

=6

4,

INVOLUTION.

7. The
which

the

6^

z=

7* is read

125.

To

234.

3 square,

53, reads

In

1. Find

number

denoting the

is to be

raised.

the 2 denotes

cube, the

number

cube

of 75.

to

the square

denotes

1 fourth power j 9^, %

raise

the

number

given

Thus, in 3 2, read
32=9.

is

exponent

299

any

the cube

Jifthpower,

to

power

of

3; hence,
of 5; hence,
etc.

power.

OPERATION.

Solution.

"

75

multipliedby

5625; this is the

multipliedby
cube

75

square
is

75.

is

"

Obtain

421875; this is the

factor as many
of the power.

productin which

times

as

75

5625

of 75.

Rule.
a

of

75

there

are

5625

75

7_5

375

28125

525

39375

5625

421875

the number

is taken

units in the

as

exponent

EVOLUTION

DEFINITIONS.

235.

is the process of
equal factors.

1. Evolution

into two
2. A

more

or

of

root

one

of

number

is

number

is

resolvinga

ber
num-

the

more

two

or

equal factors.
3. The

of

root

square

of

one

two

equal

three

equal

factors.

3 is the square

Thus,

4. The

cube

of

root

of

root

9; for

number

X 3.

is

one

of

factors.

Tlius,3 is the cube

of

root

27;

for 27

3 X

=r

3 X

3.

5. The

are
higher roots of a number
the fourth root,fifthroot,etc.
respectively

3 is the fourth

Thus,
fifth root

of

6. The

shows

for 243

radical

that

7. The

sign to

243;

root

sign

its root
index

show

the

(300)

is

of
3

81;

for 81

number

3 X

3 X

^ X

3.

3 is the

X 3.

placed

|/

is to be
a

denominated

before

number

extracted.

number
of the

placedabove
root.

the

radical

EVOLUTION.

Rem.

is

It

"

customary, however,

to

omit

2, the

index

of

the

root.

square

Thus, VH^

^"^
read

301

the

is read
fourth

the

8. A

the

is read
cube

root

perfect

root

of

root

square

of

25; hence, l/^=r5.

27; hence, f~2T"Z.

of

^T6

is

16; hence, Vl6:=2.

is

power

whose

one

be

tained
ob-

root

can

^V are

perfectcubes;

exactly.

16 and

and

25

Thus,

Jg

are

9. The

if are perfectsquares;
perfectfourth powers.

squares
in the

exhibited

and

cubes

27 and

of the first ten

numbers

are

following

TABLE.

Rem.

"

The

numbers

correspondingnumbers
in the

third

10.
obtained

An

in

the

first line

in the second

are

line,and

square roots of the


the cube roots of those

the

line.

imperfect

power

is

only approximately.

Thus, ]/Tz=: 1.41421 +.

one

whose

root

can

be

RAY'S

302

NEW

PRACTICAL

ARITHMETIC.

SQUARE
To

236.

find

the

ROOT.

number

of

figuresin

the

square

root.

1. The
100

is 10

10

all numbers

are

When

number

root consists

2. The

of

of

is

three

3. In

like

When
root

consists

of

so

And
1st.

If

on

between

figure;

one

all

fore,
there-

its square
figures^

tico

consists

three

10, and
and

the square

10000

are

all

root
bers
num-

four

and between 10
figures,
of two figures
fore,
consisting
; thereor

of three

or

four figures^its

figures.

it may

manner

number

of

is

100

consists

consists

root

of

square root
1 and 100 are

two

or

one

100

between

100;

number

of

of

root

all numbers

are

When
square

the

figure.

one

consistingof
100

1, and

is

of
consisting

consists

square

10000

and

(Art. 235, 9, Table);between


and
consistingof one or two figures,

numbers
1 and

of

root

square

be

shown

of five or

six

that,
its square
figures,

figures.

therefore,

number

be

pointedoff into periodsof tivo figures


the number
as
of periodswill be the same

each,the number
of figuresin the square root.
2d. The square of the units will be found in the first
period,the square of the tens in the second period,the
square of the hundreds in the third period,etc.

ROOT.

SQUAKE
To

237.

point

off

303

number

into

j^eriodsof

two

figureseach.
1. Point

off 368425.

2. Point

off 6.843256.

3. Point

off 83751.42963.

Rule.

Place

"

every second

6.843256
83751.429630

point over
from units

order

The

1.

Rem.

368425.

first

period on

the order
to the

the

units,and then over


leftand to the right.

left of the

integralpart of the
number
will often contain
a
singlefigure.
Rem.
2.
the first period on
When
the rightof the decimal part
contains but a singlefigure,
to complete
a
ciphermust be annexed
the period.
"

but

"

4. Point

off 864326

5. Point

off 97285.46138

To

238.

the

extract

1. Extract

the

; 4.758462

; 75300

square
root

square

; 7584.3769.
; .046827

root

of

; .0625 ; .625.

number.

of 256.
OPERATION.

Solution.

"

Point

off 256

figureseach

by placing a
(Art. 237, Rule).

into

periodsof two
2
6 and
point over

2 5

6(16

26)156
156

(Art. 235, 9, Table) is 1; its root is 1;


place the root 1 on the right and subtract the square 1 from 2;
the remainder
is 1, to which
bring down the next period 56.
Double
the root 1 and place the result 2 on
the left of 156 for a
trial divisor.
Find
how
times 2 is contained in 15 (making
many
the result is
allowance
for subsequentincrease of the trial divisor);
6; place 6 in the root on the rightof 1 and also on the rightof 2,
the trial divisor;then 26 is the complete divisor.
Multiply 26 by 6
The

and

largestsquare

subtract
256

is

the
a

in

product 156

perfect square,

from

and

156; the remainder


its square

root

is 16.

is 0.

fore,
There-

HAYS

304

PRACTICAL

NEW

GEOMETRICAL

ARITHMETIC.

EXPLANATION.

findingthat the sq. root


will contain
of the given number
two
places of figures(tens
and
units),and that the figure
in tens' place is 1 (ten),form a
square figure(A) 10 in. on each
contains
side, which
(Art. 67)
100
sq. in.; taking this sum
After

number

whole

the

from

of

156
sq. in. remain,
squares,
which
ber,
correspondto the num-

156, left after

subtracting

above.
It is obvious
it

preserve

equally;and,

that to increase

side of

the

and

Now

C,

of

contents

square

small

requirea

at the

sides of the square A.

two

in the numerical

determine

Now

figureA, and

time

same

length and breadth must be increased


square, both
since each side is 10 in. long,it will take twice 10, that

is,20 in.,to encompass


10 is doubled

the

the

breadth

A.

After

square
to

B, C, and

their contents

are

D,

the
must

this reason,

operation.
of the

addition to be made

increasingeach

(D) of the

complete

For

sa.vie

breadth

entire square;
be equal to

obtained

side
as

each

equally,it
each

hence, the
the

to

of the

will

ures
fig-

superficial

remainder,

156.

by multiplyingtheir length by

their breadth.

figurein the units' place that is,the breadth of B and


be found by trial,and it will be somewhat
less than the
must
C
of times the length of B and C (20) is contained
in the renumber
mainder
than 7 times; let us
(156). 20 is contained in 156 more
27 for the whole
length of B, C, and
try 7: 7 added to 20 makes
D, and this,multiplied
by 7, gives 189 for their superficial
contents;
than
taken
too
this being more
156, the breadth
(7). was
great.
and
breadth
6
the
of
for
to
20
6
D; adding
Next, try
length
gives
and
breadth
of
26
the
26 for the length
B, C,
D; multiplying
by
of
contents
B, C, and D.
(6) gives 156 for the superficial
Then

the

"

"

Hence,

arranged

256 sq, in.


square root of 256 is 16; or, when
in the form
of a square, each side is 16 inches.
the

are

KOOT.

SQUAKE
the

2. Extract

305

of 758.436.

8qiiare root

OPERATION.

Solution.
.

periods

into

by

Point

"

,"

of

off

two

and

Then

root

mainder
is

as

in Ex.

Rem.

last

and

square,

is 27.53

root

the

square

square

4.

root

The

root

of

Extract

Solution.

the

is .61237

Rule.

"

-f

45)2943
2 7 2 5

5503)21860
16

5 0 9

periodsof decimal ciphers,


requirednumber of decimal

more

or

to any

of

root

root

square

.375, to five decimal


of

||f

the

256

numerator

625

is 25

is 16, and

(Ex. 1); then,

the

if.

reduced

"

-4-

5 3 5 1

square

is

I .O

^ '^^

of
root
square
of the denominator

||f

"

-f.

the

"

j .i o

its

By bringing down one


the operationmight be continued
places in the root.
3. Extract

re-

"

Solution.

Therefore, 758.43G

imperfect

square

The

1.

A/o-rro

right (Art. 237,


find the figuresof

is5351.

an

left, and

the

to

the

to

Rule),
the

over

"^0U(

and

over

^ro^o^.
' ^ ""'*

each

ngures

a
placiiiijj
point

then

758.43G

to

of

decimal

root of
square
then, the square root

The

is .375,

places,is .61237 (Ex. 2);

1. Point

off the given number

into

periodsof

two

figureseach.
2, Find

3.

greatestsquare

in the

periodon
first

the

left
;

tract
in division;subrightslike a quotient
the square from, the period,
and to the remainder
bring
the next periodfor a dividend.
Double
the root found, and place it on the leftof the

place its
down

the

dividend

root

for

trial divisor

on

the

trial divisor.

is contained

the
times
many
dividend,exclusive of the

Find

in the

how

righthand, figure;place the quotientin


the rightof the trial divisor.
on

the root^ and

also

ARITHMETIC.

PRACTICAL

NEW

RAY'S

306

divisor by the last figureof the


Multiply the complete
root; subtract the product from the dividend,and to the
remainder
bring down the next periodfor a new dividend.
4.

the ivhole root

5. Double
and

the luiniher is

When

1.
"

may

be

continued

root

by bringingdown

Rem.
when

To

2.
"

both

terms

not

the

are

then

trial

divisor,

until all the

manner

the

To

the square

root

square

extract

decimal
of

root

perfectsquares,
of the

reduce

of the decimal

the

extract

denominator

root

llie square

the square

root

the

fraction to

( Ex.

of the

wh(;n

both

decimal

and

(Ex. 3); (2)


).

of

square

root

by factoring.
1. Exti-Mct

the

operation
of decimal
placesin the
ciphers(Ex. 2).
fraction: (1)
a
common

requirednumber

perfectsquares,

square

Extract

239.

new

imperfectsquare,

an

periodsof

extract

and

are

extract

to any

terms

numerator

same

broughtdown.

periodsare
Rem.

the

operationin

the

continue

found, for

root

of 441.

of

perfectsquare

KOOT.

SQUAKE
Solution.

Rule.

-441

=3X

Besolve

"

the

find the product of


Extract

the

of

one

root

of

two

is

triangle

bounded

by

three

X 7

21.

its

prime factors,and
equalfactors.

two

of

the

ani^C^eto find the third


]

into

each

v^ 44l

hence,

7;

number

square

Given

240,

7X

3X

307

sides

of

right-angledtri-

side.

plane figure
lines,called
straight
a

its sides.

Thus, D E F is

2. When

they

of the

one

form

triangle;its sides

sides is

and
right-angle,
triangle.

right-angled

are

E, E F, and

F.

to another,
perpendicular
the triangleis called a

A
Thus, in the triangle

beingperpendicularto

right-angleat C; hence,
angled triangle.

side

hypotenuse

Thus,
A

the

in

other

ABC

C, A

perpendicular.

two

is the

C, they form
is

opposite the

; the

G, the side A

the side B

3. The

right-

called

right-angleis

sides,the

base

hypotenuse, B C

and

the

the

the

pendicular.
per-

base, and

4.

of

Proposition.

Draw

The

"

square

described

on

the

the

sum

of

is equal to
triangle
right-angled

described

with

ARITHMETIC.

PRACTICAL

NEW

KAY'S

308

triangle,ABC,
right-angled

side

the

in.,and

in.; then, the side A

the square.^

sides.

the other two

on

hypotenuse

the
B

side

will be

each side of
square on
and divide each square into
the triangle,
smaller squares of 1 in. to the side. Then,
5 in.

Describe

described

the square

la-j-9

=;=

on

25

the base

squares
will contain

we
proposition

find

the

base

deduce

following

hypotenuse; To the square of


and extract
of the perpendicular,

or

the

perpendicular;From

of the hypotenusesubtract
givenside,and extract the square
base

the

the

the square
root of the sum.

square

1. The

two

add

find

inches.

square

Ist. To

"

the square
2d. To

and

this

5. From

Rules.

the
A

B will contain

on

inches, and

25

square
described

the square
root of the

the

-of the other


difference.

and

triangle
perpendicularof a right-angled
30 and
is the hj'potenuse?
40: what
50.
are
2. The hypotenuseof a right-angled
triangleis 100,
is the perpendicular?
and the base 60: what
80.
3. A castle 45 yd. high is surrounded
by a ditch 60
side
yd. wide : what length of rope will reach from the outof the ditch to the top of the castle?
75 yd.
4. A ladder 60 ft. long reaches a window
37 ft. from
the ground on
side of the street,and, without movone
ing
it at the foot,will reach one
23 ft. high on the other
side:

find the width

of the street.

102.64+

ft.

CUBE

5. A
island
from
the

top

breadth

of the

high

in

diameter;
the

the

tree

the
line

the

to

of

center

ft.

600

further

stream, the land

is 20

is the

ft. long,16

distance

long reaches

shore

Given

from

ft.

wide,

of

one

the

being

533.43

-f-ft.

and

ft.

high :

corners

to

12

the lower

find

to

square

is

side

28.28

of

area

what

each

on

circular

corner?

oppositeupper

241.

is

309

level ?

room

what

of

of

same

6. A

the

ft. in

100
the

ft.

140

tree

HOOT.

-f

ft.

its side

(Art. 67).
Biile.

Extract

1. The

area

"

the

of

length of

2. The

3. The

4. A
is the
9

times

of

as

5. What

of

square

field

lengthof

the

area.

sq. rd.

table

square

length of

contains

side?

one

circle is 4096

1. The

the side of

is the

length of

rd.

each

on

neld

square

is

the

64

yd.

side

which

what

contains
rd.

side of

square

lot

taining
con-

208.71+

find the

and

ft. 10 in.

12

one

root

number
of

1 is

10

figure; therefore,

are

ft.

ROOT.

of

figuresin

1, and

(Art. 235, 9, Table);


numbers
consistingof one,
1

rd.

rods?

square

is 10

between

79

what

yd. :

sq.

measures

many

cube

is

equal area?

CUBE

To

what

8 sq. ft. 4

1 acre?

242.

side?

of

square

of

root

field is 6241

square

is the

area
a

one

surface

sq. in.: what

side of

the square

all

between

two,

cube

the
1
or

numbers

and

the

cube

root

1000

root.

of 1000
are

all

figures,and
consistingof one

three

RAY'S

310

When
cube

number

consists

root

2. The

cube

consists

of

root

ARITHMETIC.

of

figure.

of

1000

is

and

1000

and

100

the

1000000

its
figures,

cube

root

all

are

and
figures,
consistingof two

all numbers

are

three

or

10, and

consistingof four,five,or
10

two,

one,

one

100; between

is

1000000

PRACTICAL

NEW

six

of

bers
num-

between

figures;

therefore,
When
cube

consists

root

3. In

like

When

of

on

so

consists

of

or
eight,
figures.

of

three

that.

seven,

nine

figures^

therefore,

number

If a

be shown

it may

manner

its
figures,

six

of four, five,or

figures.

two

consists

root

And

consists

number

its cube

1st.

number

be

pointedoffinto periodsof three figures


the number
as
of periodswill be the same

each,the number
in the cube root.
of figures
2d. The cube of the units will he found in the first
period,
the cube of the tens in the second period,the cube of the
in the third

hundreds

To

243.

point

period,etc.

off

number

into

periods of

three

figureseach.
1. Point

off 876453921.

2. Point

off 7.356849227.

3. Point

off 37683.5624.

Rule.

Place

"

Rem.
number
Rem.
contains
the

The

1.

"

first

will often
2.

"

When

but

one

or

7.356849227
37683.562400

point over
from units to
a

every third order

87^453921.

period on

the order

the

the

units,and

leftand

to the

then

right.

left of the

integralpart of the
contain but one
or
two
figures.
the first period on
the rightof the decimal
part
two
figures,
ciphersmust be annexed to complete

period.

4. Point

over

off^ 138975462;

3.561325482;

684536.256403.

CUBE

5. Point

244.

off 2756.56843

To

1. Extract
Solution.
13824

the

extract

the

cube

Point

"

KOOT.

cube

the

13; the

cube

bring

Square

the

of 13824.

root

result is

is

down

Multiply 2 b\^ 4
1200;

4, and

the

4X4

times

root

on

is the

1200

the

square

this

sum

cubes, and

which

in the

from

5824

8000

cu.

number

the whole
cu.

in.

in.;

are

correspondto the number


numerical
operation.

left,
5824

5824;

the

4; add the products240 and

Multiply 1456 by

5824; the
its cube

EXPLANATION.

findingthat the cube root of


the given number
will contain
two
placesof figures(tens and units),and
that the figure in the tens' place is
a
2, form
cube, A, Fig. 1, 20 (2 tens)
inches
long, 20 in. wide, and 20 in.
high; this cube will contain, (Art.

in

2.

complete divisor.

product 5824 from


a
perfectcube, and

the result,1200, is the

is contained

rightof

After

20X20X20

5824

next

GEOMETRICAL

70,)

10

"=

1456

many

1456

is

13824

240

30=

2X4X

by 30,and

and

the

i824

2X2X300=:120()

period 824.
multiply it by 300;

in the

sum

subtract

Therefore,

and
how

4; place 4

13824(24

5,
the

Find

OPERATION.

from

root

trial divisor.

of

number.

off

remainder

which

take

of

root

.0064.

into

subtract

IG to

98451.3276; .856375;

periods of
three
figures each
by
4
placing a point over
unci 3 (Art. 243, Rule).
The
largestcube in 13
(Art. 235, 9, Table) is 8;
is 2; place the
its root
2 on
the right,and
root

to

311

remainder

root

is 24.

is 0.

RAY'S

312

ARITHMETIC.

PRACTICAL

NEW

figureA, and at the sarr.e timo


each receive
preserve it a cube, the length,breadth, and heightmust
an
equal addition. Then, since each side is 20 in. long,square 20,
of square inches in each
which
400, for the number
gives20 X 20
It is obvious

that

increase

to

the

of

face

cube; and

the

sides,multiply the
because,by

addition

an

by 3, which

400

the

in

inches

square

since

3 sides.

This

gives 1200
1200

be

made

for the

is called the

of the

of it,the thickness

means

is to

additions

three

to

number

trial

of

divisor;

is determined.

By examining Fig. 2 it will be seen that,after increasingeach of


3 oblongsolids,C, C, C,
there will be required
the three sides equally,
of the

the

sides,and

each- of the

length as

same

the additional

heightare

each

D, whose

length,breadth,

additional

thickness.

same

as

and

each

solid contents

rectangularsolids,the three oblong solids,and


togetherbe equalto the remainder (5824).
find the

Now

It

additions.
.

thickness

of

times

the

is

contained

the

the

(5824). By trial,we
is

contained

proceed
the

find

to

different
of

contents

the

the
face

70);

the

are

find

1200

in

5824;

cube, must

of

The

solid

found

tions,
addi-

by

tiplying
mul-

one

of sq. in. in
thickness
(Art.

400

face of each, and


sq. in. in

first three

number

by

there

the small

sq. in. in the


400 X S
1200

Fig. 2.

"

face of the

three;then, multiplying
by

ness)
(thethick-

The solid contents


gives4800 cu. in. for their contents.
three oblong solids,
C, C, C, are found by multiplyingthe
of sq. in. in the face by the thickness;now
there are 20 X 4
in. in

one

three; then

the

(1200)

first three

are

the

as

number

contents

solids.

B, B, B,
the

the

of

same

dividend

times

the

of the

trial divisor
in

and

always be

will

less than

something

thickness

and also a cube,


thickness;

height are

the

Hence,

whose

face of

each,

and

80

240

sq. in. in

one

of the

number
80

sq.
face of the
=

multiplyingby
(thethickness),
gives 960 cu. in. for
their contents.
Lastly,find the contents of the small cube, D, by
multiplyingtogetherits length,breadth, and thickness;this gives
04 cu.
in.
4X4X4=4

ROOT.

CUBE

If the solid contents

the

added

be

additions

their

margin,

will

be

of the

number

the

5824

of

Hence,

arranged in
root

13824

in

Br=4800

in.

C=:

cubes

in.

cu.

960

D=

64

in.

"

"

"

"

5 8 2 4

Sum,

are

cube, each

side is 24

in.;that is,the cube

is 24.

It is obvious

"

ADDITIONS.

first cube,

cu.

of

the form

of 13824

Kem.

be

when

cu.

small

remaining after forming the


A.

several

together,as
sum,

313

arranged in the
2. Extract

that the

same

the

way

cube

in the

margin may readily


operationof the example.

additions

as

in the

of 413.5147.

root

OPERATION.

413.5

14

700(7.4

343

7X

7X300r^

7X

4X

14700

70514

840

30=

4X4

16

15556

74X74X300
74X

SX

3 0=

62224

1642800

8290700

11100

25

5X

8269625

1653925

2 10

Solution."

Point

placinga point over

Rule).

Then

mainder
cube

periodsof

3, and

4 and

find the

is 21075.

root

off 413.5147 into


then

figuresof

the

root

Therefore, 413.5147

is

0 to the
as
an

each by
figures
right(Art.243,

three

in Ex.

1.

The

imperfectcube,

last

and

nv

its

is 7.45 -|-.

By bringingdown one
the operationmight be continued
placesin the root.
Kem.

over

7 5

"

3. Extract

the

cube

root

or

more

to any

of

periodsof decimal ciphers


requirednumber of decimal

yVg"?-

Solution.
cube

of the

root

of the

denominator

13824

2197

numerator

is

is 13

and

the

24; (Kx. 1); then, the cube

tVbV? i" \l-

root of

Solution.

cube

the

4. Extract

i reduced

"

of

root

to

^.
is .8.

decimal

The

places is .928; (Ex. 2); then,

decimal

three

root

cube

The

"

AKITIIMETIC.

PRACTICAL

NEW

KAY'S

314

cube

root

cube

the

of .8 to

root

of

is .928 +.

Bule.

off the givennumber

1. Point

"

into

periodsof

three

figureseach.
2. Find

3.

the

firstperiodon

the

left;

tract
in division ; sublike a quotient
right,
and
the cube frojn the period,
to the remainder
bring
the next periodfor a dividend.
Square the root found, and multiplyit by 300 for a

place its
down

greatestcube in

the
root

trial divisor.

the

on

Find

how

times the trial divisor is

many

tained,
con-

dividend,and place the quotientin the root.


4. Midfiplythe preceding
or figures,
of the root by
figure,
the last and
by 30, and square the last figure;add the
in the

productsto the trial divisor ; the sum is the completedivisor.


5. Multiplythe completedivisor by the last figureof the
root; subtract the product from the dividend, and
remainder
6. Find

operationin
broughtdown.
Rem.

the next

bringdown

periodfor a

trial divisor

new

the

same

as

before,and
until

manner

new

all

dividend.
continue

the

the

to

the

periods are

imperfectcube, the operation


be continued
of decimal
to any requirednumber
places in the
may
root by bringing down
periodsof decimal ciphers. (Ex. 2).
fraction: (1)when
Rem. 2. To extract the cube root of a common
both terms
are
perfectcubes, extract the cube root of the numerator
and then of the denominator; (Ex. 3); when
both terms
not perare
fect
1.
"

When

the

number

is

the

fraction

to

an

"

cubes, reduce
root

of the

decimal.

(Ex. 4).

decimal

and

extract

the

cubo

ROOT.

CUBE

Extract

the

245.
side

Given

of

root

the

solid

of

contents

cube

to

find

its

(Art. 70).

Rule.

"

1. The
find

the

2.

is the

cubical

Find
216

The

another

much.

of

cube

root

box

of

cubical

the

cellar

solid

contents.

1953.125

are

piled

are

of

each

contains
the

512

box

in

the

of

form

side?
half-inch

cubes

450

cu.

yd.

17

cu.

23

its dimensions?
the
ft.

side

broad,

side

of

cubical

of
and

cube

48

cubical
vessel

ft.

equal

mass

high.

vessel
that

to

is 1

shall

288
144

foot:
contain

find

the

times

17.306+

ft.

what

inside?

contains

excavation

ft.

12.5

cubes

of

ft.:

cu.

side.

one

length

dimensions

are

long,
6-

of

cubical

w^hat
5.

contents

length

the
4.

the

what

3. A
are

Extract

Sixtj^-four3-incli

cube:

of

cube

315

in.
ft. :
ft.
ft.
ft.
side
as

in.

MENSURATION.)!

OF

I. MEASUREMENT

SURFACES.

DEFINITIONS.

1. A

246.

2. Lines

are

3. When
an

two

length without

has

line

either
lines

breadth

or

^^^^

curved.
or
straight
form
meet, the}'^

ness.
thick-

~^^-

angle.
Rem.

The

"

of the

point

at

which

the

lines meet

is called

angle.
either

Angles are
rightangles.
4.

5. When

two

dictular to

each

acute., obtuse^or

straightlines are
other, they form

perpen-

right

angle.
6. An

acute

angle is

less than

right

angle.
7. An

obtuse

angle

is

greater

than

rightangle.
8. When
where
9. A

without

straightlines are ever^^equallydistant they are parallel.


has
surface
length and breadth
tw^o

thickness.

(316)

the

vortex

MENSURATION.

10. Surfaces

either

arc

Thus, the surface of

plane or

table

317
curved.

floor is

or

plane; that

of

ball

or

g^lobeis curved.
11. A
bounded

plane

by

12. A

is

figure
or

one

of

plxme

plane figurebounded

surface

lines.

more

is

polygon

portion

by straight

lines.
Rem.

"

The

straightlines are
a
polygon is the

perimeter of
13. A

bounded

Rem.

"

fall upon
of the

is

triangle

by
If

three

the base

the sides of the

/1\

taken
the

from

polygon;the

of all its sides.

sum

plane figure
straightlines.

side be

one

called

for the

base, the

oppositeangle

\^

let
perpendicular

is called

the

altitude

triangle.

14. A

is

quadrilateral

planefigurebounded

by

four

straightlines.
15. There

are

the

16. A

three

kinds

and
trapezoid^

is

the

the
:
quadrilaterals
gram.
parallelo-

of

quadrilateral
v;ith no
sides parallel.
two
/
17. A
is a
trapezoid
quadrilateral
/
with
only two sides parallel.
18. A
is a
parallelogram
quadri/
its opposite sides equal
lateral with
/
and
parallel.
Rem."
fall upon

trapezium

If

one

side be

the base from

parallelogram.

the

pezium^
tra-

taken

as

the base, the

oppositeside

let
perpendicular

is called the altitude of the

KAYS

318

19. A

NEW

PKACTICAL

is

ARITHMETIC.

parallelogramwith
equal,and its angles not rightangles.
with
20. A rectangle is a parallelogram
all its anglesrightangles.
is a
21. A
rectanglewith all
square
its sides equal.
22. A polygon of five sides is called a
pentagon ; of six,a hexagon ; of eight,an
rhombus

all its sides

etc.

octagon,
23. A

is

diagonal

line

joining two

anglesnot adjacent.
is a. plane figure hounded
24. A circle
hy a curved line,every point of which
from
is equally distant
a
point within
called the

center.

25. The
the

line which

curved
The

26.

the

circle is

bounds
of

diameter

through
the

of

circumference

the

circle

figure.

straightline passing
terminated,both Avays, by

center, and

is

circumference.

27. The
the

from

of

radius

the

to

center

straightline drawn
circumference;it is half the

circle

is

diameter.

To

247.

find the

of

area

parallelogram(Art.246,

18, 19, 20, 21).


Rule.

"

Multiplythe

Explanation.
is

"

The

equal to

base
of

area

the

area

having an equalbase

by

lelogram
paral-

of

the altitude.

and the

angle,
rectsame

but the area


of the rectangle
is
altitude;
equal to its length multipliedby its
is equal to
breadth; (Art.68); hence, the area of a parallelogram
its base

multiplied
by

its altitude.

MENSURATION.

1. How

feet

square

many

319

in

floor

17

ft.

255

ft. Avide?

15

2. How

land

of

acres

many
lengthof which

rd.,and

is 120

in
the

3. How

in

acres

many

the
parallelogram,
breadth
perpendicular
a

each
field,

square

is 65 rd. ?

which

4. How

each
distance

in

acres

side

A.

26

many

sides

two

being 16

form
the

rd., and

35

measuring

between

field in the

63

A.

side

of

the

and

square,
6. A
how

in

area

each

15

difference
others

two

table

is

Solution."

ft. 4

Solution.

ft. 4

in.r=3J

in.,or

9|

rd. ?
A.

between

sq. rd.
floor 30 ft.
80

ft.;2 ft. 10 in. 2|


-i/}^oy^lJ_ ^ gq, ft. Qr

or

in.;2

ft. 10 in.

40X34^=1360

square

many

9. A

room

contain

80

10. How
floor 18
11. How

-i/ft.;

"

sq.

34

in.; 1360

in.;then the
--

144=^9

feet in

marble

face
sur-

sq. ft.

slab 5 ft. 6 in,

1 ft. 8 in. wide?

long, and

or

sq. ft.

many

8. How

to

is

table

7. How

take

3 ft. 4 in.:r=40

"

the

long and

50 sq. ydft. square.


in. long, and 2 ft. 10 in. wide:

then, the surface of the table is

64 sq.

of

sq. rd.
bus,
rhom-

sq. ft. in its surface?

many

of

65

perpendicular
3

5. Find

and

sq. ft.

rd. ?

84

long

dj-sq.

yards

square

in

ceiling25

ft.

ft. 9 in,

ft. 3 in. wide?

21

is

10 ft.

60 sq.

yd. 7

how

wide

long:

sq. ft. 27 sq. in.


it be to
must
8 ft.

sq. ft.?

yards of carpet,1^ yd. wide, will cover


ft. long and 15 ft. wide?
20 yd.
many

many

line 3

yd.

flannel,
| yd. wide, will it
6 yd.
cloth,li^ yd. wide?

yards
of

of

KAY'S

320

How

12.

lake

to

PRACTICAL

NEW

floor

cover

of

yards

mtiny

ARITHMETIC.

in.

ft. 8

21

will it

carpet, IJ yd. wide,


and

long

13

ft. G in.

wide?

25iyd.
rectangularfield

13. A

its width

find

To

248.
Rule

the

what

be

must

lOf

of

area

Multiply half

"

long:

1 A. ?

contain

to

rd.

is 15

the

rd.

trapezoid(Art.246, 17).

of

sum

the

jxiralldsides by

the altitude.
Explanation.

the

having

grarn

is one-half

area

The

"

base of

prirallcloaltitude and equal


of the parallel

same

the

sum

sides of the

trapezoid.

1. The

sides
parallel

2 ft. 11

in.; its

of

altitude

2 ft. 2 in. and

trapezoidare

is its area?

in.: what

is 11

2 sq. ft. 47^ sq. in.

field is in

2. A

sides
parallel
is 32

rd.

of

rd.,and

the

is 25

how

the form

other

19

one

rd. ; the

4 A.
3. How
in. wide

many
the

on

side,the

upper

To

249.

the

"

The

is one-half

having

the

the
same

area

of

area

the base

Explanation."

width

64 sq. rd.

yards in a pieceof roof 10 ft. 8


lower
side,and 6 ft. 2 in. wide on the
length being 12 ft.? 11 sq. yd. 2 sq. ft.

Multiplythe
product.

Rule.

the

square

find the

Ist. When

of

field?

in the

acres

many

trapezoid
;

area

of

base and

triangle.

and

altitude

base

by

of

the

are

given.

and
altitude^

parallelogram
/

half

/\

triangle

altitude.

take

/
\^
_\^/

MENSUKATION.

2d. When

1. From

sides

sum

separately,
Midtiply the half-sum

and

"

half

and

1. The

base

ft.: what

of

is its

2. One

the

extract

is 18

rd.

how

3. What

is 12

three

sides

take

ft. and

15

in the

acres

of

area

the

lot?

its altitude

2 A.

of
triangle,

altitude

of

area

5. The

sides of

sq.

which

yd. 5

trianglewhose

sides

QQ

To

other

Rule.

find the

of

area

this side
76 sq. rd.
the base

50

sq.

13, 14,

are

84 sq. ft.
ft.: what
sq. ft.

yd. 6

trapezium (Art. 246, 16)

irregular
figure.
1. Divide

"

2. Find

the

areas

1. Find

the

area

of which

the

by diagonals.
figureinto triangles
and add. them together.
of the triangles,
of

diagonalis

diagonalfrom

the

field in the
50

rd. and

of

form
the

1. To

251.

oppositeangles 30

diameter

is

"

find the circumference

given.

Multiplythe
Prac.

of

21,

diameter

by

trapezium,

to
perpendiculars

rd. and

7 A.

Rule.

sq. ft.
the perpendicula

sq. ft, 99 sq. in.

30, 40, and

area?

250.

12

ft. 9 in.?

16

triangleare

gether
to-

product.

ft.

15

is the

the

the

lot is 44 rd.,and
triangular
from the angle oppositeto

ft. 6 in. and

4. Find

remainders

90

many

is the

three

of

root

square

11

the

the

area

side of

the

triangleis

distance

or.

of

side

2.

and

given.

are

the

Rule.

each

three

the

321

3.1416.

130

20

rd.

sq. rd.

circle when

the

KAY'S

322
2.

the

Conversely:

is

Divide

"

1. The

the

diameter

the

find

to

circumference

Rule.

ARITHMETIC.

PRACTICAL

NEW

of

diameter

circle when

given.
circumferenceby

of

fl.: what

circle is 48

3.141G.

is the

fl. 9.56 in.

150
2. The

circumference

of

fl.: what

circle is 15

diameter?
diameter

of

wheel

what

fl.:

is 4

is its circumferen
fl. 6.8 in.

12
4. If

the

girthof

in.,what

fl. 5

is 12

tree

5. What

is the

of

circumference

1. To

the

find

area

of

radius

the

when
circle,

given.
Rule.

2.
area

"

root

Multiplythe

Conversely:
is given.

Rule.

of

"

Divide

the

1. Find

to

the

square

find

area

the

of

the radius

radius

2. The

the

of

area

circle whose

post

more

so

3.1416.

(drcle when

the

the square

extract

area

of

sq.

radius

yd.

long
that

he

rope
may

will
graze

it take
over

to
1 A.

is 21

fl.

8 sq. ft. 64

circle is 6 sq. fl. 98.115


and circumference?
a

2 ft. 11
3. How

by

quotient.

its diameter

are

of

by 3.1416,and

153

in.

eter
earth,the diam24856+ mi.

the

mi.?

being 7912

is

is its

3 fl. 11.43

diameter?

252.

is the

fl. 9.3 in.

3. The

cumferen
cir-

sq. in.

in.; 9
fasten

sq. in.
:

what

fl. 1.9+
a

horse

of grass, and

in.
to
no

7 rd. 2 ft. 3 in.

MEASUREMENT

4. Two
same

and

10
circles,

center

what

OF

16

is the

SOLIDS.

ft. in

diameter, have

of the

ring between

area

circumferences?
5. The

122
of

area

circle

323

is 1 square

the
their

sq. ft. 75 sq. in.


is its
foot,what

diameter?

13.54

II. MEASUREMENT

OF

in.

SOLIDS.

DEFINITIONS.

1. A

253.

solid,

has

body,

or

length,breadth, and

thickness.
2. A

which

Rem.

"

5. The

6. A

7. A

base

one

has
of

altitude

fall from

two

with

its

bases,
parallel
faces parallelograms.

cording
prism is triangular,quadrangular,etc., acthe shape of the base.

right prism

4. The

areas

solid with

and
^^olygons,

are

to

3. A

is

prism

its faces

rectangles.
prism is the perpendicularlet

upon
surface

convex

other.

the
of

prism

is the

sum

of the

of its faces.

parallelopipedon
right

solid with

six

is

prism
is

parallelopipedon

with

its bases

rectangularfaces (Art.

70).
8. A
square
9. A

which

is

cube

solid

with

six

equal

one

base,

faces.

pyramid
is

is

solid with

polygon, and

with

its faces

triangles.

lelogram
paral-

10. A

the

bodies

the

are

14. The
the

with

of

surface.
solid

with

is

distant

called

within

18. The

the

from

which

point

center.

of

volume

curved

with

point of

surface,every

equally

solid

base,

one

sphere

ing
join-

cylinderis

with
circle,and
terminatingin an apex.

17. A
curved

is

cone

line

bases.

two

surface

convex

is

surface

of the

of its curved

16. A
which

cylinderis

oi^ a

centers

area

solid

lel
paral-

two

circles,and

are

axis

15. The

is

which

cylinder^

surface.

curved

the

of

middle

the

to

the cone^ and the sphere.


13. A cylinder is a solid with

bases,

is

rightpyramid

vertex

round

three

12. The

equal.

base.

side of the

each

the

from

distance

of

height

slant

all its faces

has

right pyramid

11. The

ARITHMETIC.

PRACTICAL

NEW

RAY'S

324

body

is its

contents.

1. To

254.
Rule.

"

2. To

Rule.

find

convex

surface

Multiplythe perimeterof
find the

"

the

convex

surface

of

the base

of

rightprism.

by

the altitude,

cjdinder.

Multiply the circumferenceof

the

base

by

the

altitude.

3. To

find

cylinder.

the

entire

surface

of

prism,

or

of

MEASUREMENT

Rule.

To

"

the

OF

SOLIDS.

surface add

convex

the

325

of the

areas

two

bases.

1. Find

the

surface

of

cube,

each

side

being 37

6 sq.

2. Find

the

of

base, each side


prism is 5 ft.

the

3. Find
2

surface

ft

the

in.

4. Find

the

whicii

of

is 4 ft.; the

of

and

box

1 ft. 10

surface

of

which

sq. in.
gular
trian-

altitude

73.85

surface

wide,

yd. 3 sq. ft. 6


right prism, with a

in.

of

-f-sq.

is 3 ft. 6 in.

ft.

long,

in.

high..
42^ sq. ft.
its altitude being 5
cylinder,

ft. and

the

255.

To

Rule.

Rem.
is

The

given

find

the volume

rule for

in Art.

1. Find
which

of the

Multiply the

"

"

radius

volume

ft. 4

or

volume

of

of

cylinder.

the

altitude.

by

sq. ft.

rightparallelopipedon

of

12

of
right parallelopipedon,

width

ft.,the

ft. 3

in.,and

in.

169

ft. 3

then, the volume


cu.

prism

of the base

area

findingthe

length is

SoLTJTiON."

of

87.96

70.

the

the

ft.

base, 2

of

in.

the

cu.

the
ft.

^ or -i/ft.;
3|:or -i^3ft.; 4 ft. 4 m.
-^^^
parallelopipedonis 12 X -V"XV'^
=

ft.

2. How

many

ft. 6 in.

wide,

cubic
and

10

yards

in

ft. 7 in.

24

room

its altitude

side of the base


is 14

of

ft.: what

yd.
prism
triangular

is the

volume

cu.

ft. and

the

volume

of

cylinderwhose

the radius of the base 2 ft.

cu.

is 2

ft.

ft.;

of the

24J
4. Find

long,18

high?
174

3. Each

ft.

150.8

cu.

prism?
ft. nearly.

altitude is 12
cu.

ft. nearly.

5. How

PRACTICAL

NEW

RAY'S

326

inches

cubic

many

of the bottom

diameter

ARITHMETIC.

in

peck

in. and

being 9J

the

measure,

the

in. ?

depth 8

537.6 -f cu. in.

256.

the

find

To

1.

surface

of

the

by

convex

right

pyramid.
Multiply the perimeterof
and take half the product.
height^
Rule.

"

find the

2. To

of

surface

convex

the base

"

find

3. To

the

of

surface

entire

the slant

cone.

Multiplythe circumferenceof
and take half the jnoduct.
height^
Rule.

base

by the

slant

of

p3'ramid or

cjone.

Rule.

To

"

1. Find

the

the entire

the

surfaceadd

convex

surface of

height is

slant
6

is the
25

of

surface

convex

ft. and

the

is 5

72.3

the entire

height is

of which

cone

of the

surface

ft. 7 in. and

of

the

cone,

diameter

of the

base

27.6

To

257.
"

the volume

find

Multiply the

take one-third

1. Find

each

base

the
8 ft.

sq. ft. nearly.


of which
the slant

in.

and

-f sq. ft.

333.8

3. Find

Rule.

ft. 4 in. ; the

in.

diameter

in.?

11

base.

right pyramid, with

triangularbase, each side of which


slant heightof the pyramid is 7 fl. 6
2. What

of the

area

the

side of the

of

volume
base

of

pyramid

of the
product.

area

the

of

base

the

pyramid,

square

is 5 ft. and

by

the

altitude

ft.

+ sq. ft.

of

or

cone.

altitude^

of
21
175

which
ft.
cu.

ft.

MEASUREMENT

15

2. Find

the

volume

ft. and

the

radius

3.- A

base

4. The
37

ft. 8

of

ft.: how

SOLIDS.

base

is 477

diameter

of the base

in.,and

its altitude

the

ft.

392.7

To

258.

Rule.

"

of

ft. 9 in.

79

the

surface

Multiply the

is the

of

of

square

surface

of

the diameter

sphere,of

is 4

which

of

surface

sphere,of

suppositionthat

of

area

it be

earth's

the

To

259.

Bule.

"

find the

Multiply

of 3.1416, or
1. Find

the

o-f a

volume

the

cube

of

diameter?

in

+ sq. mi.

sphere.

the

diameter

one-sixth

by

.5236.

the

volume

of

sphere 13

ft. in

the

of

volume

sphere

diameter.

volume

of

sphere

is

cu.

CU.

ft.

eter.
ft. 6 in. in diam8.2

3. The

the

miles

1150.3
2. Find

eter
diam-

+ sq. ft.

7912

196663355.75

diameter?

sq. ft.

surface,on

perfect sphere

ft.

eter
diam-

63.6

the

is

cu.

the

which

ft. 6 in. ?

3. What

is the

3.1416.

by

3.14

is the

ft.

sphere.

is 1ft.?
2. What

what

29622 +

find

1. What

cu.

is

high ; each side of its


yards in the pyramid ?
3052800
cu.
yd.
a
conical,glasshouse, is

inclosed?

space

altitude

ft.

cubic

many

327

of which

cone,

of the

pyramid

square

is 720

OF

cu.

ft.

ft.: what
14.9

in.

nearly.
is its

nearly.

UAY\S

328

III.

AKITHMETIC.

PRACTICAL

NEW

APPLICATIONS

MENSURATION.

OF

Plastering,house-painting,
paving, paperby the square foot or square
hanging,etc.,are meaBured
yard.
2. Glazing is measured
by the square foot or by the
260.

1.

'

pane.

cuttingis measured by the square foot.


Flooring,roofing,etc., are measured
by the

3. Stone
4.

yard

by

or

the

square

of 100

square

sq. ft.

long,16 fl. 3 in. broad, 10


ft. 1 in. high : how
in it,deducting
yards of plastering
many
6 ft. 3 in. by 4 ft. 2 in. ; a door 7
a
fire-place
ft. by 4 ft. 2 in.,and
two
windows, each 6 ft. b}'3. ft.
108 sq. yd. 8 sq. ft. 6 sq. in.
3 in. ?
is 20 ft. long,14 ft. 6 in. broad, and
10
2. A
room
ft. 4 in. high: what
will the papering of the walls cost,
4 ft. by
at 27 ct. per square
3'ard,deductinga fire-place
4 ft. 4 in.,and two
windows, each 6 ft. by 3 ft. 2 in.?
1. A

is 20

room

ft. 6 in.

SI 9.73.
3. What

will

rectangularcourt, 21
5 ft.
a
foot-path,
yd. long and^VlS j". broad, in which
the whole
wide, runs
length:the path paved with flags,
at 36 ct. per square
3'ard,and the rest with bricks, at
24 ct. per square
^79.80.
yard ?
4. At 10
ct. a square
yard, what will it cost to paint
both
sides of a partition15 ft. 6 in. long,12 ft. 6 in.
S4.31.
high?
5. A
tier:

house
the

it cost

has

three

heightof

the

to

pave

tiers

of

first tier

in a
window^s, seven
is 6 ft. 11 in.; of the

ft. 4 in. ; of the third,4 ft. 3 in. ; each window


what
will the glazingcost, at
is 3 ft. 6 in. wide:

second, 5
16

ct. per

square

foot?

$64.68.

APPLICATIONS

floor

6. A

will it cost

to

long,16 ft.
$3 a square?

lay it,at

long,the

ft.

40

MENSUKATION.

ft. 3 in.

is 36

what

S3.50 jDer square,

7. At
roof

OF

rafters

will

in. wide

the
side

"

18

261,

1.

lumber

ft. 6 in.

inch

is sawed

to

in

measuring all

boards, planks,etc.
is 1 foot long, 1 foot wide,

measure,

the

find

1. Find

"

number

of

feet in

the

surfaceof the hoard


surfaceby the thickness

Multiplythe
many

board,

feet in

inch

an

board

in square

feet

many

long and

in inches.

16

ft.

long

many
in. wide, and
4. How

and

20

ft.

other

many
in.

two-inch

plank

12

wide,

56

piece of scantling15

feet of

feet in
at

inch

one

an

will

boards

ft. square

long,
ft.

stick of timber

make?

inch

end,

board,

and

11

ft.

576
12

in.

ft. 6 in.
wide

at

long
the

13i|
MASONS'

1. Stone

AND

BRICKLAYERS'

masonry

is

fl.

WORK.

usuallymeasured
perch,which is 24f or 24.75 cu. ft. (Art. 70).
2. Bricklaying is commonly
measured
by
bricks.

ft.

ft.

15

end?

262.

ft. 6 in.

in. thick?

long and

5. How
1 ft. 3

feet in

many

ft.

in

ft. 3 in. wide?

3. How

12

we

feet.

in. wide?

2. How

into

used

following

1. How
ft. 3

is

thick.

Hence,
the

MEASURE.

Measure

foot, board

Rule.
2.

Board

which

2. A

have

$51.80.
BOARD

3.

of

cost

long?

and

what

$17.94.

be

each

on

329

by
the

the

1000

RAY'S

330

PRACTICAL

NEW

ARITHMETIC.

wall

perches in a stone
many
ft. 3 in. high,2 ft. 3 in. thick?

1. How
18

is the cost of

2. What
ft. 6 in.

ft. thick,at ^2.25

high,2

3. How
ft. 6 in.

in

bricks

many

53 ft. G in.

ft. 4

48

ft. 6 in. thick,allowing20

high,1

in.

long,16

bricks to the
23925.

ft.?

cubic

bricks,each 8 in. long,4 in.


many
120 ft.
in. thick, will be requiredfor a wall

wide, 2.25

4. How

the

5. Find

of

cost

thick,at $3.25
long,4 in. wide, and 2

Rule.

"

the

Find

240

1000, each

per

ft.

long,6 ft.
brick being 9

in. thick.

S336.96.

OR

BUSHELS

BY

find the

1. To

263.

wall

buildinga

MEASUREMENT

ft.

34560.

ft.

high, 3

long,8

1 ft. 6 in. thick?

high,and

in.

long,12
$121.59.

perch?

wall

long,
161.6

wall

stone

97 ft. 5 in.

of bushels

number

volume

GALLONS.

divide

inches , and

cubic

in

(Art.61).

by

2150.4.
2. To

find the

Rule.

"

number

the

Find

of

volume

gallons(Art.64).

in cubic

divide

and
iiiches,

by

231.
1. How
and

bushels

many

wide, and
3. How

diameter

15

ft.

long,5

ft.

gallonsin
ft. deep?
bushels

many
and

the

in

trough

10

ft.

barrels,of

depth

of
cylinder,

ft.?

long,5
1496

tub
cylindincal

8 ft. deep?

many
of
form

wide,

241

many

4. How

and

bin

ft. deep?

2. How

in the

in

31 J

gal.each, in

which

the

+.
ft.

+.

ft. in

181.76+.
a
cistern,

diameter

is 4 ft.

17.9+

bl.

I. ARITHMETICAL

PROGRESSION,

264.

1. An

Arithmetical

numbers

which

increase

2. If the

if it

series

1,

8,

5,

7,

20,

17,

14,

11,

3. The

numbers

the first and

every

the

the

are

series

called

terms;

the other

terms,

are

extremes;

i\\Q firstterm,

(1)

(2)

the last term,


of terms, and

the number

ered
consid-

(3) the
(5) the

mon
com-

sum

the terms.

CASE

To

265.

1. I

bought

yard, 7
:

find the

what

ct.

10

yd.

for the

did

I.

last term,

and
difference,

common

on

increasingseries.
decreasingseries.

series,five things are

arithmetical

difference^
(4)

an

11, etc.,is an

forming

last terms

difference.
series;
increasing

common

5, etc., is

series of

means.

4. In

of

by

it is called
increase,
series.
decrease,a decreasing

Thus,

the

decrease

or

is

Progression

the
of

when

number

muslin,

second,

11

the first term, the


of terms

at

ct.

ct. for the

given.

are

for

the

third,

and

the last A^ard cost ?

(331)

first
so

Solution.
add

ct.

of the

the

4 ct.

of the

last term

3 ct.

so

add

on;

-f 3 6

36

3 9

hence, to find the

cost

to the cost of the

times

-f 36 ct.

39

ct.

ct.,the

decreasingscries

4, and

difference

operation.

first;but
of the

cost

last

progression.
of

first term

jourth,

and
first,

4 ct. nine

yard,

the cost of the

to

the

of

second

first;to find the

ct. twice

ct.,and

36

are

2. The
mon

cost

cost

yard,add

of the tenth

yard,or

the

to the cost of the

times

times

cost of the

third, add

find

first;to
three

to

once

of the

cost

fihd the

To

"

ARITHMETIC.

PRACTICAL

NEW

KAY'S

332

number

the

is

of terms

39, the
10

com-

find the

last term.
OPERATION.

Solution.
9

he

4 must

will

39, which

from

times

this case,

In

"

for the

give 3

4 X

subtracted
last

3 9

3 6

==:

36=

"

term.

Rule.

of

1.

"

less

terms

If

2.

term;

Multiplythe

difference
by the number

common

one.

the product to the first


add
increasingseries,
subtract the product from the
decreasingseries,

an

if a

firstterm.
3. Find
the

the last term

first term

number

is

of terms

4. What

2, the

5. What

is

140, and

is

of

term

To
and

of

term

329, and

1. The
number

find
the

the

common

common

number

first term
of terms

the

3, and

of
7:

series in
decreasing

difference

common

CASE

266.

difference

common

in which

149.

is the 99th

the first term

increasingseries

an

50.

is the 54th

the first term

of

of

what

which

2?

series
decreasing
difference ^?

34.
in which

243J.

II.

when
difference,
terms
are
given.

series is
is the

the

2, the last 20, and


common

difference?

tremes
ex-

the

FROGKESSION.

AKITHMETICAL

Solution.
and

is

less 1, is 3, the

terms

Rule.

of

less

terms

goes

5 mi. the

first

day, and

the

day by

To

when

the

50

mi.

find

is

term

find

Solution.
In

of

Rule.

of terms;
2. The
24

strike

in

eled
trav-

the

miles; on

last

5 mi.

III.

terms

of

terms

of

the

of terms
the

series

given.

are

series whose

first

11.

is
are

and

12^

the same,

obviously as

by

take

extremes
sum

12^

their

sum

12^ 12^

12;

12.

is twice the first series;

sum

times

many

terms; hence, the

are

12, the
of the

of the

sum

series is 6

tremes,
ex-

times

36.

Multiplythe

find the

3. How

da. ; he

10

dailyincrease.

the number
6

in

it is

series
is

there

"

of terms

series is

order

sum

72 divided

ber
num-

the distance

of

of all the

last term

The

the two

as

12

sum

of the two

their

the

sum

Since
but

"

by

number

Bangor

increases

the

sum

the

inverted

The

the

and

1, and

18

33.

number

same

extremes

1. Find

the extremes

to

CASE

267.

"

diiferenco.

common

goes

20

18^6=

300 ; the

3 and

are

Boston

he

day

difference
of

from

each

of

difference.

travels

3. A

operation.

one.

extremes

find the

number

the

by 6,

common

the

Divide

"

2. The

10

divided

18

18;

20

difference of the extremes

The

"

333

sum

half the
are

of the

strokes
many
12 hours?

of the extremes
product.

and

50 ;

the

by

number

the

number

of

624.

series.
does

terms,

the

hammer

of

clock
78.

PRACTICAL

NEW

RAY'S

334

ARITHMETIC.

apples in a right line,3 yd. fi-om each


basket:
what
distance will
3 yd. from
a
other, the first,
a
gathersthem singlyand placesthem
boy travel who
17 mi. 69 rd. ^ yd.
in the basket?
5. A
weight,if not resisted
body fallingby its own
by the air,would descend in the first second a space of
4. Place

ft. 1

16

in.;

the

second, 3

next

that

times

space;

space ; the fourth,7 times, etc. : at


it fall in 1 minute?
rate, through what space would
57900

II.

268.

1. A

numbers

by

is

Progression,

series of

or
multiplier^
decreasing

common

1,

3,

9,

27,

81, is

48,

24,

12,

6,

3, is

or
multiplier

common

an
a

increasinggeometric series.
decreasinggeometric series.
is
divisor,

common

called

ratio.

Thus, in
second,

(1) the
;

first of

numbers

first and
4. In

the

the

above

series,the

ratio

is

3; in the

2.

3. The

terms

Geometrical

divisor.

common

2. The

ft.

PROGRESSION.

OEOMETRICAL

by
increasing

Thus,

the

the

that

times

third,5
that

100

forming

last terms

are

the

series

are

the

terms; the

extremes; the others,means.

five thingsare
considered :
geometricseries,
firstterm ; (2) the last term ; (3) the number of
(4) the ratio ; (5) the sum of the terms.
every

CASE

269,

ratio,and

To

find the
the

number

last

I.

term, when

of terms

are

the

first

giv^en.

term, the

GEOMETRICAL

1. The

Solution.

the

Each

as

is the

first term

after the

as

factor;but 3, taken

2. The

192;

first term

the

Solution.
the

third

The

"

is 192

Rule.

2.

the

If

this power;
3. The

2;

divided

the

-f-

of

3^

is the
is

factor,is the 4tt\\power

162.

fourth

series

is

term?

192; the second


the

four

fourth

is

term

is 192

192-7-2;

divided

by

24.

of

to

power

whose

increasingseries

an

terms,
of

is the

ratio, 3: what

find the

13:

exponent is

an

tenth

8192.

last term.
is

; the

4.

10;

the

196830.

term?

II.

of all the

sum

ratio,

is 262144

increasingseries

CASE

find the

find the

is 2 ; the

last term.

decreasingseries

of

first term

To

=.

as

of the

3 taken

decreasinggeometric

of terms, 9

; number

270.

by
multipliedby

of terms.
series be increasing,
multiplythe firstterm by
divide the firstt^m^by the power.
if decreasing,

first term

5. The

times

multipliedby

the number

the number

number

ratio,4

as

the ratio

first term

4. The

is denoted

by 2X2;

2=* =

1. Raise

"

less than

one

of

first term

that is,192

2X2;

the third,
second, 2X3;
and the fifth,
2X3X3X3X

is 2 X

what

ratio,2:

2; the

consists of 2

Hence, the fifthterm

of 3.

is

less one,

term; then, the fifth term


times

fifth term

consists of the first term


first,

times

many

335

increasinggeometric series,is

an

fourth,2X8X3X3;

term

the ratio

what

The

"

2X^X3;
3.

of

first term

ratio,3

2 ; the

PROGRESSION.

terms

of

geometric

series.
1. Find
whose

the

of

sum

first term

is

4, and

terms

of

ratio 3.

the

geometricseries,

PvAY'S

336

Solution.
each

right,thus

12

times

upper

line

have
the

of

the divisor

2 is

972

the

the

once

be

+ 97

sum

=r^

sum

is twice

the

the

=:=^

multiply

term

toward

of the

series.

the

of the
divided

4=^968
In

this

given

series

the

the ratio less

the

least

the

the remainder

series.

if the

divided

Performing

the

this

is 484,

quotient

is the

972

lower

hence,

sum;

by 2;

process,

by

series,and

the

lower, and

sum

3.

of the

sum

difference

"

equal to

When

"

product

of

the

ratio, 4 is the least term, and

one.

the

by
divide

term;

ratio; from

the

remainder

the

by

doubled

what
4.

did

I sold
the

his

gave

it the

sum

the

decreasing,and

is 10 ;

is the

what

father

is

number

of terms

she

first

16

the
of

sum

ratio, 3
the

daughter
of

day

the

number

series?
T^ew

on

Year's

month

every

of

10930.

day $1;

for

receive?

lb. of

second,

year:

84095.

gold

at

ct. for

the

ct. for

the

third,etc.

first
:

; 4

oz.

what

ct.

did

sum

$55924.05.

get?
5. Find

finite,
in-

is 0.

first term

terms, 7:
3. A

series

last term

2. The

324

from

will

subtract

the

for

then

ratio less 1.

Rem.

he

below;

as

product one

Multiply the greatest term

"

product
the

is

series.

of

greatest term

324

their

quotient

sum

8 +

subtracted

be

the

Rule.

line

upper

operation,we
the

the

remove

108

the sum,

three

by 2,

36

the

Since

series

12-f36-flO

ARITHMETIC.

of the

the terms

ratio,and

the

by

term

Write

"

PRACTICAL

NEW

the

greatest

term

+ T0V0'

^tc.

sum

of

is .3 and

infinite

an

the

series, of

ratio,10;

that

which

is,of

yu

the
~r

toj^

^.
^

6. Find

the

sum

of the

infinite series

i, i, 2V?

7. Find

the

sum

of the

infinite

i, ^, J, etc.

series

^^c.

J.
1.

Potrebbero piacerti anche